You are on page 1of 909

909 PAGES

OBS & GYN PART 1


SHEET AL-NET COMPILED FOR SUDAN MEDICAL SPECIALIZATION BOARD OBST & GYN PART 1

BASIC SCIENCE MATERIAL AND QUESTIONS


ABDOMINAL WALL MUSCLES

ANTERIOR ABDOMINAL WALL

External oblique *

• ORIGIN: Outer surface of lower 8 ribs


• INSERTION: Xiphoid process, linea alba, pubic crest and pubic
tubercle and the anterior half of the iliac crest

Internal oblique *

• ORIGIN: Lumbar fascia, anterior 2/3 of iliac crest and lateral 2/3 of
inguinal ligament

• INSERTION: Lower border of lower 3 ribs and costal cartilages,


xiphoid process, linea alba and symphysis pubis

• Lower free border gives rise to cremasteric fascia

Transversus abdominis *

• ORIGIN: Deep surface of lower 6 costal cartilages, lumbar fascia,


anterior 2/3 of iliac crest and lateral 1/3 of inguinal ligament

• INSERTION: Xiphoid process, linea alba and symphysis pubis

• Conjoint tendon formed by insertion of internal oblique and


transversus abdominis onto pubic crest and pectineal line

Rectus abdominis *

• ORIGIN: Symphysis pubis and pubic crest

• INSERTION: 5th,6th and 7th costal cartilages and xiphoid process

• Lateral margins form the Linea semilunaris

Pyramidalis *
• ORIGIN: Anterior surface of pubis
• INSERTION: Linea alba

NERVES
• External oblique, internal oblique and transversus: Lower 6 thoracic
nerves and L1 (ileoinguinal and ileohypogastric nerves)

• Rectus abdominis: Lower 6 thoracic nerves

• Pyramidalis: T12 *

1
FUNCTION

• External / internal oblique / transversus - laterally flex and rotate the


trunk, relax during inspiration to accommodate abdominal viscera,
contract during micturiction, defecation and vomiting.

• Rectus abdominis - flexes the trunk and stabilises the pelvis

RECTUS SHEATH *****

Above costal margin


• Anterior wall: Aponeurosis of external oblique *
• Posterior wall: thoracic wall. *

Between costal margin and anterior superior iliac spine *

• Anterior wall: Aponeurosis of external and internal oblique

• Posterior wall: Aponeurosis of internal oblique and transversus


abdominis

• Note that the aponeurosis of the internal oblique splits to enclose the
rectus abdominis

Between anterior superior iliac spine and pubis *

• Anterior wall: Aponeuroses of external, internal oblique and


transversus

• Posterior wall: Transversalis fascia

• Arcuate line *: Site where the aponeuroses of the posterior wall pass
anterior to the rectus at the level of the anterior superior iliac spine. The
inferior epigastric artery enters the rectus sheath at this point and lies
posterior to the rectus abdominis. Pyramidalis lies within the rectus
sheath.

POSTERIOR ABDOMINAL WALL

PSOAS MAJOR *****

• Origin: Roots of transverse processes, sides of vertebral bodies and


inter-vertebral discs T12 - L5
• Insertion: Lesser trochanter of the femur
• Nerve: Lumbar plexus

• Action: Flexes thigh on trunk. If thigh if flexed, flexes trunk on thigh.

2
ILIACUS

• Origin: Iliac fossa

• Insertion: Lesser trochanter of the femur

• Nerve: Femoral nerve

• Action: Flexes thigh on trunk. If thigh is flexed, flexes trunk on thigh

QUADRATUS LUMBORUM

• Origin: Iliolumbar ligament, iliac crest and tip of transverse processes


of lower lumbar vertebrae

• Insertion: Lower border of 12th rib and transverse processes of upper


4 lumbar vertebrae

• Nerve: Lumbar plexus

• Action: Laterally flexes spine to same side, fixes or depresses 12th


rib during respiration

The Pfannestiel incision cuts through *****

• Skin and superficial fascia including Scarpa’s fascia

• The rectus sheath which is formed at this level by the aponeuroses of


the external, internal oblique and transversus abdominis

• The linea alba is split to separate the recti abdominis which are NOT
cut

• The transversalis fascia

• The parietal peritoneum

• Incisions should be made along Langer’s lines which run horizontally


in the lower abdomen *

3
AORTIC ARCH

• Runs upwards, backwards and to the left, initially anterior then to the
left of the trachea

Branches

• Brachiocephalic artery which divides into the right common carotid and
right subclavian arteries
• Left common carotid artery
• Left subclavian artery

DESCENDING THORACIC AORTA

• Begins on the left side of the lower border of T4


• Descends in the posterior mediastinum and ends at the aortic opening
of the diaphragm at the level of T12 where it continues as the
abdominal aorta

Branches

• Posterior intercostal arteries


• Subcostal artery
• Pericardial, oesophageal and bronchial arteries

THE PULMONARY TRUNK

• 5cm long begins at the pulmonary valve, runs upwards, backwards


and to the left, terminating in the concavity of the aortic arch by dividing
into the left and right pulmonary arteries. Lies anterior to the trachea
• Connected to the aortic arch by the ligamentum arteriosum (remnant
of the ductus arteriosus)
• The left recurrent laryngeal nerve hooks around the ligamentun
arteriosum

INTERNAL THORACIC ARTERY *****

• Branch of the first part of the subclavian artery


• Supplies the anterior wall of the body from the clavicle to the umbilicus
• Descends on the pleura deep to the costal cartilages
• Ends at the 6th intercostal space dividing into the superior epigastric
and musculophrenic arteries

1
• Gives off two anterior intercostal arteries to the upper 6 intercostal
spaces
• Pericardiophrenic artery to the pericardium
• Mediastinal arteries to the anterior mediastinum including the thymus
• Perforating arteries to thoracic and anterior abdominal wall
• The internal thoracic vein drains into the brachiocephalic vein

SUPERIOR VENA CAVA *****

• Formed from the left and right brachioceplalic veins (which are formed
from the subclavian and internal jugular veins)
• Receives the azygos vein just before piercing the pericardium
• Lies anterior to the trachea and aorta in the superior mediastinum

THORACIC DUCT *****

• Begins in the abdomen as the cysterna chyli


• Enters thorax through aortic opening of the diaphragm on the right side
of the descending aorta
• Crosses the mid-line posterior to the oesophagus
• Ascends on the left side of the oesophagus within the superior
mediastinum
• Drains into the left brachiocephalic vein
• Receives the LEFT bronchomediastinal, jugular and subclavian lymph
trunks
• Conveys lymph from the lower limbs, pelvis, abdomen, left side of the
thorax, left side of the head and neck and left upper limb into the blood
stream

Lymph from the right side of the head and neck, right upper limb and right side of the
thorax is drained by the right jugular, subclavian and bronchomediastinal trunks
respectively. These may unite to form the right thoracic duct which drains into the
right brachiocephalic vein or drain independently in to the vain.

2
TRUE PELVIS

• That part of the pelvis between the pelvic inlet and the pelvic outlet

• The pelvic inlet is oval in shape in the Gynaecoid pelvis and has the
following borders:

1) Posterior: Sacral promontory *


2) Lateral: Iliopectineal lines *
3) Anterior: Symphysis pubis *

• The pelvic outlet is diamond shaped with the following borders:

1) Posterior: Coccyx *
2) Lateral: Ischial tuberosities *
3) Anterior: Pubic arch formed by the simphysis pubis and the ischio-pubic
rami *

• Part of the lateral border is formed by the sacro-spinous and sacro-


tuberous ligaments which convert the greater and lesser sciatic
notches into the greater and lesser sciatic foramina *
• The pelvic cavity has a shallow anterior wall and a deeper posterior
wall. The ischial spines are at the level of the mid-cavity
• The acetabulum is formed from the ilium, ischium and pubic bones *

THE SACRUM *****

• 5 rudimentary vertebrae
• The anterior and upper margin of the first sacral vertebra bulges
forward as the posterior margin of the pelvic inlet and forms the sacral
promontory *
• Articulates with innominate bone at the sacro-iliac joint, with the 5th
lumbar vertebra superiorly (where it is tilted forwards at the lumbo-
sacral angle) and with the coccyx inferiorly
• The laminae of S5 and occasionally S4 fail to fuse in the mid-line,
forming the sacral hiatus

1
• The vertebral foramina form the sacral canal which contains the
cauda equine, filum terminale and meninges as far as the lower border
of S2 while the rest of the canal contains the filum terminale and lower
sacral and coccygeal nerves *
• The anterior / posterior surfaces contain 4 foramina for the anterior
and posterior rami of the upper 4 sacral nerves respectively *
• The piriformis muscle arises from the front of the lateral mass of the
sacrum and leaves the pelvis through the greater sciatic foramen to
insert onto the greater trochanter *

THE SACRO-ILIAC JOINT

• Synovial joint - the irregular articular surfaces of the joint make a


contribution to joint stability but this is mainly maintained by the very
strong posterior and inter-osseous sacro-iliac ligaments.

• The sacro-spinous and sacro-tuberous ligaments also contribute to


joint stability

• Supplied by branches of the sacral plexus and POSTERIOR rami of


S1

Differences between the Male and Female Pelvis *****

The female pelvis:

• Less massive
• The anterior iliac spines more widely separated - greater lateral
prominence of the hips.
• Wider pelvic inlet - both antero-posterior and transverse diameters
• Pelvic inlet more circular
• More shallow
• The sacrum is shorter wider, and its upper part is less curved

The obturator foramina are triangular in shape and smaller in size

• The outlet is larger and the coccyx more movable.


• The sciatic notches are wider and shallower
• The ischial spines are less prominient.
• The pubic symphysis is less deep, and the pubic arch is wider and
more rounded than in the male

2
3
EXTERNAL GENITALIA

• Labia majora: prominent hair-bearing folds of skin extending from


the mons pubis to fuse posteriorly in the mid-line

• Labia minora: hairless folds of skin within the labia majora; unite
posteriorly to form the fourchette; split anteriorly to enclose the clitoris,
forming an anterior prepuce and a posterior frenulum

• Vestibule: triangular area bounded laterally by the labia minora


with the clitoris at its apex and the fourchette at its base

• The urethra opens within the vestibule posterior to the clitoris

THE ANAL TRIANGLE

• Bounded posteriorly by the tip of the coccyx and laterally by the


ischial tuberosities
• Skin supplied by inferior rectal nerve
• Lymphatic drainage is to the medial group of superficial inguinal
nodes

THE ANAL CANAL *****

• ~4cm long, extends downwards and backwards from the rectal


ampulla to the anus
• LATERAL walls kept in apposition by the levator ani and anal
sphincter except during defecation
• Anterior relations: perineal body, urogenital diaphragm and
perineal body *

• Posterior relations: anococcygeal body and the coccyx *

UPPER HALF

1
• Derived from hind-gut endoderm *
• Lined by columnar epithelium *
• Thrown into vertical folds called anal columns *
• Autonomic supply from the inferior hypogastric plexus, sensitive to
stretch only *

• Blood - superior rectal branch of inferior mesenteric artery *


• Lymphatics - inferior mesenteric nodes *
• Dentate line - boundary between endodermal and ectodermal origin of
anal canal

LOWER HALF

• Derived from the ectoderm of the proctodeum *


• Lined by stratified squamous epithelium
• No anal columns
• Blood - inferior rectal artery, branch of internal pudendal artery *
• Lymphatics - medial group of superficial inguinal nodes *
• Nerve - inferior rectal nerve - sensitive to temperature, pain, touch and
pressure *

ANAL SPHINCTER *****

Internal

• Thickening of the inner circular layer of smooth muscle in the upper


half of the anal canal
• Lies deep to the outer layer of longitudinal smooth muscle

External

2
• Skeletal muscle *
• Sub-cutaneous part - lower half of anal canal, no bony attachments *
• Superficial part - attached to the perineal body anteriorly and the
anococcygeal body posteriorly *
• Deep part - no bony attachments, blends with puborectalis *

ISCHIO-RECTAL FOSSA *****

• Wedge shaped space filled with dense fat


• Base formed by skin
• Medial wall formed by anal canal and levator ani muscles
• Lateral wall formed by lower part of obturator internus muscle
• Contains the pudendal nerve and internal pudendal vessels within the
pudendal canal on the lateral wall

3
INGUINAL LIGAMENT *****

• FORMED BY THE APONEUROSIS OF THE EXTERNAL OBLIQUE


MUSCLE

• ATTACHED MEDIALLY TO THE PUBIC TUBERCLE

• FORMS THE INFERIOR WALL OR FLOOR OF THE INGUINAL CANAL

INGUINAL CANAL *****

• 4CM LONG IN ADULTS

• DEEP RING: - OVAL SHAPED HOLE IN TRANSVERSALIS FASCIA,


1.3CM ABOVE INGUINAL LIGAMENT, MID-WAY BETWEEN ANTERIOR
SUPERIOR ILIAC SPINE AND SYMPHYSIS PUBIS. MEDIAL RELATION -
INFERIOR EPIGASTRIC VESSELS. GIVES RISE TO THE INTERNAL
SPERMATIC FASCIA OR THE INTERNAL COVERING OF THE ROUND
LIGAMENT OF THE UTERUS *

• SUPERFICIAL RING:- TRIANGULAR IN SHAPE, DEFECT IN


EXTERNAL OBLIQUE APONEUROSIS, BASE FORMED BY PUBIC
CREST, GIVES RISE TO EXTERNAL SPERMATIC FASCIA *

• THE FOLLOWING PASS THROUGH THE SUPERFICIAL RING: ROUND


LIGAMENT, ILIOINGUINAL NERVE, GENITAL BRANCH OF THE
GENITOFEMORAL NERVE, LYMPHATICS AND SYMPATHETIC
PLEXUS *

• ANTERIOR WALL - APONEUROSIS OF EXTERNAL OBLIQUE,


REINFORCED LATERALLY BY ORIGIN OF INTERNAL OBLIQUE *

• POSTERIOR WALL - TRANSVERSALIS FASCIA, REINFORCED


MEDIALLY BY CONJOINT TENDON - COMMON INSERTION OF
INTERNAL OBLIQUE AND TRANSVERSUS TO THE PUBIC CREST AND
PECTINEAL LINE *

• FLOOR - INGUINAL LIGAMENT *

• ROOF - INTERNAL OBLIQUE AND TRANSVERSUS ABDOMINIS *

• THE FEMORAL NEUROVASCULAR BUNDLE AND LATERAL


CUTANEOUS NERVE OF THE THIGH PASS BENEATH THE INGUINAL
LIGAMENT. *

• THE FEMORAL BRANCH OF THE GENITOFEMORAL NERVE ENTERS


THE THIGH BEHIND THE MIDDLE OF THE INGUINAL LIGAMENT. *

1
• THE ILIOINGUINAL NERVE ENTERS THE THIGH THROUGH THE
SUPERFICIAL INGUINAL RING. *

• THE SUPERFICIAL EPIGASTRIC VESSELS CROSS THE INGUINAL


LIGAMENT *

INGUINAL CANAL IN MALE CONTAINS: *

• VAS DEFERENS

• TESTICULAR VESSELS

• LYMPHATICS

• SYMPATHETIC FIBRES FROM THE RENAL OR AORTIC


SYMPATHETIC PLEXUSES

• REMAINS OF THE PROCESUS VAGINALIS

• CREMASTERIC ARTERY - BRANCH OF THE INFERIOR EPIGASTRIC


ARTERY

• ARTERY TO THE VAS - BRANCH OF INFERIOR VESICAL ARTERY

• GENITAL BRANCH OF GENITOR-FEMORAL NERVE - SUPPLIES


CREMASTER MUSCLE

INGUINAL CANAL IN THE FEMALE CONTAINS: *


• ROUND LIGAMENT OF THE UTERUS

• LYMPHATICS

• REMAINS OF THE PROCESSUS VAGINALIS

• GENITAL BRANCH OF THE GENITOFEMORAL NERVE

CREMASTER MUSCLE / CREMASTERIC REFLEX


• DERIVED FROM INTERNAL OBLIQUE

• SUPPLIED BY CREMASTERIC ARTERY - BRANCH OF INFERIOR


EPIGASTRIC

• SUPPLIED BY GENITAL BRANCH OF GENITOR-FEMORAL NERVE

• CREMASTERIC REFLEX - CREMASTER MUSCLE CONTRACTS WHEN


SKIN ON MEDIAL ASPECT OF THIGH IS STROKED. AFFERENT -
FEMORAL BRANCH; EFFERENT - GENITAL BRANCH OF GENITOR-
FEMORAL NERVE

2
SCROTUM *****
• MADE UP OF FOLLOWING LAYERS (SUPERFICIAL - DEEP):
• SKIN
• DARTOS MUSCLE - SMOOTH MUSCLE, CONTINUOUS WITH FATTY /
MEMBRANEOUS LAYER OF SUPERFICIAL FASCIA OF ANTERIOR
ABDOMINAL WALL; INNERVATED BY SYMPATHETIC FIBRES AND
RESPONSIBLE FOR WRINKLING OF OVERLYING SKIN

• COLLES- FASCIA CONTINUOUS WITH SCARPA’S FASCIA

• EXTERNAL SPERMATIC FASCIA - EXTERNAL OBLIQUE

• CREMASTERIC FASCIA - INTERNAL OBLIQUE

• INTERNAL SPERMATIC FASCIA - TRANSVERSALIS FASCIA

• TUNICA VAGINALIS - PROCESUS VAGINALIS - COVERS ANTERIOR


AND LATERAL BUT NOT POSTERIOR ASPECT OF TESTIS

• TUNICA ALBUGINEA - FIBROUS CAPSULE OF TESTIS - COVERS


ENTIRE TESTIS

• BOTH DARTOS MUSCLE AND COLLES - FASCIA CONTRIBUTE TO


MEDIAN PARTITION OF SCROTUM.

LYMPHATIC DRAINAGE: *****

• SKIN AND SUPERFICIAL FASCIA (INCLUDING TUNICA VAGINALIS) -


SUPERFICIAL INGUINAL NODES

• TESTIS / EPIDIDYMIS - PARA-AORTIC NODES (L1)

• FOR SUCCESSFUL SPERMATOGENESIS, THE TEMPERATURE OF


THE TESTIS IS MAINTAINED AT 3C BELOW BODY TEMPERATURE BY
VARIOUS MECHANISMS INCLUDING A COUNTER-CURRENT HEAT
EXCHANGE SYSTEM FORMED BY THE PAMPINIFORM PLEXUS
(TESTICULAR VEIN).

BLOOD SUPPLY TO TESTIS *****

• TESTICULAR ARTERY - LEAVES AORTA AT L1

• RIGHT TESTICULAR VEIN - DRAINS INTO IVC

• LEFT TESTICULAR VEIN - DRAINS INTO LEFT RENAL VEIN

• EPIDIDYMIS - TESTICULAR ARTERY

3
4
CECUM *****

• 6cm long, intra-peritoneal

• Has 3 bands of outer longitudinal muscles (Tenia coli) which


converge on the base of the appendix

• Appendix arises from its postero-medial aspect and its lumen is


in direct communication with the cavity of the cecum

• Related anteriorly to coils of small intestine, greater omentum


and anterior abdominal wall

• Related posteriorly to the psoas, iliacus, femoral nerve and


lateral cutaneous nerve of the thigh (and usually the appendix)

• Blood supply - anterior and posterior cecal arteries - branches


of the ileocolic artery which arises from the superior mesenteric
artery

• Nerves- sympathetic and parasympathetic (vagus) from the


superior mesenteric plexus

• Lymphatics - superior mesenteric nodes

APPENDIX *****

• 8-13cm long

• Arises from the postero-medial aspect of the cecum

• Lies in RIF with its base located 1/3 of the way up the line
joining the anterior superior iliac spine to the umbilicus
(McBurney’s point)

• Identified within the abdomen by followint the Tenia coli of


the caecum to its base

• Arterial supply - appendicular artery, branch of posterior


cecal artery

Lymphatics - superior mesenteric nodes

• Nerve - parasympathetic (vagus) and sympathetic from


superior mesenteric plexus
• Fibres transmitting visceral pain signals enter the spinal
cord at T10 (dermatome is in the region of the umbilicus, the
site of referred pain during acute appendicitis)

1
ASCENDING COLON *****

• 13cm long, from the cecum to the inferior surface of the


right lobe of the liver

• retro-peritoneal

• Related anteriorly to coils of small intestine, the greater


omentum and the anterior abdominal wall

• Related posteriorly to the psoas, iliacus, quadratus


lumborum, origin of the right transversus abdominis muscle;
iliohypogastric and ilioinguinal nerves; lower pole of RIGHT
kidney

• Blood supply - right colic and iliocolic branches of


theSUPERIOR mesenteric artery

• Lymphatics - superior mesenteric nodes

• Nerves - parasympathetic (vagus) and sympathetic from the


superior mesenteric plexus

TRANSVERSE COLON *****

• 38cm long, from the right colic flexure to the left colic flexure,
which is attached to the diaphragm by the phrenico-colic ligament

• Intra-peritoneal - transverse mesocolon is attached to


its SUPERIORborder

• The POSTERIOR layer of the greater omentum is attached to


its inferior border

• Typically hangs down into the pelvis

• Related posteriorly to the second part of the duodenum, head


of pancreas, coils of jejunum and ileum

• Blood - proximal 2/3 - middle colic artery (superior mesenteric);


distal 1/3 - left colic (inferior mesenteric)

• Lymphatics - proximal 2/3 - superior mesenteric nodes; distal


1/3 - inferior mesenteric nodes

2
• Nerves - proximal 2/3 - superior mesenteric plexus (vagus -
parasympathetic); distal 1/3 - inferior mesenteric plexus
(sympathetic) and pelvic splanchnic nerves (parasympathetic)

• NOTE that the junction between the proximal 2/3 and distal 1/3
of the transverse colon is the junction between the mid-gut and
the hind-gut, hence the arrangement of blood, nerve supply and
lymphatics.

DESCENDING COLON *****

• 25cm long, retroperitoneal, extends from left colic flexure to pelvic


brim

• Related anteriorly to greater omentum, coils of small intestine and


the anterior abdominal wall

• Related posteriorly to the lateral border of the left kidney, LEFT


psoas, iliacus, iliac crest, quadratus lumborum and origin of the left
transversus abdominis; iliohypogastric, ilioinguinal and femoral nerves
and lateral cutaneous nerve of the thigh

• Blood - left colic artery and sigmoid arteries (inferior mesenteric)

• Lymphatics - inferior mesenteric nodes

• Nerves - sympathetic - inferior mesenteric plexus; parasympathetic -


pelvic splanchnic nerves

DIFFERENCES BETWEEN SMALL AND LARGE BOWEL *****

• Longitudinal smooth muscle - continuous in small bowel,


3 bands (teniae coli) in large bowel

• Fatty tags - appendices epiploicae - present in large


bowel, absent in small bowel

• Payer’s patches - lymphoid tissue within mucosa -


present in small bowel, absent in large bowel

• Plicae circularis - folds of mucous membrane - present in


small bowel, absent in large bowel

• Muscle wall - smooth in small bowel, sacculated in large


bowel

3
4
LIVER *****

• The falciform ligament divides the liver into right and left
lobes and splits on the superior surface of the liver, the right
layer forming the upper layer of the coronary ligament while
the left layer forms the upper layer of the left triangular
ligament

• The peritoneal layers forming the coronary ligaments are


widely separated, leaving part of the liver devoid of
peritoneum – the - bare - area

• The ligamentum teres (remnant of umbilical vein) runs from


the umbilicus to the left branch of the portal vein

• The ligamentum venosum (remnant of ductus venosus) runs


from the left branch of the portal vein to the inferior vena cava

• The right lobe is further divided into the quadrate lobe


(between the ligamentum venosus and inferior vena cava,
superior to the gall bladder) and the caudate lobe (between
the ligamentum teres, porta hepatis and gall bladder

• The quadrate and caudate lobes are functionally part of the


left lobe and there is very little overlap in the blood supply of
the two sides

• The hepatic artery (30%, oxygenated) and the portal vein


(70%, de-oxygenated) supply blood to the liver. Venous
drainage (right and left hepatic vein) is into the inferior vena
cava.

• Although anatomically part of the right lobe, the caudate and


quadrate lobes are supplied by the LEFT hepatic artery

• The hepatic lobule contains a central vein (tributary of the


hepatic vein) while the spaces between the lobules (portal
canals) contain branches of the hepatic artery, portal vein and
a tributary of a bile duct

• Lymphatic drainage of most of the liver is to the celiac


nodes but the bare area drains into posterior mediastinal
nodes

1
The porta hepatis contains the following structures *****

• Right and left hepatic arteries

• Right and left branches of the portal vein

• Right and left hepatic ducts

• The upper part of the free edge of the lesser omentum is


attached to the liver at this point

• Sympathetic and parasympathetic nerves

• Lymphatics and nodes draining the gall bladder

BILIARY TREE *****

• The bile canniliculi drain into interlobular ducts located at


the periphery of the hepatic lobule. The interlobular ducts join
to form larger ducts and eventually form the right and left
hepatic ducts at the porta hepatis

• The caudate and quadrate lobes are functionally part of the


left lobe of the liver and are drained by the left hepatic duct

• The common hepatic duct is formed from the left and right
hepatic ducts

• The cystic duct runs from the gall bladder to join the RIGHT
side of the common hepatic duct, forming the common bile
duct

• The common bile duct is located in the right free edge of the
lesser omentum, anterior to the epiploic foramen, in front of
the portal vein and to the right of the hepatic artery

• The common bile duct then runs behind the first part of the
duodenum (to the right of the gastro-duodenal artery) then
behind the head of the pancreas to open half way down the
second part of the duodenum into the ampulla of Varter

• The common bile duct is usually joined by the main


pancreatic duct close to its opening into the duodenum. The
opening has a smooth muscle sphincter - the sphincter of
Oddi

2
GALL BLADDER *****

• Has a fundus, body and neck which is continuous with the


cystic duct

• The fundus is at the level of the tip of the 9th right costal
cartilage

• Related anteriorly to the anterior abdominal wall and the


visceral surface of the liver

• Related posteriorly to the transverse colon and the first and


second parts of the duodenum

• Arterial supply - cystic artery, branch of the right hepatic


artery

• Venous drainage - cystic vein, into the portal vein

• Lymphatics - cystic lymph node located near the neck, then


to the celiac nodes

• Nerves - celiac plexus

3
THE PLEURA

• Parietal - lines the thoracic wall, thoracic surface of the


diaphragm and lateral aspect of mediastinum

• The cervical pleura extends ~3cm above the medial third of


the clavicle to line the under surface of the suprapleural
membrane

• Supplied by the intercostal nerves (costal pleura and


peripheral part of diaphragmatic pleura); phrenic nerve
(mediastinal pleura and diaphragmatic pleura over dome of
diaphragm)

• Visceral - surrounds the lungs and receives autonomic


supply

THE LUNGS

• Right - three lobes with a horizontal and an oblique fissure

• Left - two lobes only, with an oblique but no horizontal


fissure. Has a cardiac notch

• Blood supply to the bronchi and connective tissue of the


lung is from the bronchial arteries, branches of the descending
aorta

• Alveoli receive de-oxygenated blood from the pulmonary


arteries and oxygenated blood leaves via the pulmonary veins

• Lymphatic drainage is to the pulmonary nodes close to the


hilum and then into the bronchomediastinal lymph trunks.
These two trunks ascend on either side of the trachea and
drain into the brachiocephalic vein (left) or the thoracic or right
lymphatic duct

• Parasympathetic supply is from the vagus nerve - produce


bronchoconstriction, vasodilation and increased glandular
secretion

• Sympathetic supply is from the thoracic sympathetic trunk,


cause bronchodilation, vasoconstriction and decreased
glandular secretion

1
2
PYRIFORMIS

• Origin: Lateral mass of sacrum

• Leaves pelvis through greater sciatic foramen

• Insertion: Upper border of greater trochanter

• Action: Laterally rotates femur at hip joint

• Nerve: Sacral plexus

OBTURATOR INTERNUS *****

• Origin: Pelvic surface of obturator membrane and adjoining


part of the hip bone

• Leaves pelvis through Lesser sciatic foramen *

• Covered by the obturator fascia which gives rise to the


origin of the levator ani muscle *

• Forms the lateral wall of the roof of the ischio-rectal fossa


(medial wall of the roof formed by the levator ani) *

• Insertion: Greater trochanter of femur *

• Action: Laterally rotates femur at hip joint

• Nerve: nerve to obturator internus from sacral plexus *

LEVATOR ANI MUSCLE *****

• Origin: Back of body of pubis, obturator fascia and ischial


spine

• Insertion: Anterior fibres (sphincter vaginae) - perineal body

Intermediate fibres (puborectalis) - median raphe and


anococcygeal body
Posterior fibres (iliococcygeus) - anococcygeal body and
coccyx
• Action: Support pelvic viscera and resist rise in intra-pelvic
pressure during straining; sphincter action at anorectal
junction and vagina

• Nerve: Perineal branch of S4 and perineal branch of the


pudendal nerve

1
COCCYGEUS

• Origin: ischial spine

• Insertion: lower end of sacrum and upper part of coccyx

• Action: supports pelvic viscera, flexes coccyx

• Nerve: branch of S4 and S5

2
THE VAGUS NERVES *****

Right

• Enters the thorax posterior-lateral to the brachiocephalic artery

• Descends lateral to the trachea and medial to the terminal part


of the azygos vein

• Passes posterior to the root of the right lung

• Descends onto the posterior surface of the oesophagus to


enter the abdomen and posterior surface of the stomach

Left

• Enters the thorax between the left common carotid and left
subclavian arteries

• Crosses the left side of the aortic arch

• Is crossed by the left phrenic nerve

• Descends posterior to the root of the left lung

• Then passes onto the anterior surface of the oesophagus to


enter the abdomen

Branches

• Both nerves contribute to their respective pulmonary and


oesophageal plexuses

• The right vagus gives off a cardiac branch

• The left vagus gives off the left recurrent laryngeal nerve at
the level of the ligamentum arteriosum. The nerve hooks
around the ligament to ascend between the trachea and the
oesophagus on the left side and supplies all the muscles of
the left vocal cord except the cricothyroid muscle (external
laryngeal branch of the vagus)

1
THE PHRENIC NERVES *****

Right

• Arises in the neck from the anterior rami of C3,4,5

• Enters the thorax on the right side of the right


brachiocephalic vein and inferior vena cava

• Descends anterior to the root of the right lung

• Is separated by the pericardium from the right atrium

• Leaves the thorax through the caval opening of the


diaphragm on the right side of the inferior vena cava

Left

• Enters the thorax on the left side of the left subclavian artery

• Crosses the left side of the aortic arch and the left vagus
nerve

• Descends anterior to the root of the left lung

• Is separated from the left ventricle by the pericardium

• Terminal branches pierce the muscle of the diaphragm and


supply the central part of the peritoneum on its under surface

• Provide motor supply to the diaphragm and sensory supply


to the pericardium, mediastinal pleura, the central part of the
diaphragmatic pleura and peritoneum. *

THORACIC SYMPATHETIC TRUNK *****

• Continuation of the cervical sympathetic trunk, descends on


the head of the ribs

• Arranged in 11/12 segmental ganglia

• Enters the abdomen on the side of the body of T12 by


passing behind the medial arcuate ligament

• First ganglion often fused with the inferior cervical ganglion


to form the stellate ganglion

2
• Gives grey rami communicantes to the thoracic spinal
nerves

• Receives white rami communicantes from the thoracic spinal


nerves

• Upper five ganglia give POST-ganglionic fibres to the heart,


lungs, oesophagus and aorta

• Lower eight ganglia give PRE-ganglionic fibres which form


the greater (5th-9th), lesser (10th & 11th) and lowest (12th)
splanchnic nerves

3
THE LUMBAR PLEXUS

• Formed within psoas major from the ANTERIOR rami of


L1,2,3&4

• Receives grey rami communicantes from the sympathetic trunk

• L1&2 give off white rami communicantes to the sympathetic trunk

• Branches emerge from the lateral and medial borders and the
anterior surface of psoas major

LATERAL BRANCHES

• Iliohypogastric - L1; skin of lower part of anterior abdominal wall

• Ilioinguinal - L1; enters inguinal canal and supplies skin of groin,


scrotum / labium majus

• Lateral cutaneous nerve of the thigh - L2&3; crosses iliac fossa


anterior to iliacus, supplies skin on lateral aspect of thigh

• Femoral nerve - L2,3&4; enters thigh behind inguinal ligament


lateral to femoral sheath and canal

MEDIAL BRANCHES

• Obturator nerve - L2,3&4; crosses pelvic brim in front of sacro-


iliac joint and behind common iliac vessels

• 4th lumbar root of the lumbosacral trunk contributes to the sacral


plexus

ANTERIOR BRANCH

• Genito-femoral nerve - L1&2; genital branch enters spermatic


cord and supplies cremaster muscle; femoral branch supplies skin
on medial aspect of thigh

ABDOMINAL SYMPATHETIC TRUNK *****

• Enters the abdomen through the medial arcuate ligament and


runs downwards along the medial border of psoas major and
enters the pelvis behind the common iliac vessels. The right trunk
lies posterior to the inferior vena cava

1
• Made up of 4 segmentally arranged ganglia; the upper two
ganglia receive white ramus communicans from the 1st and 2nd
lumbar nerves

• Give off grey rami communicans to the lumbar spinal nerves

• Also gives off branches to the aortic sympathetic plexuses


andthe hypogastric plexus

CELIAC PLEXUS *****

• Two celiac ganglia and a network of fibres surrounding the origin


of the celiac artery

• Receive sympathetic pre-ganglionic fibres from the greater and


lesser splanchnic nerves and gives off POST-ganglionic fibres to
the fore-gut

• Receives parasympathetic pre-ganglionic fibres from the vagus


and these form synapses within the wall of the organs supplied

• The proximal 2/3 of the transverse colon is mid-gut (superior


mesenteric plexus)

2
SACRAL PLEXUS *****

• Formed from anterior rami of L4&5 (lumbosacral trunk) and


anterior rami of S1,2,3&4

• Related anteriorly to internal iliac vessels + branches and


the rectum

• Related posteriorly to the piriformis muscle

Branches

To lower limb - leave the pelvis through GREATER sciatic foramen

• Sciatic nerve - L4,5, S1,2&3; largest nerve in the body

• Superior gluteal nerve - gluteus medius, minimus and tensor


fascia lata

• Inferior gluteal nerve - gluteus maximus

• Nerve to obturator internus - also supplies superior


gamellus muscle

• Nerve to quadratus femoris - also supplies inferior gamellus


muscle

• Posterior cutaneous nerve of the thigh

Branches to pelvic organs

• Pudendal nerve - S2,3&4 - leaves the pelvis through the


greater sciatic foramen and enters the perineum through the
lesser sciatic foramen *

• Nerve to piriformis

• Pelvic splanchnic nerves - S2,3&4 - sacral part of


parasympathetic system *

• Perforating cutaneous nerve - skin of lower medial part of


buttock *

1
PELVIC SYMPATHETIC TRUNK *****

• Continuous with the abdominal part behind the common


iliac vessels

• Lies posterior to the rectum *

• Lies anterior to the sacrum *

• Lies medial to the anterior sacral foramina *

• Has 4-5 segmentally arranged ganglia *

• Gives off grey rami communicantes to the sacral and


coccygeal spinal nerves

• Gives off fibres to the pelvic (hypogastric) plexuses *

• No white rami communicantes join this part of the


sympathetic trunk

HYPOGASTRIC PLEXUSES

Superior hypogastric plexus

• Retroperitoneal, between common iliac arteries and in front


of the sacral promontory

• Formed from aortic sympathetic plexus and branches from


the lumbar sympathetic ganglia

• Parasympathetic fibres from the pelvic splanchinc nerves


join the inferior hypogastric plexus and ascend to the superior
hypogastric and eventually inferior mesenteric plexus to
supply the hind gut

Right and left inferior hypogastric plexuses

• Branches of superior hypogastric plexus

• Descend medial to internal iliac artery

• Descend lateral to rectum

• Receive parasympathetic fibres from the pelvic splanchnic


nerves

The descending colon (hindgut) is supplied by the inferior


mesenteric plexus

2
PELVIC SPLANCHNIC NERVES *****

• Formed from the ventral (anterior) primary rami


of S2 throughS4. *

• These are the ways in which parasympathetic


neurons reach the hypogastric plexus, and therefore the
pelvic viscera and distal colon. *

• The parasympathetic part of the autonomic nervous system


is the "craniosacral" part. Parasympathetic innervation to most
of the gut comes from the "cranio-" half of that, i.e., the vagus
nerve. The rest, to colon distal to the splenic flexure and to
pelvic viscera, is from the "-sacral" half, via the pelvic
splanchnic nerves

• There are thoracic, lumbar, sacral, and pelvic splanchnic


nerves.

• "Splanchnic" refers to nerves that supply viscera.

• Thoracic, lumbar and sacral splanchnic nerves emerge from


sympathetic ganglia and carry sympathetic fibers

• Pelvic splanchnic nerves are parasympathetic *

• Contain pre-ganglionic fibres *

• Join inferior hypogastric plexus *

• Some fibres ascend to the superior hypogastric and


eventually inferior mesenteric plexus and supply the hind-gut *

• Provide parasympathetic supply to the pelvic viscera

• Afferent impulses from the pelvic viscera are transmitted


mainly by the sympathetic pathway *

• Parasympathetic nerves innervate detrusor and internal


sphincter via the pelvic splanchnic nerves (S2,3,4) and also
innervate the external sphincter via the pudendal nerve ?
initiate micturiction by inhibiting internal sphincter activity and
stimulating detrusor contraction *

3
• Parasympathetic supply to the ovary is from the vagus
nerve. Sympathetic innervation is from the L1/2 segment *

• Pain from uterine contractions goes back to T10-L1. This


means that uterine contraction pain is sympathetic. But for the
lower portion of the uterus and upper vagina (the cervix) - it is
parasympathetic back to the pelvic splanchnic nerves *

OBTURATOR NERVE *****

• Branch of lumbar plexus - L2,3,4 - anterior divisions *

• Descends through psoas major and emerges on its medial


border to enter the pelvis anterior to the sacro-iliac joint *

• Passes behind the common iliac vessels and on the lateral


side of the internal iliac vessels and the ureter. Separated
from the ureter by the internal iliac vessels *

• Runs on the lateral wall of the pelvis above and infront of


the obturator vessels

• Separated from the ovary within the ovarian fossa by


parietal peritoneum only

• Leaves pelvis to enter the thigh through the obturator


foramen, accompanied by the obturator artery and vein *

• Supplies parietal peritoneum on lateral pelvic wall *

• Splits into anterior and posterior divisions *

• Anterior division supplies gracilis, adductor brevis, adductor


longus, the hip joint and occasionally pectineus. Also supplies
skin on medial aspect of thigh *

• Posterior division supplies obturator externus, adductor part


of the adductor magnus, the knee joint and occasionally
adductor brevis *

4
PUDENDAL NERVE *****

• Branch of sacral plexus (S2,3,4) *

• Leaves pelvic cavity through greater sciatic foramen *

• Enters perineum through lesser sciatic foramen *

• Enters perineum below and medial to the ischial spine *

• Lies medial to the internal pudendal artery as it exits from the pelvis
into the perineum *

Branches

• Inferior rectal nerve - supplies external anal sphincter and mucous


membrane of the lower half of the anal canal *

• Dorsal nerve of the clitoris (penis) *

• Perineal nerve - supplies muscles of the urogenital triangle and the


skin on the posterior surface of the labia majora

Internal pudendal artery *****

• Leaves the pelvis through the greater sciatic foramen, and emerges
between the Piriformis and Coccygeus *

• Crosses the ischial spine, and enters the perineum through the lesser
sciatic foramen. *

• Then crosses the Obturator internus, along the lateral wall of the
ischiorectal fossa, ~ 4 cm above the lower margin of the ischial
tuberosity. *

• Passes forward between the two layers of the fascia of the urogenital
diaphragm

• Runs forward along the medial margin of the inferior ramus of the pubis
behind the pubic arcuate ligament *

• Pierces the inferior fascia of the urogenital diaphragm *

• Branches include the inferior rectal artery (supplies lower half of anal
canal) and branches to the penis or labia and clitoris *

1
SUPERFICIAL PERINEAL POUCH

• Bounded inferiorly by the membranous layer of the superficial fascia

• Bounded above by the urogenital diaphragm

• Closed posteriorly by the fusion of its upper and lower walls

• Communicates anteriorly with the potential space between the


superficial fascia of the anterior abdominal wall and the abdominal wall
muscles

• Contains the structures forming the root of the clitoris: the bulb of the
vestibule, bulbospongiosus and ischiocavernosus muscles; superficial
transverse perineal muscle with their nerve supply (perineal branch of
the pudendal nerve), blood supply (branches of the internal pudendal
artery) and the perineal body

SUPERFICIAL TRANSVERSE PERINEAL MUSCLE

• Located in the posterior part of the superficial perineal pouch

• Origin: ischial ramus

• Insertion: perineal body

• Action: fixes perineal body in the centre of the perineum

• Nerve: perineal branch of the pudendal nerve

BULBOSPONGIOSUS MUSCLE *

• Surrounds vaginal orifice and covers the bulb of the vestibule

• Origin - perineal body

• Inserted onto the corpus spongiosus of the clitoris

• Compresses dorsal vein of the clitoris and assists in erection

2
ISCHIOCAVERNOSUS MUSCLE *

• Origin - ischial tuberosity

• Insertion - fascia covering corpus cavernosus

• Nerve - perineal branch of the pudendal nerve

• Action - assists in erection of the clitoris

UROGENITAL DIAPHRAGM *****

• Musculo-fascial diaphragm with a superior and inferior fascial layer


containing the deep transverse perineal muscle and the sphincter
urethrae

• The two layers fuse anteriorly, leaving a gap beneath the simphysis
pubis

• The two layers fuse posteriorly with the perineal body

• The two layers are attached laterally to the pubic arch

• The enclosed space between the two fascial is the deep perineal
pouch

DEEP PERINEAL POUCH

Enclosed space between superior and inferior fascial layers of the


urogenital diaphragm

Contains:

1) The sphincter urethrae and the deep transverse perineal muscle


2) Part of the vagina

3) Part of the urethra

4) Internal pudendal vessels

5) Dorsal nerve of the clitoris

3
MUSCLES OF THE DEEP PERINEAL POUCH

• Deep transverse perineal muscle

• Origin - ischial ramus

• Insertion - perineal body

• Nerve - perineal branch of pudendal nerve

• Action - fixes perineal body

Sphincter urethrae *****

• Origin - pubic arch

• Insertion - surrounds urethra

• Nerve - perineal branch of pudendal nerve

• Action - voluntary control of micturiction

THE CLITORIS *****

• Located at the apex of the vestibule anterior to the opening of the


urethra

• Has a root made up of three masses of erectile tissue ? the bulb of the
vestibule (divided into two by the vaginal orifice) and the left and right
crura

• The bulb is covered by the bulbospongiosus muscle and becomes the


corpus spongiosus and the glans clitoris anteriorly

• The crura are covered by the ischiocavernosus muscles and form the
corpora cavernosa anteriorly

4
PERITONEUM *****

• The central part of the diaphragmatic peritoneum is supplied by


the phrenic nerve while the peripheral parts are supplied by the lower
intercostals nerves*

• The parietal peritoneum of the anterior and lateral abdominal wall


is supplied segmentally by the lower six thoracic and first lumbar nerve.
It is sensitive to pain, temperature, touch and pressure while the
visceral peritoneum is supplied by autonomic nerves and is sensitive to
streatch*
• The pelvic parietal peritoneum is supplied by the obturator nerve. *
• The ureters are retro-peritoneal throughout their course

PERITONEAL LIGAMENTS *****

• Falciform ligament - anterior surface of liver to anterior abdominal


wall and diaphragm *
• Ligamentum Teres - free border of falciform ligament, contains
obliterated umbilical VEIN *
• Median umbilical ligament - urachus (remnant of the allantois), apex
of bladder to umbilicus *
• Lateral umbilical ligament - INTERNAL iliac artery to umbilicus,
obliterated umbilical ARTERY *
• Gastrosplenic ligament - greater curvature of stomach to spleen

GREATER OMENTUM *****

• Anterior layer attached to the greater curvature of the stomach


• Posterior layer attached to the inferior border of the transverse
colon
• The lower part of the lesser sac lies within it
• The right gastro-epiploic artery runs along the greater curvature of
the stomach in the upper border of the greater omentum

LESSER OMENTUM

• Runs from the lesser curvature of the stomach to the porta hepatis
• Its right free border forms the ANTERIOR margin of the opening
into the lesser sac (epiploic foramen) and contains the RIGHT gastric
artery

1
Boundaries of the epiploic foramen *****

• Anterior - right free border of LESSER omentum (containing bile


duct to the right and in front, hepatic artery to the left and in front and
portal vein posteriorly)

• Posterior - inferior vena cava

• Superior - caudate lobe of liver

• Inferior - first part of duodenum

PARACOLIC GUTTERS

• There are 4 paracolic gutters - left and right medial / lateral

• The right lateral extends from the pelvis to the right posterior sub-
phrenic space *

• The right medial is closed off from the pelvis by the mesentry of the
small intestine *

• The left lateral is separated from the spleen by the phrenicocolic


ligament extending from the left colic flexure to the diaphragm

2
THE OVARY *****

• 4x2 cm, attached to the posterior aspect of the broad ligament


by a mesentery - the mesovarium *

• Attached to the lateral pelvic wall by the suspensory ligament of


the ovary (infundibulo-pelvic ligament) *

• Attached to the upper part of the lateral uterine wall by the


round ligament of the ovary (remnant of upper part of the
gubenaculum) *

• Position variable, but usually lies within ovarian fossa in lateral


pelvic wall. The ovarian fossa is bounded superiorly by the
external iliac vessels, inferiorly by the ureter and internal iliac
vessels and its floor is crossed by the obturator nerve *

• The ovary is surrounded by a thin fibrous capsule - the tunica


albuginae *

• Blood supply - ovarian artery - branch of abdominal aorta at


L1 *

• Venous drainage - LEFT -left ovarian vein drains into left renal
artery; RIGHT - right ovarian vein drains into inferior vena cava *

• Lymphatics - para-aortic nodes *

• Nerve - aortic plexus

1
FALLOPIAN TUBES *****

• ~10cm long, 4 parts

• Infundibulum - funnel-shaped lateral end, projects beyond the


broad ligament with fimbriae at its free end

• Ampulla - widest and longest part, site of fertilisation *

• Isthmus - narrowest part, just lateral to the uterus

• Intra-mural part - pierces uterine wall

• Three coats: serous, muscular, and mucous.

• The external or serous coat is peritoneal.


The middle or muscular coat consists of an external longitudinal
and an internal circular layer of smooth muscle fibers continuous
with those of the uterus*

• The mucosa is thrown into longitudinal folds, which in the


ampulla are much more extensive than in the isthmus. Lined by
ciliated columnar epithelium *

• Blood - ovarian and uterine arteries

• Lymphatics - aortic and internal iliac nodes (follow arteries)

• Nerves - inferior hypogastric plexus

2
THE UTERUS *****

• 8cm long x 5cm wide x 2.5cm thick

• Covered by peritoneum except anteriorly below the reflection of


the utero-vesical fold of peritoneum and laterally between the
layers of the broad ligament *

• Fundus - that part of the uterus above the entrance of the


uterine tubes

• Cavity - triangular in coronal section, cleft in saggital section

• Anteverted uterus - long axis of uterine body at 90degrees to


long axis of vagina

• Ante-flexed uterus - long axis of the body of the uterus bent


forward at the level of the internal os

• Retroverted uterus - body and fundus bent backwards on the


vagina to lie within the pouch of Douglas

• Anterior relations: utero-vesical pouch and superior surface of


bladder *
• Posterior: Pouch of Douglas, sigmoid colon and coils of ileum *

• Laterally: uterine vessels, ureter *

• Nerve - inferior hypogastric plexuses (Parasympathetic via the


pelvic splanchnic nerves, sympathetic via the lumbar splanchnic
nerves). Pain sensation is transmitted via the sympathetic nerves
and the lumbar splanchnic nerves *

• Lymph: *
1)Fundus - accompany ovarian artery to para-aortic nodes at the level of L1

2) Body and cervix - internal and external iliac nodes

THE UTERINE ARTERY *****

• Branch of the anterior division of the internal iliac artery *

• Runs medially in the base of the broad ligament

• Reaches the cervix at the level of the internal os *

• Crosses the ureter *

3
• Ascends within the broad ligament on the lateral aspect of the
uterus
• Gives off descending branch to the cervix and vagina *

• Supplies uterus and fallopian tube *

LIGAMENTS *****

• Transverse cervical ligament: fibro-muscular condensations of


pelvic fascia pass from the cervix and upper end of the vagina to
the lateral pelvic wall

• Utero-sacral ligament: cervix and upper end of vagina to the


lower end of the sacrum - form two ridges on either side of the
pouch ofDouglas

• Pubo-cervical ligament: cervix to posterior surface of pubis

• Round ligament: Of the ovary - from medial margin of ovary to


upper part of lateral wall of uterus. Of the uterus - from upper part
of lateral uterine wall to deep inguinal ring

The uterus is supported mainly by the tone of the pelvic floor muscles (levator
ani) which are partly inserted onto the perineal body and condensations of
pelvic fascia forming the transverse cervical, pubo-cervical and utero-sacral
ligaments.

THE BROAD LIGAMENT *****

• Two layered fold of peritoneum extending from the lateral


uterine wall to the lateral pelvic wall
• Has an upper free edge which contains the fallopian tube

• The layers of peritoneum separate inferiorly to cover the pelvic


floor
• Has ovary attached to its posterior surface by the mesovarium

• Uterine artery crosses the ureter at the base (lower attached


border)

• Round ligament of the uterus forms a ridge an the anterior


surface
• Contains vestigial structures: epoophron and paroophron
(remnant of the mesonephric system)

• Uterine and ovarian blood vessels and lymphatics run within it

4
CERVIX *****

• Lower, narrow portion of the uterus, connected to the uterine fundus by


the uterine isthmus - upper limit is the internal os. Made up mainly of
fibrous tissue with very little smooth muscle *

• Protrudes through the upper anterior vaginal wall

• Approximately half its length is visible in the vagina (vaginal cervix), the
rest being above the vagina (supra-vaginal cervix) *

• The vaginal cervix ~3 cm long and 2.5 cm wide. Size and shape varies
widely with age, hormonal state, and parity - bulkier and the external
with a wider and more slit-like external os in multiparous women.

• Ectocervix - portion of the cervis beyond the external os - lined by


stratified squamous non-keratinising epithelium. *

• Endocervical canal - links external and internal os - lined by columnar


epithelium

• The squamo-columnar junction - variable location - high up the


endocervical canal before puberty and in the post-menopausal women.
Site of origin of squamous cell carcinoma of the cervix *

• The external os is bounded by two lips, an anterior and a posterior, of


which the anterior is the shorter and thicker, although due to the slope
of the cervix, it projects lower than the posterior. Both lips are in
contact with the posterior vaginal wall

• The supravaginal cervix is separated in front from the bladder by


fibrous tissue (parametrium), which extends also on to its sides and
laterally between the layers of the broad ligaments. Not covered by
peritoneum on the anterior aspect *

• The uterine arteries reach the margins of the cervix within the
parametrium *

• The ureter runs downward and forward 2 cm lateral to the supravaginal


cervix *

• Posteriorly, the supravaginal cervix is covered by peritoneum, which


extends on to the posterior vaginal wall, when it is reflected on to the
rectum, forming the Pouch of Douglas which may contain coils of small
intestine.*

• The vaginal cervix projects free into the anterior wall of the vagina
between the anterior and posterior fornices.

5
Blood supply *****

• Uterine artery, branch of internal iliac

• Cervical and vaginal branches supply the cervix and upper vagina.

• The cervical branches of the uterine arteries descend on the lateral


aspects of the cervix at 3 and 9 o'clock. Venous drainage parallels the
arterial supply, eventually emptying into the hypogastric venous plexus.

Lymphatics *****

Regional lymph nodes for the cervix include: paracervical, parametrial, presacral,
sacral, external iliac, common iliac, hypogastric (obturator), internal iliac..

Support and innervation

• Mainly the cardinal (transverse cervical) and uterosacral ligaments.

• These attach to the lateral and posterior aspects of the supra-vaginal


cervix and extend laterally and posteriorly bony pelvis.

• The uterosacral ligaments are the conduits of the main nerve supplying
to the cervix, derived from the hypogastric plexus.

• Sensory, sympathetic, and parasympathetic fibers are present in the


cervix *

• Sensory fibres travel via the parasympathetic nerves (S2,3,4 - pelvic


splanchnic nerves) *

• Dilatation of the cervix using dilators may result in a vasovagal attack


with reflex bradycardia

• The endocervix has a plentiful supply of sensory nerve endings, while


the ectocervix is relatively lacking in these. Small cervical biopsies and
cryotherapy can be performed in most patients without the use of
anaesthesia *

6
THE VAGINA *****

• ~8cm long, axis directed upwards and backwards from the


vulva. Posterior wall longer than anterior wall *

• Lined by stratified squamous epithelium which undergoes


changes during the menstrual cycle. Does not secrete mucus *

• Has anterior and posterior walls which are normally in


apposition, and four fornices (anterior, posterior, left and right
lateral)

• Upper half lies above the level of the pelvic floor

Relations *****

• Anterior: bladder, urethra *

• Posterior: upper third - pouch of Douglas; middle third - ampulla


of the rectum; lower third - perineal body *

• Lateral: upper part - ureter, middle part - anterior fibres of


levator ani; lower part - uro-genital diaphragm and the bulb of the
vestibule*

• Blood: vaginal artery, branch of internal iliac artery

• Lymphatics: upper third - internal and external iliac nodes;


middle third - internal iliac nodes; lower third - superficial inguinal
nodes *

• Nerve:Upper two thirds - inferior hypogastric plexuses.


Parasympathetic from pelvic splanchnic nerves, sympathetic from
lumbar splanchnic nerves (L1 & 2) *

• Autonomic innervation to the lower third - pudendal nerve *

• Somatic sensation is present mainly in the lower thirs and is


carried by the pudendal nerve *

7
VAGINAL SUPPORT *****

The vagina is supported by

1) Levator ani muscles, transverse cervical, pubo-cervical and utero-sacral


ligaments in its upper part

2) The uro-genital diaphragm in its middle part

3) The perineal body in its lower part

8
THE RIGHT KIDNEY *****

• Lower level than the left due to the bulk of the right lobe of the liver

• Anterior relations: suprarenal gland, liver, second part of


duodenum, right colic flexure

• Posterior relations: Diaphragm, costo-diaphragmatic recess


12th rib. Psoas, quadratus lumborum and transversus abdominis
Sub-costal (T12), iliohypogastric and ilioinguinal nerves

LEFT KIDNEY *****

• Descends 2.5cm on INSPIRATION

• Anterior relations: LEFT suprarenal gland, spleen, stomach, left


colic flexure and coils of jejunum

• Posterior relations: 11th and 12th ribs, diaphragm and costo-


diaphragmatic recess. Psoas, quadratus lumborum and transversus
abdominis
Sub-costal, iliohypogastric and ilioinguinal nerves

KIDNEY - BLOOD SUPPLY *****

• Renal artery - branch of abdominal aorta. Enters the hilus of the


kidney behind the renal vein; has 5 segmental branches at the hilus,
four in front and one behind the renal pelvis

• Subsequent branches are as follows: segmental - lobar - interlobar -


arcuate - interlobular arteries - afferent glomerular arterioles

• Renal vein drains y into the inferior vena cava directly

THE URETER *****

• 25cm long, leaves renal pelvis behind the renal vein and descends on
psoas major which separates it from the tips of the transverse
processes of the lumbar vertebrae *

• Adherent to parietal peritoneum *

• Enters the pelvis in front of the sacro-iliac joint and the bifurcation of
the common iliac artery *

1
• Supplied by the renal, testicular / ovarian and superior vesical
arteries *

• Lymphatic drainage is to the para-aortic and iliac nodes *

• Nerve - renal, testicular / ovarian and hypogastric plexuses. Afferent


(sensory) fibres travel with the sympathetic nerves and enter the spinal
cord at L1&2 *

• Within the pelvis, the ureter runs at first downward on the lateral wall
of the pelvic cavity, along the anterior border of the greater sciatic
notch and under cover of the peritoneum. *

• It lies in front of the internal iliac artery and medial to the obturator
nerve obturator, inferior vesical, and middle rectal arteries *

• In the female, the ureter forms the posterior boundary of the ovarian
fossa,in which the ovary is situated and is separated from the ovary by
pelvic peritoneum *

• In the region of the ischial spine, it runs medially and forward beneath
the base of the broad ligament on the lateral aspect of the cervix and
upper part of the vagina to reach the bladder. *

• It is crossed by the uterine artery ~2cm lateral to the supra-vaginal


cervix and the lateral vaginal fornices *

• At the bladder base, the ureters are ~5 cm apart in the full bladder
and 2.5cm apart when the bladder is empty. The ureter lies anterior to
the anterior vaginal fornix as it reaches the bladder *

• Finally, the ureters run obliquely for about 2 cm. through the wall of
the bladder and open by slit-like apertures into the cavity of the viscus
at the lateral angles of the trigone. *

• The ureter is narrowed at the pelvi-ureteric junction, where it bends


into the pelvis at the pelvic brim and where it passes into the bladder
wall - renal calculi are likely to lodge at these points *

2
During pelvic surgery, the ureters can be injured at the following
points:*****

• The ureteric tunnel where it is crossed by the uterine artery during


clamping & ligation of the uterine artery pedicle *

• At the bladder base during vaginal surgery - anterior colporrhaphy /


vaginal hysterectomy or during colpo-suspension *

• Anterior to the vagina as it courses forward to enter the bladder and


can be injured while excising the upper vagina during radical
hysterectomy *

• At the ovarian fossa where it may be involved in an ovarian mass *

• At the pelvic brim where it may be confused with the infundibulo-


pelvic ligament *

RIGHT URETER *

• Anterior relations include: second part of duodenum, terminal ileum,


right colic and ileocolic vessels, right testicular / ovarian vessels and
the root of the mesentery of the small intestine.

LEFT URETER *

• Anterior relations: sigmoid colon, sigmoid mesocolon, left testicular /


ovarian vessels, left colic vessels

• Medial relation: inferior mesenteric vessels

Structure *****

• Three coats - fibrous, muscular and mucosa

• The fibrous coat is continuous with the renal capsule and the fibrous
structure of the bladder

• Muscular coat arranged in two layers- outer longitudinal and inner


circular *

• The mucosa is smooth lined by transitional epithelium *

3
SUPRARENAL GLANDS *****

• Right - pyramidal, Left - crescenteric in shape *

• Right related anteriorly to: right lobe of liver and lateral border of
inferior vena cava *

• Left related anteriorly to: pancreas, lesser sac and stomach *

• Both glands rest posteriorly on the diaphragm *

• Blood - suprarenal branches of aorta, inferior phrenic and renal


arteries

• Single suprarenal vein drains into renal vein on the left or inferior
vena cava on the right *

• Lymphatics - para-aortic nodes *

Nerves - predominantly sympathetic pre-ganglionic fibres derived from the


splanchnic nerves *

4
THORAX - SURFACE ANATOMY

• Supra-sternal notch: Upper margin of manubrium


sterni, palpable in mid-line between medial ends of
clavicles

• Sternal angle (angle of Louis): angle between


manubrium and body of sternum; lies at the level of the
second costal cartilage opposite the intervertebral disc
between the 4th and 5th thoracic vertebrae

• Xiphisternal joint: lies opposite the body of the 9th


thoracic vertebra

• Costal margin: formed from costal cartilages of 7th ?


10th ribs and the ends of the 11th and 12th cartilages.
Lowest point formed by the 10th rib and lies at the level of
L3

• The first rib is not palpable

• The apex beat of the heart is located in the 5th left


intercostal space 9cm from the mid-line

• The first spinous process that is palpable is that of the


7th cervical vertebra (vertebra prominens)

• The superior angle of the scapula lies at the level of the


spine of the 2nd thoracic vertebra while the inferior angle
lies at the level of the spine of the 7th thoracic vertebra.
The root of the spine of the scapula lies at the level of the
spine of the 3rd thoracic vertebra

THE RIB

• A typical rib has a head which articulates with the body


of the corresponding vertebra and the vertebra
immediately above; a neck between the head and the
tubercle; a tubercle which articulates with the transverse
process of the corresponding vertebra; a shaft or body
and an angle where the shaft turns sharply forwards.

• The costal groove is located on the inferior border of


the shaft

1
FIRST RIB

• Has the scalene tubercle on its medial border for the


attachment of the scalenus anterior muscle

• The subclavian vein crosses the rib anterior to the


scalene tubercle

• The subclavian artery and brachial plexus lie posterior


to the scalene tubercle

• Has a head which articulates with the body of the 1st


thoracic vertebra. Unlike other ribs, the first rib does not
articulate with the body of the vertebra above it (7th
cervical)

• Has a tubercle which articulates with the transverse


process of the 1st thoracic vertebra

• Is not palpable - lies deep to the clavicle

INTERCOSTAL MUSCLES

• EXTERNAL - fibres directed downwards and forwards


from the inferior border of the rib above to the inferior
border of the rib below; extends from the tubercle to the
costochondral junction where it is replaced by the
anterior intercostal membrane

• INTERNAL - fibres directed downwards and backwards,


extends from the sternum to the angle of the rib
posteriorly where it is replaced by the posterior
intercostal membrane

• TRANSVERSUS THORACIS - extends over more than


one intercostal space

INTERCOSTAL NEUROVASCULAR BUNDLE

• Runs within the costal groove between the internal


thoracic and transversus thoracis muscles

• Arranged from above downwards: Vein, Artery and


Nerve (VAN)

2
INTERCOSTAL VESSELS

• Each intercostal space has two anterior and one


posterior arteries

• 1st and 2nd posterior intercostal arteries are branches


of the superior intercostal artery, a branch of the
costocervical trunk of the subclavian artery

• Lower 9 posterior intercostal arteries - branches of the


thoracic aorta

• Upper 6 anterior intercostal arteries - branches of the


internal thoracic artery

• Lower 5 anterior intercostal arteries - branches of the


musculophrenic artery (terminal branch of the internal
thoracic artery)

• Intercostal arteries supply parietal pleura, intercostal


muscles and skin of thoracic wall

• Posterior intercostal veins drain into azygos or


hemiazygos veins

• Anterior intercostal veins brain into musculophrenic


and internal thoracic veins

INTERCOSTAL NERVES *****

Anterior rami of upper 11 thoracic spinal nerves

• Enter intercostal space between the parietal pleura and


the posterior intercostal membrane

• Run in the subcostal groove between the internal


intercostal and transversus thoracis muscles and inferior
to the intercostal vein and artery (VAN)

• Upper 6 supply intercostal spaces

• Lower 5 also supply anterior abdominal wall

3
Branches *****

• Rami communicantes - give white rami and receive grey


rami from the sympathetic trunk

• Lateral cutaneous branch - divides into anterior and


posterior branches and supplies the skin

• Anterior cutaneous branch - divides into medial and


lateral branches

• Collateral branch - runs forward below the main nerve

• Muscular, pleural and peritoneal branches

• First intercostal nerve - the equivalent of the lateral


cutaneous branch joins the brachial plexus. There is no
anterior cutaneous branch

• Second intercostal nerve - the equivalent of the lateral


cutaneous branch forms the intercostobrachial nerve
which joins the medial cutaneous nerve of the arm to
supply the skin of the armpit and medial side of the arm

4
SIGMOID COLON *****

• Continuous with the descending colon in front of the LEFT


external iliac artery, and with the rectum at the level of S3

• Intra-peritoneal with an inverted V-shaped mesentery -


one limb running on the medial side of the left external iliac
artery and the other from the bifurcation of the left common
iliac artery to S3. The recess of the pelvic mesocolon is
located at the apex of the V and the left ureter lies beneath
it *

• Related anteriorly to the bladder in the MALE and the


uterus and upper part of the vagina in the female *

• Related posteriorly to the sacrum and rectum

• Blood supply - inferior mesenteric artery

• Venous drainage - inferior mesenteric vein - to portal vein

• Lymphatics - inferior mesenteric plexus

• Nerve - inferior hypogastric plexuses

THE RECTUM *****

• ~13cm long
• Upper third - covered by peritoneum on its anterior and
lateral surfaces, middle third covered by peritoneum on the
anterior surface only and lower third devoid of peritoneum *

• The three teniae coli of sigmoid colon come together to


form a broad band of longitudinal smooth muscle on its
anterior and posterior surfaces *

• Three transverse folds, two on the left and one on the


right *

• Anterior relations: sigmoid colon and small intestine within


pouch of Douglas (upper third); Posterior surface of vagina
(lower third) *

• Posterior relations: sacrum, coccyx, piriformis and


coccygeus muscles, lavatory ani, sacral plexus and pelvic
sympathetic trunk *

1
Blood supply: *****

1) Superior rectal artery - continuation of inferior mesenteric artery - main


blood supply to mucosa

2) Middle rectal artery - branch of internal iliac artery - muscular coat mainly

3) Inferior rectal artery - branch of internal pudendal artery

• Veins correspond to arteries and form an important portal-


systemic anastomosis

• Lymphatic drainage: Upper two thirds - inferior mesenteric


nodes; lower third - internal iliac nodes *

Nerve supply - inferior hypogastric plexus *

2
STOMACH *****

• Lies in the left HYPOCHONDRIUM extending onto the


epigastric and umbilical regions

• Has a fundus, body, antrum and pylorus

• Body extends from the level of the cardiac orifice to the


incisura angularis on the lesser curvature

• Connected to the liver by the lesser omentum

• Three muscle layers - outer longitudinal, inner circular and


innermost oblique. Intra-peritoneal (has peritoneum on all
surfaces)

• The pyloric sphincter is an anatomical sphincter but a


physiological sphincter only exists at the cardiac orifice

RELATIONS OF THE STOMACH

Anterior

• Anterior abdominal wall, left costal margin, diaphragm, left


lung and pleura, left lobe of liver

Posterior

• Lesser sac, diaphragm, spleen, splenic artery, pancreas, left


suprarenal gland and upper part of left kidney, transverse
colon and transverse mesocolon

BLOOD SUPPLY *****

• Left gastric artery - branch of celiac artery - lower third of


oesophagus and upper RIGHT part of stomach

• Right gastric artery - branch of hepatic artery - lower right


part of stomach

• Short gastric arteries - branches of splenic artery - fundus

• Left gastro-epiploic - branch of splenic artery - runs along


GREATER curvature

• Right gastro-epiploic - branch of gastroduodenal artery -


lower part of greater curvature

1
• Left / right gastric veins - drain into portal vein directly

• Short gastric and left gastro-epiploic veins - drain into the


splenic vein

• Right gastro-epiploic vein - drains into the superior


mesenteric vein

NERVES AND LYMPHATICS *****

• Lymphatics follow arteries, all drain into celiac nodes. Those


from the fundus of the stomach initially drain into nodes in the
hilus of the spleen then to the celiac nodes

• Nerves: Anterior vagal trunk - Left vagus nerve mainly; has


hepatic branch to the liver from which the pyloric branch arises

• Posterior vagal trunk - Right vagus mainly; has branch to


celiac and superior mesenteric plexuses which supply foregut
and mid-gut (splenic flexure)

• Vagal impulses are secretomotor to the glands and motor to


the muscle (but inhibitory to the pyloric sphincter)

• Sympathetic supply - from celiac plexus - motor to the


pyloric sphincter

DUODENUM *****

• 25cm long, first 2.5cm are intra-peritoneal. Divided into 4


parts

• First part - 5 cm long on transpyloric plane

• Anterior: quadrate lobe of liver and gall bladder

• Posterior: Lesser sac, gastroduodenal artery, bile duct,


portal vein, inferior vena cava

• Superior: Epiploic foramen

• Inferior: Head of pancreas

• Second part - 8cm long

• Anterior: gallbladder, right lobe of liver, transverse colon and


small intestine

2
• Posterior: Hilus of right kidney and right ureter *

• Lateral: Ascending colon, right colic flexure, right lobe of


liver

• Medial: Head of pancreas

• Pancreatic and bile ducts open into second part of


duodenum

• Third part - 8cm long

• Anterior: Superior mesenteric vessels, root of mesentery of


small intestine and jejunum

• Posterior: Right ureter, right psoas, inferior vena cava and


aorta*

• Superior: Head of pancreas

• Inferior: Jejunum

• Fourth part - 5cm long

• Anterior: Jejunum

• Posterior: Aorta and left psoas

• Ligament of Trietz attaches the duodeno-jejunal junction to


the right crus of the diaphragm

SMALL INTESTINE *****

• Jejunum and Ileum, 6m long

• Attached to the posterior abdominal wall by a mesentery

• Mucosa has CIRCULAR folds called plicae circularis which


are more prominent in the jejunum

• Aggregations of lymphoid tissue are present in the mucous


membrane of the lower ileum along the antimesenteric border
(Payer’s patches)

• Blood - Superior mesenteric vessels

• Lymphatics - superior mesenteric nodes


• Nerve - sympathetic and parasympathetic (vagus) from the
superior mesenteric plexus

3
SURFACE ANATOMY - ABDOMEN *****

The abdomen is divided into 9 regions:

• Right hypochondrium, Epigastrium, left hypochondrium


• Right lumbar, umbilical, left lumber
• Right iliac, hypogastrium, left iliac; by:

1) Two vertical planes through the mid-point between the anterior


superior iliac spine and the symphysis pubis and

2) Two horizontal planes:

I )The sub-costal plane - joins lowest points of costal margin on both


sides (inferior margin of 10th costal cartilage; level L3)

ii) The inter-tubercular plane - joins tubercles of iliac crests (level L5)

Trans-pyloric plane:

• Through tips of 9th costal cartilages (point where linea


semilunaris intersects with costal margin). Passes through the
pylorus, nexk of pancreas, hili of kidneys and duodeno-jejunal
junction.

Abdominal wall

Dermatomes *****

• Xiphoid process - T7
• Umbilicus - T10
• Pubis - L1

Blood supply: *****


• Medial aspects - Superior epigastric artery (above umbilicus
- continuation of the internal thoracic artery, branch of the first
part of the subclavian artery. Enters the rectus sheath
between the sternal and costal origins of the diaphragm and
descends behind rectus muscle)*

• Inferior epigastric artery (below umbilicus - branch of


external iliac artery just above inguinal ligament. It pierces the
transversalis fascia to enter the rectus sheath anterior to the
arcuate line; runs behind rectus muscle). There is no
anastomosis between the arteries of the left and right side *
• Lateral aspects - intercostals, lumbar arteries and deep
circumflex iliac artery (branch of external iliac artery above the
inguinal ligament). *

1
Venous drainage: *****

• Above umbilicus - axillary vein via lateral thoracic vein *

• Below umbilicus - femoral vein via superficial epigastric and


great saphenous vein *

• Few para-umbilical veins drain into the portal vein via the
ligamentun teres, forming a portal-systemic anastomosis.
Distended veins may indicate portal hypertension *

• Deep veins follow the arteries with the posterior intercostals


veins draining into the azygous vein and the lumbar veins into
the inferior vena cava

Lymphatics: *****

• Above umbilicus - anterior axillary nodes

• Below umbilicus - superficial inguinal nodes

• Deep lymphatics follow the arteries into the internal thoracic,


external iliac, posterior mediastinal and para-aortic nodes

HERNIAS

INGUINAL

• Indirect commoner than direct

• Indirect more common in males and on the right side (right


testis descends later than the left)

• Direct hernias commoner in (old) males

• Indirect: Enters inguinal canal through deep inguinal ring,


lateral to inferior epigastric vessels. Can descent into the
scrotum or labia majora. Sac is formed by the remains of the
processus vaginalis

• The sac of all inguinal hernias lies above and medial to the
pubic tubercle

2
FEMORAL

• Commoner in females protrudes through the femoral canal


medial to the femoral vein, below and lateral to the pubic
tubercle.

UMBILICAL

• Congenital - exomphalos

• Acquired infantile - weakness in scar of umbilicus

• Acquired adult - para-umbilical, weakness in linea alba


above or below umbilicus, commoner in females

EPIGASTRIC

• Weakness in linea alba above umbilicus

RICHCTER’S

• A knuckle of the side-wall of the bowel is incarcerated in the


sac but the continuity of the bowel is maintained and there is
no obstruction.

3
THE DIAPHRAGM *****

Has a peripheral muscular part and a central tendon

Three origins:

• Sternal - from the posterior surface of the xiphoid process *


• Costal - from the deep surfaces of the lower 6 ribs and their costal
cartilages *
• Vertebral - right crus from the sides of the bodies and inter-vertebral
discs of the upper 3 lumbar vertebrae; left crus from the sides of the
bodies and intervertebral disc of the first 2 lumbar vertebrae. Also has
an origin from the medial and lateral arcuate ligaments. The median
arcuate ligament joins the crura *
• Insertion - central tendon

Action

• Contracts during inspiration - most important muscle for inspiration *


• Contraction raises intra-abdominal pressure during micturiction,
defecation or parturition *
• Thoraco-abdominal pump - contraction lowers intra-thoracic and raises
intra-abdominal pressure, aiding venous return from the inferior vena
cava to the right atrium *
• Nerve supply - motor : phrenic nerve. Sensory supply to parietal
pleura and peritoneum covering the central tendon is from the phrenic
nerve. Sensory supply to the peripheral part is from the lower 5
intercostal nerves

OPENINGS *****

• Aortic - anterior to T12 between the crura, transmit the aorta, thoracic
duct and azygos vein *
• Oesophageal - level of T10, also transmits right and left vagi,
oesophageal branch of the left gastric vessels and lymphatics from the
lower third of the oesophagus *
• Caval - level of T8 in the central tendon, also transmits terminal
branch of the right phrenic nerve *
• The superior epigastric vessels pass between the sternal and costal
origins ; the left phrenic nerve pierces the left dome; the greater, lesser
and lowest splanchnic nerves pierce the crura and the sympathetic
trunk lies posterior to the medial arcuate ligament. *

1
2
THE RIGHT ATRIUM

• Has a main cavity and an auricle

• The superior and inferior vena cava and coronary sinus


open into the right atrium

• Drains into right ventricle via the tricuspid valve

• Fetal remnants include the fossa ovalis (on the atrial


septum, remnant of foramen ovale); annulus ovalis (upper
margin of fossa ovalis, lower edge of septum secundum)

THE RIGHT VENTRICLE

• Communicates with the right atrium (tricuspid valve) and the


pulmonary artery (pulmonary valve with three semilunar
cusps). Pumps de-oxygenated blood to the lungs

• Cavity contains muscular projections called trabeculae


carneae:

1) Papillary muscles - connected by chordae tendinae to the


cusps of the tricuspid valve

2) Muscular projections, one of which traverses the cavity and


transmits the right branch of the atrio- ventricular bundle
(moderator band)

THE LEFT ATRIUM

• Has a main cavity which is smooth and an auricle with


muscular ridges

• Lies posterior to the right atrium and forms the base of the
heart. Separated from the oesophagus by the pericardium

• Receives 4 pulmonary veins and opens into the left ventricle


via the mitral (bicuspid) valve

THE LEFT VENTRICLE

• Communicates with the left atrium (mitral valve, two cusps)


and with the aorta through the aortic valve (three cusps)
• Myocardium is three times thicker than that of the right
ventricle
• Has trabeculae carneae and two large papillary muscles

• There is no moderator band (right ventricle only)

1
CONDUCTING SYSTEM OF THE HEART *****

Sino-atrial node *

• Specialised cardiac muscle of the right atrium to the right of


the opening of the superior vena cava

• Pacemaker

• Supplied by the left and right coronary arteries

Atrio-ventricular node *

• Situated in the lower part of the atrial septum

• Supplied by the right coronary artery

Atrio-ventricular bundle

• Conducts impulses from the atrio-ventricular node to the


ventricles

• Only muscular connection between atrial and ventricular


myocardium

• Descends to reach the inferior border of the membranous


part of the inter-ventricular septum

• Supplied by the right coronary artery

Right and left atrio-ventricular bundles *

• Formed from the atrio-ventricular bundle in the upper border


of the muscular part of the ventricular septum

• Right branch enters the moderator band

• Continuous with the Purkinje plexus

• Right branch is supplied by the right coronary artery, left


branch by the left and right coronary arteries

2
BLOOD SUPPLY TO THE HEART *****

Right coronary artery

• From anterior aortic sinus

Descends in the atrio-ventricular groove, supplying right atrium


and ventricle

• Gives off marginal branch at the inferior border of the heart


and a posterior interventricular branch

Left coronary artery

• From the left posterior aortic sinus

• Gives off anterior interventricular and circumflex branches

• Anterior interventricular branch supplies right and left


ventricles and interventricular septum

• Circumflex branch supplies left atrium and ventricle

Venous drainage

• Coronary sinus, a continuation of the great cardiac vein.


Drains into the right atrium

• Small cardiac and middle cardiac veins drain into the


coronary sinus

• The anterior cardiac vein drains directly into the right atrium

3
THE MEDIASTINUM

• Divided into superior and inferior mediastinum by a plane


passing from the sternal angle to the lower border of T4

• Inferior mediastinum divided into middle mediastinum containing


the heart, posterior and anterior mediastinum posterior and anterior
to the heart respectively

• Superior mediastimun has the following structures from front to


back: thymus, large veins, large arteries, trachea, oesophagus +
thoracic duct, sympathetic trunk

• Inferior mediastinum has the following structures from front to


back: thymus, heart + phrenic nerves, oesophagus + thoracic duct,
descending aorta, sympathetic trunk

TRACHEA

• 13cm tube with a fibro-elastic wall within which are embedded U-


shaped bars of hyaline cartilage

• Commences in the neck below the cricoid cartilage of the larynx


(C6) and ends at the level of the angle of Louis (lower border of T4)

• Anterior relations: sternum, thymus, left brachiocephalic vein,


arch of the aorta and the origins of the brachiocephalic and left
common carotid arteries

• Posterior relations: oesophagus, left recurrent laryngeal nerve

• Right side: azygos vein, right vagus nerve

• Left side: arch of the aorta with left common carotid and
subclavian arteries, left vagus and left phrenic nerves.

1
MAIN BRONCHI

RIGHT

• Shorter, wider and more vertical than the left

• Gives off the superior lobar bronchus before entering the hilum of
the lung where it divides into the middle and inferior lobe bronchus

LEFT

• Passes to the left below the aortic arch, anterior to the


oesophagus

• At the hilum of the left lung divides into superior and inferior lobe
bronchi

2
THE OESOPHAGUS

•25cm long, continuous with the laryngeal part of the pharynx


opposite C6

Relations in the neck

•Posterior: vertebral column

•Anterior: trachea and recurrent laryngeal nerves

•Lateral: lobes of thyroid gland

Relations in the thorax

•Anterior: trachea, left recurrent laryngeal nerve, left main


bronchus, left atrium separated by pericardium

•Posterior: thoracic vertebral bodies, thoracic duct, azygos vein,


descending thoracic aorta

•Right side: terminal part of azygos vein and mediastinal pleura

•Left side: left subclavian artery, aortic arch, thoracic duct and
mediastinal pleura

Blood supply

•Upper third: inferior thyroid artery / vein

•Middle third: descending thoracic aorta / azygos vein

•Lower third: left gastric artery / vein (portal vein)

Lymphatic drainage

•Upper third: deep cervical nodes

•Middle third: posterior mediastinal nodes

•Lower third: celiac nodes

Oesophageal opening of the diaphragm

•Level T10

•Left vagus (anterior), right vagus (posterior)


•Transmits left gastric vessels and lymphatics

1
2
THE PANCREAS *****

• Exocrine (acini) and endocrine (islets of Langerhans) gland

• Head, neck, body and tail. The uncinate process of the head lies
posterior to the superior mesenteric vessels. The neck lies anterior to
the origin of the superior mesenteric artery and portal vein

• Related anteriorly to the transverse colon and mesocolon, lesser sac


and stomach

• Related posteriorly to the bile duct, portal vein, splenic vein, inferior
vena cava and aorta, origin of superior mesenteric artery, left psoas,
left suprarenal gland, left kidney and hilus of spleen

• The splenic artery runs along the upper border of the pancreas

• Drained by main pancreatic duct, opening half way down the second
part of the duodenum. The accessory pancreatic duct, where present,
opens above the opening of the main duct

• Arterial supply - splenic, superior and inferior pancreatico-duodenal


arteries

• Lymphatics - celiac nodes

• Nerves - parasympathetic (vagus) and sympathetic from the celiac


plexus

SPLEEN *****

• Lies in the left hypochondrium under the 9th, 10th and 11th ribs with
its long axis along the shaft of the 10th rib. Extends to the mid-axillary
line and the normal spleen is not palpable. It is intra-peritoneal *

• Has a notched anterior border *

• Anterior relations: stomach, tail of pancreas and left colic flexure *

• Medial relation: left kidney *

• Posterior relations: diaphragm, left costo-diaphragmatic recess, left


lung, 9th,10th and 11th ribs *

• Blood supply - splenic artery, branch of celiac artery

• Lymphatics - celiac nodes *

Nerves - celiac plexus *

1
2
THE URINARY BLADDER *****

• Lined by transitional epithelium - the upper part is derived


from the yolk sac and is continuous with the urachus *

• The trogone is derived from the mesonephric duct and is


lined by cells of mesodermal origin. It is believed that these
are later replecad by cells of endodermal origin *

• The bladder wall is made up of a syncytium of smooth


muscle fibres called the detrusor - contraction results in
simultaneous reduction in all dimensions of the bladder *

• The female urethra has an intrinsic and an extrinsic


component: the intrinsic component is made up of epithelial,
vascular and connective tissue and the rhabdosphincter
which is a circular ring of striated muscle with slow twitch
fibres. The ring is well developed anteriorly, thins laterally and
is virtually absent posteriorly *

• The extrinsic sphincter mechanism is made up of the


striated muscles of the levator ani - mainly fast twitch fibres *

• Main nerve supply to the detrusor is parasympathetic -


causes contraction (S2,3 4) *

• Beta-sympathetic innervation causes relaxation (L 1&2) *

• The urethral smooth muscle has alpha sympathetic


innervation which causes contraction - there is however,
significant overlap *

• The rhabdosphincter is supplied by somatic nerves (S2,3,4)


via the pelvic splanchnic nerves *

• Levator ani supplied by S2,3,4 through the perineal branch


of the pudendal nerve *

• Sensory innervation (stretch) is via both sympathetic and


parasympathetic pathways *

1
PHARMACOLOGY *****

• INHIBIT DETRUSOR CONTRACTILITY - Anti-cholinergic


agents, calcium channel blockers, beta-agonists, CNS
depressants like chlorpromazine cause voiding dysfunction *

• STIMULATE DETRUSOR CONTRACTILITY - Cholinergic


agonists *

• INCREASE URETHRAL RESISTANCE - alpha agonists *

URODYNAMIC PARAMETERS *****

• MAXIMUM FLOW RATE - measured with a full bladder - at


least 15ml/s. Reduced with hypotonic detrusor, outflow
obstruction or an inadequate voided volume *

• A graph of flow rate against time is bell-shaped

• BLADDER CAPACITY - 400 - 600ml *

• First sensation to void occurs at 150 - 250ml - decreased in


sensory urgency and detrusor instability; increased in
overflow incontinence *

• Detrusor pressure rise should be less than 15cm H2O


during filling - increased pressure occurs in low compliance
bladder and detrusor instability *

• Leakage occurs if detrusor pressure exceeds urethral


pressure. If there is no abnormal pressure rise on filling and
the woman leaks because of a rise in intra-abdominal
pressure without a rise in detrusor pressure then a diagnosis
of GENUINE STRESS INCONTINENCE is made. Leakage
with a rise in detrusor pressure occurs in detrusor instability
and fistulas

• Cystometry is required for a diagnosis of detrusor instability


- detrusor contraction associated with symptoms during
bladder filling when the individual is trying to inhibit
micturiction

• High urethral pressure profile occurs in outflow obstruction.


Low pressure in GSI.

2
3
Question 1: With respect to the scrotum and its contents

a. Colles? fascia is a continuation of Scarpa?s fascia


True False
b. Dartos muscle does not contribute to the median partition of the
scrotum
True False
c. Dartos muscle is innervated by the genital branch of the genitor-
femoral nerve
True False
d. The tunica albuginea does not cover the posterior surface of the testis
True False

Question 2: The iliac crest

a. Ends anteriorly at the anterior superior iliac spine


True False
b. Ends posteriorly at the sacral promontory
True False
c. Has its highest point at the level of the second lumbar vertebral body
True False
d. Has a tubercle which lies at the level of the body of L5
True False

Question 3: With respect to abdominal hernias

a. Umbilical hernias are always congenital


True False
b. Para-umbilical hernias are more common in males
True False
c. The neck of a femoral hernia lies medial to the femoral vein
True False
d. A Richter?s hernia occurs through the arcuate line
True False

1
Question 4: Prolactin

a. Is not produced by the fetal membranes


True False
b. Concentration in amniotic fluid decreases with gestation from 20
weeks
True False
c. Is important in the regulation of amniotic fluid electrolyte balance
True False
d. Production by the decidua is inhibited by dopamine agonists
True False

Question 5: Concerning the abdominal wall

a. The umbilicus is located in the L1 dermatome


True False
b. The left and right epigastric arteries anastomose
True False
c. Distended veins radiating from the umbilicus are indicative of portal
hypertension
True False
d. Langer?s lines run vertically in the lower abdomen
True False

Question 6: With respect to the blood supply to the testis

a. The testicular artery is a branch of the aorta at the level of L5


True False
b. The left testicular vein drains into the left renal vein
True False
c. The right testicular vein drains into the inferior vena cava
True False
d. The epididymis is supplied by the artery to the vas, a branch of the
inferior epigastric artery
True False

2
Question 7: With respect to the scrotum and its contents

a. Dartos muscle is smooth muscle


True False
b. The external spermatic fascia lies superficial to Dartos muscle
True False
c. Cremasteric fascia lies deep to the external spermatic fascia
True False
d. The tunica vaginalis covers the anterior, posterior and lateral aspects
of the testis
True False

Question 8: When making a low transverse (Pfannestiel) incision, the


following structures are divided

a. The aponeurosis of the internal oblique


True False
b. The rectus abdominis muscle
True False
c. The visceral peritoneum
True False
d. Linea alba
True False

Question 9: With respect to the muscles of the anterior abdominal wall

a. The rectus abdominis originates from symphysis pubis and the pubic
crest
True False
b. The rectus abdominis inserts onto the 5th, 6th and 7th costal cartilages
and the xiphoid process (T)pyramidalis lies deep to rectus abdominis
True False
c. The external oblique, internal oblique and transversus abdominis are
supplied by the lower 6 thoracic nerves and the ileoinguinal and
ileohypogastric nerves
True False
d. Pyramidalis is supplied by L1
True False

Question 10: The inguinal ligament

3
a. Is formed by the aponeurosis of the internal oblique muscle
True False
b. Is attached laterally to the anterior superior iliac spine
True False
c. Is attached medially to the posterior superior iliac spine
True False
d. Forms the posterior wall of the inguinal canal
True False

Question 11: The following structures pass beneath the inguinal


ligament

a. Ilioingiunal nerve
True False
b. Femoral artery
True False
c. Femoral branch of the genitor-femoral nerve
True False
d. Lateral cutaneous nerve of the thigh
True False

Question 12: A normal distribution

a. Is symmetrical about the mode


True False
b. Has a median which is always less than the mean
True False
c. Has 75% of its values above the upper quartile
True False
d. Cannot contain negative values
True False

Question 13: With respect to the blood supply and lymphatic drainage of
the skin of the anterior abdominal wall

4
a. The superior epigastric artery is a branch of the aorta
True False
b. The inferior epigastric artery is a branch of the external iliac artery
True False
c. The lateral aspects of the abdominal wall are supplied by the lumbar
arteries and the deep circumflex iliac artery
True False
d. There is no connection between the veins of the anterior abdominal
wall and the portal vein
True False

Question 14: The following structures are present within the inguinal
canal in the male

a. Testicular artery
True False
b. Vas derefens
True False
c. Inguinal nerve
True False
d. Genital branch of the genitor-femoral nerve
True False

Question 15: With respect to the inguinal canal

a. The pubic crest forms the base of the superficial inguinal ring
True False
b. The margins of the superficial inguinal ring form the external spermatic
fascia
True False
c. The anterior wall is formed by the aponeurosis of the external oblique
muscle
True False
d. The anterior wall is reinforced laterally by the origin of the internal
oblique muscle
True False

5
Question 1: With respect to the lumbar plexus

a. The femoral nerve is derived from L1, L1 & L3

True False
b. The femoral nerve enters the thigh anterior to the inguinal ligament

True False
c. The femoral nerve enters the thigh lateral to the femoral sheath

True False
d. The femoral nerve enters the thigh medial to the femoral canal

True False

Question 2: The abdominal aorta

a. Enters the abdomen anterior to the 12th thoracic vertebra

True False
b. Lies to the right of the cisterna chyli

True False
c. Bifurcates into the left and right common iliac arteries at the level of the first lumbar vertebra

True False
d. Lies to the left of the inferior vena cava

True False

Question 3: The portal vein

a. Is formed by the union of the superior and inferior mesenteric veins

True False
b. Lies anterior to the first part of the duodenum

True False
c. Lies within the lesser omentum

True False
d. Lies posterior to the opening into the lesser sac

True False

Question 4: With respect to the lumbar plexus

a. The lumbar plexus is formed from the posterior rami of the upper 4 lumbar nerves

True False
b. The nerves of the lumbar plexus receive grey rami communicantes from the sympathetic trunk

True False
c. The upper 3 lumbar nerves give off white rami communicantes to the sympathetic trunk

True False
d. The lumbar plexus is formed anterior to the psoas and iliacus muscles

True False
Question 5: With respect to the blood supply to the gastro-intestinal tract

a. The inferior pancreatico-duodenal artery is a branch of the celiac artery

True False
b. The middle colic artery is a branch of the inferior mesenteric artery

True False
c. The ileocolic artery is a branch of the superior mesenteric artery

True False
d. The inferior mesenteric artery is the artery to the hind-gut

True False

Question 6: The abdominal part of the sympathetic trunk

a. e) The upper two sympathetic ganglia receive a white ramus communicans from the first and
second lumbar nerves

True False
b. f) Gives off grey rami communicantes to the lumbar spinal nerves

True False
c. g) The left sympathetic trunk is posterior to the inferior vena cava

True False
d. h) Enters the abdomen behind the medial arcuate ligament

True False

Question 7: With respect to the blood supply to the gastro-intestinal tract

a. The celiac artery is the artery to the fore-gut

True False
b. The celiac artery supplies the lower third of the oesophagus to the duodeno-jejunal junction
(ligament of Treitz)

True False
c. The inferior mesenteric artery is the artery to the mid-gut

True False
d. The superior mesenteric artery supplies the distal half of the second part of the duodenum to
the junction between the proximal 2/3 and distal 1/3 of the transverse colon

True False

Question 8: The (common) hepatic artery

a. Runs within the lesser omentum

True False
b. Lies posterior to the opening into the lesser sac

True False
c. Lies to the right of the bile duct

True False
d. Lies posterior to the portal vein

True False

Question 9: The celiac plexus


a. Gives off pre-ganglionic sympathetic fibres to the fore-gut

True False
b. Receives the greater and lesser splanchnic nerves

True False
c. Receives parasympathetic post-ganglionic fibres from the vagus nerve

True False
d. Supplies the proximal 2/3 of the transverse colon

True False

Question 10: With respect to the blood supply to the gastro-intestinal tract

a. The left colic artery is a branch of the inferior mesenteric artery

True False
b. The superior rectal artery is a continuation of the inferior mesenteric artery

True False
c. The splenic artery is a branch of the celiac artery

True False
d. The short gastric arteries are branches of the splenic artery

True False

Question 11: With respect to the lumbar plexus

a. The genito-femoral nerve is derived from L1 & L2

True False
b. The genito-femoral nerve emerges from the lumbar plexus on the anterior surface of psoas
major

True False
c. The genital bramch of the genito-femoral nerve enters the spermatic cord

True False
d. The femoral branch of the genito-femoral nerve supplies the cremaster muscle

True False

Question 12: The inferior vena cava within the abdomen

a. Lies to the right of the aorta

True False
b. Pierces the central tendon of the diaphragm at the level of T12

True False
c. Lies medial to the right ureter

True False
d. Is related anteriorly to the opening into the lesser sac

True False

Question 13: The celiac artery


a. Is the artery of the mid-gut

True False
b. Lies anterior to the lesser sac

True False
c. Is a branch of the abdominal aorta at the level of the 12th thoracic vertebra

True False
d. Has three branches ? the right gastric artery, the splenic artery and the hepatic artery

True False

Question 14: With respect to the lumbar plexus

a. The obturator nerve is derived from L2,L3 & L4

True False
b. The obturator nerve emerges from the lumbar plexus on the medial border of psoas major

True False
c. The obturator nerve crosses the pelvic brim infront of the sacral promontory

True False
d. The obturator nerve crosses the pelvic brim anterior to the common iliac vessels

True False

Question 15: The inferior vena cava

a. Drains into the left atrium

True False
b. Lies to the left of the cysterna chyli

True False
c. Is related posteriorly to the pancreas

True False
d. Originates behind the left common iliac vein

True False
Question 1: With respect to the bladder and urodynamic studies

a. Cystometry is not always necessary to make a diagnosis of detrusor


instability
True False
b. Urinary leakage occurring without a rise in detrusor pressure is
consistent with genuine stress incontinence
True False
c. During bladder filling, the detrusor pressure rise should be < 15mmHg
True False
d. High urethral pressure profile occurs in detrusor instability
True False

Question 2: With respect to the bladder and urodynamic studies

a. High urethral pressure profile occurs in genuine stress incontinence


True False
b. A peak urinary flow rate of <15ml/s may occur if only a small volume of
urine is passed
True False
c. A peak urinary flow rate of <15ml/s may occur with a hypotonic
bladder
True False
d. A first sensation to void at 50ml is consistent with sensory urgency
True False

Question 3: With respect to the blood supply to the rectum

a. The superior rectal artery is a branch of the internal iliac artery


True False
b. The middle rectal artery is a branch of the internal iliac artery
True False
c. The inferior rectal artery is a branch of the superior vesical artery
True False
d. The lower third of the rectum is supplied by a branch of the internal
pudendal artery
True False

1
Question 4: With respect to the uterus

a. In an anteverted uterus, the long axis of the uterus is typically at


45degrees to the long axis of the vagina
True False
b. In an ante-flexed uterus, the long axis of the body of the uterus is bent
forward at the level of the external os
True False
c. The body of a retroverted uterus lies within the utero-vesical pouch
True False
d. The body of an anteverted uterus lies within the pouch of Douglas
True False

Question 5: The broad ligament

a. Has the ovary attached to its posterior surface by the mesovarium


True False
b. Is a two layered fold of peritoneum
True False
c. Has the ureter crossing the uterine artery at its base
True False
d. Contains the epoophron
True False

Question 6: With respect to the bladder and urodynamic studies

a. A first sensation to void at 200ml is consistent with detrusor instability


True False
b. Urinary leakage with a rise in detrusor pressure during bladder filling is
consistent with genuine stress incontinence
True False
c. A first sensation to void ar 500ml is consistent with overflow
incontinence
True False
d. Cystometry is required to measure peak urinary flow rate
True False

2
Question 7: The sigmoid colon

a. Is retro-peritoneal
True False
b. Is continuous with the rectum at the level of S3
True False
c. Is continuous with the descending colon in front of the right external
iliac artery
True False
d. Is related anteriorly to the uterus
True False

Question 8: The ovary

a. Lies on the obturator nerve within the ovarian fossa


True False
b. Is surrounded by a fibrous capsule, the tunica albuginae
True False
c. Is supplied by the ovarian artery, a branch of the abdominal aorta at
L1
True False
d. Lymphatic drainage of the ovary is to the internal iliac nodes
True False

Question 9: The uterus

a. Is completely covered by peritoneum


True False
b. Is related anteriorly to the bladder
True False
c. Is anterior to the pouch of Douglas
True False
d. Is related posteriorly to the sigmoid colon
True False

3
Question 10: With respect to the uterus

a. The fundus is that part of the uterus above the entrance of the fallopian
tubes
True False
b. The uterus is 8cm long x 2cm wide by 5cm thick
True False
c. In saggital section, the cavity of the uterus is triangular in shape
True False
d. In coronal section, the cavity of the uterus is represented by a cleft
True False

Question 11: The following structures provide support to the uterus

a. Perineal body
True False
b. Transverse cervical ligament
True False
c. Utero-sacral ligament
True False
d. Pubo-cervical ligament
True False

Question 12: The following normal structures can be palpated through


the vagina

a. The cervix
True False
b. The bladder
True False
c. The appendix
True False
d. The perineal body
True False

4
Question 13: The ovary

a. Is attached to the anterior surface of the broad ligament


True False
b. Is retro-peritoneal
True False
c. Is attached to the pelvic side wall by the round ligament of the ovary
True False
d. Is attached to the upper end of the lateral wall of the uterus by the
suspensory ligament of the ovary
True False

Question 14: The cervix

a. Communicates with the cavity of the uterus through the internal os


True False
b. Has a spindle-shaped cervical canal
True False
c. Communicates with the vagina through the external os
True False
d. Is divided into a supra-vaginal and a vaginal part
True False

Question 15: The broad ligament

a. Contains the paroophron


True False
b. Has the fallopian tube in its upper free border
True False
c. Has the ovarian artery in its lower attached border
True False
d. Has the round ligament forming a ridge on its posterior surface
True False

5
Question 1: The pelvic splanchnic nerves

a. Contain afferent fibres for the ovary


True False
b. Conduct pain from the body of the uterus
True False
c. Originate from L 2,3,4 spinal segments
True False
d. Are adrenergic
True False

Question 2: With respect to the levator ani muscle

a. Anterior fibres are inserted into the perineal body


True False
b. The posterior fibres (iliococcygeus) are inserted into the anococcygeal
body and the coccyx
True False
c. Intermediate fibres (puburectalis) are inserted into the anococcygeal
body and the median raphe
True False
d. Form a smooth muscle sphincter at the ano-rectal junction
True False

Question 3: The pyriformis muscle

a. Originates from the sacral promontory


True False
b. Leaves the pelvis through the lesser sciatic foramen
True False
c. Is inserted onto the lesser trochanter of the femur
True False
d. Medially rotated the femur at the hip joint
True False

1
Question 4: The levator ani muscle

a. Originates from the back of the pubic bone anteriorly


True False
b. Originates from the obturator fascia laterally
True False
c. Originates from the ischial spine
True False
d. The anterior fibres are inserted onto the symphysis pubis
True False

Question 5: The obturator internus muscle

a. Forms the roof of the ischio-rectal fossa


True False
b. Is innervated by the femoral nerve
True False
c. Has fascia on its pelvic surface which gives rise to the levator ani
muscle
True False
d. Is a medial rotator of the thigh
True False

Question 6: The obturator nerve

a. Passes through the greater sciatic foramen


True False
b. Arises from the sacral plexus
True False
c. Descends through psoas major
True False
d. Mainly supplies the abductor muscles of the thigh
True False

2
Question 7: The obturator nerve

a. Emerges from the lateral border of the psoas muscle


True False
b. Is formed from the posterior division of L 2,3 4
True False
c. Passes lateral to the internal iliac vessels
True False
d. Lies below the obturator artery in the obturator foramen
True False

Question 8: The pelvic part of the sympathetic trunk

a. Receives white rami communicantes from the sacral nerves


True False
b. Is continuous with the abdominal part of the sympathetic trunk anterior
to the common iliac vessels
True False
c. Supplies the descending colon
True False
d. Is part of the sacral plexus
True False

Question 9: The obturator internus muscle

a. Originates from the pelvic surface of the obturator membrane


True False
b. Leaves the pelvis through the greater sciatic foramen
True False
c. Is inserted onto the greater trochanter of the femur
True False
d. Laterally rotates the femur at the hip joint
True False

3
Question 10: The levator ani muscle

a. Is supplied by the perineal branch of S4


True False
b. Is supplied by the nerve to obturator internus
True False
c. Is supplied by the perineal branch of the pudendal nerve
True False
d. Support the pelvic viscera and resist the rise in intra-pelvic pressure
during straining
True False

Question 11: The pelvic surface of the sacrum

a. Gives origin to the piriformis muscle


True False
b. Gives origin to the levator ani muscle
True False
c. Is broader in the male than in the female
True False
d. Transmits the dorsal rami of the spinal nerves
True False

Question 12: With respect to the hypogastric plexuses

a. The inferior hypogastric plexuses lie medial to the rectum


True False
b. The inferior hypogastric plexuses lie lateral to the internal iliac artery
True False
c. The inferior hypogastric plexuses contain both sympathetic and
parasympathetic fibres
True False
d. The superior hypogastric plexus supplies the descending colon
True False

Question 13: The pelvic splanchnic nerves

4
a. Join the inferior hypogastric plexus directly
True False
b. Join the superior hypogastric plexus directly
True False
c. Provide parasympathetic supply to the hind-gut
True False
d. Provide sympathetic supply to the bladder
True False

Question 14: The pelvic splanchnic nerves

a. Supply afferent fibres


True False
b. Intermingle with branches of the sympathetic pelvic plexus
True False
c. Are pre-ganglionic fibres
True False
d. Supply the bladder sphincter with motor fibres
True False

Question 15: With respect to the true pelvis

a. The inlet is bounded posteriorly by the sacral promontory


True False
b. The lateral border of the inlet is formed by the lacunar ligament
True False
c. The anterior border of the inlet is formed by the symphysis pubis
True False
d. The posterior border of the outlet is formed by the sacro-spinous
ligament
True False

5
Question 1: The external anal sphincter

a. Is made up of skeletal muscle


True False
b. Has a sub-cutaneous component
True False
c. Has a deep component which is attached to the perineal body
True False
d. Has a superficial component which is attached to the anococcygeal
body
True False

Question 2: The deep transverse perineal muscle

a. Lies anterior to the sphincter urethrae


True False
b. Originates from the ischial ramus
True False
c. Is inserted onto the perineal body
True False
d. Is supplied by the perineal branch of the pudendal nerve
True False

Question 3: The superficial perineal pouch in the female

a. Communicates anteriorly with the potential space between the


superficial fascia of the anterior abdominal wall and the anterior
abdominal wall muscles
True False
b. Contains the bulbospongiosus muscle
True False
c. Contains the ischiocavernosus muscle
True False
d. Contains the superficial transverse perineal muscle
True False

1
Question 4: The superficial perineal pouch in the female

a. Is bounded inferiorly by the upper surface of the urogenital diaphragm


True False
b. Is closed posteriorly by the fusion of its upper and lower walls
True False
c. Communicates with the cavity of the pelvis laterally
True False
d. Is closed laterally by the attachment of the membraneous layer of
superficial fascia to the pubic arch
True False

Question 5: The internal anal sphincter

a. Is superficial to the longitudinal muscle fibres of the anal canal


True False
b. Is made up of smooth muscle
True False
c. Has a deep, superficial and sub-cutaneous component
True False
d. Is attached to the perineal body
True False

Question 6: The bulbospongiosus muscle

a. Surrounds the orifice of the urethra


True False
b. Surrounds the orifice of the vagina
True False
c. Covers the bulb of the vestibule
True False
d. Is attached to the crus of the clitoris
True False

2
Question 7: The superficial perineal pouch in the female

a. Contains the bulb of the vestibule


True False
b. Lies between the two layers of the urogenital diaphragm
True False
c. Contains the perineal branch of the pudendal nerve
True False
d. Contains branches of the internal pudendal artery
True False

Question 8: The urogenital diaphragm

a. Is attached anteriorly to the symphysis pubis


True False
b. Is attached posteriorly to the perineal body
True False
c. Is attached laterally to the pubic arch
True False
d. Contains the deep perineal pouch
True False

Question 9: The anal canal

a. Is 2cm long
True False
b. Has its axis directed downwards and forwards from the rectal ampulla
True False
c. Has its anterior and posterior walls in apposition by the tone of the
levator ani muscles
True False
d. Is related posteriorly to the anococcygeal body
True False

3
Question 10: The ischiocavernosus muscle

a. Originates from the ischial spine


True False
b. Is inserted onto the corpus cavernosus
True False
c. Is supplied by the dorsal nerve of the clitoris
True False
d. Assists in erection of the clitoris
True False

Question 11: The sphincter urethrae

a. Is made of skeletal muscle


True False
b. Originates from the pubic arch
True False
c. Surrounds the membranous part of the urethra
True False
d. Is supplied by the perineal branch of the pudendal nerve
True False

Question 12: The superficial transverse perineal muscle

a. Lies in the anterior part of the superficial perineal pouch


True False
b. Originates from the ischial tuberosity
True False
c. Is inserted onto the perineal body
True False
d. Is supplied by the perineal branch of the pudendal nerve
True False

4
Question 13: The ischio-rectal fossa

a. Is bounded inferiorly by the ischial tuberosities


True False
b. Contains the pudendal nerve
True False
c. Contains the internal pudendal artery
True False
d. Is bounded laterally by the obturator internus muscle
True False

Question 14: The deep perineal pouch in the female

a. Contains part of the vagina


True False
b. Contains the deep transverse perineal muscle
True False
c. Contains the superficial transverse perineal muscle
True False
d. Contains the branches of the internal pudendal artery
True False

Question 15: The pudendal nerve

a. Enters the perineum lateral to the ischial spine


True False
b. Supplies the internal anal sphincter
True False
c. Gives off the inferior rectal nerve
True False
d. Gives off the dorsal nerve of the clitoris
True False

5
Question 1: The pulmonary trunk

a. Lies posterior to the trachea


True False
b. Lies superior to the aortic arch
True False
c. Conveys de-oxygenated blood from the heart
True False
d. Divides into right and left pulmonary veins
True False

Question 2: With respect to the conducting system of the heart

a. The atrio-ventricular bundle is the only muscular connection between


the myocardium of the atria and the ventricular myocardium
True False
b. Cardiac impulses are relayed from the atrio-ventricular node to the
ventricles by the atrio-ventricular bundle
True False
c. The sino-atrial node is supplied by the left and right coronary arteries
True False
d. The atrio-ventricular node is supplied by the left coronary artery
True False

Question 3: The superior vena cava

a. Is formed from the right brachiocephalic and left internal jugular veins
True False
b. Receives blood from the azygos vein
True False
c. Lies anterior to the aorta in the superior mediastinum
True False
d. Lies posterior to the trachea in the superior mediastinum
True False

1
Question 4: The trachea

a. Lies anterior to the thymus in the superior mediastinum


True False
b. Is posterior to the arch of the aorta
True False
c. Lies anterior to the origin of the left common carotid artery
True False
d. Is related anteriorly to the oesophagus
True False

Question 5: The right ventricle

a. Has a pulmonary out-flow tract (pulmonary valve) with two semilunar


cusps
True False
b. Contracts during ventricular diastole
True False
c. Communicates directly with the inferior vena cava
True False
d. Pumps oxygenated blood into the pulmonary artery
True False

Question 6: The left ventricle

a. Has trabeculae carneae


True False
b. Has a moderator band
True False
c. Has three papillary muscles
True False
d. Has a thicker wall than that of the right ventricle
True False

2
Question 7: The arch of the aorta has the following branches

a. The left common carotid artery


True False
b. The left subclavian artery
True False
c. The internal thoracic artery
True False
d. Oesophageal artery
True False

Question 8: The right vagus nerve

a. Enters the thorax medial to the brachiocephalic artery


True False
b. Lies medial to the trachea in the superior mediastinum
True False
c. Lies medial to the azygos vein in the superior mediastinum
True False
d. Descends anterior to the root of the right lung
True False

Question 9: The arch of the aorta has the following branches

a. The right brachiocephalic artery


True False
b. The left brachiocephalic artery
True False
c. The right subclavian artery
True False
d. The right common carotid artery
True False

3
Question 10: Within the mediastinum

a. The heart is anterior to the oesophagus


True False
b. The thymus is anterior to the heart
True False
c. The thoracic duct lies anterior to the heart
True False
d. The oesophagus lies posterior to the descending aorta
True False

Question 11: The left lung

a. Receives de-oxygenated blood from the left pulmonary artery


True False
b. Has Lymphatic drainage to the para-aortic nodes
True False
c. Receives parasympathetic supply from the vagus nerve
True False
d. Sympathetic stimulation results in bronchodilation and vasoconstriction
True False

Question 12: The right vagus nerve

a. Enters the abdomen on the anterior surface of the oesophagus


True False
b. Contributes to the right pulmonary plexus
True False
c. Gives off the right recurrent laryngeal nerve within the thorax
True False
d. Stimulation results in vasodilation of the bronchial vessels
True False

Question 13: The right ventricle

4
a. Communicates with the pulmonary vein
True False
b. Has papillary muscles
True False
c. Is traversed by the moderator band
True False
d. Has papillary muscles which are connected to the cusps of the
tricuapid valve by the chordae tendinae
True False

Question 14: The left atrium

a. Communicates with the left ventricle via the tricuspid valve


True False
b. Receives blood from two pulmonary arteries
True False
c. Forms the base of the heart
True False
d. Has an auricle
True False

Question 15: The right atrium

a. Has a main cavity and an auricle


True False
b. Opens into the right ventricle via the mitral valve
True False
c. Has the fossa ovalis located on the atrial septum
True False
d. The fossa ovalis is the remnant of the septum secundum
True False

5
Question 1: The diaphragm

a. Has a left crus originating from the lateral arcuate ligament


True False
b. Has the median arcuate ligament between the medial and the lateral
arcuate ligaments
True False
c. Contracts during expiration
True False
d. Contracts during micturiction and defecation
True False

Question 2: The following statements about the intercostal arteries are


true

a. The anterior intercostal arteries of the lower 5 spaces are branches if


the superior epigastric artery
True False
b. The intercostal arteries supply the parietal pleura
True False
c. The intercostal arteries supply the visceral pleura
True False
d. The intercostal arteries supply the intercostal muscles and skin of the
thoracic wall
True False

Question 3: A needle inserted in the mid-axillary line over the 8th


intercostal space into the pleural space would traverse the following
structures

a. Serratus anterior muscle


True False
b. External intercostal muscle
True False
c. External oblique muscle
True False
d. Internal intercostal muscle
True False

1
Question 4: The internal thoracic artery

a. Supplies the pericardium


True False
b. Supplies the thymus gland
True False
c. Gives off one anterior intercostal artery to the upper 6 intercostal
spaces
True False
d. Gives off perforating branches
True False

Question 5: The first rib

a. Articulates with the first thoracic vertebral body


True False
b. Articulates with the body of the 7th cervical vertebra
True False
c. Is crossed by the subclavian vein anterior to the scalene tubercle
True False
d. Is crossed by the subclavian artery posterior to the scalene tubercle
True False

Question 6: The diaphragm

a. Causes a reduction in venous return during contraction


True False
b. Has the right and left vagus nerves entering the abdomen through the
oesophageal opening
True False
c. Has a caval opening at the level of the 8th thoracic vertebra
True False
d. Has the peritoneum on its inferior surface supplied by the phrenic
nerve
True False

2
Question 7: The diaphragm

a. Is covered by visceral pleura on its superior surface


True False
b. Receives motor nerve supply from the phrenic nerve
True False
c. Receives sensory supply from the phrenic nerve
True False
d. Receives sensory supply from the lower 5 intercostal nerves
True False

Question 8: With respect to the intercostals muscles

a. The fibres of the external intercostal muscles are directed downwards


and forwards
True False
b. The external intercostal muscles extend from the tubercle of the ribs to
the sternum
True False
c. The internal intercostal muscles extend from the sternum to the
tubercle of the rib posteriorly
True False
d. The transversus thoracis muscle extends over more than one
intercostal space
True False

Question 9: A needle inserted in the mid-axillary line over the 8th


intercostal space into the pleural space would traverse the following
structures

a. Transversus thoracis muscle


True False
b. The costal origin of the diaphragm
True False
c. Visceral pleura
True False
d. The posterior intercostal membrane
True False

Question 10: With respect to the lymphatic drainage of the thoracic wall

3
a. Lymphatic drainage of the skin of the anterior chest wall is to the
anterior axillary nodes
True False
b. Lymphatic drainage of the skin of the posterior chest wall is to the
posterior axillary nodes
True False
c. Lymphatic drainage of the anterior part of the intercostal spaces is to
the internal thoracic nodes
True False
d. Lymphatic drainage of the posterior part of the intercostal spaces is to
the para-aortic nodes
True False

Question 11: The first rib

a. Has the scalenus anterior muscle inserted onto the scalene tubercle
True False
b. Is crossed by the brachial plexus anterior to the scalenus anterior
muscle
True False
c. Articulates with the transverse process of the 7th cervical vertebra
True False
d. Is palpable throughout its length
True False

Question 12: With respect to the lymphatic drainage of the breast

a. Lymphatic vessels cross the mid-line


True False
b. Lymphatic drainage of the lateral part of the gland is to the anterior
axillary nodes
True False
c. Lymphatic drainage of the superior part of the gland is to the supra-
clavicular nodes
True False
d. Lymphatic drainage of the medial part of the gland is to the internal
thoracic nodes
True False

Question 13: The diaphragm

4
a. Has a central muscular part and a peripheral tendinous part
True False
b. Has a sternal origin from the posterior surface of the body of the
sternum
True False
c. Has a costal origin from the deep surfaces of the lower 4 ribs and their
costal cartilages
True False
d. Has a right crus originating from the medial arcuate ligament
True False

Question 14: The intercostal nerves

a. Are the posterior rami of the first 11 thoracic spinal nerves


True False
b. Enter the intercostal space between the internal intercostal muscle and
the transversus thoracis
True False
c. Run superior to the intercostal vein
True False
d. Give off grey rami communicantes to the sympathetic trunk
True False

Question 15: With respect to the surface anatomy of the thorax

a. The xiphisternal joint lies opposite the body of the 9th thoracic vertebra
True False
b. The lowest part of the costal margin is formed by the 12th rib
True False
c. The lowest part of the costal margin lies at the level of the 3rd lumbar
vertebra
True False
d. The first rib is palpable throughout its length
True False

5
Question 1: The ascending colon

a. Is related posteriorly to the right ureter

True False
b. Is related superiorly to the right lobe of the liver

True False
c. Is related anteriorly to the greater omentum

True False
d. Has parasympathetic supply from the vagus nerve via the inferior mesenteric plexus

True False

Question 2: With respect to the biliary tree

a. The bile canaliculi drain into the interlobular ducts

True False
b. The interlobular ducts are located at the centre of the hepatic lobule

True False
c. The right hepatic duct drains bile from the caudate lobe of the liver

True False
d. The left hepatic duct drains bile from the quadrate lobe of the liver

True False

Question 3: The cecum

a. Is in direct communication with the lumen of the appendix

True False
b. Is related posteriorly to the psoas and iliacus muscles

True False
c. Is related posteriorly to the femoral nerve

True False
d. Is related posteriorly to the lateral cutaneous nerve of the thigh

True False

Question 4: The ureter

a. Lies 5cm lateral to the lateral vaginal fornix

True False
b. Crosses superior to the uterine artery in the broad ligament

True False
c. Is supplied by the uterine artery within the pelvis

True False
d. Lies more than 5cm lateral to the supra-vaginal cervix

True False

Question 5: The duodenum

1
a. Is 25cm long

True False
b. Is retro-peritoneal throughout its length

True False
c. The first part is 5cm long and lies on the intercristal plane

True False
d. The first part is related anteriorly to the quadrate lobe of the liver and the gall bladder

True False

Question 6: The pancreas

a. Is related anteriorly to the transverse colon and the transverse mesocolon

True False
b. Lies anterior to the lesser sac

True False
c. Is posterior to the stomach

True False
d. Is related posteriorly to the splenic vein

True False

Question 7: The cecum

a. Is related posteriorly to the right ureter

True False
b. Receives its blood supply from the inferior mesenteric artery

True False
c. Receives its blood supply from branches of the ileocolic artery

True False
d. Is supplied by the left and right caecal arteries

True False

Question 8: The ureter

a. Enters the pelvis posterior to the bifurcation of the common iliac artery

True False
b. Enters the pelvis in front of the sacro-iliac joint

True False
c. Is supplied by the renal artery

True False
d. Is supplied by the testicular / ovarian artery

True False

Question 9: With respect to the blood supply to the kidneys

2
a. The lobar arteries supply the renal pyramids

True False
b. The interlobular arteries arise from the lobar arteries

True False
c. The efferent glomerular arteriols arise from the interlobular arteries

True False
d. The renal vein drains directly into the inferior vena cava

True False

Question 10: The ureter

a. Is 35cm long

True False
b. Leaves the renal pelvis anterior to the renal vein

True False
c. Is adherent to visceral peritoneum

True False
d. Is separated from the tips of the transverse processes of the lumbar vertebrae by psoas major

True False

Question 11: With respect to the transverse colon

a. Parasympathetic supply to the proximal 2/3 is from the vagus via the inferior mesenteric
plexus

True False
b. Parasympathetic supply to the distal 1/3 is from pelvic splanchnic nerves

True False
c. Sympathetic supply to the proximal 2/3 is from the superior mesenteric plexus

True False
d. Sympathetic supply to the distal 1/3 is from the inferior mesenteric plexus

True False

Question 12: The descending colon

a. Is supplied by the left colic artery, a branch of the inferior mesenteric artery

True False
b. Lymphatic drainage is to the inferior mesenteric nodes

True False
c. Receives sympathetic supply from the pelvic splanchnic nerves

True False
d. Receives parasympathetic supply from the pelvic splanchnic nerves

True False

Question 13: With respect to the nerve supply and lymphatic drainage of the stomach

3
a. The lymphatics drain into the superior mesenteric nodes

True False
b. The anterior vagal trunk is derived mainly from the left vagus nerve

True False
c. The posterior vagal trunk supplies the pancreas

True False
d. The posterior vagal trunk supplies the intestine as far as the splenic flexure

True False

Question 14: The transverse colon

a. In the standing subject, lies along the transpyloric plane

True False
b. Is intra-peritoneal

True False
c. Has the transverse mesocolon attached to its inferior border

True False
d. Has the anterior layer of the greater omentum attached to its inferior border

True False

Question 15: The following structures are present at the porta hepatis

a. Right and left hepatic ducts

True False
b. Right and left branches of the hepatic artery

True False
c. Right and left branches of the hepatic vein

True False
d. The upper part of the free edge of the lesser omentum

True False

4
Question 1: With respect to the scrotum and its contents

a. Colles? fascia is a continuation of Scarpa?s fascia


True False
b. Dartos muscle does not contribute to the median partition of the
scrotum
True False
c. Dartos muscle is innervated by the genital branch of the genitor-
femoral nerve
True False
d. The tunica albuginea does not cover the posterior surface of the testis
True False

Question 2: The iliac crest

a. Ends anteriorly at the anterior superior iliac spine


True False
b. Ends posteriorly at the sacral promontory
True False
c. Has its highest point at the level of the second lumbar vertebral body
True False
d. Has a tubercle which lies at the level of the body of L5
True False

Question 3: With respect to abdominal hernias

a. Umbilical hernias are always congenital


True False
b. Para-umbilical hernias are more common in males
True False
c. The neck of a femoral hernia lies medial to the femoral vein
True False
d. A Richter?s hernia occurs through the arcuate line
True False

1
Question 4: Prolactin

a. Is not produced by the fetal membranes


True False
b. Concentration in amniotic fluid decreases with gestation from 20
weeks
True False
c. Is important in the regulation of amniotic fluid electrolyte balance
True False
d. Production by the decidua is inhibited by dopamine agonists
True False

Question 5: Concerning the abdominal wall

a. The umbilicus is located in the L1 dermatome


True False
b. The left and right epigastric arteries anastomose
True False
c. Distended veins radiating from the umbilicus are indicative of portal
hypertension
True False
d. Langer?s lines run vertically in the lower abdomen
True False

Question 6: With respect to the blood supply to the testis

a. The testicular artery is a branch of the aorta at the level of L5


True False
b. The left testicular vein drains into the left renal vein
True False
c. The right testicular vein drains into the inferior vena cava
True False
d. The epididymis is supplied by the artery to the vas, a branch of the
inferior epigastric artery
True False

2
Question 7: With respect to the scrotum and its contents

a. Dartos muscle is smooth muscle


True False
b. The external spermatic fascia lies superficial to Dartos muscle
True False
c. Cremasteric fascia lies deep to the external spermatic fascia
True False
d. The tunica vaginalis covers the anterior, posterior and lateral aspects
of the testis
True False

Question 8: When making a low transverse (Pfannestiel) incision, the


following structures are divided

a. The aponeurosis of the internal oblique


True False
b. The rectus abdominis muscle
True False
c. The visceral peritoneum
True False
d. Linea alba
True False

Question 9: With respect to the muscles of the anterior abdominal wall

a. The rectus abdominis originates from symphysis pubis and the pubic
crest
True False
b. The rectus abdominis inserts onto the 5th, 6th and 7th costal cartilages
and the xiphoid process (T)pyramidalis lies deep to rectus abdominis
True False
c. The external oblique, internal oblique and transversus abdominis are
supplied by the lower 6 thoracic nerves and the ileoinguinal and
ileohypogastric nerves
True False
d. Pyramidalis is supplied by L1
True False

Question 10: The inguinal ligament

3
a. Is formed by the aponeurosis of the internal oblique muscle
True False
b. Is attached laterally to the anterior superior iliac spine
True False
c. Is attached medially to the posterior superior iliac spine
True False
d. Forms the posterior wall of the inguinal canal
True False

Question 11: The following structures pass beneath the inguinal


ligament

a. Ilioingiunal nerve
True False
b. Femoral artery
True False
c. Femoral branch of the genitor-femoral nerve
True False
d. Lateral cutaneous nerve of the thigh
True False

Question 12: A normal distribution

a. Is symmetrical about the mode


True False
b. Has a median which is always less than the mean
True False
c. Has 75% of its values above the upper quartile
True False
d. Cannot contain negative values
True False

Question 13: With respect to the blood supply and lymphatic drainage of
the skin of the anterior abdominal wall

4
a. The superior epigastric artery is a branch of the aorta
True False
b. The inferior epigastric artery is a branch of the external iliac artery
True False
c. The lateral aspects of the abdominal wall are supplied by the lumbar
arteries and the deep circumflex iliac artery
True False
d. There is no connection between the veins of the anterior abdominal
wall and the portal vein
True False

Question 14: The following structures are present within the inguinal
canal in the male

a. Testicular artery
True False
b. Vas derefens
True False
c. Inguinal nerve
True False
d. Genital branch of the genitor-femoral nerve
True False

Question 15: With respect to the inguinal canal

a. The pubic crest forms the base of the superficial inguinal ring
True False
b. The margins of the superficial inguinal ring form the external spermatic
fascia
True False
c. The anterior wall is formed by the aponeurosis of the external oblique
muscle
True False
d. The anterior wall is reinforced laterally by the origin of the internal
oblique muscle
True False

5
Question 1: With respect to the lumbar plexus

a. The femoral nerve is derived from L1, L1 & L3

True False
b. The femoral nerve enters the thigh anterior to the inguinal ligament

True False
c. The femoral nerve enters the thigh lateral to the femoral sheath

True False
d. The femoral nerve enters the thigh medial to the femoral canal

True False

Question 2: The abdominal aorta

a. Enters the abdomen anterior to the 12th thoracic vertebra

True False
b. Lies to the right of the cisterna chyli

True False
c. Bifurcates into the left and right common iliac arteries at the level of the first lumbar vertebra

True False
d. Lies to the left of the inferior vena cava

True False

Question 3: The portal vein

a. Is formed by the union of the superior and inferior mesenteric veins

True False
b. Lies anterior to the first part of the duodenum

True False
c. Lies within the lesser omentum

True False
d. Lies posterior to the opening into the lesser sac

True False

Question 4: With respect to the lumbar plexus

a. The lumbar plexus is formed from the posterior rami of the upper 4 lumbar nerves

True False
b. The nerves of the lumbar plexus receive grey rami communicantes from the sympathetic trunk

True False
c. The upper 3 lumbar nerves give off white rami communicantes to the sympathetic trunk

True False
d. The lumbar plexus is formed anterior to the psoas and iliacus muscles

True False
Question 5: With respect to the blood supply to the gastro-intestinal tract

a. The inferior pancreatico-duodenal artery is a branch of the celiac artery

True False
b. The middle colic artery is a branch of the inferior mesenteric artery

True False
c. The ileocolic artery is a branch of the superior mesenteric artery

True False
d. The inferior mesenteric artery is the artery to the hind-gut

True False

Question 6: The abdominal part of the sympathetic trunk

a. e) The upper two sympathetic ganglia receive a white ramus communicans from the first and
second lumbar nerves

True False
b. f) Gives off grey rami communicantes to the lumbar spinal nerves

True False
c. g) The left sympathetic trunk is posterior to the inferior vena cava

True False
d. h) Enters the abdomen behind the medial arcuate ligament

True False

Question 7: With respect to the blood supply to the gastro-intestinal tract

a. The celiac artery is the artery to the fore-gut

True False
b. The celiac artery supplies the lower third of the oesophagus to the duodeno-jejunal junction
(ligament of Treitz)

True False
c. The inferior mesenteric artery is the artery to the mid-gut

True False
d. The superior mesenteric artery supplies the distal half of the second part of the duodenum to
the junction between the proximal 2/3 and distal 1/3 of the transverse colon

True False

Question 8: The (common) hepatic artery

a. Runs within the lesser omentum

True False
b. Lies posterior to the opening into the lesser sac

True False
c. Lies to the right of the bile duct

True False
d. Lies posterior to the portal vein

True False

Question 9: The celiac plexus


a. Gives off pre-ganglionic sympathetic fibres to the fore-gut

True False
b. Receives the greater and lesser splanchnic nerves

True False
c. Receives parasympathetic post-ganglionic fibres from the vagus nerve

True False
d. Supplies the proximal 2/3 of the transverse colon

True False

Question 10: With respect to the blood supply to the gastro-intestinal tract

a. The left colic artery is a branch of the inferior mesenteric artery

True False
b. The superior rectal artery is a continuation of the inferior mesenteric artery

True False
c. The splenic artery is a branch of the celiac artery

True False
d. The short gastric arteries are branches of the splenic artery

True False

Question 11: With respect to the lumbar plexus

a. The genito-femoral nerve is derived from L1 & L2

True False
b. The genito-femoral nerve emerges from the lumbar plexus on the anterior surface of psoas
major

True False
c. The genital bramch of the genito-femoral nerve enters the spermatic cord

True False
d. The femoral branch of the genito-femoral nerve supplies the cremaster muscle

True False

Question 12: The inferior vena cava within the abdomen

a. Lies to the right of the aorta

True False
b. Pierces the central tendon of the diaphragm at the level of T12

True False
c. Lies medial to the right ureter

True False
d. Is related anteriorly to the opening into the lesser sac

True False

Question 13: The celiac artery


a. Is the artery of the mid-gut

True False
b. Lies anterior to the lesser sac

True False
c. Is a branch of the abdominal aorta at the level of the 12th thoracic vertebra

True False
d. Has three branches ? the right gastric artery, the splenic artery and the hepatic artery

True False

Question 14: With respect to the lumbar plexus

a. The obturator nerve is derived from L2,L3 & L4

True False
b. The obturator nerve emerges from the lumbar plexus on the medial border of psoas major

True False
c. The obturator nerve crosses the pelvic brim infront of the sacral promontory

True False
d. The obturator nerve crosses the pelvic brim anterior to the common iliac vessels

True False

Question 15: The inferior vena cava

a. Drains into the left atrium

True False
b. Lies to the left of the cysterna chyli

True False
c. Is related posteriorly to the pancreas

True False
d. Originates behind the left common iliac vein

True False
Question 1: The ascending colon

a. Is related posteriorly to the right ureter

True False
b. Is related superiorly to the right lobe of the liver

True False
c. Is related anteriorly to the greater omentum

True False
d. Has parasympathetic supply from the vagus nerve via the inferior mesenteric plexus

True False

Question 2: With respect to the biliary tree

a. The bile canaliculi drain into the interlobular ducts

True False
b. The interlobular ducts are located at the centre of the hepatic lobule

True False
c. The right hepatic duct drains bile from the caudate lobe of the liver

True False
d. The left hepatic duct drains bile from the quadrate lobe of the liver

True False

Question 3: The cecum

a. Is in direct communication with the lumen of the appendix

True False
b. Is related posteriorly to the psoas and iliacus muscles

True False
c. Is related posteriorly to the femoral nerve

True False
d. Is related posteriorly to the lateral cutaneous nerve of the thigh

True False

Question 4: The ureter

a. Lies 5cm lateral to the lateral vaginal fornix

True False
b. Crosses superior to the uterine artery in the broad ligament

True False
c. Is supplied by the uterine artery within the pelvis

True False
d. Lies more than 5cm lateral to the supra-vaginal cervix

True False

Question 5: The duodenum

1
a. Is 25cm long

True False
b. Is retro-peritoneal throughout its length

True False
c. The first part is 5cm long and lies on the intercristal plane

True False
d. The first part is related anteriorly to the quadrate lobe of the liver and the gall bladder

True False

Question 6: The pancreas

a. Is related anteriorly to the transverse colon and the transverse mesocolon

True False
b. Lies anterior to the lesser sac

True False
c. Is posterior to the stomach

True False
d. Is related posteriorly to the splenic vein

True False

Question 7: The cecum

a. Is related posteriorly to the right ureter

True False
b. Receives its blood supply from the inferior mesenteric artery

True False
c. Receives its blood supply from branches of the ileocolic artery

True False
d. Is supplied by the left and right caecal arteries

True False

Question 8: The ureter

a. Enters the pelvis posterior to the bifurcation of the common iliac artery

True False
b. Enters the pelvis in front of the sacro-iliac joint

True False
c. Is supplied by the renal artery

True False
d. Is supplied by the testicular / ovarian artery

True False

Question 9: With respect to the blood supply to the kidneys

2
a. The lobar arteries supply the renal pyramids

True False
b. The interlobular arteries arise from the lobar arteries

True False
c. The efferent glomerular arteriols arise from the interlobular arteries

True False
d. The renal vein drains directly into the inferior vena cava

True False

Question 10: The ureter

a. Is 35cm long

True False
b. Leaves the renal pelvis anterior to the renal vein

True False
c. Is adherent to visceral peritoneum

True False
d. Is separated from the tips of the transverse processes of the lumbar vertebrae by psoas major

True False

Question 11: With respect to the transverse colon

a. Parasympathetic supply to the proximal 2/3 is from the vagus via the inferior mesenteric
plexus

True False
b. Parasympathetic supply to the distal 1/3 is from pelvic splanchnic nerves

True False
c. Sympathetic supply to the proximal 2/3 is from the superior mesenteric plexus

True False
d. Sympathetic supply to the distal 1/3 is from the inferior mesenteric plexus

True False

Question 12: The descending colon

a. Is supplied by the left colic artery, a branch of the inferior mesenteric artery

True False
b. Lymphatic drainage is to the inferior mesenteric nodes

True False
c. Receives sympathetic supply from the pelvic splanchnic nerves

True False
d. Receives parasympathetic supply from the pelvic splanchnic nerves

True False

Question 13: With respect to the nerve supply and lymphatic drainage of the stomach

3
a. The lymphatics drain into the superior mesenteric nodes

True False
b. The anterior vagal trunk is derived mainly from the left vagus nerve

True False
c. The posterior vagal trunk supplies the pancreas

True False
d. The posterior vagal trunk supplies the intestine as far as the splenic flexure

True False

Question 14: The transverse colon

a. In the standing subject, lies along the transpyloric plane

True False
b. Is intra-peritoneal

True False
c. Has the transverse mesocolon attached to its inferior border

True False
d. Has the anterior layer of the greater omentum attached to its inferior border

True False

Question 15: The following structures are present at the porta hepatis

a. Right and left hepatic ducts

True False
b. Right and left branches of the hepatic artery

True False
c. Right and left branches of the hepatic vein

True False
d. The upper part of the free edge of the lesser omentum

True False

4
Question 1: The pelvic splanchnic nerves

a. Contain afferent fibres for the ovary


True False
b. Conduct pain from the body of the uterus
True False
c. Originate from L 2,3,4 spinal segments
True False
d. Are adrenergic
True False

Question 2: With respect to the levator ani muscle

a. Anterior fibres are inserted into the perineal body


True False
b. The posterior fibres (iliococcygeus) are inserted into the anococcygeal
body and the coccyx
True False
c. Intermediate fibres (puburectalis) are inserted into the anococcygeal
body and the median raphe
True False
d. Form a smooth muscle sphincter at the ano-rectal junction
True False

Question 3: The pyriformis muscle

a. Originates from the sacral promontory


True False
b. Leaves the pelvis through the lesser sciatic foramen
True False
c. Is inserted onto the lesser trochanter of the femur
True False
d. Medially rotated the femur at the hip joint
True False

1
Question 4: The levator ani muscle

a. Originates from the back of the pubic bone anteriorly


True False
b. Originates from the obturator fascia laterally
True False
c. Originates from the ischial spine
True False
d. The anterior fibres are inserted onto the symphysis pubis
True False

Question 5: The obturator internus muscle

a. Forms the roof of the ischio-rectal fossa


True False
b. Is innervated by the femoral nerve
True False
c. Has fascia on its pelvic surface which gives rise to the levator ani
muscle
True False
d. Is a medial rotator of the thigh
True False

Question 6: The obturator nerve

a. Passes through the greater sciatic foramen


True False
b. Arises from the sacral plexus
True False
c. Descends through psoas major
True False
d. Mainly supplies the abductor muscles of the thigh
True False

2
Question 7: The obturator nerve

a. Emerges from the lateral border of the psoas muscle


True False
b. Is formed from the posterior division of L 2,3 4
True False
c. Passes lateral to the internal iliac vessels
True False
d. Lies below the obturator artery in the obturator foramen
True False

Question 8: The pelvic part of the sympathetic trunk

a. Receives white rami communicantes from the sacral nerves


True False
b. Is continuous with the abdominal part of the sympathetic trunk anterior
to the common iliac vessels
True False
c. Supplies the descending colon
True False
d. Is part of the sacral plexus
True False

Question 9: The obturator internus muscle

a. Originates from the pelvic surface of the obturator membrane


True False
b. Leaves the pelvis through the greater sciatic foramen
True False
c. Is inserted onto the greater trochanter of the femur
True False
d. Laterally rotates the femur at the hip joint
True False

3
Question 10: The levator ani muscle

a. Is supplied by the perineal branch of S4


True False
b. Is supplied by the nerve to obturator internus
True False
c. Is supplied by the perineal branch of the pudendal nerve
True False
d. Support the pelvic viscera and resist the rise in intra-pelvic pressure
during straining
True False

Question 11: The pelvic surface of the sacrum

a. Gives origin to the piriformis muscle


True False
b. Gives origin to the levator ani muscle
True False
c. Is broader in the male than in the female
True False
d. Transmits the dorsal rami of the spinal nerves
True False

Question 12: With respect to the hypogastric plexuses

a. The inferior hypogastric plexuses lie medial to the rectum


True False
b. The inferior hypogastric plexuses lie lateral to the internal iliac artery
True False
c. The inferior hypogastric plexuses contain both sympathetic and
parasympathetic fibres
True False
d. The superior hypogastric plexus supplies the descending colon
True False

Question 13: The pelvic splanchnic nerves

4
a. Join the inferior hypogastric plexus directly
True False
b. Join the superior hypogastric plexus directly
True False
c. Provide parasympathetic supply to the hind-gut
True False
d. Provide sympathetic supply to the bladder
True False

Question 14: The pelvic splanchnic nerves

a. Supply afferent fibres


True False
b. Intermingle with branches of the sympathetic pelvic plexus
True False
c. Are pre-ganglionic fibres
True False
d. Supply the bladder sphincter with motor fibres
True False

Question 15: With respect to the true pelvis

a. The inlet is bounded posteriorly by the sacral promontory


True False
b. The lateral border of the inlet is formed by the lacunar ligament
True False
c. The anterior border of the inlet is formed by the symphysis pubis
True False
d. The posterior border of the outlet is formed by the sacro-spinous
ligament
True False

5
Question 1: The external anal sphincter

a. Is made up of skeletal muscle


True False
b. Has a sub-cutaneous component
True False
c. Has a deep component which is attached to the perineal body
True False
d. Has a superficial component which is attached to the anococcygeal
body
True False

Question 2: The deep transverse perineal muscle

a. Lies anterior to the sphincter urethrae


True False
b. Originates from the ischial ramus
True False
c. Is inserted onto the perineal body
True False
d. Is supplied by the perineal branch of the pudendal nerve
True False

Question 3: The superficial perineal pouch in the female

a. Communicates anteriorly with the potential space between the


superficial fascia of the anterior abdominal wall and the anterior
abdominal wall muscles
True False
b. Contains the bulbospongiosus muscle
True False
c. Contains the ischiocavernosus muscle
True False
d. Contains the superficial transverse perineal muscle
True False

1
Question 4: The superficial perineal pouch in the female

a. Is bounded inferiorly by the upper surface of the urogenital diaphragm


True False
b. Is closed posteriorly by the fusion of its upper and lower walls
True False
c. Communicates with the cavity of the pelvis laterally
True False
d. Is closed laterally by the attachment of the membraneous layer of
superficial fascia to the pubic arch
True False

Question 5: The internal anal sphincter

a. Is superficial to the longitudinal muscle fibres of the anal canal


True False
b. Is made up of smooth muscle
True False
c. Has a deep, superficial and sub-cutaneous component
True False
d. Is attached to the perineal body
True False

Question 6: The bulbospongiosus muscle

a. Surrounds the orifice of the urethra


True False
b. Surrounds the orifice of the vagina
True False
c. Covers the bulb of the vestibule
True False
d. Is attached to the crus of the clitoris
True False

2
Question 7: The superficial perineal pouch in the female

a. Contains the bulb of the vestibule


True False
b. Lies between the two layers of the urogenital diaphragm
True False
c. Contains the perineal branch of the pudendal nerve
True False
d. Contains branches of the internal pudendal artery
True False

Question 8: The urogenital diaphragm

a. Is attached anteriorly to the symphysis pubis


True False
b. Is attached posteriorly to the perineal body
True False
c. Is attached laterally to the pubic arch
True False
d. Contains the deep perineal pouch
True False

Question 9: The anal canal

a. Is 2cm long
True False
b. Has its axis directed downwards and forwards from the rectal ampulla
True False
c. Has its anterior and posterior walls in apposition by the tone of the
levator ani muscles
True False
d. Is related posteriorly to the anococcygeal body
True False

3
Question 10: The ischiocavernosus muscle

a. Originates from the ischial spine


True False
b. Is inserted onto the corpus cavernosus
True False
c. Is supplied by the dorsal nerve of the clitoris
True False
d. Assists in erection of the clitoris
True False

Question 11: The sphincter urethrae

a. Is made of skeletal muscle


True False
b. Originates from the pubic arch
True False
c. Surrounds the membranous part of the urethra
True False
d. Is supplied by the perineal branch of the pudendal nerve
True False

Question 12: The superficial transverse perineal muscle

a. Lies in the anterior part of the superficial perineal pouch


True False
b. Originates from the ischial tuberosity
True False
c. Is inserted onto the perineal body
True False
d. Is supplied by the perineal branch of the pudendal nerve
True False

4
Question 13: The ischio-rectal fossa

a. Is bounded inferiorly by the ischial tuberosities


True False
b. Contains the pudendal nerve
True False
c. Contains the internal pudendal artery
True False
d. Is bounded laterally by the obturator internus muscle
True False

Question 14: The deep perineal pouch in the female

a. Contains part of the vagina


True False
b. Contains the deep transverse perineal muscle
True False
c. Contains the superficial transverse perineal muscle
True False
d. Contains the branches of the internal pudendal artery
True False

Question 15: The pudendal nerve

a. Enters the perineum lateral to the ischial spine


True False
b. Supplies the internal anal sphincter
True False
c. Gives off the inferior rectal nerve
True False
d. Gives off the dorsal nerve of the clitoris
True False

5
Question 1: With respect to the bladder and urodynamic studies

a. Cystometry is not always necessary to make a diagnosis of detrusor


instability
True False
b. Urinary leakage occurring without a rise in detrusor pressure is
consistent with genuine stress incontinence
True False
c. During bladder filling, the detrusor pressure rise should be < 15mmHg
True False
d. High urethral pressure profile occurs in detrusor instability
True False

Question 2: With respect to the bladder and urodynamic studies

a. High urethral pressure profile occurs in genuine stress incontinence


True False
b. A peak urinary flow rate of <15ml/s may occur if only a small volume of
urine is passed
True False
c. A peak urinary flow rate of <15ml/s may occur with a hypotonic
bladder
True False
d. A first sensation to void at 50ml is consistent with sensory urgency
True False

Question 3: With respect to the blood supply to the rectum

a. The superior rectal artery is a branch of the internal iliac artery


True False
b. The middle rectal artery is a branch of the internal iliac artery
True False
c. The inferior rectal artery is a branch of the superior vesical artery
True False
d. The lower third of the rectum is supplied by a branch of the internal
pudendal artery
True False

1
Question 4: With respect to the uterus

a. In an anteverted uterus, the long axis of the uterus is typically at


45degrees to the long axis of the vagina
True False
b. In an ante-flexed uterus, the long axis of the body of the uterus is bent
forward at the level of the external os
True False
c. The body of a retroverted uterus lies within the utero-vesical pouch
True False
d. The body of an anteverted uterus lies within the pouch of Douglas
True False

Question 5: The broad ligament

a. Has the ovary attached to its posterior surface by the mesovarium


True False
b. Is a two layered fold of peritoneum
True False
c. Has the ureter crossing the uterine artery at its base
True False
d. Contains the epoophron
True False

Question 6: With respect to the bladder and urodynamic studies

a. A first sensation to void at 200ml is consistent with detrusor instability


True False
b. Urinary leakage with a rise in detrusor pressure during bladder filling is
consistent with genuine stress incontinence
True False
c. A first sensation to void ar 500ml is consistent with overflow
incontinence
True False
d. Cystometry is required to measure peak urinary flow rate
True False

2
Question 7: The sigmoid colon

a. Is retro-peritoneal
True False
b. Is continuous with the rectum at the level of S3
True False
c. Is continuous with the descending colon in front of the right external
iliac artery
True False
d. Is related anteriorly to the uterus
True False

Question 8: The ovary

a. Lies on the obturator nerve within the ovarian fossa


True False
b. Is surrounded by a fibrous capsule, the tunica albuginae
True False
c. Is supplied by the ovarian artery, a branch of the abdominal aorta at
L1
True False
d. Lymphatic drainage of the ovary is to the internal iliac nodes
True False

Question 9: The uterus

a. Is completely covered by peritoneum


True False
b. Is related anteriorly to the bladder
True False
c. Is anterior to the pouch of Douglas
True False
d. Is related posteriorly to the sigmoid colon
True False

3
Question 10: With respect to the uterus

a. The fundus is that part of the uterus above the entrance of the fallopian
tubes
True False
b. The uterus is 8cm long x 2cm wide by 5cm thick
True False
c. In saggital section, the cavity of the uterus is triangular in shape
True False
d. In coronal section, the cavity of the uterus is represented by a cleft
True False

Question 11: The following structures provide support to the uterus

a. Perineal body
True False
b. Transverse cervical ligament
True False
c. Utero-sacral ligament
True False
d. Pubo-cervical ligament
True False

Question 12: The following normal structures can be palpated through


the vagina

a. The cervix
True False
b. The bladder
True False
c. The appendix
True False
d. The perineal body
True False

4
Question 13: The ovary

a. Is attached to the anterior surface of the broad ligament


True False
b. Is retro-peritoneal
True False
c. Is attached to the pelvic side wall by the round ligament of the ovary
True False
d. Is attached to the upper end of the lateral wall of the uterus by the
suspensory ligament of the ovary
True False

Question 14: The cervix

a. Communicates with the cavity of the uterus through the internal os


True False
b. Has a spindle-shaped cervical canal
True False
c. Communicates with the vagina through the external os
True False
d. Is divided into a supra-vaginal and a vaginal part
True False

Question 15: The broad ligament

a. Contains the paroophron


True False
b. Has the fallopian tube in its upper free border
True False
c. Has the ovarian artery in its lower attached border
True False
d. Has the round ligament forming a ridge on its posterior surface
True False

5
Question 1: The pulmonary trunk

a. Lies posterior to the trachea


True False
b. Lies superior to the aortic arch
True False
c. Conveys de-oxygenated blood from the heart
True False
d. Divides into right and left pulmonary veins
True False

Question 2: With respect to the conducting system of the heart

a. The atrio-ventricular bundle is the only muscular connection between


the myocardium of the atria and the ventricular myocardium
True False
b. Cardiac impulses are relayed from the atrio-ventricular node to the
ventricles by the atrio-ventricular bundle
True False
c. The sino-atrial node is supplied by the left and right coronary arteries
True False
d. The atrio-ventricular node is supplied by the left coronary artery
True False

Question 3: The superior vena cava

a. Is formed from the right brachiocephalic and left internal jugular veins
True False
b. Receives blood from the azygos vein
True False
c. Lies anterior to the aorta in the superior mediastinum
True False
d. Lies posterior to the trachea in the superior mediastinum
True False

1
Question 4: The trachea

a. Lies anterior to the thymus in the superior mediastinum


True False
b. Is posterior to the arch of the aorta
True False
c. Lies anterior to the origin of the left common carotid artery
True False
d. Is related anteriorly to the oesophagus
True False

Question 5: The right ventricle

a. Has a pulmonary out-flow tract (pulmonary valve) with two semilunar


cusps
True False
b. Contracts during ventricular diastole
True False
c. Communicates directly with the inferior vena cava
True False
d. Pumps oxygenated blood into the pulmonary artery
True False

Question 6: The left ventricle

a. Has trabeculae carneae


True False
b. Has a moderator band
True False
c. Has three papillary muscles
True False
d. Has a thicker wall than that of the right ventricle
True False

2
Question 7: The arch of the aorta has the following branches

a. The left common carotid artery


True False
b. The left subclavian artery
True False
c. The internal thoracic artery
True False
d. Oesophageal artery
True False

Question 8: The right vagus nerve

a. Enters the thorax medial to the brachiocephalic artery


True False
b. Lies medial to the trachea in the superior mediastinum
True False
c. Lies medial to the azygos vein in the superior mediastinum
True False
d. Descends anterior to the root of the right lung
True False

Question 9: The arch of the aorta has the following branches

a. The right brachiocephalic artery


True False
b. The left brachiocephalic artery
True False
c. The right subclavian artery
True False
d. The right common carotid artery
True False

3
Question 10: Within the mediastinum

a. The heart is anterior to the oesophagus


True False
b. The thymus is anterior to the heart
True False
c. The thoracic duct lies anterior to the heart
True False
d. The oesophagus lies posterior to the descending aorta
True False

Question 11: The left lung

a. Receives de-oxygenated blood from the left pulmonary artery


True False
b. Has Lymphatic drainage to the para-aortic nodes
True False
c. Receives parasympathetic supply from the vagus nerve
True False
d. Sympathetic stimulation results in bronchodilation and vasoconstriction
True False

Question 12: The right vagus nerve

a. Enters the abdomen on the anterior surface of the oesophagus


True False
b. Contributes to the right pulmonary plexus
True False
c. Gives off the right recurrent laryngeal nerve within the thorax
True False
d. Stimulation results in vasodilation of the bronchial vessels
True False

Question 13: The right ventricle

4
a. Communicates with the pulmonary vein
True False
b. Has papillary muscles
True False
c. Is traversed by the moderator band
True False
d. Has papillary muscles which are connected to the cusps of the
tricuapid valve by the chordae tendinae
True False

Question 14: The left atrium

a. Communicates with the left ventricle via the tricuspid valve


True False
b. Receives blood from two pulmonary arteries
True False
c. Forms the base of the heart
True False
d. Has an auricle
True False

Question 15: The right atrium

a. Has a main cavity and an auricle


True False
b. Opens into the right ventricle via the mitral valve
True False
c. Has the fossa ovalis located on the atrial septum
True False
d. The fossa ovalis is the remnant of the septum secundum
True False

5
Question 1: The diaphragm

a. Has a left crus originating from the lateral arcuate ligament


True False
b. Has the median arcuate ligament between the medial and the lateral
arcuate ligaments
True False
c. Contracts during expiration
True False
d. Contracts during micturiction and defecation
True False

Question 2: The following statements about the intercostal arteries are


true

a. The anterior intercostal arteries of the lower 5 spaces are branches if


the superior epigastric artery
True False
b. The intercostal arteries supply the parietal pleura
True False
c. The intercostal arteries supply the visceral pleura
True False
d. The intercostal arteries supply the intercostal muscles and skin of the
thoracic wall
True False

Question 3: A needle inserted in the mid-axillary line over the 8th


intercostal space into the pleural space would traverse the following
structures

a. Serratus anterior muscle


True False
b. External intercostal muscle
True False
c. External oblique muscle
True False
d. Internal intercostal muscle
True False

1
Question 4: The internal thoracic artery

a. Supplies the pericardium


True False
b. Supplies the thymus gland
True False
c. Gives off one anterior intercostal artery to the upper 6 intercostal
spaces
True False
d. Gives off perforating branches
True False

Question 5: The first rib

a. Articulates with the first thoracic vertebral body


True False
b. Articulates with the body of the 7th cervical vertebra
True False
c. Is crossed by the subclavian vein anterior to the scalene tubercle
True False
d. Is crossed by the subclavian artery posterior to the scalene tubercle
True False

Question 6: The diaphragm

a. Causes a reduction in venous return during contraction


True False
b. Has the right and left vagus nerves entering the abdomen through the
oesophageal opening
True False
c. Has a caval opening at the level of the 8th thoracic vertebra
True False
d. Has the peritoneum on its inferior surface supplied by the phrenic
nerve
True False

2
Question 7: The diaphragm

a. Is covered by visceral pleura on its superior surface


True False
b. Receives motor nerve supply from the phrenic nerve
True False
c. Receives sensory supply from the phrenic nerve
True False
d. Receives sensory supply from the lower 5 intercostal nerves
True False

Question 8: With respect to the intercostals muscles

a. The fibres of the external intercostal muscles are directed downwards


and forwards
True False
b. The external intercostal muscles extend from the tubercle of the ribs to
the sternum
True False
c. The internal intercostal muscles extend from the sternum to the
tubercle of the rib posteriorly
True False
d. The transversus thoracis muscle extends over more than one
intercostal space
True False

Question 9: A needle inserted in the mid-axillary line over the 8th


intercostal space into the pleural space would traverse the following
structures

a. Transversus thoracis muscle


True False
b. The costal origin of the diaphragm
True False
c. Visceral pleura
True False
d. The posterior intercostal membrane
True False

Question 10: With respect to the lymphatic drainage of the thoracic wall

3
a. Lymphatic drainage of the skin of the anterior chest wall is to the
anterior axillary nodes
True False
b. Lymphatic drainage of the skin of the posterior chest wall is to the
posterior axillary nodes
True False
c. Lymphatic drainage of the anterior part of the intercostal spaces is to
the internal thoracic nodes
True False
d. Lymphatic drainage of the posterior part of the intercostal spaces is to
the para-aortic nodes
True False

Question 11: The first rib

a. Has the scalenus anterior muscle inserted onto the scalene tubercle
True False
b. Is crossed by the brachial plexus anterior to the scalenus anterior
muscle
True False
c. Articulates with the transverse process of the 7th cervical vertebra
True False
d. Is palpable throughout its length
True False

Question 12: With respect to the lymphatic drainage of the breast

a. Lymphatic vessels cross the mid-line


True False
b. Lymphatic drainage of the lateral part of the gland is to the anterior
axillary nodes
True False
c. Lymphatic drainage of the superior part of the gland is to the supra-
clavicular nodes
True False
d. Lymphatic drainage of the medial part of the gland is to the internal
thoracic nodes
True False

Question 13: The diaphragm

4
a. Has a central muscular part and a peripheral tendinous part
True False
b. Has a sternal origin from the posterior surface of the body of the
sternum
True False
c. Has a costal origin from the deep surfaces of the lower 4 ribs and their
costal cartilages
True False
d. Has a right crus originating from the medial arcuate ligament
True False

Question 14: The intercostal nerves

a. Are the posterior rami of the first 11 thoracic spinal nerves


True False
b. Enter the intercostal space between the internal intercostal muscle and
the transversus thoracis
True False
c. Run superior to the intercostal vein
True False
d. Give off grey rami communicantes to the sympathetic trunk
True False

Question 15: With respect to the surface anatomy of the thorax

a. The xiphisternal joint lies opposite the body of the 9th thoracic vertebra
True False
b. The lowest part of the costal margin is formed by the 12th rib
True False
c. The lowest part of the costal margin lies at the level of the 3rd lumbar
vertebra
True False
d. The first rib is palpable throughout its length
True False

5
MAGNETIC RESONANCE IMAGING

• Uses radiowaves and magnetic fields

• Non-ionising

• The patient is placed in a magnet - the strength of the magnetic field is measured in tesla (T)
and is set at 0.15 - 2 T. This is 10,000 - 20,000 x the earth’s magnetic field. Inside the magnet,
the patient becomes slightly magnetised, generating a combined or net longitudinal
magnetism (M)

• A radio frequency lasting 1ms or less is then sent into the patient by a transmitter - this
produces a rapidly alternating magnetic field which excites protons within the tissues, causing
resonance

• Only mobile protons produce a magnetic resonance signal - those immobilised in large
molecules or in bone do not. The signal is mainly due to water, free or bound. Air has no
hydrogen and produces no signal - appears black. Fat has a higher proton density than other
soft tissue

• The magnetic resonance signal is dependent on the proton density of the tissue and is
greatest immediately after the radiofrequency signal is switched off and decays thereafter as a
result of two independent methods of energy loss or -relaxation - Spin - lattice or T1
relaxation & spin-spin or T2 relaxation

• T1 is the time it takes for the net longitudinal magnetism to recover to 63% of its maximum
value. T1 relaxation is caused by thermodynamic jostling of the excited protons by nearby
molecules - heavy molecules are more effective than light molecules at removing energy from
excited protons

• Thus water bound to the surface of proteins or protons in fat have a shorter T1 while free
water, urine, amniotic fluid, CSF and salt solutions have a long T1. The greater the proportion
of free water in a tissue, the longer is T1

• Atoms in solids and rigid macromolecules are least effective at removing energy and have a
very long T1
• Spin-spin or T2 relaxation is caused by the excited protons (initially in phase) becoming
progressively de-phased as some rotate faster / slower than others

• T2 is the time for the magnetic resonance signal to fall to 37% of its maximum value. De-
phasing occurs because each spinning proton produces a magnetic field which affects
neighbouring protons

• The local variation in magnetic fields is greatest in solid and rigid macromolecules. Compact
bone, teeth, tendons, calculi have a very short T2 and do not produce a lasting signal

• Free water, amniotic fluid, CSF have lighter molecules with rapid thermal motion which
smooths out magnetic fields, producing a longer T2

• Water bound to protein or other large molecules and protons in fat have a shorter T2

• T2 is always shorter than T1

• T1 of tissues increases with magnetic field strength whereas T2 is more-or-less unaffected

• The MRI image maps three properties - the proton density (determines the brightness of the
pixel), T1 and T2

• In T1 weighted images, the shorter the T1 of the tissue, the stronger the signal and the
brighter the pixel - fat is bright as is fatty bone marrow while water and CSF are dark

• In T2 weighted images, the the longer the T2 of the tissue, the stronger the signal and the
brighter the pixel - water and CSF appear brighter than fat

• Generally, tissues with long T1 also have long T2 and those with short T1 have short T2.
Images cannot be weighted for both T1 and T2

• To image small areas, radiofrequency coils (transmitters / receivers) may be placed as close
as possible to the part being imaged. These are similar to head coils used to image the brain
SAFETY

• Magnetic field should not exceed 2.5T in patients, 0.2T in the arms and hands of staff and
0.02T within the body of staff. Strong magnetic fields may result in a sensation of taste and
flashes of light on the retina

• The magnetic field should not be built up too quickly - usually 1-5T per second and no faster
than 20T per second

• Not thought to be any adverse effects on fetal development but MRI is generally not
performed in the first trimester and pregnant staff may be re-deployed

• Switching of magnetic fields may induce eddy currents in nerve fibres causing involuntary
movements, breathing difficulties and ventricular fibrillation. Contra-indicated in patients with
an implanted pacemaker.

• Radiofrequency fields may result in microwave heating - the cornea and testes are particularly
at risk and heating of metallic implants may occur

• Rectal and skin temperature rise should not exceed 1C

• Radiofrequencies may affect pacemakers, computers and other equipment relying on electron
flow in a vacuum

• Ferromagnetic objects may be converted into projectiles - scissors / scalpels in particular.


Aneurism clips may be displaced and non-magnetic material should always be used. The
fringe magnetic field may also affect some watshes, destroy data on computer discs and credit
cards
RADIOACTIVITY

• Unstable nuclei have a neutron excess or deficit, are radioactive and decay spontaneously
until they become stable

• Nuclides with a neutron excess decay by releasing a high energy electron (negative beta
particle) while those with a neutron deficit decay by emiting a high energy positive electron
(positive beta particle)

• The product nucleus is usually left with excess energy which is lost by the emission of one or
more gamma photons

• Gamma rays emitted by a particular radionuclide have specific energies whereas beta rays
are emitted with a continuous spectrum of energies up to a maximum which is characteristic
for the radionuclide

• Beta rays are directly ionising and have a range of about 2mm in tissue

• The quantity of radioactive material is measured in the number of disintergrations per second -
the SI unit is the Becquerel (1Bq = 1 disintergration per second). 1 curie (1Ci ) = 3.7 x 10E10
disintergrations per second

• The half-life of a radionuclide is the time taken for its activity to decay to half its original value.
It is not affected by temperature, pressure, electricity or chemical reactions

STABLE ISOTOPES

• Atoms with same number of protons (and electrons) but different number of neutrons with a
stable conformation - these atoms do not decay and do not generate radioactivity

• Examples include Deuterium (heavy water), 13C, 15N, 18O, 34S, 37Cl, 41K

RADIATION HAZARDS

• Effects on tissues include


a) Ionisation - causes the majority of immediate chemical changes in tissues

b) Free radical generation - particularly the hydroxyl radical

c) Molecular changes in DNA, RNA and enzymes

d) Sub-cellular damage to cell membranes, nuclei and chromosomes

e) Cell death

f) Cell transformation - cancer induction has a latent period of a few years for leukaemia and up to 40
years for solid cancers. Children are more radio-sensitive than adults

• Beta particles / electrons have a short range in tissues and are easily absorbed by several mm
of Perspex or thin sheets of metal

• X-rays and gamma rays do not have a maximum depth of penetration in tissues but simply
undergo continuous attenuation - a residual beam always remains

• The biological damaging effects of heavy particles such as neutrons and alpha particles is 10-
20 times greater than that of electrons as their energy is completely deposited in a shorter
range of tissue

• The EQUIVALENT DOSE of radiation is the absorbed dose multiplied by a radiation weighting
factor which is a measure of the effectiveness of the radiation in inducing cancers compared to
electrons - measured in Joules /Kg (SI unit = seivert, Sv)

• The EFFECTIVE DOSE is the sum of the weighted equivalent doses

• There are no documented risks to the fetus during the first three weeks of gestation (based on
LMP) - that is, before the first missed period. However, high dose procedures such as barium
enemas and abdominal / pelvic CT scans may pose a risk

EFFECTIVE DOSE FOR DIFFERENT PROCEDURES


DOSE EXAMPLE

EXPOSURE

5 - 50mSv Barium enema, gallium scan, CT scan

HIGH
0.5 - 5mSv IVU, Barium meal, X-ray lumbar / thoracic spine,
abdomen, pelvis, 99mTc scans of brain, bone, kidney,
Medium liver; lung perfusion scan, thyroid

scan

0.05 - 0.5mSv Chest X-ray, dental and skull X-ray

Low
ELECTROMAGNETIC RADIATION

• Name given to energy travelling in empty space - all have same velocity = speed of light.
Made up of packets / quanta of energy

• Include radio waves (lowest frequency), infrared waves, visible light, ultraviolet light and X &
gamma rays (highest frequency)

• Photon energy is proportional to frequency and is measured in electron Volts (eV).

• The intensity of the radiation is inversely proportional to the square of the distance from a
point source - Inverse square law

• IONISATION - of air and other gases (makes these rays electrically conducting) and of atoms
in cells - produces adverse biological effects

Other ionising radiation includes some ultraviolet radiation, beta particles and alpha rays

X-RAYS

• Produced when fast-moving electrons impact on a metal target - 99% of the energy of the
electrons is converted into heat and 1% into X-rays

• The absorbed dose is the photon energy deposited per unit mass of a stated material and is
measured in Joules per Kg - the SI unit is the gray (Gy). 1Gy = 1J/Kg

• Dose rate is measured in grays / sec

• 1Gy = 100 rad (the rad was the old unit for absorbed dose)

• Particulate forms of radiation such as alpha particles cause more ionisation - a multiplier Q is
used to to express the relative effectiveness of ionisation - Q = 1 for X, Gamma and Beta
radiation, Q = 10 for alpha particles. The product Q. Gy is the sievert (Sv).
• When a beam of X-rays or gamma rays (similar to X-rays but originating from the nucleus)
encounters an object, they can be:

1) Transmitted - pass through unaffected

2) Scattered - diverted in a new direction

3) Absorbed - transferring part or all of their energy into the object

• Attenuation refers to the reduction in photon energy as the beam passes through an obstacle

• The thickness of a material that will reduce the photon energy to half its initial value is called
the half-value layer (HVL) and is a measure of the penetrating power of X-rays or gamma rays

GAMMA IMAGING

• Patient is given an appropriate radionuclide usually intravenously

• Metastable Technetium-99 is most commonly used - emits 140keV gamma rays

• Gamma camera is used to image the distribution of the radionuclide in the patient - the
camera has heavy lead shielding to attenuate unwanted background radiation coming from the
patient or the environment

• Unlike X-rays, the radiation dose delivered to the patient is not dependent on the number of
images taken and is not confined to the region of diagnostic interest

• The ideal radionuclide for imaging should have the following properties:

1) A physical half life of a few hours

2) Decay to a stable isotope or one with a very long half life

3) Emit gamma rays of energy 50-300keV - high enough to exit the patient but low enough to be easily
measured. The rays should be mono-energetic

4) Should not emit alpha or beta particles or low energy photons

5) Have a high specific activity

6) Be readily and cheaply available on the hospital site

7) Localise quickly to the tissue of interest


8) Be eliminated from the body with an effective half life similar to the duration of the examination -
reduces subsequent dose to the patient

CT SCANNING

• Images a slice of the patient - avoids superposition of adjacent structures that occurs with
conventional X-ray

• Uses X-rays

• Contrast resolution (ability to detect small differences in the attenuation coefficient of adjacent
structures) is better than with conventional X-rays but spatial resolution is poorer.

• The CT number is calculated from the linear attenuation coefficient of each tissue Ut and that
of water Uw as: CT number = 1000(Ut - Uw)/Uw

• The CT number is also known as the Hounsfield number and is -1000 for air and zero for
water

• The radiation dose to the patient from CT is much greater than from conventional X-rays - CT
scans account for 2% of X-ray examinations but 20% of the radiation dose delivered to
patients in the UK.
Question 1: During magnetic resonance imaging
Ionising radiation is not used
HTMLCONTROL Forms.HTML:Option.1 True
HTMLCONTROL Forms.HTML:Option.1
False Radiofrequency waves are used
HTMLCONTROL Forms.HTML:Option.1 True
HTMLCONTROL Forms.HTML:Option.1 False The patient is
placed in an electromagnetic field
HTMLCONTROL Forms.HTML:Option.1 True
HTMLCONTROL Forms.HTML:Option.1 False The brightness
of a pixel is dependent on the proton density of the tissue
HTMLCONTROL Forms.HTML:Option.1 True
HTMLCONTROL Forms.HTML:Option.1 False
Question 2: With respect to ultrasound imaging
The resolution along the direction of the beam is finer than that
across the width of the beam
HTMLCONTROL Forms.HTML:Option.1 True
HTMLCONTROL Forms.HTML:Option.1 False The scanning
jelly is only used so that the probe will slide along the patient?s
skin
HTMLCONTROL Forms.HTML:Option.1 True
HTMLCONTROL Forms.HTML:Option.1 False Bowel gas
helps transmission of sound waves and enables deeper tissues
to be visualised
HTMLCONTROL Forms.HTML:Option.1 True
HTMLCONTROL Forms.HTML:Option.1 False Using a
transducer of higher frequency will result in better detail but
only at a shallower depth
HTMLCONTROL Forms.HTML:Option.1 True
HTMLCONTROL Forms.HTML:Option.1 False
Question 3: Ultrasound
Cannot be focused
HTMLCONTROL Forms.HTML:Option.1 True
HTMLCONTROL Forms.HTML:Option.1 False Can travel
through a vacuum
HTMLCONTROL Forms.HTML:Option.1 True
HTMLCONTROL Forms.HTML:Option.1 False Can be
converted into electrical impulses by a transducer
HTMLCONTROL Forms.HTML:Option.1 True
HTMLCONTROL Forms.HTML:Option.1 False Is
electromagnetic
HTMLCONTROL Forms.HTML:Option.1 True
HTMLCONTROL Forms.HTML:Option.1 False
Question 4: With respect to ultrasound imaging
Penetration increases with increasing ultrasound frequency
HTMLCONTROL Forms.HTML:Option.1 True
HTMLCONTROL Forms.HTML:Option.1 False Axial
resolution increases with increasing ultrasound frequency
HTMLCONTROL Forms.HTML:Option.1 True
HTMLCONTROL Forms.HTML:Option.1 False Lateral
resolution is dependent on the diameter of the transducer
HTMLCONTROL Forms.HTML:Option.1 True
HTMLCONTROL Forms.HTML:Option.1 False Lateral
resolution is decreased by focusing the ultrasound beam
HTMLCONTROL Forms.HTML:Option.1 True
HTMLCONTROL Forms.HTML:Option.1 False
Question 5: During magnetic resonance imaging
Air produces excellent magnetic resonance signals
HTMLCONTROL Forms.HTML:Option.1 True
HTMLCONTROL Forms.HTML:Option.1 False Fat has a
lower proton density than skeletal muscle
HTMLCONTROL Forms.HTML:Option.1 True
HTMLCONTROL Forms.HTML:Option.1 False The magnetic
resonance signal is lowest immediately after the radiofrequency
signal is switched off
HTMLCONTROL Forms.HTML:Option.1 True
HTMLCONTROL Forms.HTML:Option.1 False T1 relaxation
is caused by thermodynamic jostling of excited protons by
surrounding molecules
HTMLCONTROL Forms.HTML:Option.1 True
HTMLCONTROL Forms.HTML:Option.1 False
Question 6: Ultrasound
Is a high frequency magnetic field above the audible range
HTMLCONTROL Forms.HTML:Option.1 True
HTMLCONTROL Forms.HTML:Option.1 False Medical
ultrasound utilises frequencies in the 1-20MHz range
HTMLCONTROL Forms.HTML:Option.1 True
HTMLCONTROL Forms.HTML:Option.1 False Medical
ultrasound utilises frequencies of ~3MHz
HTMLCONTROL Forms.HTML:Option.1 True
HTMLCONTROL Forms.HTML:Option.1 False Piezoelectric
crystals are used in ultrasound scanning probes to generate a
focused beam of ultrasound
HTMLCONTROL Forms.HTML:Option.1 True
HTMLCONTROL Forms.HTML:Option.1 False
Question 7: With respect to ultrasound imaging
Ultrasound exposure in-utero is associated with an increased
risk of dyslexia
HTMLCONTROL Forms.HTML:Option.1 True
HTMLCONTROL Forms.HTML:Option.1 False There has
been a rise in the occurrence of birth defects with increasing
use of ultrasound
HTMLCONTROL Forms.HTML:Option.1 True
HTMLCONTROL Forms.HTML:Option.1 False The process of
microbubble formulation in the presence of an ultrasonic field is
known as cavitation
HTMLCONTROL Forms.HTML:Option.1 True
HTMLCONTROL Forms.HTML:Option.1 False Cavitation has
been demonstrated in human systems during medical imaging
HTMLCONTROL Forms.HTML:Option.1 True
HTMLCONTROL Forms.HTML:Option.1 False
Question 8: With respect to the use of radiation for medical
imaging
The effective dose from a chest X-ray is higher than from a
chest CT scan
HTMLCONTROL Forms.HTML:Option.1 True
HTMLCONTROL Forms.HTML:Option.1 False The effective
dose from a barium meal is lower than from a barium enema
HTMLCONTROL Forms.HTML:Option.1 True
HTMLCONTROL Forms.HTML:Option.1 False The effective
dose from a pelvic X-ray is lower than from a chest X-ray
HTMLCONTROL Forms.HTML:Option.1 True
HTMLCONTROL Forms.HTML:Option.1 False The effective
dose from a lung perfusion scan is lower than for a chest X-ray
HTMLCONTROL Forms.HTML:Option.1 True
HTMLCONTROL Forms.HTML:Option.1 False
Question 9: With respect to SI units
The energy of X-rays and Gamma reya is measured in electron
volts (eV) (T)
HTMLCONTROL Forms.HTML:Option.1 True
HTMLCONTROL Forms.HTML:Option.1 False The amount of
a radioactive substance is measured in Grays (Gy) (F)
HTMLCONTROL Forms.HTML:Option.1 True
HTMLCONTROL Forms.HTML:Option.1 False The absorbed
dose of radiation from x-rays and nuclear medicine
examinations is measured in grays (Gy) (T)
HTMLCONTROL Forms.HTML:Option.1 True
HTMLCONTROL Forms.HTML:Option.1 False Radiation
doses to individuals can be expressed in Sieverts (Sv) to
indicate the degree of risk (T)
HTMLCONTROL Forms.HTML:Option.1 True
HTMLCONTROL Forms.HTML:Option.1 False
Question 10: The following are examples of stable isotopes
35S
HTMLCONTROL Forms.HTML:Option.1 True
HTMLCONTROL Forms.HTML:Option.1 False 32P
HTMLCONTROL Forms.HTML:Option.1 True
HTMLCONTROL Forms.HTML:Option.1 False 2H (D-2)
HTMLCONTROL Forms.HTML:Option.1 True
HTMLCONTROL Forms.HTML:Option.1 False 125I
HTMLCONTROL Forms.HTML:Option.1 True
HTMLCONTROL Forms.HTML:Option.1 False
Question 11: During magnetic resonance imaging
The patient is placed in a magnetic field which is about 2x the
earth?s magnetic field
HTMLCONTROL Forms.HTML:Option.1 True
HTMLCONTROL Forms.HTML:Option.1 False The strength
of the magnetic field should not exceed 0.1T
HTMLCONTROL Forms.HTML:Option.1 True
HTMLCONTROL Forms.HTML:Option.1 False Patients with
ferromagnetic aneurysm clips can safely be imaged
HTMLCONTROL Forms.HTML:Option.1 True
HTMLCONTROL Forms.HTML:Option.1 False The cornea
may be damaged by microwave heating
HTMLCONTROL Forms.HTML:Option.1 True
HTMLCONTROL Forms.HTML:Option.1 False
Question 12: Accoustic attenuation
Is measured in megahertz (MHz)
HTMLCONTROL Forms.HTML:Option.1 True
HTMLCONTROL Forms.HTML:Option.1 False Increases with
increasing ultrasound frequency
HTMLCONTROL Forms.HTML:Option.1 True
HTMLCONTROL Forms.HTML:Option.1 False Is high in
water and blood
HTMLCONTROL Forms.HTML:Option.1 True
HTMLCONTROL Forms.HTML:Option.1 False Is low in air
HTMLCONTROL Forms.HTML:Option.1 True
HTMLCONTROL Forms.HTML:Option.1 False
Question 13: During magnetic resonance imaging
Free water, amniotic fluid, and CSF have a longer T2 compared
to fat
HTMLCONTROL Forms.HTML:Option.1 True
HTMLCONTROL Forms.HTML:Option.1 False Tissues with a
long T1 have a short T2
HTMLCONTROL Forms.HTML:Option.1 True
HTMLCONTROL Forms.HTML:Option.1 False Tissues with a
short T2 have a short T1
HTMLCONTROL Forms.HTML:Option.1 True
HTMLCONTROL Forms.HTML:Option.1 False Images can be
weighted for both T1 and T2
HTMLCONTROL Forms.HTML:Option.1 True
HTMLCONTROL Forms.HTML:Option.1 False
Question 14: In Doppler ultrasound imaging
The frequency shift is within the audible range
HTMLCONTROL Forms.HTML:Option.1 True
HTMLCONTROL Forms.HTML:Option.1 False A continuous
wave Doppler probe contains two transducers
HTMLCONTROL Forms.HTML:Option.1 True
HTMLCONTROL Forms.HTML:Option.1 False During pulsed
Doppler imaging, only a small fraction of time is spent in the
Doppler mode
HTMLCONTROL Forms.HTML:Option.1 True
HTMLCONTROL Forms.HTML:Option.1 False Continuous
wave Doppler can distinguish between flow in two overlapping
vessels at different depths
HTMLCONTROL Forms.HTML:Option.1 True
HTMLCONTROL Forms.HTML:Option.1 False
Question 15: With respect to ultrasound imaging
In colour Doppler imaging, the frequency shift of the echo is
coloured to indicate the direction of blood flow towards or away
from the probe
HTMLCONTROL Forms.HTML:Option.1 True
HTMLCONTROL Forms.HTML:Option.1 False At the power
levels used for medical imaging, no harmful effects to human
tissue have been demonstrated
HTMLCONTROL Forms.HTML:Option.1 True
HTMLCONTROL Forms.HTML:Option.1 False The power
output of ultrasound equipment is measured in megahertz
(MHz)
HTMLCONTROL Forms.HTML:Option.1 True
HTMLCONTROL Forms.HTML:Option.1 False The power
output into the patient should be less than 100megaWatts per
square cm
HTMLCONTROL Forms.HTML:Option.1 True
HTMLCONTROL Forms.HTML:Option.1 False
DNA REPLICATION *****

 Occurs during the S phase of the cell cycle resulting in


doubling of genetic material *
 During the G2 phase, the cell is tetraploid. The G2 phase
is followed by the M (mitosis) phase when the genetic
material is halved and diploid daughter cells are formed.
The G1 phase follows the M phase and some cells stop
dividing and enter a G0 phase *
 Both DNA strands are copied - replication is semi-
conservative and bi-directional. *
 DNA helix is unwound by DNA helicase at 20-80 sites
forming replicons or replication units *
 There are 5 DNA polymerase enzymes which are
responsible for replication:
 Polymerase - alpha synthesizes the lagging strand
via Okazaki fragments
 Polymerase beta and epsilon repair DNA
 Polymerase delta synthesizes the leading strand
 Polymerase gamma replicates mitochondrial DNA
 Telomerase is a DNA polymerase with an integral RNA
that acts as its own primer and replicates DNA at the
ends of chromosomes (telomeres)
 Replication is initiated by the synthesis of a primer
sequence of RNA. Chain elongation occurs in the 5' to 3'
direction on the leading strand and in Okazaki fragments
on the lagging strand (discontinuous). The fragments are
joined by DNA ligase
 Histones do not dissociate from DNA prior to replication.
Histones are also synthesized during the S phase and the
old nucleosomes stay with the daughter DNA molecule
containing the leading strand while new nucleosomes
assemble on the daughter molecule containing the
lagging strand.
 Rapidly dividing (malignant) cells are most sensitive to
anti-metabolite chemotherapy during the S phase of the
cell cycle *
DNA SYNTHESIS FROM RNA *****

• Reverse transcription

• Catalysed by the enzyme reverse transcriptase - found in


retroviruses but not in DNA viruses and not present in
eukaryotic cells *

• Requires the presence of an oligo (d)T primer


(complementary to the poly A tail on mRNA and
deoxyribonucleotides

POLYMERASE CHAIN REACTION - PCR *****

• Enables the rapid amplification of DNA sequences in small


biological samples *

• Used for pre-natal diagnosis and forensic medicine *

• Based on the use of DNA polymerases which function at


high temperatures *

• Require the presence of DNA primers which flank the DNA


sequence to be amplified and deoxyribonucleotides

• Begins with the denaturation of DNA by heating to 90C *

• Primers anneal to the sequences to be amplified at 50C

• Extension of the primer sequence occurs at 70C

• The sequence is repeated usually over 30-35 cycles

• The DNA is separated by electrophoresis and the amplified


sequences are visualised *

• Based on the primers used, the size of the PCR product can
be predicted and this is used in identification

• Further identification of PCR products requires sequencing


to confirm their identity or use of restriction endonucleases
which should cut the sequence at a predicted point, with
products of predictable sizes *

RESTRICTION ENDONUCLEASES *****

• Enzymes which recognise specific DNA sequences and cut


DNA at specific sites *
• Recognise palindromic sequences - sequences that are
identical when read from left to right and from right to left -
AATTAA

• Cut DNA leaving 'blunt' ends - no bases sticking out, or


'sticky' ends with bases sticking out, allowing complementary
pairing with another chain

• May be used for the identification of PCR products as they


cut at predictable sites, and should produce fragments of
predictable sizes if the identity of the PCR product is correct *

DETECTION OF MACROMOLECULES *****

• Western blotting - protein

• Northern blotting - RNA

• Southern blotting - DNA

VECTORS

• DNA species which can enter cells readily, remain stable and
replicate within these cells

• Include plasmids and bacteriophages

• Plasmids are small circular DNA sequences which occur


naturally in bacteria in which they replicate independent of
chromosomal DNA

• Plasmid entry into bacteria is facilitated by exposure of


bacteria to an electric field (electroporation) or treatment with
CaCl2 and exposure to 40C heat. Plasmid infected cells are
transformed and replicate forming a clone

• Plasmids confer different phenotypic characteristics to


bacteria, including antibiotic resistance

• Viruses that infect bacteria are Bacteriophages


NUCLEIC ACID STRUCTURE *****

NUCLEOTIDES

• Base - purine (guanine -G and adenine -A) or pyrimidine (thymine -


T, cytosine -C and uracil -U) *
• Pentose (5-carbon) sugar - ribose (RNA) or deoxyribose (DNA)
Phosphate *
• Uracil is found only in RNA and replaces thymine *
• Nucleotides are linked by 3-5-phosphodiester bonds to form nucleic
acids. The 3-end of the chain has a free OH group while the 5- end has a
phosphate group. During synthesis, nucleotides are added to the 3- end
• RNA is single stranded while DNA has a double helix structure with
purine pairing with pyrimidine bases (A with T, G with
C).* Complementary nucleotides are held together by non-covalent
forces -mainly hydrogen bonds and the two chains can be separated by
heating to 90C. This process is reversible and the two strands re-anneal
on cooling *
• The double helix is right-handed with 10 bases per turn and a
complete turn every 3.4nm
• The DNA molecule can be cut using restriction enzymes. These
enzymes recognise specific DNA sequences and therefore cut at specific
locations producing fragments of specific lengths *

CHROMOSOME STRUCTURE

• The DNA in chromosomes is associated with several proteins, of


which histones are the most abundant

• HISTONES are highly basic, positively charged arginine / lysine rich


proteins which interact with the negatively charged phosphate groups on
DNA *

• There are 5 types of histone protein: H1, H2A, H2B, H3& H4

• DNA is wound around a histone octamer made up of two molecules


of H2A, H2B,H3 & H4 to form beads called nucleosomes

• Nucleosomes are joined by threads of double stranded (linker) DNA


which is bound to histone H1 giving a -beads on a string- appearance

• The ratio of DNA to histones in chromosomes on a weight basis is ~


1:1

• Nucleosomes are organised into 30nm fibres which are then attached
to a central protein scaffolding a series of radial loops.
MESSANGER RNA *****

• Single stranded and monocistronic (one mRNA molecule


only codes for one protein). Synthesized from DNA in the
process of transcription by RNA polymerase. Mature mRNA
does not contain introns and exons - introns are removed
during splicing before transport into the cytoplasm *

• The size of RNA molecules is quoted in Svedberg units (S)

• Sequence is written in 5- to 3- direction. Has a 5- methylated


cap followed by a leader sequence then the start codon (AUG)
followed by the translated region and the 3- poly-A tail *

• Relatively unstable and is broken down rapidly after


transcription

• Uracil replaces Thymine in RNA and the sugar unit is


RIBOSE *

• Detected by Northern blotting or by PCR *

RIBOSOMAL RNA *****

• Makes up 80% of total cellular RNA *

• Relatively stable due to its association with proteins

• Essential for protein synthesis *

• Has secondary structure with base-pairing within the


molecule allowing the formation of helical regions and hairpin
loops

• Contains several modified nucleotides, especially 2-


methylation on ribose

• RIBOSOMES: Eukaryotic ribosomes are 80S and dissociate


into 40S and 60S subunits

• The 40S subunit has an 18S rRNA molecule and about 34


different proteins *

• The 60S subunit has 5S, 5.8S and 28S rRNA molecules and
about 50 different proteins *

TRANSFER RNA *****

• Makes up 15% of total RNA


• Binds specific amino acids and carries them to the
polyribosomes for protein synthesis

• There is a specific tRNA for each amino acid but some amino
acids are recognised by more than one specific tRNA and
therefore have more than one codon *

• Has a clover-leaf secondary structure with an amino acid


binding site at the 3- end and an anticodon site on one of the
clover leaves. The anticodon recognises the triplet codon on
mRNA *

TRANSCRIPTION *****

• Process by which the genetic information in DNA is


converted to RNA. Occurs in the nucleus *

• Catalysed by RNA polymerases *

• RNA polymerase I - synthesizes rRNA

• RNA polymerase II - synthesizes mRNA

• RNA polymerase III - synthesizes rRNA and tRNA

• Only the anti-sense DNA strand is transcribed. The mRNA


molecule is synthesized in the 5- to 3- direction

• RNA polymerase does not have a nuclease activity and


cannot remove any incorrect bases. A primer is not required for
RNA synthesis *

• INITIATION of transcription occurs when RNA polymerase


recognises promoter regions up-stream of the start site.
Enhancer sequences also play a role in the initiation of
transcription and may confer specificity to tissue or organ
responses to specific stimuli, such as hormones

• Transcription factors are proteins that are required for RNA


polymerase to recognise promoter sites

• ELONGATION of the RNA chain occurs in the 5- to 3-


direction. There is no specific stop codon and elongation stops
at varying distances downstream of the gene, releasing pre-
mRNA

• RNA processing then occurs to produce mature mRNA ready


for translation *
1) Addition of 5- methylated cap - protects the 5- end against
ribonuclease degradation and plays a role in the initiation of protein
synthesis

2) Addition of 3- poly-A tail -cleavage and addition of 200-250 A


residues - protects against nuclease degradation and increases
translational efficiency

3) SPLICING to remove introns: the coding regions of DNA (EXONS)


are separated by non-coding regions (INTRONS). After transcription,
the introns are excised in SPLICEOSOMES. All introns have a
sequence which begins with 5-GU and ends with AG-3-. The introns
are excised and the exons joined. *

4) Alternative splicing occurs when one or more exons are excised


together with the introns in certain tissues. Non-coding regions are
never present in mature mRNA

5) RNA editing - the mRNA sequence may be changed after synthesis


and processing and nucleotides may be substituted, added or deleted
Biochemistry :. Nucleotides & nucleic acids :. Module 1: 60 Random questions for MCQ Test
Question 1: During DNA replication

a. Replication is initiated by the synthesis of an RNA primer

True False
b. The complementary DNA chain is synthesized in the 3 to 5 direction

True False
c. Replication of the leading DNA strand is discontinuous

True False
d. Replication of the lagging DNA strand occurs in Okazaki fragments

True False

Question 2: With respect to transcription

a. The primary mRNA transcript is produced only from the exons on the DNA molecule

True False
b. Introns have a sequence that begins with 5?-GU

True False
c. Introns have a sequence that ends with AG-3?

True False
d. The removal of introns occurs in spliceosomes

True False

Question 3: Messanger RNA

a. Is very stable and has a long half life within cells

True False
b. Acts as a template for protein synthesis

True False
c. Contains introns and exons

True False
d. Contains Thymine

True False

Question 4: Messanger RNA

a. Has a neuclotide sequence which is written in the 3? to 5? direction

True False
b. Contains deoxyribose

True False
c. Is synthesised by RNA polymerase

True False
d. Is synthesized during translation

True False

Question 5: The synthesis of DNA from mRNA

a. Requires the presence of a primer


True False
b. Is called reverse transcription

True False
c. Is mediated by the enzyme DNA polymerase

True False
d. Occurs normally in the human nucleus

True False

Question 6: With respect to the structure of nucleic acids

a. The 3? end of a nucleotide chain has a free OH group

True False
b. The 5? end of a nucleotide chain has a phosphate group

True False
c. In DNA, cytosine pairs with adenine

True False
d. Base-pairing occurs through the formation of 3?5?phosphodiester bonds

True False

Question 7: Messanger RNA in eukaryotic cells

a. Is polycistronic

True False
b. Has a 5? methylated cap

True False
c. HAS A 3? poly-A tail

True False
d. Is double-stranded

True False

Question 8: The following are components of nucleotides

a. Purine or pyrimidine base

True False
b. Pentose sugar

True False
c. Phosphate group

True False
d. Histones

True False

Question 9: During the normal cell cycle

a. DNA is principally synthesised during the G1 phase

True False
b. A tetraploid quantity of DNA is present during the G2 phase

True False
c. G2 is the post-mitotic resting phase

True False
d. Cells are generally sensitive to anti-metabolites in the S phase

True False

Question 10: With respect to transcription

a. Transcription factors are specific DNA sequences that are essential for the initiation of
transcription

True False
b. The RNA molecule is synthesized in the 3? to 5? direction

True False
c. The 5? methylated cap is added to mRNA within the cytoplasm

True False
d. The 3? poly-A tail is added to mRNA within the nucleus

True False

Question 11: The synthesis of DNA from RNA

a. Requires the presence of ribonucleotides

True False
b. Occurs in DNA viruses

True False
c. Occurs in retroviruses

True False
d. Is used for fluorescent in-situ hybridisation

True False

Question 12: Histones

a. Are acidic proteins

True False
b. Are arginine and lysine ? rich proteins

True False
c. Are synthesized in the nucleus

True False
d. Are highly negatively charged proteins

True False

Question 13: The following techniques are used to detect the macromolecules listed

a. Northern blotting - DNA

True False
b. Western blotting - protein

True False
c. Southern blotting - mRNA

True False
d. Fluorescent in-situ hybridisation - DNA
True False

Question 14: With respect to transcription

a. RNA syntheses requires the presence of a primer

True False
b. RNA polymerase transcribes the sense strand of the DNA template

True False
c. Alternative splicing may result in some introns persisting in cytoplasmic mRNA

True False
d. The RNA molecule is synthesized in the 3? to 5? direction

True False

Question 15: Ribosomal RNA

a. Has a secondary structure

True False
b. Has methylated bases

True False
c. Is essential for protein synthesis

True False
d. Makes up 20% of total cellular RNA

True False
ESSENTIAL AMINO ACIDS *****

• These are amino acids which cannot be synthesised by the body and
must be absorbed from the diet
• Include: Histidine, Leucine, Isoleucine, Lysine, Methionine,
Phenylalanine, Threonine, Tryptophan, Valine *

Classification according to side-chain *****

• Aliphatic - alanine, glycine, isoleucine, leucine, proline, valine

• Aromatic - phenylalanine, tryptophan, tyrosine

• Acidic - aspartic acid, glutamic acid

• Basic - arginine, histidine, lysine

• Hydroxylic - serine, threonine

• Sulphur-containing - cysteine, methionine, taurine

• Amidic (containing amide group) - asparagine, glutamine

• Methionine + serine are converted to cysteine in the liver, a precursor for


glutathione synthesis. Cystathionase, is the enzyme that is necessary to convert
cystathionine into cysteine, and is present in humans post-natally *

• Cystathionase is not present in human fetal liver or brain. *

• Cysteine is not an essential amino acid in adults but is an essential amino acid
for the human fetus, and for prematurely born and full term infants for a short period
after birth. *
• Cysteine concentration in maternal plasma is greater than or equal to that in
fetal plasma

• Methionine is also a precursor for taurine synthesis

AMINO ACID METABOLISM *****

• Excess amino acids are neither stored nor excreted but are converted
into glucose, fatty acids or ketone bodies or oxidised for ATP production

• The initial process of amino acid catabolism is transamination -the alpha


amino group is transferred to alpha-ketoglutarate, converting it to glutamate
with the formation of the corresponding alpha-keto-acid

• Transamination is catalysed by aminotransferases which are dependent


on a pyridoxal phosphate prosthetic group (derived from vitamin B6) *
• Alpha-keto-acids are converted into glucose (glucogenic amino acids) or
ketone bodies (ketogenic amino acids) *

• Leucine and lysine are the only strictly ketogenic amino acids *

• Isoleucine, tryptophan, phenylalanine and tyrosine are ketogenic and


glucogenic *

• The other amino acids are glucogenic *

• During starvation, muscle protein is broken down into amino acids which
are de-aminated and metabolised into pyruvate. Pyruvate can be converted
into alanine, transported to the liver and used for gluconeogenesis *

• Amino acids are filtered in the glomerullus and actively absorbed in the
proximal convoluted tubule *
PROTEIN STRUCTURE *****
• Primary structure - linear sequence of amino acids, including disulphide bonds

• Secondary structure - protein folding into alpha helix and pleated sheets

• Tertiary structure - regional folding between alpha helix and pleated sheets -
dependent on non-covalent forces

• Quaternary structure - only present in proteins with more than one sub-unit - such as
haemoglobin

COLLAGEN *****

• Predominant protein in the body. Synthesised within the cytoplasm in rough


endoplasmic reticulum and then released into the extracellular matrix where further
modifications occur

• Produced mainly by fibroblasts *

• Each collagen peptide is referred to as an alpha chain and is a left handed helical
polypeptide with the basic structure (Gly-A-B) n where one third of A residues are
proline and one third of B residues are hydroxyproline. Collagen is also rich in lysine
and hydroxylysine *

• Alpha chains are synthesised as pro-alpha chains and a signal peptide is excised.
Proline and lysine residues are hydroxylated in the endoplasmic reticulum and golgi
apparatus in a reaction which is dependent on vitamin C as cofactor *

• Glycosylation of alpha chains occurs and three chains are intertwined to form a right
handed triple helix *- tropocollagen which is released into the extracellular matrix

• The propeptide in most forms of collagen is excised in the extracellular matrix and
tropocollagen molecules spontaneously aggregate to form polymeric fibrils held
together through cross-linking between lysine and hydroxylysine residues. These
have a banded appearance on electron microscopy *

• Polymeric fibrils aggregate to form polymeric fibres visible by light microscopy

• Collagen synthesis is inhibited by glucocorticoids *

• Collagen type I: two identical and one different alpha chain. Makes up 90% of total
collagen and is found in skin, bone, tendon *

• Collagen type II: three identical alpha chains - cartilage and inter-vertebral disc *

• Collagen type III: three identical alpha chains - interstitial collagen in skin and
organs *

• Collagen type IV: two identical and one different alpha chain - basement
membrane *
PROTEIN SYNTHESIS *****

Occurs in the RIBOSOMES within the cytoplasm during TRANSLATION *

Four stages: initiation, elongation, translocation, termination.

RIBOSOMES

Eukaryotic ribosomes are 80S and dissociate into 40S and 60S subunits

The 40S subunit has an 18S rRNA molecule and about 34 different proteins

The 60S subunit has 5S, 5.8S and 28S rRNA molecules and about 50 different
proteins

Ribosomes may be found free in the cytoplasm (synthesize cytosolic proteins) or


attached to the endoplasmic reticulum (synthesize membrane proteins or proteins for
export) *

THE GENETIC CODE *****

Triplet codon derived from 4 bases giving a total of 64 possible codons

There are 20 amino acids, each coded by a different specific codon

However, one amino acid can have more than one codon - the genetic code is
DEGENERATE *

The mRNA molecule is translated from the 5- end to the 3- end

For an amino acid to be added to the protein chain, it must be bound to a tRNA
molecule in a reaction catalysed by aminoacyl tRNA synthetase. There are 20
enzymes, each specific for a particular amino acid. During this reaction, ATP is
hydrolysed to AMP + 2Pi *

The binding of tRNA molecules to the mRNA molecule is less stringent than the
pairing of complementary DNA strands. Pairing may still occur when the third
position on the mRNA codon read from the 5- to 3- end and the first position on the
anticodon on the aminoacyl-tRNA is not an exact match the WOBBLE hypothesis

INITIATION - Protein synthesis begins when the ribosomal subunits come together
with the mRNA molecule and the initiator tRNA molecule carrying the amino acid
METHIONINE *

The start codon is AUG and the first amino acid to be added is always
METHIONINE. This is usually excised during post-translational modification

There are two sites on the ribosome - the P (Peptidyl) site occupied by the peptide
chain and the A (aminoacyl) site to which new aminoacyl-tRNA molecules are added
ELONGATION - two processes - peptide bond formation and translocation

New aminoacyl-tRNAs are added to the A site and a peptide bond is formed between
the carbonyl atom of the P site aminoacyl tRNA and the alpha amino group of the A
site amino acid - catalysed by PEPTIDYL TRANSFERASE

There are mechanisms to check that the correct aminoacyl-tRNA is added to the A
site. Once the peptide bond is formed, the wrong amino acid cannot be removed

Translocation is the process of movement down the mRNAmolecule so that the A site
is emptied for the next aminoacyl-Trna - mediated by TRANSLOCASE and GTP
hydrolysis

TERMINATION - a termination code (UGA, UAA, UAG) is encountered. Peptidyl


transferase switches to a hydrolase function and hydrolyses the carbonyl end of the
peptide chain. Release factors catalyse the release of the peptide chain from the
ribosomal complex which dissociates.

The polypeptide chain is synthesized from the N- to the C- terminal *

POST-TRANSLATIONAL MODIFICATIONS *****

Excision of signal sequences - these sequences target proteins to specific sites,


mitochondria for instance and are removed once the target is reached. Nuclear
proteins have arginine and lysine -rich sequences

Disulphide bond formation

Proteolysis - for instance the conversion of pro-hormones to hormones

Glycosylation - three types: N-linked; O-linked and addition of the glycosyl


phosphatidylinositol anchor to membrane proteins

O-linked glycosylation occurs by the sequential addition of monosaccharide units to


the OH group of serine or threonine sidechains

N-linked glycosylation is the addition of oligosaccharide molecules to the NH2


groups of asparagine sidechains

Proteins may be modified during translation - for instance proline is hydroxylated


during collagen synthesis
URIC ACID *****

• Produced from the breakdown of purine bases in the liver *

• Plasma concentration higher in males (202 - 406 microM) than in


females (148 - 357 microM) *

• Weak acid with a pK of 5.8. At pHs below the pK, it exists


predominately in a nonionized form

• The urate ion is more soluble than the nonionized molecule.

• Urate ions are about 5% protein bound.

• Urate is filtered at the glomerulus. *

• The renal tubule can reabsorb or secrete. Typically, net reabsorption


occurs in infants and children while net secretion occurs in adults *

• The kidneys excrete two-thirds of the uric acid produced daily; the
remaining one-third is excreted in the stool

• In early pregnancy uric acid levels fall by about one-third but rise to
non-pregnant levels by term *

Causes of raised Plasma uric acid concentration *****

• Renal diseases with renal failure and prerenal renal failure (e.g.
dehydration)

• Gout.

• Drugs - diuretics, pyrazinamide, ethambutol, nicotinic acid, and aspirin


in low doses. *

• Excessive cell death - neoplasia, even before as well as following


chemotherapy and radiotherapy, especially lymphoma and leukemia;
hemolytic anemia, resolving pneumonia and other inflammation;
polycythemia, myeloma, pernicious anemia, infectious mononucleosis,
congestive heart failure, large myocardial infarct. *

• Endocrine conditions: hypothyroidism, hypoparathyroidism,


hyperparathyroidism, pseudohypoparathyroidism; diabetes insipidus of
nephrogenic type, Addison disease.

• Lead poisoning

• Hyperlipidaemia
• Acidosis: lactic acidosis, diabetic ketoacidosis, alcoholic ketosis.

• Pre-eclampsia *

• Dieting, weight loss, fasting or starvation. *

• Decreased urate clearance: cyclosporine-induced hyperuricemia.

Low uric acid concentration *****

• Drugs - allopurinol, corticosteroids and probenecid cause low


uric. Massive doses of vitamin C are uricosuric

• Poor dietary intake of purines and protein; tea, coffee.

• Renal tubular defects, Fanconi syndrome, late


in Wilson disease

• UREA *****

• Excess nitrogen is converted to ammonia and excreted as urea


which is formed in the urea cycle

• Molecule contains two amine groups {CO(NH2)2}*

• The urea cycle occurs in the liver. Blood urea is then taken up and
excreted by the kidneys

• Ammonia is conbined with CO2 to form carbamoyl phosphate

• Carbamoyl phosphate conbines with orinthine to form citrulline

Citrulline + aspartate to form arginosuccinate

• Arginosuccinate splits to release arginine and fumarate

• Arginine then splits to form urea and regenerate ornithine

• First 2 reactions occur in the mitochondria, rest occur in cytosol

• Fumarate formed in the urea cycle can enter the citric acid cycle

• 3 molecules of ATP are consumed per molecule of urea formed in the


urea cycle

• Freely filtered at the renal glomerulus. *

• About 50% of the filtered urea is reabsorbed passively in the proximal


tubule by a paracellular route
• Urea is secreted into the tubule fluid in the descending limb of the loop
of Henle by carrier mediated transport *

• Final urea excretion is dependent on the additional amount reabsorbed


by carrier-mediated transport in the medullary collecting ducts. This
reabsorption increases at low urine flows and decreases at high urine flows
and is controlled by ADH. *

• Urea concentration increases with age and is higher in males than in


females

• The urinary excretion of urea is proportional to protein intake and is


increased on a high protein diet. Urea excretion is decreased during
growth and pregnancy or due to action of insulin, growth hormone and
testosterone *

• The enzyme system necessary for urea synthesis in human fetuses is


functional when mesonephric glomeruli are present and urea is produced
by the fetus *
Biochemistry :. Amino acids & proteins :. Module 2: 60 Random questions for MCQ Test
Question 1: With respect to protein synthesis from mRNA

a. Each codon is specific for a particular amino acid

True False
b. One amino acid can be coded by more than one codon

True False
c. There are a total of 48 possible codons

True False
d. During the synthesis of aminoacyl-tRNA from amino acids and tRNA, ATP is converted to ADP
and phosphate

True False

Question 2: The following are essential amino acids

a. Histidine

True False
b. Isoleucine

True False
c. Leucine

True False
d. Lysine

True False

Question 3: Excess amino acids in the body

a. Are excreted in urine

True False
b. Are stored in the liver

True False
c. Are converted to glucose

True False
d. Are converted to ketone bodies

True False

Question 4: During protein synthesis

a. The reaction between transfer RNA and amino acids is catalysed by the enzyme aminoacyl-
tRNA synthetase

True False
b. There are 64 different aminoacyl tRNA synthetase enzymes

True False
c. The protein chain is synthesized from the C-terminal to the N-terminal

True False
d. A codon is made up of a sequence of four bases

True False

Question 5: During protein synthesis


a. The bases on the tRNA anticodon have to be exactly complementary to the mRNA codon for
pairing to occur

True False
b. The ?Wobble? hypothesis explains the ability of different tRNAs to pair with a specific mRNA
codon

True False
c. New aminoacyl tRNAs are added to the P-site of the ribosomal complex

True False
d. Peptide bond formation is catalysed by the enzyme Peptidyl transferase

True False

Question 6: Collagen

a. Has a double helical structure

True False
b. Synthesis is inhibited by glucocorticoids

True False
c. Shows a regular banding pattern on electron microscopy

True False
d. Is not found within basement membrane

True False

Question 7: Uric acid

a. Is formed from the breakdown of purines

True False
b. Is raised in serum during normal pregnancy

True False
c. Is raised in serum during treatment with thiazide diuretics

True False
d. Concentration in plasma increases in acute leukaemia

True False

Question 8: Urea

a. Is formed mainly in the kidneys

True False
b. Contains two amine groups

True False
c. Is formed by trans-amination from ketoglutarate

True False
d. Excretion is unrelated to protein intake

True False

Question 9: Methionine

a. Is an essential amino acid


True False
b. Is a sulphur-containing amino acid

True False
c. Cannot be converted to cystine by the fetal liver

True False
d. Is reabsorbed in the proximal convoluted tubule of the kidney

True False

Question 10: The following are recognised post-translational modifications of proteins

a. Formation of disulphide bonds

True False
b. Excision of signal sequences

True False
c. Addition of oligosaccharides to asparagines residues

True False
d. Proteolysis

True False

Question 11: The following amino acids are ketogenic

a. Leucine

True False
b. Lysine

True False
c. Isoleucine

True False
d. Tryptophan

True False

Question 12: With respect to protein synthesis from mRNA

a. The start codon is located at the 3? end of the mRNA molecule

True False
b. The last amino acid of every protein is glutamine

True False
c. Protein synthesis takes place on the ribosomes

True False
d. Transfer RNA molecules are specific for particular amino acids

True False

Question 13: Excess amino acids in the body

a. Are converted to fatty acids

True False
b. Are oxidised for ATP production

True False
c. Undergo transamination with the conversion of alpha-ketoglutarate to glutamate

True False
d. Undergo transamination which converts them into glucose

True False

Question 14: Collagen

a. Has proline making up 50% of the amino acid residues

True False
b. Has hydroxyproline making up one third of the amino acid residues

True False
c. Is synthesised in ribosomes attached to the endoplasmic reticulum

True False
d. Synthesis requires vitamin C which acts as a cofactor

True False

Question 15: Uric acid

a. Concentration in plasma falls with low dose aspirin therapy

True False
b. Concentration in plasma is raised by corticosteroid therapy

True False
c. Concentration in plasma is higher in males than in females

True False
d. Concentration in plasma characteristically falls during starvation

True False
CARBOHYDRATES

MONOSACCHARIDES *****

• Have a general formula (CH2O)n where n is 3 or more

• Have an aldehyde (aldose) or ketone (ketose) group *

• The free aldehyde or ketone can reduce Cu2+ to Cu+ - this forms the basis of Fehling’s and
Benedict’s tests. All monosaccharides are reducing sugars

• Include glucose, fructose, galactose

• Carbon atoms are numbered beginning with the C carrying the aldehyde or ketone group

• Glucose is an aldose while fructose is a ketose *

• Glucose has 6 carbon atoms and is therefore a hexose *

• D and L forms refer to the configuration of the asymmetry of the C atom furthest away from
the aldehyde or ketone group *

DISACCHARIDES *****

• Formed from two monosaccharides by glycosidic bonds

• Lactose = glucose + galactose: reducing

• Maltose = glucose + glucose: reducing


• Sucrose = glucose + fructose: non-reducing as the carbon atoms carrying the aldehyde /
ketone groups are involved in glycosidic bond formation

CHOLESTEROL *****

• Component of cell membranes and precursor of steroid hormones and vitamin D *

• C27 compound, all derived from acetyl CoA. The acetate units are initially converted to C5
isoprene units *

• Can be obtained from diet or synthesised in the liver *

• High levels of cholesterol and its metabolites reduce the activity of HMG coA reductase,
the enzyme catalysing the committed step in cholesterol synthesis

• Cholesterol is mainly excreted as bile salts and bile acids - glycocholate (glycine
conjugate) or taurocholate (taurine conjugate) *

TRIGLYCERIDES *****

• Three faty acid chains esterified to a glycerol backbone *

• Simple triglycerides have three identical fatty acids while mixed triglycerides have 2 or 3
different fatty acids *

• Major dietary lipid in humans and major energy store

• Synthesised from glycerol-3-phosphate (glycolytic intermediate) and fatty acyl CoA

• Fatty acids removed from triglycerides by the action of lipase and the free fatty acids
degraded by beta-oxidation
• Glucagons, adrenaline and noradrenaline increase plasma free fatty acid concentration
while insulin has the opposite effect *

• May accumulate in the liver (fatty liver) *

LIPOPROTEINS *****

• Globular particles consisting of a hydrophobic core and a hydrophilic surface. Contain


triglycerides, cholesterol, phospholipids, apoproteins

• Five different types

• With respect to density: Least dense: Chylomicron < VLDL < LDL < HDL most dense *

Chylomicrons *****

• Largest of the lipoproteins

• Synthesized by the gut and transport dietary lipids to tissues via peripheral circulation. *

• Transport cholesterol to the liver *

• Have a molecular mass of ~ 400kDa with a density of <0.95g/ml *

• Protein content is 1.5 - 2.5%

• Major lipid is triglycerides *

• At tissues, the triglycerides are hydrolysed by lipoprotein lipase

VLDL *****
• Synthesized in the liver and transports lipids from the liver to other tissues.

• Molecular weight ~10-80kDa with a density of <1.006g/ml *

• Protein content 5-10%

• Contain triglycerides, phospholipids and cholesterol ester *

• Triglycerides are hydrolysed by lipoprotein lipase. As triglycerides are removed and cholesterol
is esterified with the loss of apoproteins order than apoB-100, VLDL is converted to IDL and
then to LDL

LDL *****

• Binds to LDL receptor * and is internalised by receptor mediated endocytosis. Cholesterol is


released by the action of lysosomal lipase.

• Molecular weight ~2.3kDa, density 1.019 - 1.063g/ml

• Protein content 20-25% *

• Major lipids are cholesterol ester and phospholipids *

High intracellular cholesterol concentrations reduce the expression of the LDL receptor and inhibit
cellular biosynthesis of cholesterol from acetate *

Nascent HDL *****

• Produced by the liver. * More HDL is formed within the circulation. *

• Molecular weight 175-360Da, density 1.063 - 1.21g/ml


• 40-55% protein.*

• Major lipids are phospholipids and cholesterol ester *

• HDL extracts cholesterol from cell membranes, converting it into cholesterol esters which
are then transferred to VLDL. VLDL and HDL are taken up by the liver by receptor
mediated endocytosis and cholesterol is excreted in bile in the form of bile salts - the liver
is the only organ that can dispose of significant quantities of cholesterol. *
CITRIC ACID (TRI-CARBOXYLIC ACID) CYCLE *****

• Oxidation of pyruvate (derived from glycolysis) to carbon dioxide and


water

• Also oxidises acetyl CoA from fatty acid breakdown and amino acid
degradation products

• Occurs within the mitochondria under aerobic conditions *

• Glycolysis breaks 1 glucose into 2 pyruvate, producing 6 ATP.

• Pyruvate is used to make acetyl-CoA, the starting product for the citric
acid cycle.

• Each turn of the cycle oxidizes 1 pyruvate - 2 turns to completely


oxidize 1 glucose.

• Each turns produces 3 NADH, 1 FADH2, and 1 GTP. NADH and


FADH2 are then oxidatively phosphorylated producing ATP. *

• For each turn of the cycle, 12 molecules of ATP are produced and one
molecule of GTP *

• Two molecules of CO2 are produced per molecule of acetyl CoA *

• The citric acid cycle will operate to a reduced extent unless new
oxaloacetate is formed, because acetyl CoA cannot enter the cycle
unless it condenses with oxaloacetate

• Oxaloacetate is formed by the carboxylation of pyruvate, in a reaction


catalyzed by the biotin-dependent enzyme pyruvate carboxylase

• The disruption of pyruvate metabolism is the cause of


beriberi secondary to thiamine deficient - Thiamine pyrophosphate is
the prosthetic group of three important enzymes: pyruvate
dehydrogenase, α-ketoglutarate dehydrogenase, and transketolase *

• The low transketolase activity of red cells in beriberi is an easily


measured and reliable diagnostic indicator of the disease

• The citric acid cycle also produces intermediates for other biosynthetic
pathways. For instance: *

1) Citrate for fatty acid synthesis

2) Alpha-ketoglutarate which is trans-aminated for amino acid synthesis


3) Alpha-ketoglutarate and oxaloacetate used for the synthesis of purines and
pyrimidines

4) Oxaloacetate used for glucose synthesis - gluconeogenesis

5) Succinyl-coA used for the synthesis of the porphyrin ring of heme groups.

REGULATION

• Mainly regulated by substrate availability and


product inhibition
• Citrate synthase is inhibited by citrate and ATP
• Pyruvate dehydrogenase is inhibited by NADH and
acetyl CoA
• Alpha-ketoglutarate dehydrogenase is inhibited by
NADH and succinyl coA
• The cycle speeds up when cellular energy levels
are low (low ATP and NADH, high ADP) and slows
down when ATP accumulates
GLYCOGEN *****

• Branched chain polysaccharide containing glucose residues *

• Stored mainly in the liver and skeletal muscle - muscle glycogen provides energy for prolonged
muscle contraction while liver glycogen is used to maintain blood glucose levels *

• Glycogen is stored in granules in the cytosol *

GLYCOGENOLYSIS *****

• Phosphorylase catalyses the removal of glucose units by the phosphorylysis of alpha 1,4
glycosidic bonds up to 4 residues away from a branch point with the release of glucose-1
phosphate *

• Glucose-1-phosphate is converted to glucose-6-phosphate by phosphoglucomutase and this


can then enter glycolysis *

• The liver, intestines and kidneys have glucose-6-phosphatase which can release glucose from
glucose-6-phosphate. Glycogen in skeletal muscle cannot be converted into glucose as
glucose-6-phosphate cannot leave cells *

• The three residues leading up to a branch point on glycogen are transferred to another part of
the molecule by transferase

• Alpha-1,6-glycosidic bonds at branch points are cleaved by alpha-1,6 glucosidase

GLYCOGEN SYNTHESIS *****

• Glucose must initially be activated by conversion to UDP-glucose (Uridine diphosphate-


glucose)

• Glycogen synthase catalyses the addition of glucose from UDP-glucose to the non-reducing
end of the glycogen chain, forming alpha-1,4-glycosidic bonds. Glucose residues can only be
added to a chain of five or more residues

• When the chain contains over 11 residues, a branching enzyme transfers seven residues to an
interior site, forming a branch point which must be at least four residues from any pre-existing
branch points

REGULATION OF GLYCOGEN METABOLISM *****

• Regulated by the phosphorylation and dephosphorylation of phosphorylase (glycogenolysis)


and glycogen synthase (glycogen synthesis)

• Phosphorylase exists as a phosphorylated active 'a' form and a dephosphorylated inactive 'b'
form

• Glycogen synthase exists as a phosphorylated INACTIVE 'b' form and a dephosphorylated


active 'a' form

• Phosphorylation / dephosphorylation is regulated by the activity of phosphorylase kinase and


protein phosphatase I

• GLUCAGON / ADRENALINE - stimulate the phosphorylation of inactive phosphorylase ′ b′,


converting it into active phosphorylase ′a′ andglycogenolysis is stimulated *

• Phosphorylation of glycogen synthase converts the active 'a' form to the inactive 'b' form
and glycogen synthesis is inhibited
• INSULIN - results in the phosphorylation and activation of protein phosphatase I *

• Phosphorylated and active phosphorylase 'a' is de-phophorylated and inactivated -


glycogenolysis is inhibited *

• Phosphorylated and inactive glycogen synthase 'b' is dephosphorylated and activated -


glycogen synthesis is activated *

• Calcium ions released during muscle contraction stimulate glycogenolysis. Skeletal muscle
glycogen cannot be converted into glucose. *
GLYCOLYSIS *****

• Occurs in the cytoplasm and does not require oxygen. Principal energy generating mechanism
in the absence of oxygen of in cells such as RBC which do not have mitochondria *

• Initial reactions involve two phosphorylation steps: initially by hexokinase to form glucose-6-
phosphate (converted to fructose-6-phosphate by isomerase) and then to fructose 1,6
bisphosphate by phosphofructokinase with the hydrolysis of ATP to ADP.

• All reactions in the glycolysis pathway are reversible except those catalysed by hexokinase,
phosphofroctokinase and pyruvate kinase.

• Glycolysis can be summarised by:

Glucose + 2Pi + 2ADP + 2NAD+ = 2pyruvate + 2H2O + 2ATP + 2NADH + 2H+ *

• Pyruvaye can be reduced to Lactate, converted to Acetyl CoA or Oxaloacetate

• Fructose and galactose can both be metabolised by glycolysis *

GLUCONEOGENESIS *****

• Formation of glucose from non-carbohydrate precursors

• Occurs mainly in the liver but also in the renal cortex

Substrates include: *
1) Pyruvate

2) Lactate

3) Oxaloacetate

4) Some amino acids

5) Glycerol

Two molecules of pyruvate are converted to one molecule of glucose with the
hydrolysis of 4 molecules of ATP and two molecules of GTP *

• Gluconeogenesis occurs during conditions of deficient dietary intake

REGULATION OF GLYCOLYSIS AND


GLUCONEOGENESIS *****

• Unregulated glycolysis and gluconeogenesis would result in the futile consumption of ATP

• Glycolysis is inhibited when ATP and synthetic intermediates are in excess and
gluconeogenesis inhibited when ATP levels are low and ADP levels are high
• The main sites of regulation are at the reactions catalysed by phosphofructokinase (PFK -
glycolysis) and fructose 1,6 bisphosphatase (gluconeogenesis)

• AMP activates PFK (glycolysis and ATP synthesis) but inhibits fructose 1,6 bisphosphatase

• ATP inhibits PFK (glycolysis) and pyruvate kinase (glycolysis)

• Citrate (citric acid cycle) inhibits PFK (glycolysis) and stimulates fructose 1,6 bisphosphatase

• Glucagon released during starvation inhibits the synthesis of fructose 2,6 bisphosphate while
insulin stimulates its synthesis. Fructose 2,6 bisphosphate activates PFK (glycolysis) and
inhibits fructose 1,6 bisphosphatase

• The amino acid alanine inhibits pyruvate kinase (glycolysis)

CORI CYCLE

• Occurs during strenuous exercise - NADH formation by glycolysis exceeds the capacity of the
respiratory chain to oxidise it back into NAD+

• Pyruvate is produce by glycolysis in skeletal muscle and converted into lactate by lactate
dehydrogenase with the NADH to NAD+, allowing glycolysis to continue to provide ATP

• Lactate diffuses out of skeletal muscle into blood, is taken up by the liver and converted into
glucose by gluconeogenesis

• Hepatic glucose is then utilised by muscle for glycolysis


FATTY ACID SYNTHESIS *****

• Takes place in the cytosol

• Mediated by a single polypeptide enzyme - fatty acid synthase with several enzymatic domains

• The committed step in the pathway is the conversion of acetyl CoA to malonyl CoA by acetyl
CoA carboxylase

• Occurs by the repetitive addition of two carbon units from malonyl CoA which has a three
carbon unit, with the release of CO2

• ATP is consumed during fatty acid synthesis

TRIGLYCERIDE SYNTHESIS

• Alpha-glycerol phosphate and fatty acyl CoA are the substrates

• Alpha-glycerol phosphate reacts with 2 molecules of fatty acyl CoA to form phosphatidate

• The phosphate group is removed to form diglycerides

• Diglycerides + one fatty acyl CoA to form triglycerides

BREAKDOWN OF TRIGLYCERIDES *****

• Occurs in the cytoplasm

• Mediated by hormone sensitive lipase which is activated by glucagon, adrenaline,


noradrenaline and ACTH through phosphorylation

• Insulin causes dephosphorylation and inactivation of hormone sensitive lipase

• Triglycerides are converted to one molecule of glycerol and three fatty acids

• Glycerol can enter glycolysis through phosphorylation to glycerol-3-phosphate

FATTY ACID OXIDATION *****

• Begins in the cytoplasm by the conversion of fatty acids to fatty Acyl CoA - ATP is hydrolysed
to AMP + 2Pi and CoA is required. The reaction is catalysed by Acyl CoA synthetase *

• Fatty Acyl CoA is carried into the mitochondria for beta-oxidation.* This shuttling requires the
presence of carnitine. Acyl Carnitine is formed in the cytosol, carried into the mitochondria then
converted to carnitine + Acyl CoA

• In addition to ATP, NADH and FADH2 are also produced during beta-oxidation

• If the rate of fatty acid oxidation exceeds the rate of carbohydrate breakdown, more acetyl CoA
is generated than can be converted to oxaloacetate - acetyl CoA is then converted to ketone
bodies: acetoacetate and hydroxybutyrate *

• Ketone bodies are produced mainly by the liver and used as fuel particularly by the heart,
muscle and renal cortex. During starvation, the brain may also utilise ketone bodies. The brain,
however, metabolises mainly glucose *
• After a three-day fast, 30% of the energy used by the brain is from ketone bodies. And after 40
days, this goes up to 70%

• Both acetoacetate and beta-hydroxybutyrate are acidic, and if levels of ketone bodies are too
high, ketoacidosis occurs. This happens in untreated Type I diabetes and in alcoholics after
binge drinking and subsequent starvation*

• Ketone bodies are water soluble and may be excreted in the breath (acetone) or in urine
causing ketonuria *

• The fetal brain is an obligate glucose user and cannot utilise ketone bodies*
STARVATION *****

POST-ABSORPTIVE PHASE

• Insulin concentrations fall while glucagon concentrations rise *

• Phosphorylation of enzymes occurs: Glycogen synthase -inactivated,


glycogen phosphorylase - activated, hormone sensitive lipase - activated *

• Triglyceride breakdown is stimulated in adipose tissue and glycogenolysis


is stimulated with the release of glucose and fatty acids into blood. *

GLUCONEOGENIC PHASE *****

• Glycolysis is inhibited by the phosphorylation and inactivation of


phosphofroctokinase and increased activity of fructose-1,6-bisphosphatase

• Fatty acid oxidation is increased in the liver

• Amino acids are released from skeletal muscle and utilised by the liver for
gluconeogenesis

KETOTIC PHASE *****

• Acetyl CoA in the liver is converted to ketone bodies which are used as
fuel mainly by the heart and renal cortex

• Ketone bodies stimulate insulin secretion which prevents excessive


protein hydrolysis

PENTOSE PHOSPHATE PATHWAY

• Occurs in the cytoplasm

• Starting substrate is glucose-6-phosphate

• Oxidative pathway results in the conversion of glucose-6-phosphate to


ribolose-5-phosphate with the production of NADPH

• Ribolose-5-phosphate is converted to Ribose-5-phosphate which is a


precursor for DNA, RNA, ATP, NAD+, FAD and CoA synthesis *

• Non-oxidative pathway results in the conversion of three molecules of


ribolose-5-phosphate to 2 molecules of fructose-6-phosphate and one
molecule of glyceraldehyde-3-phosphate. These can then enter the
glycolytic pathway
Biochemistry :. Carbohydrate & lipid metabolism :. Module 3: 60 Random questions for MCQ Test
Question 1: Plasma low density lipoprotein

a. Are less dense than chylomicron

True False
b. Predominantly contain non-esterified fatty acids

True False
c. Contain more protein by dry weight than high density lipoprotein

True False
d. Attach to specific cell membrane receptors

True False

Question 2: During triglyceride synthesis

a. Free fatty acid molecules are added directly to glycerol

True False
b. Phosphatidate is produced by the combination of alpha-glycerol phosphate and two molecules
of free fatty acid

True False
c. Diglycerides are produced following the dephosphorylation of phosphatidate

True False
d. Diglycerides combine with one molecule of fatty acyl CoA to form triglycerides

True False

Question 3: With respect to the regulation of glycogen metabolism

a. Insulin stimulates glycogen synthesis

True False
b. Glucagon inhibits glycogenolysis

True False
c. Adrenaline stimulates glycogenolysis

True False
d. Phosphorylase a is phosphorylated and active

True False

Question 4: Glycogen

a. Is a polymer of glucose residues

True False
b. Is predominantly found in cytoplasm

True False
c. Is mainly stored in the liver

True False
d. Is cleaved by phosphorylase to glucose-1-phosphate

True False

Question 5: During the gluconeogenic phase of starvation


a. Glycolysis is stimulated

True False
b. Amino acids are released from skeletal muscle protein

True False
c. There is increased fatty acid synthesis

True False
d. Phosphofructokinase is inactivated

True False

Question 6: During glycogen synthesis

a. Glucose must be converted to glucose-6-phosphate in order to be added to the glycogen


molecule

True False
b. The enzyme glycogen synthase catalyses the addition of glucose units to the non-reducing end
of a chain

True False
c. Glycogen synthase can only add glucose residues to a chain containing three or more
residues

True False
d. New branch points are created at least four residues away from any pre-existing branch points

True False

Question 7: With respect to lipoprotein synthesis and metabolism

a. Chylomicrons are lipoproteins

True False
b. Lipoproteins do not contain any cholesterol

True False
c. Lipoproteins have a hydrophilic core and a hydrophobic surface

True False
d. VLDLs are synthesized in the liver

True False

Question 8: Muscle glycogen

a. Metabolism cannot yield free glucose

True False
b. Metabolism is independent of the enzyme phosphorylase

True False
c. Metabolism only generates ATP under aerobic conditions

True False
d. Is entirely intra-cellular

True False

Question 9: The oxidation of fatty acids

a. Begins in the cytoplasm


True False
b. Requires the presence of carnitine

True False
c. Does not require mitochondria

True False
d. May result in the production of ketone bodies

True False

Question 10: During the cori cycle

a. Pyruvate is produced by glycolysis in skeletal muscle

True False
b. Pyruvate is converted to lactate in skeletal muscle

True False
c. Lactate is taken up by hepatocytes and converted to alanine

True False
d. Glucose formed from hepatic gluconeogenesis is utilised by skeletal muscle for glycolysis

True False

Question 11: Cholesterol

a. Is synthesised in the liver

True False
b. Is a C19 compound

True False
c. Synthesised from acetate

True False
d. Predominantly excreted unchanged in urine

True False

Question 12: Chylomicrons

a. Are synthesised in adipose tissue

True False
b. Are a major component of bile

True False
c. Contain triglycerides

True False
d. Are predominantly composed of free fatty acids

True False

Question 13: With respect to fatty acid metabolism

a. Hormone sensitive lipase is inhibited by ACTH

True False
b. Glucagon results in the phosphorylation of hormone sensitive lipase

True False
c. Insulin causes dephosphorylation and inactivation of hormone sensitive lipase

True False
d. Glycerol from triglycerides cannot be used for glycolysis

True False

Question 14: During the post-absorptive phase of starvation

a. Insulin concentration increases while glucagon concentration falls

True False
b. Glycogen synthase is phosphorylated

True False
c. Glycogen synthase is activated

True False
d. Glycogen phosphorylase is phosphorylated

True False

Question 15: With respect to the regulation of glycolysis and gluconeogenesis

a. AMP activates phosphofructokinase but inhibits fructose 1,6 bisphosphatase

True False
b. ATP activates phosphofructokinase

True False
c. Citrate inhibits phosphofructokinase

True False
d. Citrate inhibits fructose 1,6 bisphosphatase

True False
PROTEIN DIGESTION *****

MOUTH - mechanical disintegration only

STOMACH

• HCL secreted by Oxyntic cells denatures proteins

• Pepsinogen is secreted by Chief cells - converted into active pepsin when pH


falls < 5 (autocatalysis) and by proteolysis by pepsin. Endopeptidase and
cleaves peptide bonds on the NH2 side of aromatic amino acids. Optimal pH
= 2-3

SMALL INTESTINE

• Enterokinase - secreted by duodenal mucosa: proteolytic conversion of


trypsinogen to trypsin

• Bicarbonate secreted by the pancreas and biliary tree provides the optimal pH
for enzyme action

• Trypsin also activates trypsinogen in addition to other zymogens including


chymotrypsinogen, pro-elastase and pro-carboxypeptidase

• Trypsin, chymotrypsin and elastase are endopeptidases - digest proteins from


within the polypeptide chain

• Carboxypeptidases release amino acids from the carboxy terminal of the


polypeptide chain

• Endopeptidases, aminopeptidases and dipeptidases are present in the brush-


border of intestinal epithelial cells and hydrolyse oligopeptides

• Amino acids, dipeptides and tripeptides are absorbed. Further breakdown of di-
and tri-peptides occurs within intestinal epithelial cells

CARBOHYDRATE DIGESTION *****

MOUTH

• Salivary amylase, hydrolyses alpha 1,4 glycosydic bonds

• Salivary amylase is inactivated by the acidic environment in the stomach.


Enzymatic digestion of carbohydrates does not occur in the stomach

• Pancreatic amylase is secreted by the pancreas and continues the hydrolysis of


glycosidic bonds. Bicarbonate provides optimal pH for enzyme action
• Ologosaccharides and disaccharides are further digested by enzymes within the
brush border of intestinal epithelial cells: Sucrase: sucrose to glucose +
fructose; Lactase: Lactose to glucose + galactose; Maltase: maltose to glucose

• Lactase deficiency results in impaired lactose digestion. Lactose is fermented


by bacteria in the colon producing gas and osmotically active products which
cause flatulence and diarrhoea

LIPID DIGESTION *****

• Begins in the stomach by the action of lingual lipase secreted by glands at the
back of the tongue and active in the acidic pH of the stomach. Hydrolyses
triglycerides to fatty acids, mono - / di-acylglycerol and glycerol *

• Main enzymatic digestion of fats occurs in the small intestine

• Pancreas secretes pancreatic lipase, lipid esterase and phospholipase A2 with


bicarbonate providing the optimal pH for enzyme action. Pancreatic lipase is
inhibited by bile acids but activated by colipase -secreted by the pancreas as
procolipase and activated by trypsin. Lipid esterase and phospholipase A2
require bile acids for optimal function *

• Lipids are converted to free fatty acids, glycerol and cholesterol

• Bile acids are synthesized by the liver and are amphipathic molecules - have
both hydrophilic and hydrophobic parts. Form micelles in solution with the
hydrophobic parts facing inwards and the hydrophilic parts facing outwards *

• Free fatty acids, monoacylglycerol, phospholipids and cholesterol are


incorporated into micelles to form mixed micelles

• Lipids are absorbed from the micelles mainly in the jejunum. The majority of
bile salts are absorbed in the terminal ileum and there is an effective entero-
hepatic circulation *

• Within intestinal epithelial cells, fatty acids containing less than 12 carbon
atoms pass directly into the circulation *

• Fatty acids containing over 12 carbon atoms are metabolised into triglycerides
and incorporated into chylomicrons by the endoplasmic reticulum and golgi
apparatus *

• Chylomicrons enter the intestinal lymphatics and enter the circulation via the
thoracic duct *

• In tissues, triglycerides within chylomicrons are hydrolysed by lipoprotein


lipase to release fatty acids which are taken up. The chylomicron remnant is
taken up by the liver by receptor mediated endocytosis *
ENZYME INHIBITORS *****

Can reversible or irreversible. Reversible inhibition can be competitive or non-


competitive

• IRREVERSIBLE INHIBITION *- permanent enzyme inhibition usually due


to covalent binding of the inhibitor to the enzyme

REVERSIBLE INHIBITION - bind non-covalently to the enzyme and can


be removed by dialysis

• COMPETITIVE INHIBITION *- competes with the substrate for its


binding site on the enzyme. Usually a substrate analogue and may be
converted to product. Action can be overcome by increasing substrate
concentration. There is an apparent increase in the Km of the enzyme (more
substrate required to achieve a particular initial rate) but no change in the
Vmax. The Lineweaver-Burk plot therefore shows no change in the intercept
on the y-axis (1/Vmax) BUT the slope (Km/Vmax) is increased

• NON-COMPETITIVE INHIBITION *- binds to the enzyme at a site other


than the active site. The substrate may still bind to the enzyme but is not
converted to product. Enzyme is effectively removed from the reaction. The
Km is unchanged but the Vmax is reduced, hence the y-intercept and the slope
of the Lineweaver-Burk plot are both altered.

ALLOSTERIC ENZYMES

• Enzymes whose activity is altered by the binding of ligands to sites other than
the substrate binding site. Ligand binding may result in activation or inhibition
of enzyme activity. Ligands are allosteric activators or inhibitors *

• K-class: ligand binding results in ALTERED Km but no change in Vmax (see


competitive inhibition). Lineweaver-Burk plot: y-intercept unchanged but
slope is changed

• V-class: ligand binding results in ALTERED Vmax but unchanged Km (see


non-competitive inhibition). Lineweaver-Burk plot shows altered y-intercept
and slope

• If the enzyme is an oligomer (more than one identical subunits), the individual
units are known as protomers. Homotropic interaction is when an allosteric
ligand increases the affinity of another protomer for the same ligand. The
interaction is heterotropic if the affinity for a different ligand is increased.

COFACTORS *****

Non-protein molecules that are necessary for enzyme action. Tow main groups
• COENZYMES - may be metal ions such as Mn2+ or Zn2+ or or complex organic
molecules such as NAD+, NADP+, FAD and Flavine mononucleotide
(FMN) - they are non-covalently bound to the enzyme

• PROSTHETIC GROUPS - are covalently bound to the enzyme at its active site and
include metal ions and organic molecules such as biotin

The protein part of the enzyme without its cofactor is called an apoenzyme

ISOENZYMES

• Different forms of an enzyme

• Catalyse the same reaction and have the same substrate specificity

• Have different physical or kinetic properties such as substrate affinity optimal


pH, susceptibility to inhibitors, isoelectric point

• Usually different gene products and are expressed in different cells or tissues

REGULATION OF ENZYME ACTIVITY IN VIVO *****

• Regulation of enzyme protein synthesis - the total amount of enzyme present


is a balance between synthesis and degradation

• Feed-back inhibition - end-product often feeds back to inhibit the committed


step earlier on in the same pathway

• Allosteric modification - enzyme activity altered by the binding of allosteric


activators / inhibitors

• Reversible covalent modification - most commonly phosphorylation /


dephosphorylation

• Proteolytic activation - digestive enzymes / coagulation pathway

• Enzymes are not activated by the products of their reaction otherwise a vicious
cycle would develop. Irreversible covalent modification does not regulate
enzyme activity in vivo but is the mechanism of action of drugs / toxins
PROPERTIES OF ENZYMES *****

• Proteins which catalyse chemical reactions but are unchanged at the end of the
reaction

• Increase the rate at which equilibrium is reached but do not alter the direction
of the reaction or the equilibrium constant

• Reduce the activation energy required for the reaction

• Have an optimum pH for their action - pH affects enzyme structure, degree of


ionisation of substrate and enzyme-substrate interaction *

• Rate of enzyme catalysed reaction increases with increased temperature but


above 40C, the enzyme becomes progressively inactivated *

• Enzymes are specific for specific substrates and for the reaction catalysed.
They have an active site which undergoes a conformational change on contact
with substrate to accommodate the substrate (induced fit model). The enzyme-
substrate complex is held together by non-covalent forces - hydrogen and
hydrophobic bonds, electrostatic and van der Waals forces *

• Some enzymes are regulated by phosphorylation / dephosphorylation

• Activity of some enzymes is modified by the presence of co-factors *

ENZYME KINETICS *****

• VELOCITY: The rate of an enzyme-catalysed reaction. Highest at time t=0


(initial velocity, V0) because the substrate concentration is maximum and
there is no product formed (product may have feed-back inhibition on enzyme
activity)

• If the substrate concentration is progressively increased for a fixed enzyme


concentration, there comes a point where all the active sites on the enzyme are
occupied and the initial velocity of the reaction cannot be increased further.
This is the maximum velocity V max
• If there is an excess of substrate, the initial velocity is proportional to the
enzyme concentration and vice-versa

• The initial velocity is given by the Michaelis-Menten equation:

V0 = (Vmax[S]) /([S] +Km): Km = Michaelis constant which is the substrate


concentration at which the initial velocity of the reaction is half the maximum
velocity

• The Km is a measure of the affinity of the enzyme for its substrate - the higher
the Km, the lower the affinity

• The plot of initial velocity against substrate concentration gives a rectangular


hyperbole. There are various methods of deriving a linear equation from the
Michaelis-Menten equation. One of these, the Lineweaver-Burk plot used a
plot of 1/V0 against 1/[S]. The intercept on the y-axis = 1/Vmax and the slope
of the line = Km/Vmax

• The SI unit of enzyme activity is the katal (kat) = the quantity of enzyme in
the presence of which one mole of substrate is converted to product in one
second under optimal conditions for the activity of that enzyme

• The standard unit of enzyme activity U is the amount of enzyme catalysing the
conversion of 1 micromol of substrate per minute under optimal conditions for
the activity of that enzyme
Biochemistry :. Enzymes & digestion :. Module 4: 60 Random questions for MCQ Test
Question 1: With respect to the enzymatic digestion of protein

a. The optimal pH for the action of pepsin is 5.0

True False
b. Pepsin cleaves peptide bonds on the NH2 side of sromatic amino acids

True False
c. Enterokinase is secreated by the pancreas

True False
d. Enterokinase converts trypsinogen to trypsin

True False

Question 2: Enzymes

a. Increase the rate at which a reaction reaches equilibrium

True False
b. Shift the equilibrium point of a reaction towards the synthesis of product

True False
c. Are inactivated at the end of the reaction

True False
d. Increase the equilibrium constant of the reaction catalysed

True False

Question 3: With respect to the enzymatic digestion of protein

a. Dipeptidases are secreted by the pancreas

True False
b. Dipeptides and tripeptides are absorbed by intestinal epithelial cells

True False
c. Trypsin is an endopeptidase

True False
d. Further hydrolysis of peptide bonds occurs within intestinal epithelial cells

True False

Question 4: Isoenzymes

a. Catalyse the same reaction

True False
b. Have different substrate specificities

True False
c. Have different substrate affinities

True False
d. Have identical susceptibility to inhibitors

True False

Question 5: The following are recognised classes of enzymes

a. Digestive enzymes
True False
b. Polymerases

True False
c. Oxidoreductases

True False
d. Transferases

True False

Question 6: Allosteric enzymes

a. Of the V class have an altered Km but unchanged Vmax in the presence of allosteric ligand

True False
b. Of the V class, in the presence of allosteric ligand have a Lineweaver-Burke plot with an
altered y-intercept

True False
c. Of the V class, in the presence of allosteric ligand have a Lineweaver-Burk plot with an altered
slope

True False
d. Show homotropic interaction if the ligand increases the affinity of another protomer for the
same ligand

True False

Question 7: With respect to the digestion of lipids

a. Phospholipase A2 is inhibited by bile acids

True False
b. Lipid esterase requires bile acids for optimal function

True False
c. Bile salts are hydrophobic molecules

True False
d. Bile salts form micelles in solution

True False

Question 8: The following are recognised classes of enzymes

a. Hydrolylases

True False
b. Lyases

True False
c. Isomerases

True False
d. Ligases

True False

Question 9: With respect to the kinetics of enzyme reactions

a. If the substrate concentration is in excess, the initial rate of the reaction is proportional to the
enzyme concentration
True False
b. The initial rate of the reaction is proportional to the substrate concentration for a given enzyme
concentration

True False
c. The Km of an enzyme is the substrate concentration at whch the initial velocity of the reaction
is half the maximum velocity

True False
d. When the substrate concentration = the Km, the initial velocity of the reaction = Vmax

True False

Question 10: Enzyme activity is regulated in-vivo by the following mechanisms

a. Reversible covalent modification

True False
b. Proteolytic activation

True False
c. Regulation of enzyme protein synthesis

True False
d. Irreversible covalent modification

True False

Question 11: Enzyme cofactors

a. May be covalently attached to the enzyme

True False
b. Include NAD+

True False
c. Include NADP+

True False
d. Include Flavin adenine dinucleotide

True False

Question 12: With respect to the enzymatic digestion of protein

a. Trypsin converts trypsinogen to trypsin

True False
b. Chymotrypsinogen is activated by trypsin

True False
c. Chymotrypsin is a carboxypeptidase

True False
d. Elastase is an endopeptidase

True False

Question 13: With respect to the digestion of lipids

a. Lipid digestion occurs by the action of lingual lipase in the stomach

True False
b. Glycerol is produced by the action of lingual lipase on triglycerides
True False
c. Monoglycerides are produced by the action of lingual lipase on triglycerides

True False
d. Lipid esterase is secreted by the pancreas

True False

Question 14: With respect to the digestion of lipids

a. Colipase is secreted by the intestinal epithelial cells as procolipase

True False
b. Phospholipase A2 is secreted by the intestinal epithelial cells

True False
c. Pancreatic lipase is inhibited by bile acids

True False
d. Colipase binds to and activates lipase

True False

Question 15: Enzymes

a. Are specific for specific substrates

True False
b. Bind substrates by covalent bonds

True False
c. Undergo a conformational change when the active site comes in contact with substrate

True False
d. Can be activated by phosphorylation

True False
Vitamin A (retinol) *****

 Retinol

 Fat soluble vitamin

 Found in dairy products, liver, fish and leafy vegetable

 Synthesised in the intestinal wall from beta-carotene *

 Not destroyed by cooking

 Recommended daily requirements 750, 500 and 1200 micrograms for adults,
children and lactating women respectively

 High doses of vitamin A are terratogenic *

Vitamin A deficiency

 Common in developing countries and common cause of blindness

 Plays important role in vision. Rhodopsin* is formed from a protein called


opsin and a vitamin A-dependent compound called 11-cis retinal. As light
strikes the rods and cones it is absorbed by the pigment molecules, and
retinal is split off from opsin

 Impaired dark adaptation and eventually night blindness

 Dryness of the conjunctiva and cornea - xeropthalmia

 Bitot’s spots - white plaques of keratinised epithelial cells on the conjunctiva

 Follicular keratosis -thickening and dryness of the skin

Vitamin D (caliciferol) *****

 Produced predominantly in the skin by photoactivation of 7-


dehydrocholesterol to form cholecalciferol

 In the liver, cholecalciferol is concerted to 25[OH] Cholecalciferol

 In the kidneys, 25[OH]Cholecalciferol is converted to 1,25[OH]2


Cholecalciferol. this is regulated by parathyroid hormone, phosphate,
calcium levels and feed-back inhibition by 1,25[OH]2 Cholecalciferol

 If vitamin D supplies are adequate, the kidneys also produce 24,25[OH]2


Cholecalciferol

 Sarcoid and lymphomatous tissue can also produce 1,25[OH]2


Cholecalciferol
 Transported bound to vitamin D binding protein

 Acts via intracellular receptor interacting with DNA to alter gene expression

Vitamin D deficiency

 Associated with raised serum alkaline phosphatase due to increased bone


turnover

 Causes growth failure and rickets in children *

 Causes osteomalacia in adults *

 Commonest in Asian populations in UK when the skin is not exposed to sun


light. Increased melanin in the skin decreases vitamin D production

 Treated by increased exposure to sun light, oral vitamin D

Vitamin K *****
 Fat soluble vitamin - deficiency occurs in fat malabsorption
 Present in plant foods as phylloquinone (Vitamin K1)
 Synthesised by large intestinal bacteria as menaquinone (Vitamin K2) -
broad spectrum antibiotics destroy intestinal flora and reduce vitamin K
production *
 Co-factor in the synthesis of clotting factors - prothrombin (Factor II),
Factors VII, IX, X *
 Not stored in significant amounts and deficiency occurs within a few days of
cessation of bile secretion *
 Deficiency results in prolonged prothrombin time.
 Increased prothrombin time caused by liver failure is not corrected by
vitamin K administration
 Should be offered to all neonates (orally) to prevent haemorrhagic disease of
the new born. Intramuscular administration recommended only in neonates
at high risk of haemorrhagic disease

Vitamin E (tocopherol) *****

 Alpha-tocopherol

 Fat-soluble vitamin

 Occurs mainly in vegetable oils and fish

 Anti-oxidant

 Role in nutrition uncertain but severe deficiency causes anaemia, haemolysis


and muscle disorders

Thiamine (B1) *****


 Water soluble vitamin

 Co-factor in several reactions, especially those involved in the glycolytic


pathway

 Not stored in significant quantities and deficiency occurs within days -weeks
of inadequate intake

 Destroyed by cooking

 Deficiency typically occurs in chronic alcoholics, as Beriberi in children


when the only food consumed is polished rice and in starved patients
including hyperemesis gravidarum

 Thiamine should be administered to chronic alcoholics BEFORE the


administration of intravenous glucose

 Deficiency should be suspected in endemic areas and can be confirmed by


measuring red cell transketolase activity

 Deficiency treated by im or iv thiamine (with other vitamin supplements)


followed by oral supplementation

Beriberi *

 Caused by thiamine deficiency

 Confined to developing countries, especially in Asia where the main food is


polished rice

 Dry beriberi presents with heaviness and stiffness of the lower limbs,
weakness, numbness, loss of ankle reflexes. Polyneuropathy may involve the
trunk. Cerebral involvement results in Wernicke-Korsakoff syndrome

 Wet beriberi associated with oedema initially due to vasodilatation but


cardiac failure subsequently develops. Ascites and pleural effusion may
occur

 Infantile beriberi may occur acutely in breastfed infants at about 3 months of


age

 Wet beriberi shows an acute response within hours of thiamine


administration.
Riboflavine (B2) *

 Water soluble vitamin

 Found in many animal and plant foods including dairy products, leafy
vegetables and offal
 Not destroyed by cooking

 Destroyed by sun light

 Co-factor in oxidative reactions

 Deficiency is associated with angular stomatitis (cheilosis) and seborrhoeic


dermatitis involving the face, scrotum and vulva

Niacin (B) *

 Nicotinic acid and nicotinamide

 Nicotinamide found in the nucleotides NAD and NADP involved in redox


reactions

 Found in both animal and plant foods. Lost by removing bran from cereal

 Synthesised in humans from the amino acid tryptophan this reaction requires
the presence of vitamin B6 (Pyridoxine)

 Deficiency causes Pellagra, *characterised by Dementia, Diarrhoea and


Dermatitis. Angular stomatitis also occurs

 Deficiency occurs in communities where maize is the main food (contains


niacytin which is not absorbed), Hartnup disease (in-born error of
metabolism with malabsorption of amino acids including tryptophan),
therapy with isoniazide (Leads to B6 deficiency), carcinoid syndrome and
phaeochromocytoma (tryptophan metabolism is diverted)

Vitamin B6 (Pyridoxine)

 Found in plant and animal foods

 Deficiency associated with peripheral neuropathy

 Deficiency occurs in celiac disease and following treatment with isoniazide,


hydralazine and penicillamine

Vitamin C (ascorbic acid) *****

 Ascorbic acid

 Simple sugar but humans and other primates cannot produce it from glucose

 Anti-oxidant with important role in the regulation of redox potential within


cells
 Important in the hydroxylation of praline to form hydroxyproline during
collagen synthesis *

 Present in fresh fruit and vegetable - lost during storage and readily leached
out of vegetables placed in water

 Oxidised during cooking

 Vitamin C transporters are present in the placenta

 Deficiency causes scurvy - mainly due to impaired collagen synthesis and


characterised by: spontaneous haemorrhages, especially sub-periosteal
haemorrhages, swollen bleeding gums, poor wound healing, anaemia,
keratosis of hair follicles *

Vitamin B12 (cyancobalamin) *****

(Cobalamine / Hydroxycobalamine)

 Synthesised by microbes and animals but not by humans

 Not present in plant foods

 Not destroyed by cooking

 Enough is stored in the liver to last for up to 5 years

 Important co-enzyme in several reactions including the metabolism of


methyltetrahydrofolate which is important for DNA synthesis

 Absorption in the ileum requires the presence of Intrinsic factor, a


glycoprotein produced by the gastric parietal cells

 Transported in the circulation bound to Transcobalamine I (in serum), II (in


erythrocytes and delivered to bone marrow) and III

 Transcobalamine I & III are produced by granulocytes - serum vitamin B12


levels rise in myeloproliferative disorders

Vitamin B12 deficiency *

Occurs in:

 Low dietary intake - Vegans

 Impaired absorption -pernicious anaemia, gastrectomy, ileal disease /


resection, celiac disease, tropical sprue, pancreatic disease and Zollinger
Ellison syndrome
 Nitrous oxide inhalation - inactivates vitamin B12

Vitamin B12 deficiency (Pernicious anaemia) - features *

 Anaemia - megaloblastic

 Weight loss

 Hyperbilirubinaemia due to haemolysis

 Neurological manifestations

a) Sub-acute combined degeneration of the spinal cord - symmetrical paraesthesia,


weakness, loss of vibration and joint position sense, ataxia

b) Dementia

c) Optic atrophy

TRACE ELEMENTS *****

 Copper - deficiency occurs in Menke’s kinky hair syndrome - sex-linked


abnormality with growth failure, learning disability, brittle hair, anaemia and
neutropaenia

 Iodine - endemic goitre -occurs in areas with a daily intake of less than
150micrograms per day

 Zinc - Acrodermatitis enteropathica: inherited disorder due to zinc


malabsorption - diarrhoea, growth restriction, hair loss, candida + bacterial
infections

 Selenium - deficiency associated with growth restriction


 Fluorine - dental caries in areas where fluorine levels in drinking water are
below 1 part per million

HYPERVITAMINOSIS *****

 Excessive administration of vitamins. It is a particular problem with certain


fat soluble vitamins (A, D, and E) which are not readily cleared by the body
and with vitamin K in newborn infants. Riboflavine, biotin and vitamin C
are the least likely to cause problems
 Hypervitaminosis A*: Acutely - may occur following a large overdose
(eating polar bears liver). Headaches, nausea & vomiting, diarrhoea, double
vision, lethargy and loss of consciousness with evidence of raised intra-
cranial pressure. Skin desquamation, anaemia and thrombocytopaenia may
occur. Chronic Hypervitaminosis A - increased intracranial pressure
(hydrocephalus), skin disorders, loss of hair, skeletal deformities with
hypercalcaemia, and psychiatric side-effects. High doses of vitamin A are
potentially teratogenic.
 Hypervitaminosis D *-toxic at levels which are only marginally greater than
the recommended daily requirement. Acute effects include abdominal
cramps, nausea, vomiting. Chronic effects include hypercalciuria, and often
metastatic calcification. Renal toxicity and renal stones may also occur.
 Hypervitaminosis K *- administration of large doses of vitamin K to infants,
particularly premature infants. It may be associated with
hyprebilirubinaemia, haemolytic anaemia, hepaomegaly, and, in severe
cases, death.
 Hypervitaminosis E *- headache, fatigue, double vision, diarrhoea and
inhibition of uptake of other fat-soluble vitamins
 Vitamin B 6 - toxic at doses about 1000 times the recommended daily
allowance - difficulty in walking & paraesthesia.
 Vitamin C toxicity *-increased risk of oxalate stone formation
Biochemistry :. Nutrition :. Module 5: 60 Random questions for MCQ Test
Question 1: Niacin

a. Deficiency occurs in phaeochromocytoma

True False
b. Deficiency causes beriberi

True False
c. Deficiency causes angular stomatitis

True False
d. Deficiency causes dementia, diarrhoea and dermatitis

True False

Question 2: Vitamin B12

a. Can be synthesised de-novo by humans

True False
b. Is present in plant foods

True False
c. Is not stored by humans in significant amounts

True False
d. Is destroyed by cooking

True False

Question 3: Vitamin A

a. Can be synthesised in the skin

True False
b. Is derived from beta-carotene

True False
c. Is toxic to tissues in high doses

True False
d. Is required for rhodposin synthesis

True False

Question 4: Vitamin B6 (Pyridoxine)

a. Is found in both plant and animal foods

True False
b. Deficiency causes peripheral neuropathy

True False
c. Deficiency occurs in patients treated with hydralazine

True False
d. Deficiency occurs in patients treated with penicillin

True False

Question 5: Niacin

a. Production from tryptophan requires the presence of pyridoxine


True False
b. Deficiency occurs in patients treated with ethambutol

True False
c. Deficiency occurs in Hartnup disease

True False
d. Deficiency occurs in carcinoid syndrome

True False

Question 6: Vitamin K

a. Should be administered to all neonates by intra-muscular injection

True False
b. Deficiency is treated by intra-muscular injection of phytomenadione

True False
c. Deficiency causes Beriberi

True False
d. Is stored in the liver in sufficient quantities to provide for up to 6 months requirement

True False

Question 7: Vitamin C (Ascorbic acid)

a. Is a simple sugar

True False
b. Is an oxidising agent

True False
c. Is involved in the hydroxylation of proline in the synthesis of collagen

True False
d. Can be synthesised from glucose in humans

True False

Question 8: Beriberi

a. Typically occurs in chronic alcoholics

True False
b. Is associated with peripheral vasodilatation

True False
c. Is associated with brisk ankle reflexes

True False
d. May result in Wernicke-Korsakoff syndrome

True False

Question 9: Vitamin E

a. Is a water-soluble vitamin

True False
b. Is mainly found in meat

True False
c. Is an anti-oxidant

True False
d. Deficiency results in xeropthalmia

True False

Question 10: Vitamin B12 deficiency occurs

a. In Crohn?s disease

True False
b. In ulcerative colitis

True False
c. In pregnancy

True False
d. In the Zollinger-Ellison syndrome

True False

Question 11: Riboflavine

a. Deficiency is associated with angular stomatitis

True False
b. Deficiency is associated with the Wernicke-Korsakoff syndrome

True False
c. Deficiency is associated with seborrhoeic dermatitis

True False
d. Is a water soluble vitamin

True False

Question 12: Niacin

a. Is essential for the production of nicotinamide adenine dinucleotide

True False
b. Is found exclusively in fish oils

True False
c. Is lost by removing bran from cereal

True False
d. Can be synthesised from tryptophan in humans

True False

Question 13: The neurological manifestations of vitamin B12 deficiency include

a. Optic neuritis

True False
b. Exaggerated ankle reflexes

True False
c. Asymmetrical lower limb weakness

True False
d. Generalised paresis of the insane
True False

Question 14: Deficiency of the following are matched with the correct disease

a. Thiamine ? Pellagra

True False
b. Cyanocobalamine ? Microcytic anaemia

True False
c. Niacin ? Beriberi

True False
d. Ascorbic acid ? Night blindness

True False

Question 15: Vitamin C (Ascorbic acid)

a. Deficiency is associated with sub-periosteal haemorrhages

True False
b. Does not cross the placenta

True False
c. Deficiency is associated with poor wound healing

True False
d. Deficiency is associated with loss of the enamel of teeth

True False
PRE-IMPLANTATION DEVELOPMENT *****

• The Zygote begins a series of mitotic divisions within 24h of fertilization


- cleavage. The number of cells increases but the size of the embryo
remains constant within the zona pellucida - results in a decrease in
mean cell volume. By the 32 cell stage, the embryo is called a morula *

• The cells (blastomeres) become segregated into the inner cell mass
which forms the embryo proper and the outer cell mass which forms
the placenta and membranes *

• Fluid collects between the cells of the inner cell mass, forming a
blastocyst cavity - day 4 of development

The blastocyst enters the uterine cavity ~day 4, hatches from the zona
Pellucida *

PRIMITIVE STREAK *****

• Appears during the third week and is clearly visible by day 15-16

• Narrow groove on the epiblast with slightly bulging sides

• Cephalic end forms primitive node - elevated area surrounding a small


pit, the primitive pit

• Cells from the epiblast migrate towards the primitive streak, detach
from and slip underneath the epiblast to form a third layer between the
epiblast and hypoblast - the intra-embryonic mesoderm

• These cells also form most, or all of the intra-embryonic endoderm

1
• This process is known as gastrulation, at the end of which the
remaining epiblast forms the ectoderm

• The ectoderm and endoderm remain in contact, without intervening


mesoderm in two regions - the buccopharyngeal membrane and the
cloacal membrane

NERVOUS SYSTEM *****

• Development begins during the third week *

• A thickening of the ectoderm forms the neural plate with elevated


lateral edges forming the neural folds and the depressed mid-region
forming the neural groove *

• Fusion of the neural folds begins in the region of the future neck (4th
somite) and proceeds in ceplalic and caudal directions, forming the
neural tube

• The neural tube remains temporarily open to the amniotic cavity at the
anterior and posterior neuropores *

• The anterior neuropore closes on day 25 and the posterior on day 27 *

• Abnormal fusion of the neural tube results in neural tube defects -


spina bifide / anencephaly. Risk reduced by use of folic acid 12 weeks
before to 12 weeks after fertilisation *

2
NEURAL CREST *****

• Ectodermal cells at the edge of the neural grove, become detached


and located initially between the closed neural tube and the overlying
ectoderm *
• Contribute to several important structures including

1) The truncoconal septum of the heart - migrate via pharyngeal arches. Form
the connective tissue , muscle and parasympathetic ganglia *

2) Dorsal root ganglia of spinal nerves *

3) Sensory ganglia of 5th, 7th, 9th and 10th cranial nerves *

4) Aortic sympathetic ganglia *

5) Sympathetic chain *

6) Parasympathetic ganglia *

7) Melanocytes *

8) Cartilage of pharyngeal arches *

9) Odontoblasts *

10) Schwann cells and meninges *

3
SPERMATOGENESIS
• Begins at puberty under the influence of testosterone *
• Sertoli cells differentiate and the sex cords become canalised and converted
into seminiferous tubules *
• The dormant primordial germ cells divide by mitosis and then differentiate
into spermatogonia located immediately beneath the basement membrane of
the seminiferous tubules *
• Germ cells are translocated from the basement membrane to the lumen of the
seminiferous tubule as spermatogenesis progresses
• Spermatogonium - (mitotic division) -primary spermatocyte - (first meiotic
division) - secondary spermatocyte - (second meiotic division) - spermatid -
(spermiogenesis) - spermatozoon *
• Spermatozoon obtain full motility in the epididymis *
• Spermatogenesis takes 64 days *
• Primordial germ cells - diploid (2N) *
• Spermatogonia - diploid (2N) *
• Primary spermatocytes - diploid (4N) *
• Secondary spermatocytes - haploid (2N) *
• Spermatides - haploid (N) *

SPERMATOZOON *****
• Head - contains a condensed nucleus capped by the acrosome which contains
hydrolytic enzymes
• Middle-piece - contains large helical mitochondria which generate the power
for swimming
• Tail - contain microtubules forming part of the propulsion system
• Sperm morphology is not important in determining fertility and abnormal
spermatozoa are commonly present in fertile semen *
• Capacitation - changes in the acrosome in preparation for release of hydrolytic
enzymes required to penetrate the zona pellucida - occurs in the female genital
tract

OOGENESIS *****
• Begins during intra-uterine life. The primordial germ cells undergo mitosis
and then differentiate into oogonia *
• By the 5th month in-utero, the number of oogonia is at a maximum - 7
million *
• Oogonia enlarge into primary oocytes. These are surrounded by flattened
follicular cells forming primordial follicles
• Primary oocyte enters prophase of meiosis I and becomes arrested at the
dictyotene stage. Meiosis I is not completed until puberty *
• Primordial follicles begin to degenerate such that at birth, there are 700,000 - 2
million, and at puberty there are 40,000 *
• Several primordial follicles (10-12) begin to mature with each menstrual
cycle and one dominant follicle becomes selected by mechanisms which are
not fully understood *

1
• Follicular cells become cuboidal, forming the primary follicle. A layer of
acellular mucopolysaccharide becomes deposited between the developing
oocyte and the follicular cells, forming the zona pellucida

• Resumption of meiosis is triggered by the ovulatory LH / FSH surge with


formation of the secondary oocyte with most of the cytoplasm and the first
polar body *

• Ovulation occurs the moment the secondary oocyte shows spindle formation
and the second meiotic division is only completed if fertilisation occurs *

• The polar body also completes meiosis II, resulting in one definitive oocyte
and three polar bodies *

FERTILIZATION *****
• Occurs in the ampulla of the fallopian tube *

• The second meiotic division in the oocyte is only completed if fertilization


occurs *

• The spermatozoan undergo capacitation within the female genital tract, during
which seminal plasma proteins and a glycoprotein coat are removed from the
plasma membrane overlying the acrosome

• The spermatozoan force their way through the cumulus oophorus to reach the
zona pellucida to which they are attached aided by sperm receptors

• The acrosome reaction results in the release of hydrolytic enzymes and one
spermatozoan enters the oocyte. This causes immediate release of cortical
granules into the perivitelline space (between the oocyte cell membrane and
the zona pellucida, preventing penetration by other sperm *

2
Question 1: During gametogenesis

a. The first meiotic division in the male is completed before birth


True False
b. The first meiotic division in the female is completed before birth
True False
c. DNA replication occurs before the second meiotic division
True False
d. The second polar body does not contain any DNA
True False

Question 2: The following are derived from the ectoderm

a. The lens
True False
b. The inner ear
True False
c. The middle ear
True False
d. The neural creast
True False

Question 3: With respect to the process of oogenesis

a. The zona pellucida develops after ovulation


True False
b. The first meiotic division is completed before puberty
True False
c. The second meiotic division is completed before ovulation
True False
d. The second meiotic division is completed only if the oocyte is fertilised
True False

1
Question 4: The following are derived from the ectoderm

a. The central nervous system


True False
b. The peripheral nervous system
True False
c. The epidermis, hair and nails
True False
d. The mammary glands
True False

Question 5: The following are derivatives of the neural crest

a. Odontoblasts
True False
b. Meninges
True False
c. The truncoconal septum of the heart
True False
d. Peripheral spinal nerves
True False

Question 6: Fertilisation

a. Occurs in the ampulla of the fallopian tube


True False
b. Results in completion of the first meiotic division in the oocyte
True False
c. Results in the cortical granules of the oocyte being released into the
cytoplasm
True False
d. Occurs before the completion of the second meiotic division in the
oocyte
True False

2
Question 7: The primordial germ cells

a. Originate from the primitive endoderm


True False
b. Are located in the wall of the yolk sac in the 6 week embryo
True False
c. Migrate to the developing gonad by amoeboid movement
True False
d. Undergo the first meiotic division during their migration to the primitive
gomad
True False

Question 8: With respect to the development of the pituitary gland

a. The anterior pituitary gland develops from the ectoderm of the


forebrain
True False
b. Rathke?s pouch is located on the dorsal aspect of the stomatodeum
True False
c. The posterior pituitary develops as a downward extension of the
diencephalons
True False
d. The pituitary stalk and the pars tuberalis are developmentally parts of
the anterior pituitary gland
True False

Question 9: During the development of the nervous system

a. The neural plate is derived from ectoderm


True False
b. Fusion of the neural tube begins in the region of the future neck and
proceeds in a cephalic and caudal direction
True False
c. Closure of the posterior neuropore occurs before closure of the anterior
neuropore
True False
d. Closure of the anterior neuropore occurs on day 45
True False

Question 10: With respect to the process of oogenesis

a. The total number of oogonia increases between birth and puberty

3
True False
b. The first meiotic division is completed during intra-uterine life
True False
c. Primary oocytes are haploid cells
True False
d. The dictyotene stage occurs between the end of the first and the
beginning of the second meiotic divisions
True False

Question 11: The primitive streak

a. Appears on the surface of the hypoblast on days 15-16


True False
b. Has the primitive node located at its caudal end
True False
c. Formation leads to gastrulation and the formation of the extra-
embryonic mesoderm
True False
d. Formation leads to gestrulation and the formation of the intra-
ambryonic endoderm
True False

Question 12: In spermatogenesis

a. Spermatides are haploid


True False
b. The secondary spermatocyte contains 23 chromosomes
True False
c. One secondary spermatocyte forms two spermatides
True False
d. The seminiferous tubules contain motile spermatozoa
True False

4
Question 13: Concerning oogenesis

a. The primary oogonia undergo mitotic division prior to meiosis


True False
b. The primary oocyte enters meiosis at puberty
True False
c. At birth, primary oocytes have completed the first meiotic division
True False
d. Secondary oocytes complete the second meiotic division at ovulation
True False

Question 14: Following fertilisation in humans

a. The first polar body is formed


True False
b. The first cleavage division occurs within 12h
True False
c. The zona pellucida is shed at the second cleavage division
True False
d. The mean cell volume decreases with each cleavage division
True False

Question 15: During the pre-implantation development of the embryo

a. The blastocyst hatches from the zona pellucida within the fallopian
tube
True False
b. The blastocyst enters the uterine cavity on day 7 after fertilization
True False
c. The outer cell mass gives rise to the embryo proper
True False
d. The inner cell mass gives rise to the placenta and membranes
True False

5
PHARYNGEAL ARCHES

• First arch - cartilage forms incus and malleus, mesenchyme


gives rise to the maxilla, mandible, zygomatic bone and part of
the temporal bone

• Second arch - cartilage forms the stapes and styloid


process of the temporal bone and part of the hyoid bone

• Third arch - part of the hyoid bone

• Fourth to sixth arches - thyroid, cricoid, arytenoids and


cuneiform cartilages

• Pharyngeal arches develop during the 4th-5th weeks. The


arches receive neural crest cells which form the skeletal
components while the mesenchyme forms the muscle.

NERVE SUPPLY TO PHARYNGEAL ARCHES

• First arch - mandibular branch of the trigerminal nerve

• Second arch - facial nerve

• Third arch - glossopharyngeal nerve

• Fourth arch - superior laryngeal branch of the vagus

• Sixth arch - recurrent laryngeal branch of the vagus

1
PHARYNGEAL POUCHES

• First pouch - tympanic cavity and pharyngo-tympanic tube

• Second pharyngeal pouch - palatine tonsil

• Third pharyngeal pouch - inferior parathyroid gland and the


thymus gland

• Fourth pharyngeal pouch - superior parathyroid gland

• Fifth pharyngeal pouch - parafollicular or C cells of the


thyroid gland

• The thyroid gland develops from an epithelial proliferation in


the floor of the pharynx (represented by the foramen cecum)
and subsequently descends in front of the pharyngeal gut,
hyoid bone and laryngeal cartilages to reach its final position
in the 7th week.

• Thyroxine is produced from the 12th week.

• The first pharyngeal cleft forms the external auditory


meatus. The others do not form any definitive structures.

2
DEVELOPMENT OF THE GUT *****

FOREGUT

• The pharynx extends from the buccopharyngeal membrane to the


tracheo-bronchial diverticulum

• The tracheo-bronchial diverticulum develops on the ventral wall of the


foregut during the 4th week and then becomes separated from it by the
tracheo-oesophageal septum. The respiratory primordium is therefore
developed from the foregut

• The stomach develops as a dilatation and rotates 90 degrees


clockwise along its longitudinal axis. The left side comes to lie
anteriorly such that the left vagus nerve forms the anterior vagal trunk

• Rotation of the stomach leads to the formation of the lesser sac

• The liver develops as an outgrowth of the endoderm of the distal


foregut, as does the gall bladder, bile and cystic ducts. The
haemopoietic cells, Kupffer cells and connective tissue are derived
from the mesoderm of the septum transversum

• The pancreas develops from a dorsal and a ventral pancreatic bud


from the distal foregut. With rotation of the duodenum, the ventral
pancreatic bud migrates dorsally.

• The dorsal bud forms the head, body and tail while the ventral bud
forms the uncinate process of the pancreas.

• The main pancreatic duct is formed from the distal part of the dorsal
pancreatic duct and the entire ventral duct. Exocrine pancreatic glands
are derived from the foregut endoderm.
• Insulin secretion begins in the fifth month. The origin of the Islets is
controversial.

1
MIDGUT

• Begins distal to the entrance of the bile duct into the duodenum and
ends at the junction between the proximal 2/3 and distal 1/3 of the
transverse colon

• During the 6th week, the midgut herniated into the extra-embryonic
coelom in the umbilical cord and rotates 90 degrees counter clockwise
along the axis of the superior mesenteric artery when viewed from in
front

• The apex of the intestinal loop remains in open connection with the
yolk sac through the vitelline duct, remnants of which form the
Meckel?s diverticulum

• During the 10th week, the midgut retracts into the abdomen,
undergoing a further 180 degrees rotation

HINDGUT

• Extends to the upper two thirds of the anal canal. The distal third of
the anal canal is formed from the ectoderm of the cloaca and the
junction is marked by the pectinate line

• The urogenital septum divides the cloaca into an anterior urogenital


sinus (forming the bladder, pelvic urethra and external genitalia) and
the anorectal canal

• The gut forms a solid cord during the 6th week with re-canalisation
during the 7th - 8th weeks

• The smooth muscle and mesentery of the gut is derived from


mesoderm

2
3
DEVELOPMENT OF THE TESTIS *****

• The gonads do not develop male / female differentiation until the end
of the 6th week *

• Primordial germ cells develop from the yolk sac endoderm and migrate
via the dorsal mesentery of the hindgut to reach the gonadal ridge in
the 6th week *

• Failure of migration results in gonadal agenesis

• The gonadal ridge develops medial to the mesonephros by


proliferation of the coelomic epithelium and condensation of the
underlying mesenchyme, forming the primitive sex cords

• The sex-determining region of the Y chromosome has a testis


determining factor, transcription of which triggers male development *

• Primitive sex cords proliferate into medullary cords with Sertoli cells
which produce Mullerian Inhibiting Factor which acts on the ipsilateral
Mullerian tube only, causing degeneration. Mullerian remnants in the
male include the appendix testis and the Utriculus prostaticus *

• Leydig (Interstitial) cells develop from the mesenchyme and produce


testosterone, influencing development of the duct system and external
genitalia. The enzyme 5-alpha reductase is essential for the
development of the external genitalia but not for the development of the
duct system *

• The distal excretory tubules of the mesonephros persist, forming the


ductuli efferentes while the mesonephric duct elongates and forms the
eipdidymis and vas deferens *

• The medullary cords remain solid until puberty when they become
canalised, forming the seminiferous tubules *

• The first meiotic division and spermatogenesis do not commence until


puberty *

• The seminal vesicles develop as outgrowth of the mesonephric duct


while the prostate and bulbo-urethral glands develop from the prostatic
urethra

DESCENT OF THE TESTIS *****

1
• During the 7th week, the testis begins to detach from the surrounding
mesenchyme and develops a tough connective tissue coat - tunica
albuginae

• A thickening of mesenchyme, the Gubernaculum testis runs from the


testis to the genital swellings

• As a result of the growth of the body relative to the gubernaculums, the


testis descends to lie in the inguinal region during the 12th week *

• During this process, an evagination of the coelomic epithelium forms


the processus vaginalis which follows the descent of the testis

• The final descent of the testis into the scrotum occurs in the 7th-9th
months*

• The processus vaginalis is obliterated during the first year of life,


forming the tunica vaginalis

DEVELOPMENT OF THE OVARY *****

• The primitive sex cords disintegrate and the centre of the developing
ovary becomes replaced by a vascular stroma, forming the ovarian
medulla *

• The surface epithelium continues to proliferate, producing cortical


cords which split into isolated cell clusters surrounding the primitive
germ cells

• The primitive germ cells differentiate into oogonia which undergo


several mitotic divisions. The number of oogonia reaches its maximum
(~7 million) during the 5th month after which degeneration begins *

• The oogonia become surrounded by a layer of follicular cells from the


surface epithelium and develop into primary oocytes. Primary oocytes
surrounded by follicular cells form primordial follicles

• The primary oocytes duplicate their DNA and enter the first meiotic
division and become arrested during Prophase I until puberty

• There are 700,000 - 2 million primary oocytes at birth and 40,000 at


puberty*

• The descent of the ovary is less extensive, coming to lie within the
pelvis. The round ligaments of the ovary and uterus are the equivalent
of the gubernaculums testis *

2
GENITAL DUCTS IN THE FEMALE *****

• The Mullerian ducts persist in the female while the Mesonephric ducts
degenerate. The Mullerian duct gives rise to the Fallopian tube, body of
the uterus, cervix and upper third of the vagina *

• The Mullerian ducts fuse and grow into the urogenital sinus, forming
the sinovaginal bulb which proliferates to form a solid plate of tissue
between the uterus and the urogenital sinus. This becomes canalised
at the end of the 5th month to form the vagina *

• The epoophron and paroophron and Gartners cyst are remnants of the
Mesonephric duct *

• BLADDER & URETHRA*****

• Development occurs in the 4th to 7th week

• Urogenital septum divides the cloaca into the anorectal canal and the
primitive urogenital sinus - endodermal in origin *

• Cloacal membrane divided into urogenital membrane and anal


membrane *

• The upper part of the urogenital sinus forms the bladder (except the
trigone) which is initially continuous with the allantois (obliterated to
form the urachus) *

• The trogone is formed from the mesoderm of the mesonephric ducts *

• The narrow pelvic part of the urogenital sinus forms the prostatic and
membranous urethra in the male *

• The definitive urogenital sinus which forms the external genitalia

EXTERNAL GENITALIA *****

• Cloacal folds develop on either side of the cloacal ggmembrane


during the 3rd week and fuse to form the genital tubercle cranial to the
cloaca

• With partitioning of the cloaca, the folds form the urethral folds
anteriorly and the anal folds posteriorly

1
• The male / female genitalia are indistinguishable at the end of the 6th
week *

• Under the influence of testosterone (and 5-alpha reductase), the


genital tubercle elongates rapidly in the male, forming the penis, pulling
the urethral folds forward forming the urethral groove

• The urethral groove closes over at the end of the 12th week, forming
the penile urethra. The lining is endodermal in origin. The external
urethral meatus is formed from ectodermal cells from the tip of the
glans which penetrate inwards, forming a cord which is later
canalised *

• The genital swellings, which form on either side of the urethral


swellings form the scrotum in the male and the labia majora in the
female

• In the female, there is only slight elongation of the genital tubercle,


forming the clitoris. The urethral folds do not fuse and form the labia
minora. The urogenital groove is open to the surface and forms the
vestibule *

• The epithelium of the male and female urethra is endodermal


(urogenital sinus) in origin apart from the most distal tip in the male
which is ectodermal in origin. The proximal part of the urethra in the
female forms the urethral and paraurethral glands and greater
vestibular (Bartholin's) glands *

2
KIDNEY AND URETER *****

• Three overlapping systems

Pronephros

• Vestigial - develops from cervical nephrotomes, disappears


by the end of the 4th week

Mesonephros *****

• Develops from the intermediate mesoderm on either side of


the upper thoracic and lumbar vertebrae during the 4th week *

• The mesonephric duct appears ~day 24 on the dorso-lateral


aspect of the mesonephros and grows caudally to fuse with
the cloaca on ~day 26 *

• The mesonephros is functional between the 6th - 10th


week, producing urine. The mesonephros regresses after 10
weeks in the female *

• In the male, the mesonephric duct and a few modified


mesonephric tubules persist forming the ductus deferens and
ductuli efferentes of the testis. In the female, Gartners duct
cysts, epoophron and paroophron are mesonephric
remnants *

• Mullerian remnants in the male include the appendix testis


and the Utriculus prostaticus *

• The developing gonad lies medial to the mesonephros *

• The mesonephric ducts are derived from mesoderm

Metanephros - permanent kidney *****

• Develops in the 5th-15th week from the metanephric


mesoderm in the sacral region *

• Ureters - develop from the ureteric bud, an outgrowth of the


mesonephric duct close to its opening into the cloaca. Grows
into the metanephric mesoderm, successive divisions forming
the renal pelvis, major and minor calyces and collecting
tubules *

• The metanephric mesoderm develops into the excretory


units under inductive influence of the ureteric bud, forming the

1
Bowman’s capsule and excretory tubules. If the ureteric bud is
missing, the kidney does not develop *

• The metanephros is functional from the 10th week *

Metanephric development occurs in the pelvis and the kidney later


ascends to its location in the abdomen by the 10th week. Failure of this
ascent leads to a pelvic kidney *

LOWER URINARY TRACT *****

• The cloaca becomes divided into the anorectal canal and


the primitive urogenital sinus during the 4th - 7th week by the
urorectal septum *

• The cloacal membrane is divided into the anal and


urogenital membranes

• The upper part of the primitive urogenital sinus forms the


urinary bladder which is initially continuous with the allantois *

• The obliterated allantois forms the urachus or median


umbilical ligament *

• The distal portions of the mesonephric ducts are absorbed


into the urinary bladder such that the ureters come to open
directly into the bladder. With the ascent of the kidneys, the
mesonephric ducts come to open into the prostatic urethra as
the ejaculatory ducts *

• The part of the mucosa of the bladder derived from the


incorporation of the mesonephric ducts is mesodermal in
origin and forms the TRIGONE. The mesodermal lining of the
trigone is later replaced by cells of endodermal origin such
that at birth, the bladder is completely lined by cells of
endodermal origin *

• The narrow pelvic part of the urogenital sinus forms the


prostatic and membranous part of the urethra in the male
(membranous urethra in the female). Outgrowths of the
endoderm of the prostatic urethra into the surrounding
mesoderm form the prostate gland (urethral and para-urethral
glands in the female)*

• The definitive urogenital sinus develops into the distal


urethra and external genitalia *

2
FETAL ARTERIAL SYSTEM *****

• Each branchial arch receives its own artery during the 4-5th week -
aortic arches

• The first aortic arch disappears apart from a small part - maxillary
artery

• Second arch disappears - small hyoid artery

• Third arch - common carotid artery and first part of internal carotid
artery

• The forth arch on the left forms part of the arch of the aorta. Right -
proximal part of right subclavian artery

• Fifth arch is transient

• Sixth arch - pulmonary arch

FETAL VENOUS SYSTEM *****

• Three pairs of major veins develop in the 5th week:

• Vitelline veins - carry blood from the yolk sac to the sinus venosus -
develops into the post-hepatic portion of the inferior vena cava, the
portal vein and superior mesenteric vein

• Umbilical veins - the proximal part of both umbilical veins and the
remainder of the right vein later disappear so that the left vein is the
only one to carry blood from the placenta to the liver

• Cardinal veins: anterior cardinal veins anastomose to form the


brachiocephalic vein. The superior vena cava is formed from the right
common cardinal vein and the proximal portion of the right anterior
cardinal vein. Anastomosis of the sacrocardinal veins forms the
common iliac veins. *

The developing umbilical cord contains the following *


• Yolk sac stalk
• Two umbilical arteries and one umbilical vein
• The remnant of the allantois
• Small intestinal loops
• Extra-embryonic mesoderm

FETAL CIRCULATION *****

1
• Two umbilical arteries and one umbilical vein with in the umbilical
cord *

• The umbilical vein carries oxygenated blood from the placenta to the
fetus *

• Oxygenated blood in the umbilical vein bypasses the liver, draining


into the inferior vena cava via the ductus venosus *

• The obliterated umbilical vein forms the ligamentum teres while the
obliterated ductus venosus forms the ligamentum venosum. The
ductus venosus is formed when a direct communication develops
between the left umbilical vein and the hepatocardiac channel *

• Oxygenated blood entering the right atrium from the inferior vena cava
is directed into the left atrium through the foramen ovale *

• Desaturated blood from the superior vena cava flows via the right
ventricle to the pulmonary artery *

• Oxygenated blood then enters the aorta via the left ventricle

• Some blood from the right atrium leaves via the pulmonary artery -
mainly return from the superior vena cava. As the pulmonary circulation
has high resistance, the blood enters the descending aorta through the
ductus arteriosus which connects the pulmonary artery to the aorta *

• From the aorta, blood is supplied to the fetus, deoxygenated blood


returns to the placenta via two umbilical arteries - oxygen saturation
here is ~58% *

• The proximal part of the umbilical arteries form the superior vesical
arteries. The obliterated distal part form the medial umbilical
ligaments *

• Oxygenated and deoxygenated blood become mixed at the following


points *
1) The liver, mixing with blood returning via the portal system

2) The inferior vena cava, mixing with venous blood from the lower extremities

3) The right atrium, mixing with venous blood from the superior vena cava

4) The descending aorta, mixing with blood from the ductus arteriosus

• Blood in the ascending aorta has the highest oxygen saturation and
supplies the heart and brain *

2
• Blood in the right ventricle is therefore mostly de-oxygenated blood
from the superior vena cava. Oxygenated blood passes directly into the
left atrium via the foramen ovale and does not enter the right ventricle *

CIRCULATORY CHANGES AT BIRTH *****

• Placental blood flow ceases


• Respiration begins

• Pulmonary artery vasodilatation in response to falling PaCO2 and


rising PaO2 - rapid fall in pulmonary vascular resistance in the first few
days of life *

• Closure of the umbilical arteries - functionally closed within a few


minutes of birth, obliteration takes 2-3 months. Distal part forms the
Medial umbilical ligaments while the proximal portion remains open as
the superior vesical arteries *

• Closure of the umbilical vein and ductus venosus - occurs shortly after
closure of the umbilical arteries. The umbilical vein forms the
Ligamentum teres hepatis in the falciform ligament while the ductus
venosus forms the ligamentum venosum *

• Loss of umbilical blood supply reduces venous return via the inferior
vena cava - fall in pressure *

• Closure of the Ductus arteriosus - dependent on a rise in PaO2.


Prostaglandin F, low calcium, low glucose and high pulmonary
pressure keep ductus arteriosus open in utero. Hypoxia can cause
ductus to become patent *

• Closure of the foramen ovale - caused by decreased right atrial


pressure and increased left atrial pressure. Held shut by
haemodynamic forces only for the first few weeks. Remains potentially
patent in 25-30% of normal adults *

3
Question 1: The following are derived from the mesonephros

a. Appendix of the testis


True False
b. Efferent ductules of the testis
True False
c. Gartnet?s duct cyst
True False
d. Prostatic utricle
True False

Question 2: During the development of the gastro-intestinal system

a. The liver is derived from mid-gut endoderm


True False
b. The haemopoietic cells of the liver are derived from foregut endoderm
True False
c. The pancreas is derived from the endoderm of the midgut
True False
d. The pancreas develops from a dorsal and a ventral pancreatic buds
True False

Question 3: With respect to the circulatory system in the fetus

a. Blood flows from the left atrium into the right atrium through the
foramen ovale
True False
b. The resistance within the pulmonary vessels is high
True False
c. Oxygenated blood from the placenta is mixed with deoxygenated blood
within the right atrium
True False
d. Oxygenated blood from the placenta is mixed with deoxygenated blood
in the inferior vena cava
True False

1
Question 4: During the development of the genital system in the male

a. The male gonad is distinguishable from the female gonad by 4 weeks


of development
True False
b. Germ cells are present in the developing gonad by 5 weeks
True False
c. Gonadal development may proceed in the absence of germ cells
True False
d. Germ cells originate in the endoderm of the yolk sac
True False

Question 5: In the fetal circulatrion

a. Blood from the inferior vena cava is largely directed through the
foramen ovale
True False
b. Most blood from the superior vena cava passes directly from the right
to the left atrium
True False
c. Out-put of the right ventricle is greater than that of the left ventricle
True False
d. Blood in the descending aorta is more oxygenated than that in the
ascending aorta
True False

Question 6: With respect to the development of the urinary tract

a. The mesonephric duct disappears completely in the male


True False
b. The mesonephric duct persists in the female to form the fallopian
tubes
True False
c. The metanephros forms the permanent kidney
True False
d. The metanephros appears during the third week
True False

2
Question 7: With respect to the circulatory system in the fetus

a. The obliterated umbilical vein forms the ligamentum venosum


True False
b. The obliterated ductus venosus forms the ligamentum venosum
True False
c. The ductus venosus connects the umbilical vein to the inferior vena
cava
True False
d. The ductus arteriosus connects the pulmonary vein to the aorta
True False

Question 8: The following are derived from the pharyngeal arches

a. First pharyngeal arch ? the mandible


True False
b. First pharyngeal arch ? the maxilla
True False
c. First pharyngeal arch ? the thyroid gland
True False
d. Fourth and sixth pharyngeal arches ? thyroid and cricoid cartilages
True False

Question 9: With respect to the development of the external genitalia

a. The lining of the male external urethral meatus is ectodermal in origin


True False
b. The genital swellings give rise to the penis
True False
c. The genital tubercle gives rise to the clitoris in the female
True False
d. In the female, the urogenital groove is open to the surface and forms
the vestibule
True False

Question 10: With respect to the development of the female gonadal

3
system

a. The primitive sex cords disintergrate


True False
b. The surface epithelium of the ovary continues to proliferate, producing
cortical cords
True False
c. The germ cells develop into oogonia
True False
d. The follicular cels are derived from the mesenchyme of the gonadal
ridge
True False

Question 11: In the fetal circulatrion

a. The ductus venosus delivers blood directly into the superior vena cava
True False
b. The umbilical artery returns blood from the placenta
True False
c. The ductus arteriosus carries blood to the lungs
True False
d. Blood returning from the lungs is 90% saturated with oxygen
True False

Question 12: The following are derived from the urogenital sinus

a. Paraurethral glands
True False
b. The greater vestibular glands
True False
c. Gartner?s duct
True False
d. Urachus
True False

Question 13: With respect to the development of the urinary tract

4
a. There is an increase in the number of nephrons in each kidney after
birth
True False
b. The mesonephric kidney develops in the pelvis
True False
c. The metanephric kidney develops in the pelvis and ascends into the
abdomen
True False
d. The ureters are endonermal in origin
True False

Question 14: With respect to the development of the urinary tract

a. The ureter develops as an outgrowth of the mesonephric duct close to


its entrance into the cloaca
True False
b. The ureter develops as an outgrowth of the paramesonephric duct
close to its entrance into the cloaca
True False
c. The ureteric bud gives rise to the renal collecting tubules
True False
d. The metanephric mesoderm gives rise to the excretory units of the
kidney
True False

Question 15: With respect to the development of the female gonadal


system

a. The paramesonephric duct opens into the intra-embryonic coelom


True False
b. The mesonephric duct disintergrates under influence of oestrogen
produced by the developing ovaries
True False
c. Mullerian inhibiting factor is produced by the developing ovary
True False
d. The infundibulo-pelvic ligament is the female equivalent of the
gubernaculum testis
True False

5
DEVELOPMENT OF THE PLACENTA *****

• Fetal weight is related to placental weight. The fetal:placental weight


ratio increases with gestation age. At term, fetal weight is 5-6x
placental weight. At 32 weeks gestation, fetal weight ~ 4x placental
weight

• During implantation, the outer layer of the blastocyst proliferates to


form the outer cell mass from which the trophoblast and the placenta
develop

• The trophoblast differentiate into an inner layer of large clear


mononuclear cells, the cytotrophoblast and an outer layer of multi-
nucleated cells, the syncytiotrophoblast which forms a true syncytium

• DNA synthesis and mitosis occurs only in the cytotrophoblast layer.


The syncytiotrophoblast is formed by fusion of cells from the
cytotrophoblast layer *

• Lacunae appear within the syncytiotrophoblast between days 10 - 13


post-ovulation and are the precursors of the intervillous space. The
lacunae are separated by columns of syncytiotrophoblast called
primary villous stems (These are not villi)

DEVELOPMENT OF THE VILLOUS TREE

• Primary villous stems become infiltrated by cytotrophoblasts between


days 13-21 post-ovulation

• Villous stems are subsequently infiltrated by extra-embryonic


mysenchyme which differentiates into fetal blood vessels

1
• The distal parts of the villous stems are not vascularised. Here,
cytotrophoblasts proliferate and spread laterally to form a
cytotrophoblastic shell, splitting the syncytiotrophoblast into a definitive
syncytiotrophoblast on the fetal side and the peripheral syncytium on
the decidual side which degenerates and is replaced by fibrinoid
material (Nitabuch's layer)

• Sprouts extend from primary villous stems, initially made up of


syncytiotrophoblast and then infiltrated by cytotrophoblast and
mesenchyme - these are primary stem villi and the placenta is a true
villous structure by day 21 of gestation. These villi grow and divide into
secondary, tertiary and terminal villi

• The villi oriented towards the uterine cavity degenerate between day
21 and the 4th month to form the chorion laeve. The overlying decidua
degenerates and the chorion laeve comes in contact with the deciduas
of the opposite uterine wall

• The rest of the villi form the chorion frondosum which develops into the
definitive placenta

• Division and modification of the villous tree continues until term. First
trimester villi are larger, have a complete layer of cytotrophoblasts and
have a loose mysenchymal core which is vascularised towards the end
of the first trimester

• At term, the villi are smaller, cytotrophoblasts are few in number, the
syncytiotrophoblast is irregularly thinned. Fetal vessels are sinusoidal
and occupy most of the villous core and lie close to the
syncytiotrophoblast, forming vasculusyncytial membranes which
maximise materno-fetal transfer.

• Sometimes, the syncytiotrophoblast nuclei appear in clusters called


syncytial knots - more common in placentas from IUGR / pre-eclamptic
pregnancies *
• Maternal blood is separated from fetal blood by the
syncytiotrophoblast and the fetal capillary endothelium *

2
TROPHOBLAST INVASION *****

• Extra-villous cytotrophoblast from the trophoblastic shell break through


the outer syncytiotrophoblast layer and invade the decidua

• Some of these cells invade the decidua (interstitial trophoblasts) and


fuse to form multinucleat giant cells

• Endovascular trophoblasts invade the lumen of spiral arteries,


destroying the muscular and elastic layers of the vessels, replacing
these with fibrinoid and replacing the vascular endothelium. The
vessels are converted into wide, low resistance vascular channels *

• This invasion is initially restricted to the intra-decidual portion of the


vessels and starts at 8-12 weeks. During the 4th month (16-18 weeks),
a second wave of invasion occurs, extending to involve the intra-
myometrial segments of the spiral arterioles *

• Extravillous trophoblasts invade through the decidua, followed by a


second wave of migration ~18 weeks gestation to invade the
myometrium. Some differentiate and fuse into multinucleated giant
cells within the decidua and myometrium *

• Trophoblast cells stain positively for cytokeratin while decidual stromal


cells are negative

• The basic structure of the placenta is formed by day 20 of pregnancy

3
PLACENTAL LOBES AND LOBULES

• LOBES: Placental septae develop during thr 3rd month as protrusions


of the basal plate into the intervillous space, dividing the maternal
surface of the placenta into 15-20 lobes

• Lobes are not structural or functional units

• LOBULES: Each placental lobule is derived from a single secondary


stem villus. A cotyledon is that part of the villous tree which has arisen
from a single primary stem villus and contains 2-5 lobules

• Each placental lobule is supplied by a single utero-placental artery

THE PLACENTAL BED

• Refers to the deciduas and myometrium directly underlying the


placenta

• The fetal component is made up of extra-villous trophoblasts

• The maternal component is made up of decidualised endometrial


stromal cells, macrophages and granular lymphocytes. Residual
endometrial glands are also present.

4
AMNIOTIC FLUID *****

· Initially produced by primitive cells around the amniotic vesicle

· Later formed from transudate from fetal skin and umbilical cord
and diffusion across the amniotic membrane *

· Fetal skin becomes keratinised in the second trimester and


amniotic fluid is mainly formed from fetal urine and lung secretions.
The term fetus passes 500-700ml urine per day *

· Fetal swallowing is a major route of amniotic fluid re-circulation


and begins at 12 weeks. At term. ~500ml amniotic fluid is
exchanged / 24h *

· Amniotic fluid volume = 50ml at 12 weeks, 150ml at 16 weeks


gestation and ~1000ml at term. Peak volume is at 32-36 weeks *

· Osmolarity: 275 mOsmol/l (lower than maternal or fetal),


decreases as pregnancy progresses *

· Cells: at term, contains fetal epithelial cells, amniocytes and


dermal fibroblasts. Epithelial cells and amniocytes grow poorly in
culture. Glial cells present if neural tube defect *

· Protein: concentration increases with gestation but plateaus after


30 weeks. Mainly albumin and globulins. Also contains AFP
(1/10TH concentration in fetal blood - rises until 12 weeks then
declines). Virtually no fibrinogen. *

· Urea, creatinine and urateconcentration increases with gestation

· Amino acids: concentration similar to that in maternal plasma

· Lipids: mainly free fatty acids. Also contains phospholipids,


cholesterol and lecithin (secreted by lungs during maturation) *

· Carbohydrates: mainly glucose; concentration ~ half that of


maternal serum

· PO2 = 2-15mmHg while PCO2 = 50-60mmHg

· pH = 7.0 (acidic relative to fetal blood)

1
· Bilirubin concentration falls in the third trimester (except in
haemolytic disease)*

2
PLACENTAL HORMONE PRODUCTION *****

OESTROGENS *

• Mainly oestriol, but also oestradiol and oestrone in smaller


amounts. Oestriol is produces from DHES-sulphate from fetal
zone of the fetal adrenal gland and also from the maternal
adrenals. Fetal DHEA-S is initially hydroxylated by the fetal
liver

PROGESTERONE *

• Produced from maternal cholesterol. Pregnenolone is also


produced and is converted by the fetal adrenals into androgens
which are then converted by the placenta into oestrogens.

HCG

• Produced by the syncytiotrophoblast. Cytotrophoblasts


produce HCG in-vitro

Human Placental Lactogen - has growth-hormone -like effects and


decreases insulin-sensitivity. *

Human chorionic thyrotropin and virtually all of the hypothalamic releasing


hormones

INHIBIN & ACTIVIN *

• Produced by the feto-placental unit (mainly by the ovary in


non-pregnant state)

• Inhibin-A levels peak in early pregnancy and rise again at


term and are increased in pre-eclampsia

• Activin levels increase with gestation age and a marked


increase occurs with the onset of labour and in pre-eclampsia

1
PLACENTAL TRANSFER

FACTORS AFFECTING PLACENTAL TRANSFER:

• Utero-placental blood flow - particularly important in the


exchange of lipid soluble solutes such as O2, CO2

• Feto-placental blood-flow

• Surface area of placenta

• Activity of placental transport systems in the microvillous and


basal plasma membranes of the syncytiotrophoblast

• Molecular weight and lipid solubility of solute

• Charge of solute

• Protein binding

• Placental metabolism of solute

• Concentration of solute in maternal and fetal plasma

PLACENTAL GAS TRANSFER

O2 / CO2 *****

• Small lipid soluble molecules - transfer is by simple diffusion


and rate of transfer is dependent on maternal / fetal
concentrations (partial pressure), rate of blood flow and surface
area

• The fetal O2 - Hb dissociation curve lies to the left of the


maternal curve - fetal red cells have greater affinity for O2 - see
respiratory physiology

• A hypoxic fetus develops both respiratory and metabolic


acidosis ? CO2 excretion is impaired and anaerobic glucose
metabolism results in lactate production

• During acidosis, the fetal O2 - Hb dissociation curve is


shifted to the right, decreasing oxygen binding for a given
partial pressure and releasing more O2 in fetal tissues. De-
oxygenated Hb becomes available and acts as a better buffer
than oxygenated Hb.

2
WATER TRANSFER

• 3-4L of water is exchanged per hour between the mother and


the fetus, placenta and amniotic fluid

• Net water accumulation by the fetus continues until delivery

• Water exchange is by perfusion transfer and osmosis

• Maternal dehydration / over-hydration will affect fetal water


accumulation, although there is a time lag

CARBOHYDRATES & AMINO ACIDS *****

• Glucose transport across the microvillous and basal plasma


membranes is by facilitated diffusion. *

• This is dependent on glucose concentration gradient and the


activity of transport proteins within the placenta. In theory,
facilitated diffusion can be saturated.

• In practice, the glucose transfer capacity (especially of the


microvillous plasma membrane) is so large that it would not
approach saturation under in-vivo conditions

• Fetal amino acid concentrations are generally higher than


maternal levels *

• Transfer of amino acids from the mother to the fetus is


therefore against a concentration gradient and energy (ATP) is
required for this process *

• This energy is provided by coupling the transfer of amino


acids (up a concentration gradient) to the transfer of sodium
ions (down a concentration gradient) - secondary active
transport. These are called Na+-dependent transporters. The
sodium concentration gradient is maintained by
Na+K+ATPase.
• Other amino acid transport systems are, however, Na+-
independent.

3
ELECTROLYTE TRANSFER *****

• Na+ concentration is low while K+ is high within the


syncytiotrophoblast

• These concentrations are maintained by Na+K+ATPase


which transfers 3Na+ out for 2K+ into the syncytiotrophoblast
across the microvillous plasma membrane *

• Total and ionised calcium concentration - higher in fetal than


maternal plasma. Calcium transfer across the microvillous
plasma membrane is magnesium dependent and dependent on
1,25-dihydroxycholecalciferol *

• Phosphate transfer is Na+-dependent

4
Molecule Placental transfer

Testosterone Minimal transfer - androgens aromatised by placenta. Very


high maternal androgen concentration may virilise female
fetus
Ca2+, Mg2+ Active transfer against concentration gradient

PTH, Calcitonin Not transferred

Vitamin D Good transfer

IgA Minimal passive transfer

IgG Good active and active transfer from 7 weeks gestation

IgM No transfer

Glucose Fascilitated diffusion - excellent transfer

Amino acids Active transport - excellent transfer

Free fatty acids Very limited transfer - essential fatty acids only

Ketone bodies Excellent transfer - diffusion

Insulin, glucagons No transfer

Thyroid hormone Poor transfer - diffusion

TRH Excellent transfer

Iodine and thioamides Excellent transfer

Cortisol & aldosterone Excellent transfer

ACTH No transfer

5
6
Question 1: With respect to the transfer of solutes across the placenta

a. Glucose transfer is by simple diffusion


True False
b. Amnio-acid transfer is by active transport
True False
c. Amino acid concentrations are lower in the fetal than in the maternal
plasma
True False
d. Glucose concentration is lower in the fetal than in maternal plasma
True False

Question 2: With respect to the structure of the placenta

a. Each placental lobule is formed from a single primary stem villus


True False
b. Each placental lobe is supplied by a single utero-placental artery
True False
c. Each cotyledon has 2-5 lobules
True False
d. Each placenta is divided into 15-20 lobes by septae which originate
from the chorionic plate
True False

Question 3: With respect to the transfer of water between the mother


and the fetus

a. 3-4L of water are exchanged between the maternal and fetal


compartments per 24h
True False
b. Water crosses the placenta by osmosis
True False
c. Net water accumulation by the fetus stops at 37 weeks gestation
True False
d. Maternal dehydration has no impact on net fetal water accumulation
True False

1
Question 4: With respect to the development of the placenta

a. Trophoblasts initially invade the uterine spiral arterioles at 16-18 weeks


gestation
True False
b. Villous cytotrophoblasts form a layer beneath the syncytiotrophoblast in
early pregnancy
True False
c. The syncytiotrophoblast is the only cellular layer between maternal and
fetal blood
True False
d. Syncytial knots in the placenta are composed of aggregates of
cytotrophoblasts
True False

Question 5: The placenta

a. Produces oestriol from cholesterol


True False
b. Produces progesterone from cholesterol
True False
c. Produces more oestradiol than oestriol
True False
d. Produces Inhibin-A and Activin
True False

Question 6: The composition of amniotic fluid varies as normal


pregnancy advances in the following way

a. Glucose concentration falls


True False
b. Uric acid concentration rises
True False
c. Oestriol concentration falls
True False
d. Osmolality falls
True False

2
Question 7: The human placenta

a. Has negligible oxygen requirement


True False
b. Has syncytiotrophoblast as the major component of trophoblast at
term
True False
c. Is haemochorial
True False
d. Has anastomoses between villous vessels
True False

Question 8: During the development of the placenta

a. Interstitial trophoblasts invade the spiral arteries


True False
b. Only the intra-decidual segment of spiral arteries are converted by
trophoblast invasion
True False
c. The endothelial lining of spiral arteries is replaced by fibrinoid material
True False
d. The elastic layer of spiral arteries remains intact after trophoblast
invasion
True False

Question 9: Amniotic fluid

a. Contains cells of maternal origin


True False
b. At term, is hyperosmolar compared to fetal plasma
True False
c. Contains bilirubin
True False
d. Contains phospholipids
True False

3
Question 10: Trophoblast

a. Develops from the blastocyst


True False
b. Secrete gonardotrophins
True False
c. Is physiologically invasive
True False
d. Forms part of the chorion
True False

Question 11: With respect to the transfer of electrolytes across the


placenta

a. Na+ is taken up across the microvillous plasma membrane through the


action of Na+K+ ATPase
True False
b. The concentration of Na+ is higher within the syncytiotrophoblast than
in maternal plasma
True False
c. The concentration of K+ is lower in maternal plasma than in the
syncytiotrophoblast
True False
d. Total calcium concentration is higher in fetal than in maternal plasma
True False

Question 12: Amniotic fluid

a. Has a protein content similar to maternal plasma


True False
b. Is mainly a filtrate of maternal plasma during the second half of
pregnancy
True False
c. Has the highest bilirubin concentration during the last trimester
True False
d. Concentration of alpha-fetoprotein increases with increasing gestation
age
True False

Question 13: In the human placenta

4
a. Intervillous spaces communicate directly with the uterine arteries
True False
b. Decidual cells are derived from endometrial stromal cells
True False
c. At term, each lobe represents a single primary stem villus
True False
d. Cytotrophoblasts are in direct contact with maternal blood
True False

Question 14: With respect to the transfer of electrolytes across the


placenta

a. Ionised calcium concentration is lower in fetal than in maternal plasma


True False
b. Uptake of calcium by the microvillous plasma membrane is magnesium
dependent
True False
c. Uptake of calcium by the microvillous plasma membrane is
independent of 1,25-dihydroxycholecalciferol
True False
d. Placental ion uptake is by facilitated diffusion
True False

Question 15: During fetal acidosis

a. The O2- Hb dissociation curve is shifted to the left


True False
b. More O2 becomes bound to Hb for a given partial pressure
True False
c. The buffering capacity of oxygenated Hb is better than that of
deoxygenated Hb
True False
d. Decreased oxygen supply from the utero-placental circulation results in
respiratory, but not metabolic acidosis
True False

5
AUTONOMIC NERVOUS SYSTEM
• Provides the efferent (motor) pathway linking areas of the brain
concerned with the regulation of the internal environment to specific
effectors such as blood vessels, glands and the heart

SYMPATHETIC DIVISION *****

• EFFERENT fibres originate in cells in the inter-mediolateral column of


T1 - L2/3 segments of the spinal cord - pre-ganglionic neurons *

• Axons of pre-ganglionic fibres emerge via the ventral root of the spinal
cord together with somatic motor fibres *

• Shortly after the dorsal and ventral roots of the spinal cord fuse,
sympathetic pre-ganglionic fibres leave the spinal nerve trunk and
travel to sympathetic ganglia via white rami communicantes *

• Sympathetic pre-ganglionic fibres synapse with post-ganglionic


neurons in the sympathetic ganglia *

• Fibres entering the sympathetic ganglia high up in the thorax may


travel up the sympathetic trunk to cervical ganglia where they synapse
with post-ganglionic neurons

• Pre-ganglionic fibres may pass through the sympathetic ganglia without


synapsing - these myelinated fibres form splanchnic nerves of which
there are three - greater splanchnic nerve (5th - 9th thoracic ganglia -
pierce the diaphragm and synapse in the celiac ganglion), lesser
splanchnic nerve (10thand 11th thoracic ganglia, pierce the diaphragm
and synapse with cells in the lower part of the celiac plexus) and lowest
splanchnic nerve (12th thoracic ganglion, may be absent, pierces the
diaphragm and synapses with cells in the renal plexus) *

• A few pre-ganglionic fibres travelling in the greater splanchnic nerve


synapse directly with cells in the adrenal medulla *

• Sympathetic post-ganglionic fibres travel to target organs via grey rami


communicantes and segmental spinal nerves *

• Sympathetic pre-ganglionic fibres may therefore terminate in the


ganglion of the same segment or pass to another ganglion in the
sympathetic chain or to pre-vertebral ganglia such as the celiac
ganglion *

• Sympathetic pre-ganglionic fibres are myelinated (white) while post-


ganglionic fibres are non-myelinated (grey) *

• With the exception of the cervical region, sympathetic ganglia are


distributed segmentally as far as the coccyx *

1
• The cervical sympathetic chain is represented by the superior, middle
and inferior cervical ganglia which supply the eyes, lacrimal, salivary
glands (superior), heart and respiratory tract (middle & inferior plus
upper 3-4 thoracic ganglia) *

• Sympathetic pre-ganglionic fibres to the abdominal organs form the


splanchnic nerves which are distributed to the celiac, superior and
inferior mesenteric plexuses *

• All pre-ganglionic fibres secrete acetylcholine *

• AFFERENT myelinated fibres travel from the viscera through the


sympathetic ganglia without synapsing, enter the spinal nerve via the
white rami communicantes and reach their cell bodies in the posterior
(dorsal) root ganglion of the corresponding spinal nerve. The central
axon then enters the spinal cord and may form the afferent component
of a local reflex arc or pass to higher autonomic centres in the brain *

CHROMAFFIN CELLS *****

• Derived embryologically from the neuro-ectoderm (neural crest). During


development, chromaffin cells are widely scattered within the embryo but in the adult
can only be found in the adrenal medulla*

• Innervated by pre-ganglionic fibres from the thoracic spinal cord via the
splanchnic nerves. These fibres synapse directly with the chromaffin cells which are
homologous to sympathetic post-ganglionic neurons and can generate action
potential *

• Sympathetic pre-ganglionic fibres are myelinated while post-ganglionic fibres


are non-myelinated. All pre-ganglionic fibres secrete acetylcholine *

Synthesis of adrenaline in adrenal medulla *****

• Tyrosine converted to dihydrophenylalanine (DOPA) by tyrosine


hydroxylase *

• DOPA converted to dopamine by DOPA decarboxylase *

• Dopamine converted to noradrenaline by dopamine-beta-


hydroxylase *

• Noradrenaline converted to adrenaline by phenylethanolamine-N-


methyltransferase

2
ORGAN SYMPATHETIC PARASYMPATHETIC

Eyes - pupils Dilatation Constriction / accomodation

Lachrymal glands No effect / vasoconstriction Secretion

Heart Tachycardia & increased Bradicardia, no effect on force of


force of contraction contraction

Salivary gland Secretion of viscous fluid, Copious secretion of saliva,


vasoconstriction vasodilatation

Blood vessels Vasoconstriction. Vasodilatation in some exocrine


Vasodilatation in skeletal glands and external genitalia
muscle

Lungs Bronchodilatation Bronchoconstriction & increased


secretion

Adrenal medulla Secretion of adrenaline & No innervation


noradrenaline

GI Tract Vasoconstriction, Increased motility and secretion;


constriction of sphincters, relaxation of sphincters
decreased motility and
secretion
Kidneys Vasoconstriction & No effect
decreased urine out-put

Bladder Inhibit micturiction Initiate micturiction

Sweat glands Secretion by eccrine glands No innervation

Hair follicle Piloerection No innervation

Genitalia Ejaculation Erection

3
PELVIC SPLANCHNIC NERVES

• There are thoracic, lumbar, sacral, and pelvic splanchnic nerves.


"Splanchnic" refers to nerves that supply viscera. *

• Thoracic, lumbar and sacral splanchnic nerves emerge from


sympathetic ganglia and carry sympathetic fibers*

• Pelvic splanchnic nerves arise from the ventral (anterior) primary


rami of S2, 3, 4. *

• These are the ways in which parasympathetic neurons reach the


hypogastric plexus, and therefore the pelvic viscera and distal colon.*

• The parasympathetic part of the autonomic nervous system is the


"craniosacral" part. Parasympathetic innervation to most of the gut
comes from the "cranio-" half of that, i.e., the vagus nerve. The rest, to
colon distal to the splenic flexure and to pelvic viscera, is from the "-
sacral" half, via the pelvic splanchnic nerves.

• Intermingle with sympathetic nerves in the hypogastric plexus

• Pelvic splanchnic nerves innervate the kidneys, urinary bladder,


distal portions of the large intestine and the sex organs. Contain
EFFERENT fibres

• Stimulate smooth muscle activity in the distal large bowel

• Contract the detrusor and relax the urethral sphincter

• Vasodilatation of genital / reproductive organs

AFFERENT / EFFERENT FIBRES *****

• Each spinal nerve is connected to the spinal cord by two roots -


anterior (ventral) and posterior (dorsal) roots

• The anterior root contains nerves carrying impulses AWAY FROM the
CNS - EFFERENT fibres. Efferent fibres to skeletal muscles are called
motor fibres - cell bodies located in the anterior grey horn of the spinal
cord *

• Posterior roots contain fibres carrying impulses TO the CNS -


AFFERENT fibres - conveying information about sensation of touch,
pain, temperature, vibration - SENSORY fibres. Cell bodies are located

4
in the posterior / dorsal root ganglion with a peripheral axon to the
viscera and a central axon to the CNS *

• At each intervertebral foramen, the anterior and posterior roots unite to


form a spinal nerve *

• On emerging from the foramen, each spinal nerve divides into a large
anterior ramus and a smaller posterior ramus which passes posteriorly
to supply the skin and muscle of the back. The anterior ramus supplies
the antero-lateral body wall and limbs *

• At the root of the limbs, the anterior rami come together to form
complex nerve plexuses

REGULATION OF CATECHOLAMINE RELEASE *****

Basal secretion is very small

• Secretion mediated directly by the activity of the splanchnic nerves


and the gland becomes non-functional if these nerves are cut

• Secretion increased in stressful situations: *


1) Exercise
2) Hypoglycaemia
3) Cold
4) Haemorrhage
5) Hypotension
6) Emotional situations such as fear, anger, pain, sexual arousal
7) Glucocorticoids via cortical sinusoids stimulate conversion of noadrenaline
to adrenaline
8) Fetal adrenal responds directly to hypoxia
• Phaeochromocytoma - tumour of the chromaffin cells is characterised by
increased catecholamine production*
• Results in episodic or sustained hypertension, tremor, hyperglycaemia,
anxiety, arrhythmias, sweating and a raised metabolic rate *

5
Question 1: The parasympathetic nervous system supplies

a. Dilator fibres to the bronchioles


True False
b. Inhibitory fibres to the myocardium
True False
c. Dilator fibres to the sphincter pupillae
True False
d. Constrictor fibres to the small intestine
True False

Question 2: The pelvic splanchnic nerves

a. Are derived from posterior rami of sacral spinal nerves


True False
b. Supply the ascending colon with motor fibres
True False
c. Supply the uterus with parasympathetic fibres
True False
d. Contain parasympathetic fibres
True False

Question 3: The pelvic splanchnic nerves

a. Supply afferent fibres


True False
b. Intermingle with branches of the sympathetic pelvic plexus
True False
c. Are pre-ganglionic fibres
True False
d. Supply the bladder sphincter with motor fibres
True False

1
Question 4: Chromaffin cells

a. Are present in the celiac ganglion


True False
b. Are derived from neuro-ectoderm
True False
c. Are present in the adrenal cortex
True False
d. Are present in the adrenal medulla
True False

Question 5: The sympathetic ganglia

a. Are arranged segmentally in the cervical sympathetic chain


True False
b. Contain cholinergic pre-ganglionic nerve terminals
True False
c. Contain adrenergic pre-ganglionic nerve terminals
True False
d. Receive non-myelinated pre-ganglionic fibres through the grey rami
communicantes
True False

Question 6: The sympathetic nervous system supplies

a. Inhibitory fibres to the detrusor muscle


True False
b. Constrictor fibres to blood vessels in skeletal muscle
True False
c. Constrictor fibres to the sphincter of Oddi
True False
d. Dilator fibres to blood vessels of the external genitalia
True False

2
Question 7: The sympathetic nervous system supplies

a. Dilator fibres to the bronchial tree


True False
b. Constrictor fibres to the muscle of the small intestine
True False
c. Inhibitory fibres to the detrusor muscle
True False
d. Inhibitory fibres to the detrusor muscle
True False

Question 8: The pelvic splanchnic nerves

a. Are motor to the internal sphincter of the bladder


True False
b. Contain afferent fibres for the ovary
True False
c. Conduct pain from the body of the uterus
True False
d. Have noradrenaline as the neurotransmitter
True False

Question 9: The release of catecholamines from the adrenal medulla

a. Increases during sleep in healthy individuals


True False
b. Increases when the nerves to the adrenal gland are stimulated
True False
c. Increases following an increase in blood sugar
True False
d. Increases during acute haemorrhage
True False

3
Question 10: The pelvic splanchnic nerves

a. Supply the uterus with vasodilator fibres


True False
b. Contain sympathetic fibres
True False
c. Contribute to the inferior hypogastric plexus
True False
d. Contribute to the inferior mesenteric plexus
True False

Question 11: With respect to the nervous system

a. Efferent fibres transmit impulses away from the CNS


True False
b. Each spinal nerve has a larger posterior and a smaller anterior ramus
True False
c. The anterior (ventral) root of the spinal cord contains efferent fibres
True False
d. The posterior (dorsal) root of the spinal cord contains sensory fibres
True False

Question 12: Chromaffin cells

a. Can decarboxylate amino acids


True False
b. Are innervated by pre-ganglionic sympathetic fibres
True False
c. Can generate action potential
True False
d. Are innervated by non-myelinated nerve fibres
True False

Question 13: The sympathetic nervous system supplies

4
a. Dilator fibres to the bronchial tree
True False
b. Constrictor fibres to the muscle of the small intestine
True False
c. Inhibitory fibres to the detrusor muscle
True False
d. Constrictor fibres to the coronary arteries
True False

Question 14: The sympathetic ganglia

a. Are located within the walls of abdominal viscera


True False
b. Contain afferent myelinated fibres
True False
c. Contain the cell bodies of pre-ganglionic neurons
True False
d. Contain the cell bodies of post-ganglionic neurons
True False

Question 15: Parasympathetic stimulation results in

a. Ejaculation
True False
b. Increased secretion by eccrine sweat glands
True False
c. Viscous secretion by the salivary glands
True False
d. Renal vasoconstriction and decreased urine out-put
True False

5
EMBRYOLOGY

1 Concerning the embryology of the urinary tract:

A The detrusor has a mesodermal origin T

B The urogenital sinus is derived from the cloaca T

C The allantois gives origin to the lateral umbilical ligaments. F

D The metanephric ducts arise from the mesonephric ducts F

E The mesonephric duct remnants form the epoophoron in the T


adult female

Comments 2:
The urinary system develops from the intermediate mesoderm.
During development of the fetus there are 3 overlapping kidney
systems – the pro, meso, and metanephric systems. The
metanephros forms the permanent kidney. Bowman's capsule
and the glomerulus develop together from the mesonephros.

C. The obliterated allantois forms the urachus or median


umbilical ligament *

2 The following tissues are paired with the appropriate primary


germ cell layer of origin:

A Mammary duct epithelium: ectoderm. T

B Epithelium of the tongue: mesoderm. F

C Pineal gland: ectoderm. T


D Ovarian stroma: mesoderm. T

E Endometrium: mesoderm. F
Comments 2:
The mammary gland is derived from the ectoderm as is the tongue epithelium and the
pineal gland. The tongue muscle is derived from mesoderm origin. The mesoderm of the
urogental ridges gives rise to the ovarian stroma, the endometrium is derived from the
endoderm.

3 The following are derived from the unogental sinus:

A The bladder trigone. F

B The ureters. F

C The female urethra. T

D Greater vestibular glands. T

E Paraurethral glands. T
Comment:

The trogone is formed from the mesoderm of the mesonephric ducts *.

Ureters - develop from the ureteric bud, an outgrowth of the mesonephric duct
close to its opening into the cloaca. Grows into the metanephric mesoderm,
successive divisions forming the renal pelvis, major and minor calyces and
collecting tubules *

4 The urogenital sinus in female gives rise to the following:

A Ureter. F

B Paraurethral glands. T

C Bartholins gland. T

D Urachus. F

E Gartners duct. F
5 In the ovary

A Primordial germ cells are formed F

B Primary oocytes have completed the first mitotic division by F


birth

C The majority of primary oocytes atretic by puberty T

D Fewer than ten follicles start the process of the antrum F


formation in each ovarian cycle

E The second polar body is formed at ovulation F

6 The normal human preovulatory follicle:

A Reaches a diameter of 40-60 mm before rupture. F

B Requires luteinising hormone for development. T

C Is lined by theca interna cells on its inner surface. F

D Contains a zona-denuded oocyte. F

E Contains fluid rich in progesterone. T

Comment:

Diameter from 35-120micro m.

The theca and granulose cell around the follicle.

7 During development of the female reproductive system:

A Primordial germ cells arise in the yolk sac. T

B Ovarian development is dependent upon oestrogen activity. F

C The paramesophric ducts give rise to the cervix. T


D The greater vestibular glands arise from the urogenital sinus. T

E Differentiation of the external genetalia is dependent upon F


ovarian activity.

8 In the human testis

A Secondary spermatocytes contain 23 chromosomes T

B One secondary spermatocyte form two spermatids F

C The seminiferous tubules contain motile spermatozoa F

D The process of spermatogenesis takes 34 days. F

E Inhibin is produced by primary spermatocytes F

9 The fetal testes

A Are morphologically distinguishable 4 weeks after conception F

B Contain cells which have migrated from the wall of the yolk sac T

C Are usually intra-abdominal 12 weeks after conception T

D Are necessary for the persistence of the mesonephric (wolfian T


ducts).

E Secrete androgens T

10 In spermatogenesis:

A Primary spermatocytes undergo reduction division. T

B Each primary spermatocyte ultimately gives rise to four F


spermatids.

C The whole process of spermatogenesis in man takes 7-6 days. F

D Grossly abnormal spermatozoa may be found in fertile semen. T

E Spermatids are haploid. T

11 Concerning oogenesis:

A The primary oogonia undergo mitotic divisions prior to meiosis. T

B The primary oocytes enter meiosis at puberty. F

C At birth, primary oocytes have completed the first meiotic F


division.

D DNA synthesis does not take place as part of the second meiotic T
division.

E Secondary oocytes complete the second meiotic division at F


ovulation.

12 Following fertilization in the human

A The first polar body is formed. F

B The firat cleavage division occurs within 12 hours. F

C The zona pellucida is shed at the second cleavage division. F

D Cleavage divisions are mitotic. T

E The mean cell volume decreases with each cleavage division. T


13 Concerning gonadal development:

A The histological appearance of the primitive gonad is similar in T


both sexes until 42 days after fertilization.

B The ovary develops in the medulla of the primitive gonad. F

C The histo-differentiation of the testis begins later than that of F


the ovary.

D Primary sex cells (gonocytes) have a haploid number of F


chromosomes.

E Mitosis in oogonia is not completed by the end of the first year F


of life.

Comments 2:
The appearance of the primitive gonad is similar in both sexes
until 42 days after fertilization when seminiferous
differentiation occurs. At 10 weeks conception there is meiotic
entry of oocytes in the medulla of the primitive gonad of the
developing foetus. Gonocytes are diploid and only become
haploid in the gonad via meiosis. Mitosis in oogonia is
completed 7th month of fetal life. The ovary develops from the
primitive cortex not medulla.

14 The following are derivatives of the mesonephros:

A Appendix of the testis F

B Efferent ductules of the testis T

C Gartner's duct cyst T

D Gubernaculums testis T

E Prostatic utricle F
15 In the fetal cardiovascular system.

A The heart arises from endoderm F

B The heart is formed by fusion of endocardial tubes T

C Cardiac pulsation is present by the day 30th after fertilization T

D Oxygenated blood is transferred to the left atrium through the T


foramen ovale

E The ductus arteriosus closes during the last 4 weeks of F


pregnancy.

Comments 2:
The fetal heart arises from cardiogenic mesoderm. and is
formed by fusion of endocardial tubes. Cardiac pulsation is
present by the 4th week after fertilisation. Oxygenated blood
returning from the placenta goes from the right atrium to the
left atrium (via the foramen ovale). The ductus arteriosus closes
at birth.

16 In the development of the heart:

A The primitive heart consists of five parts. F

B The septa are formed within 6 weeks of conception. T

C The septum secundum forms a complete partition in the atrial F


cavity.

D Obliteration of the right atrioventricular orifice is frequently T


associated with a ventricular septa defect.

E Anatomical closure of the ductus arteriosus occurs within 1 hour F


of birth.
17 Concerning the development of the nervous system:

A The posterior neuropore closes before the anterior neuropore. F

B At 3 months of age, the spinal cord extends the length of the F


vertebral canal.

C At birth, the spinal canal terminates at L3. T

D At birth, the Dura mater extends as far as L3. F

E Myelin formation in the spinal cord is completed before birth. F

Comments 2:
Development of the central nervous system starts at the
beginning of the 3rd week. The neural plate is made of
thickened ectoderm. The anterior neuropore closes at the 18-20
somite stage on day 25. The posterior neuropore closes on day
27.

18 The mesoderm gives rise to:

A Straiated muscle. T

B Blood. T

C Peritoneum. T

D Transitional epithelium of the bladder. F

E Ovarian stroma. T

19 The pineal gland

A Is situated at the anterior end third ventricle F


B Is innervated by the parasympathetic nervous system F

C Produces melatonin T

D May be calcified in the adult T

E Is most active during daylight F

20 The umbilical cord

A Contains mid-gut during embryonic development T

B Is covered by amnion T

C Consists chiefly of fetal endodermal cells F

D Contains two veins and an artery F

E Is approximately 50 cm long at term T

21 Concerning embryological development:

A The amnion has an endodermal origin F

B Uterine epithelium is developed from the paramesonephric T


ducts

C The hymen develops at the junction of the synovaginal bulbs T


and the urogenital sinus

D The round ligament of the uterus is derived from the T


gubernaculum

E The adrenal cortex is derived from neural crest cells F


22 In the pituitary gland:

A The anterior lobe is smaller than the posterior lobe. F

B The posterior lobe is ectodermal in origin. T

C The acidophil cells produce oxytocin. F

D The basophil cells produce growth hormone. F

E The blood supply is derived from the internal carotid artery. T

23 The following are present in the developing umbilical cord:

A Allantois T

B Extra-embryonic mesoderm. T

C Intestinal loops. T

D Two umbilical arteries. T

E Yolk sac stalk. T


Comment:

The developing umbilical cord contains the following *

• Yolk sac stalk

• Two umbilical arteries and one umbilical vein

• The remnant of the allantois

• Small intestinal loops

• Extra-embryonic mesoderm

24 In a neonate at birth:

A Closure of the ductus arteriosus is due to increased arterial T


oxygen tension.
B The ductus arteriosus closes before the lungs are expanded. F

C Pressure in the inferior vena cava falls. T

D The foramen ovale seals immediately. F

E Closure of the foramen ovale is due to increased carbon dioxide F


tension in venous blood.

25 In the vulva:

A Sebaceous glands are not present in the labia minora. F

B Apocrine glands are present in the labia majora. T

C Both surfaces of the hymen are covered by keratinized F


epithelium.

D The bartholin's glands are mucus-secreting. T

E The clitoris is covered by columnar epithelium. F

26 After birth

A The allantois forms the median umbilical ligament. T

B The umbilical vein forms the medial umbilical ligament F

C The umbilical artery form the superior vesical artery. T

D The ductus venosus forms the ligamentum teres. F

E The ductus arteriosus forms the arch of the aorta. F

27 The seminal vesicles


A Contain spermatids F

B Secrete hyaluronidase. F

C Secrete acid phosphatase. F

D Secrete fructose T

E Secrete prostaglandins. T

28 Trophoblast

A Develops from the blastocyst. T

B Gives rise to the fetal blood vessels in the placenta. T

C Enters the maternal circulation during normal pregnancy. T

D Replaces endothelium of pregnant spiral arterioles. T

E Is genetically identical to deciduas. F

Comments 2:
Trophoblast - forms the outer layer of the blastocyst, composed
of mitotically active mononucleotide cells called cytotrophoblast
cells. Cytotrophoblast cells can fuse to form large
multinucleated masses called syncytial trophoblast (mitotically
inactive). Both types of cells contribute to the formation of the
placenta. Gives rise to the fetal blood vessels in the placenta.
Small clumps of trophoblast can break away from the villi and
enter the maternal circulation by way of the uterine veins.
Continued invasion of the endometrium by trophoblast cells
establishes direct connections between maternal arteries and
veins and the placental intervillous space. Some cytotrophoblast
cells migrate and penetrate into spiral arteries. The developing
fetus is maintained by a placenta which develops from fetal and
maternal tissues. The fetal portion, formed by the chorion, and
the maternal portion, formed by the decidua basalis – therefore
I would expect them Not to be genetically identical.

29 The ampulla of the fallopian tube

A Is the longest portion of the tube. T

B Is developed from the paramesonephric duct. T

C Has complex folding of the mucosa. T

D Is lined exclusively by ciliated columnar epithelium. F

E Has an internal longitudinal muscle coat. F

30 Concerning histological constituents:

A The round ligament contains smooth muscle fibres. T

B The bladder epithelium has no mucous glands. T

C The whole of the urethra is lined by squamous epithelium. F

D The sacroiliac joint has synovial membrane. T

E Skene's glands are present in the clitoris. F


LEUTENISING HORMONE (LH)

· Glycoprotein produced by the anterior pituitary. Protein hormones are not


significantly protein bound

· Acts via a receptor on the cell membrane

· Has an alpha and a beta subunit

· Concentrations are low in childhood, increase during puberty and are


elevated after the menopause

· Concentrations begin to rise within a few days of the onset of menstruation


but fall during the latter part of the follicular phase (negative feed-back from
rising oestrogen concentrations)

· There is a pre-ovulatory LH peak occurring about 18h before ovulation.


Without the LH surge, ovulation does not occur

· Stimulates the synthesis of androstendione and testosterone by the theca


cells which are converted to oestrogen by the granulosa cells

· Stimulates progesterone production by the granulosa cells and later the


conversion of granulosa into lutein cells

· FSH and oestrogen induce the expression of LH receptors by granulosa


cells

Secretion is regulated by:

1) GnRH from the anterior pituitary

2) Oestrogen from the ovary - initially inhibits LH secretion but induces the
pre-ovulatory surge

3) Inhibin - mainly inhibits FSH, but also LH secretion

4) Progesterone - in high concentrations inhibit LH secretion

FOLLICLE-STIMULATING HORMONE (FSH)

· Gonadotropin - produced by the anterior pituitary gland. Glycoprotein

· Has an alpha and a beta sub-unit

· Functions via a receptor on the cell membrane

1
· Main function is the stimulation of the growth of 6-12 primary follicles per
month

· Concentartions are low during childhood, increase during puberty and are
elevated after the menopause

· Concentrations begin to rise within a few days of the onset of menstruation


and then fall as the oestrogen levels rise. There is a pre-ovulatory surge in
FSH concentration

· Stimulates granulosa cell proliferation and also the ovarian stroma to form
the theca interna and externa of the vesicular follicle

Production is regulated by:

1) GnRH: from the hypothalamus - stimulates FSH production

2) Oestrogen from the ovary - negative feed-back to inhibit pituitary FSH


production

3) Inhibin: from the ovary - inhibits FSH production

4) Activin: from the ovary - stimulates FSH production

5) High levels of progesterone inhibit FSH production

GONADOTROPIN RELEASING HORMONE

· Decapeptide produced by the arcuate and preoptic nuclei of the


hypothalamus and stored in the median eminence. Synthesised as a
preprohormone

· Pulsatile release under influence of higher centres: 8-10 pulses per day in
the adult male; pulsatility in the female is dependent on the menstrual cycle

· Acts via a cell-surfece receptor

· Stimulates LH and FSH release from the anterior pituitary

· Sustained high levels cause down-regulation of pituitary GnRH receptors


and androgen / oestrogen production by the gonad becomes suppressed

· MENOPAUSE

· Average age 50.8 years

2
· Caused by ‘burn-out’ of the ovarian follicles - insufficient to produce enough
oestrogen to suppress FSH secretion or to induce LH surge

· Oestrogen and progesterone levels fall

· FSH & LH concentrations are elevated which suppress GnRH secretion

PUBERTY

· Initiated by maturation in higher centres which result in pulsatile release of


GnRH by the hypothalamus

· Prior to the onset of puberty, the hypothalamus is capable of secreting


GnRH and the pituitary and ovaries are capable of a response

· Oestrogen, progesterone, LH & FSH levels are low prior to puberty.

3
INHIBIN

• Family of peptides produced by the ovary and testis in addition to


other peripheral tissues

• Two sub-units: alpha and beta. Two types of inhibin (A & B) have
been identified. Have identical alpha sub-unit but different beta sub-
units

• Secretion by the granulosa cells is stimulated by FSH. Inhibin in turn


inhibits pituitary FSH production

• GnRH and epidermal growth factor inhibit FSH stimulated inhibin


production

• Insulin-like growth factor I enhances inhibin production

• Released in a pulsatile fashion with periodicity of 60-70 minutes

• FSH stimulates inhibin B secretion - levels rise in the early follicular


phase, peak in the mid-follicular phase and than fall to reach a nadir in
the mid-luteal phase

• There is a post-ovulatory inhibin-B peak due to release from the


ruptured follicle

• Inhibin-A concentration rises in the late follicular phase and peaks in


the mid-luteal phase

• Produced by the Sertoli calls in the male, facilitates LH stimulation of


androgen synthesis and inhibits proliferation of spermatogonia

ACTIVIN

• Member of family of peptides including Inhibin, Mullerian Inhibitory


Factor and Transforming growth factor - Beta

• Has two sub-units which are identical to the beta sub-units if Inhibin

• Increases FSH binding to gramulosa cells, augmenting FSH

stimulation and inhibin production

• Stimulates FSH release by the anterior pituitary and increases GnRH


receptor number

1
• Suppresses androgen production in the ovary / testis

• Both Inhibin A and Activin A are potent stimulators of the in-vitro


maturation of oocytes

• The anterior pituitary expresses inhibin / activin sub-units and locally


produced activin B augments FSH secretion

• Circulating levels increase in the late luteal phase and peak during
menstruation

FOLLISTATIN

• Peptide produced by several pituitary cells and also by the granulosa


cells

• Main function is inhibition of FSH synthesis and secretion and


inhibition of the FSH response to GnRH

• Production is stimulated by activin and inhibited by inhibin

2
FOLLICULAR DEVELOPMENT AND OVULATION *****

• At birth, the 2 million – ova - are surrounded by a layer of granulosa


cells forming the primordial follicle. These degenerate and by puberty,
only 400,000 are left.

PRIMORDIAL TO ANTRAL STAGES

• The ovum increases in size from < 0.5mm to 10-20mm and the layer of
granulosa cells thickens to form the primary follicle - this may occur
without gonadotropin stimulation

• The oocyte secretes glycoproteins which condense around it to form


the zona pellucida which separates the oocyte from the granulosa
cells *

• During the first few days after the onset of menstruation, the FSH and
later LH concentrations begin to rise. FSH stimulates the proliferation
of granulosa cells in 6-12 primary follicles. Stromal proliferation also
occurs to form the theca interna and externa

• The theca externa forms the capsule of the developing follicle while the
theca interna forms an inner glandular and highly vascular layer

• Granulosa cells secrete follicular fluid causing an antrum to appear


within the mass of granulosa cells - this marks the beginning of the
antral phase of development. The follicular antrum increases in size
and the oocyte becomes surrounded by a dense mass of granulosa
cells- the cumulus oophorus

1
• The antral follicle secretes increasing amounts of steroids - mainly 17-
beta oestradiol and oestrone (granulosa cells - produced de novo or by
aromatization of androgens from thecal cells). The main androgens
produced are testosterone and androstendione (thecal cells) *

• Activin suppresses androgen secretion but stimulates aromatizing


capacity of granulosa cells. Inhibin stimulates androgen secretion but
reduces aromatizing capacity of granulosa cells

• LH has a synergistic effect with FSH in stimulating the growth of


granulosa cells. Granulosa cells secrete oestrogen which stimulated
the expression of FSH receptors thereby increasing sensitivity to FSH.

OVULATION *****

• A single dominant follicle is selected within 7 days of the onset of


menstruation. The remainder undergo atresia and apoptosis

• An LH surge is essential for ovulation and occurs about 18h before


ovulation which typically occurs on day 14 of a 28 day cycle

• The follicle grows to a diameter of up to 25mm before ovulation *

• Within 2h of the LH surge, there is a transient increase in oestrogen


and androgen out-put from the follicle followed by a decline. The outer
granulosa cells begin secretion of progesterone *

• Proteolytic enzymes such as plasminogen activator and renin play a


role in ovulation

• The released oocyte is surrounded by a layer of granulosa cells which


form the cumulus oophorus. Meiosis I is completed at ovulation and the
secondary oocyte is released
• The oocyte enters meiosis II which arrests at metaphase

2
CORPUS LUTEUM *****

• Formed mainly from granulosa cells after ovulation

• The granulosa cells enlarge, develop an extensive smooth


endoplasmic reticulum and become filled with lipid inclusions giving
them a yellowish appearance (lutenisation) and secrete increasing
amounts of progesterone*

• Thecal cells form smaller lutein cells which produce progesterone and
androgens

• About 15 - 30mm in diameter by 8 days after ovulation and is functional


for 14 days after which it involutes (luteolysis) into the corpus albicals.
If fertilisation occurs, the corpus luteum persists under stimulation of
HCG produced by the placenta

• Secretes oestrogens, 17-alpha-hydroxyprogesterone and progesterone


in addition to androgens, inhibin, relaxin and oxytocin *

• Percentages of the total cell number were as follows: large luteal cells -
4.2%; small luteal cells - 18.5%; fibroblasts - 16.8%; endothelial cells
and pericytes - 52.6%. The remaining 7.9% comprised other
miscellaneous cell types including macrophages *

• The corpus luteum survives for ~14 days in a woman with a regular 28
day cycle

3
ENDOMETRIAL CYCLE

PROLIFERATIVE PHASE

• Under influence of oestrogen

• Rapid proliferation of stromal and epithelial cells with mitotic figures.


Endometrium is re-epithelialised within 3-7 days of the onset of
menstruation

• Elongation of spiral arteries. Endometrium is 3-4mm thick at ovulation

• Cervical mucus becomes copious, watery and more elastic with ferning
when dried. Easily penetrated by sperm

• Vaginal epithelial cells accumulate glycogen, become large and


cornified and their nuclei shrink

SECRETORY PHASE

• Under influence of oestrogen and progesterone from the corpus luteum

• Endometrial glands become tortuous and accumulate glycogen and


lipid vacuoles which migrate towards the lumen of the glands. Spiral
arteries become elongated and coiled

• Stroma becomes oedematous

4
• Endometrium is 5-6mm thick at the end of the secretory phase

• Percentage of cornified cells in the vaginal epithelium falls

• Cervical mucus becomes thick, viscous and less penetrable to sperm

MENSTRUAL PHASE

• Initiated by a sudden fall in oestrogen and progesterone concentrations

• There is involution followed by vasospasm in the spiral arterioles 24h


before the onset of menstruation

• There is necrosis and shedding of the superficial layers of the


endometrium.

• On average, 35ml of blood are lost per menstrual cycle.

• Menstrual blood does not clot because of the presence of fibrinolysin.

• Clotting indicates excessive bleeding.

• Following menstruation, only a thin layer of stroma and epithelial cells


in the crypts are left.

5
6
SERTOLI CELLS

• Large cells of the germinal epithelium - provide the special


environment within which spermatogenesis occurs. Activity is
dependent on FSH

• Secrete MIF during fetal life - causes regression of Mullerian duct

• Secrete oestradiol - produced from testosterone and required for


spermatogenesis

• Secrete inhibin - feed-back inhibitory effect on anterior pituitary LH &


FSH secretion

• Produce androgen binding protein - binds testosterone,


dihydrotestosterone and oestradiol with high affinity and concentrate
then within the cell

HORMONES AND SPERMATOGENESIS

• Testosterone - produced by Leydig cells - essential for


spermatogenesis

• LH & FSH - LH stimulates Leydig cells to produce testosterone while


FSH stimulates sertoli cells

• Oestrogens - produced by sertoli cells from testosterone and


essential for spermiation, the process of converting spermatides to
spermatozoa

• Prolactin at normal concentrations stimulates testosterone production


while high levels are inhibitory

1
SEMINAL FLUID CONTENT - Produced by the seminal vesicles.

• Fructose - nutrient for spermatozoa

• Prostaglandins - interact with cervical mucus and may stimulate


reverse peristalsis

• Fibrinogen

• Also contain LH & FSH, calcium and zinc, prolactin, testosterone,


oestradiol, inhibin, oxytocin and endorphins

PROSTATE SECRETION

• Alkaline secretion containing citric acid, calcium, alkaline


phosphatase, profibrinolysin - essential for neutralising acidic fluid from
vas deferens and vagina

• Sperm do not become optimally motile until surrounding pH rises to


6-6.5

NORMAL SEMEN

• Volume > 2ml

• Sperm concentration > 20million/ml

• Motility - >50% with forward progression or >25% with rapid


progression within 60minutes of ejaculation

2
• Fewer than 1million white blood cells /ml

• pH ~7.5

• Undergoes liquefaction within 15-30 minutes of ejaculation

• Spermatozoa survive for 24-48h in the female genital tract

MALE SEXUAL ACT

• Erection - parasympathetic impulses from nervi erigentes (S23&4)


causing arterial dilatation in the erectile tissue

• Lubrication - release of secretions from the urethral and


bulbourethral glands - parasympathetic

• Emission and ejaculation - sympathetic impulses leave the spinal


cord at L1&2 and reach the genital organs through the hypogastric
plexus

TESTOSTERONE

• Synthesised from cholesterole, mainly by the Leydig cells of the


testis, but also in small amounts by the adrenal cortex and ovary

• Converted to oestrogens by the sertoli cells and also in adipose


tissue and liver

• Most actions of testosterone are dependent on its conversion to


dihydrotestosterone by the action of 5-alpha reductase

3
• Production is mainly under regulation of pituitary LH. During fetal life
in the male, adult concentrations are attained. Levels fall after birth but
rise sharply during puberty

• Circulates bound to sex-hormone binding globulin and albumin. Only


1-2% of testosterone is free and metabolically active

• Metabolised in the liver, conjugated to glucuronide or sulphate and


excreted in urine or bile

• Acts via an intracellular receptor which regulates DNA transcription

ANDROGEN ACTIONS - TESTOSTERONE (T) /


DIHYDROTESTOSTERONE (DHT)

FETUS

• Stimulates development of the mesonephric duct - vas deferens and


epididymis - T

• Stimulates development of the external genitalia - DHT

• Fetal testosterone secretion is stimulated by placental HCG

• Stimulates descent of the testes

• HAIR - development of facial hair and male pattern hair distribution -


DHT

4
• SKIN - increased sebaceous gland secretion - DHT

• MUSCLE - skeletal muscle hyperplasia - increased muscle mass

• RED CELLS - increased erythropoiesis - T

• LIVER - decreased synthesis of binding globulins, including sex


hormone binding globulin, increased LDL & VLDL with decreased HDL.
Anabolic and causes positive nitrogen balance - T

• TESTIS - stimulates spermatogenesis - conversion to DHT is not


essential

• GROWTH - stimulates pubertal growth spurt; stimulates closure of


epiphyses

• Widening of the larynx with thickening of the vocal cords

5
OESTROGENS *****

• Steroid hormones synthesised from cholesterol and secreted mainly by


the ovary but also by the adrenal cortex and in large quantities by the
placenta *

• Three forms - Oestradiol, Oestrone and Oestriol. Oestradiol is the


major oestrogen secreted by the ovary. Oestrone is mainly formed from
androgens while Oestriol is an oxidative product of oestradiol and
oestrone

• Oestradiol has 12 times the oestrogenic potency of oestrone and 80


times that of oestriol

• Serum oestradiol concentrations are maximal just before ovulation on


day 14 of a regular 28 day cycle *

• Circulate bound loosely to plasma proteins (mainly SHBG and


albumin) *

• Conjugated by the liver into glucuronides and sulphates. 20% of these


are excreted in bile and 80% in urine. The liver also converts oestradiol
to oestriol.

• Inhibits FSH secretion by the anterior pituitary. Positive feed-back


effect results in pre-ovulatory LH surge

• Has two known receptors located within the cytoplasm / nucleus - ER -


alpha and beta

1
OESTROGEN - EFFECTS

• Development of secondary sexual characteristics. Pubertal hair growth


is under androgenic control *

• VAGINA: Increase in size, conversion of cuboidal to stratified


squamous epithelium at puberty. Vaginal epithelial cells accumulate
glycogen, their nuclei shrink and they become cornified

• ENDOMETRIUM - stimulates proliferation of glandular and stromal


elements. Stimulates the development of progesterone receptors in
endometrial cells

• MYOMETRIUM - increases sensitivity to oxytocin *

• Increases uterine blood flow - vasodilates uterine artery *

• FALLOPIAN TUBES - proliferation of glandular tissue; increased


number of ciliated cells and increased ciliary activity

• CERVIX - increased mucus production- thin, watery and clear with a


ferning pattern if dried on a slide. Mucus shows increased elasticity and
a drop may be stretched to 10-12cm and is easily penetrable by sperm

• BREASTS - stimulates fat deposition and growth of stroma and ducts.


Devlopment of lobules and alveoli are mainly under influence of
progesterone and prolactin. Inhibits milk secretion

• SKELETON - stimulates osteoblastic activity; stimulates fusion of the


epiphyses of long bones

2
METABOLISM

• Anabolic effect; stimulates protein deposition in specific organs.

• Increases sub-cutaneous fat and blood supply to skin.

• Stimulates sodium and water retention by the kidneys

• Increases hepatic globulin synthesis: sex hormone binding globulin,


thyroxine and cortisol binding globulins *

• Free thyroxine and free cortisol unchanged *

• Increased transferring concentrations *

• Serum albumin levels are usually decreased (along with the associated
calcium levels). *

• Reduced serum folate concentration *

• Increase insulin resistance

• Increase HDL cholesterol

3
• Decrease LDL cholesterol

• Increase total cholesterol

• Increase triglycerides

• Clotting factors - increase in factors II, V, VII, IX, X, XII and especially
VII; decrease of Antithrombin III. Therombogenic *

PROGESTERONE

• C21 steroid Synthesised from cholesterol by the corpus luteum *

• Acts via receptor located in the cytoplasm / nucleus

• Also secreted in large quantities by the placenta *

• Metabolised by the liver into pregnanediol, 10% of which is excreted in


urine

PROGESTERONE - EFFECTS

• UTERUS - secretory changes in the endometrial glands. Inhibits


uterine contractions *. Cervical mucus is thick, viscous and non-ferning

4
• FALLOPIAN TUBES - secretory changes in epithelium

• CERVIX - production of small quantity of viscous mucus which is less


penetrable to sperm

• BREASTS - proliferation of lobules and alveoli. Secretory changes in


alveoli. Does not stimulate milk secretion (Prolactin).

• RESPIRATION - increased ventilation *

• METABOLISM - catabolic hormone; thermogenic with a rise of ~0.5C in


basal body temperature*; induces sodium and water retention by the
kidneys. Promotes development of the lobules and alveoli of the breast

5
STRUCTURE OF STEROID HORMONES

• All steroids have a basic perhydrocyclopentanephenanthrene


molecule

• Has three 6-carbon rings and one 5-carbon ring

• One ring is benzene, two rings naphthalene and three rings


phenanthrene - the 5-carbon ring is cyclopentane

• Divided into three groups according to the number of carbon atoms:

1) 21 - carbon series: Includes corticosteroids and progestins - the basic


structure is the pregnane nucleus

2) 19-carbon series: Includes all androgens and the basic structure is the
androstane nucleus

3) 18-carbon series: Includes oestrogens - estrange nucleus

• All derived from cholesterol (27 carbons) which is converted to


pregnanes (21C) to androstanes (19C) and then to estranges (18C)

STEROID SYNTHESIS

• CHOLESTEROL TO PREGNENOLONE: Occurs by hydroxylation


within the mitochondria

• Tropic hormones from the pituitary increase cholesterol uptake and


hydroxylation

• Most of the cholesterol used is from mobilisation of intracellular


stores

1
• The rate limiting step in steriodogenesis is the transfer of cholesterol
from the outer to the inner mitochondrial membrane

• Long-term stimulation of steroid synthesis requires protein synthesis


while acute stimulation occurs independent of new mRNA synthesis

• PREGNENOLONE TO PROGESTERONE: Catalysed by 3-beta


hydroxylsteroid dehydrogenase which exists in 2 isoforms, types I & II,
two separate gene products on chromosome I

• The type II gene is expressed in the gonads and adrenal gland

• PROGESTERONE TO 17-alpha-hydroxy PROGESTERONE - the


immediate precursor of the C19 androgens

• PREGNENOLONE can also be directly converted to


DEHYDROEPIANDROSTERONE (DHEA)

• Both 17-alpha HYDROXYPROGESTERONE & DHEA can be


converted to ANDROSTENDIONE (C19)

• ANDROSTENDIONE is converted to TESTOSTERONE by 17-beta


hydroxysteroid dehydrogenase

• Both C19 steroids (androstendione and testostereoe) are readily


converted to the C18 steroid OESTROGEN (oestradiol ans oestrone)
by aromatisation which occurs in the smooth endoplasmic reticulum

2
• The reactions converting pregnenolone and progesterone to 17-
hydroxylated products occur in the smooth endoplasmic reticulum

• There are 2 human 21-hydroxylase genes on chromosome 6, the A


& B genes - only the B gene is active

• 17-beta-HYDROXYSTEROID DEHYDROGENASE (non-P450


enzyme, as is 5-alpha reductase)- converts oestrone to oestradiol,
androstendione to testosterone and dehydroepiandrosterone to
androstendiol and vice versa - the enzyme has four isoforms

• Type I - avtive in placenta: oestrone to oestradiol

• Types II & IV - testosterone to androstendione; oestradiol to


oestrone

• Typt II - active in testis, converts androstendione to testosterone

STEROID HORMONE TRANSPORT

• Circulate bound to albumin, sex hormone binding globulin and


corticosteroid binding globulin

• Only a small proportion is free and biologically active

• Hyperthyroidism, pregnancy and oestrogen administration increase


SHBG concentrations

• Corticosteroids, androgens, growth hormone, insulin, progesterone


and IGF-I decrease SHBG concentrations

3
• Circulating levels of SHBG are inversely related to weight

• SHBG concebtrations are a marker of hyper-insulinaemic insulin


resistance

4
OESTROGENS *****

• Oestradiol - most potent natural oestrogen *

• Major oestrogen secreted by the ovaries

• Not active when administered orally *

• Ethinyl-oestradiol - synthetic with ethinyl group at the 17 position -


active orally *

• The other synthetic oestrogen used in the COCP is mestranol - weaker


oestrogen than ethinyl-oestradiol and is converted to ethinyl-oestradiol
in the body

• Unconjugated ethinyl-oestradiol is the active oestrogen in the blood for


both mestranol and ethinyl-oestradiol

• Oestrogens increase target tissue responsiveness to itself and to


progestogens and androgens by increasing the concentration of its
receptor and the receptor for progesterone and androgens

• Progesterone and clomiphene reduce tissue responsiveness to


oestrogen by decreasing the concentration of oestrogen receptors

1
OESTROGEN METABOLISM

• Produced from androgens by aromatization *

• 17-beta-hydroxysteroid dehydrogenase converts androstendione to


testosterone

• Testosterone is rapidly de-methylated at the C19 position and


aromatised to oestradiol - the major oestrogen secreted by the ovary *

• Oestradiol is also produced from androstendione via oestrone *

• Oestriol is not secreted by the ovary and is a peripheral metabolite of


oestrone and oestradiol

• Enough oestrogens can be derived from androgens to produce


bleeding in a post-menopausal woman

• Androstendione is secreted in mg amounts while oestradiol is secreted


in microgram amounts in the non-pregnant female. 1% of
androstendione is converted to oestrone

TAMOXIFEN

• Non-steroidal partial oestrogen antagonist, similar structure to


clomiphene and diethylstilbestrol

• Binds oestrogen receptor with affinity 100 - 1,000 times LOWER than
that of oestrogen

2
• Must therefore be present in a concentration 100 - 1,000 times that of
oestrogen maintain inhibition of breast cancer cells - has cytostatic
NOT cytocidal effect

• Decreases antithrombin III, cholesterol and LDL-cholesterol


concentrations while LDL, SHBG and other binding globulin
concentrations are increased - agonist effects

• Lowers FSH levels in post-menopausal women

• Stimulates synthesis of the progesterone receptor, has oestrogenic


effects on bone, endometrium and vaginal epithelium

PROGESTOGENS *****

• There are 2 main groups of progestogens - progesterone and its


analogues (medroxyprogesterone and dydrogesterone) and
testosterone analogues (norethisterone and norgestrel)

• Levonorgestrel is the active isomer of norgestrel and has twice its


potency
• Newer progestogens desogestrel, norgestimate and gestodene are
derivatives of norgestrel

• Progesterone and its analogues are less androgenic than testosterone


derivatives

• Drospirenone is a newer progestogen which is a spironolactone


derivative
• 17-alpha-hydroxyprogesterone is an intermediate in steroid synthesis
that may be produced from 17-alpha-
hydroxypregnenolone or progesterone. It can be converted into either
androstenedione or 11-deoxycortisol and has progestogenic activity

• Cyproterone acetate is an anti-androgen which also has progestogenic


activity

3
THERAPEUTIC USE OF PROGESTOGENS *****

• Contraception - combined oral contraceptive pill / progestogen-only pill


/ progestogen implants and injectables / progestogen containing intra-
uterine system

• Treatment of endometriosis

• Treatment of menorrhagia - less effective compared to tranexamic acid


or mefenamic acid

• Hormone replacement therapy in women with a uterus

• Endometrial support in women undergoing IVF

• Treatment of endometrial cancer and pre-malignant endometrial


lesions (endometrial hyperplasia). Surgery +/- radiotherapy are the
mainstay treatment for endometrial cancer

PROGESTERONE METABOLISM

• Peripheral conversion of steroids to progesterone does NOT occur in


the non-pregnant female

• Secreted by the adrenal cortex and ovary

• Pre-ovulation production rate <1mg/day; post-ovulation rate = 20-


30mg/day
• 10-20% excreted as pregnanediol *

• pregnanetriol is the main urinary metabolite of 17-alpha


hydroxyprogesterone ? important in congenital adrenal hyperplasia *

4
RU 486 - MIFEPRISTONE

• 19-nortestosterone derivative

• Progesterone antagonist - has some agonist activity

• Binds to the progesterone receptor with 5x the affinity of progesterone

• Has a long half life and active netabolites

• Does not bind to the oestrogen receptor

• Binds weakly to the androgen receptor

• Binds to the glucocorticoid receptor with very high affinity but a high
dose is required to produce effects because of the high circulating
levels of cortisol (1,000x concentration of progesterone)

ANDROGEN METABOLISM

• Main androgens secreted by the ovary are DHEA and androstendione -


by stromal cells *

• Normal accumulation of stromal tissue in mid-cycle results in a rise in


androstendione and testosterone at ovulation

5
• Adrenal sex steroid production is less important in normal women

• 50% daily DHEA and androstendione production is from adrenal *

• 50% androstendione is from ovary; DHEA - 25% ovarian, 25%


peripheral conversion *

• Testosterone - 0.2-0.3mg /day production: 25% ovarian, 25% adrenal,


50% peripheral conversion mainly from androstendione

• Major androgen excreted in urine is 17-ketosteroid

• Testosterone binding capacity is decreased by androgens - hence


lower in males than females

• Only free hormone is biologically active *

• Conversion of testosterone to dihydrotestosterone by 5-alpha


reductase significantly increases its androgenic potency *

• 5-alpha reductase has 2 isoenzymes - Type I is found mainly in skin


and type II mainly in the reproductive tract - separate gene products

• The metabolic product of dihydrotestosterone 3-alpha androstanediol


glucuronide can be neasured in plasma to indicate the level of activity
of target tissues converting testosterone to dihydrotestosterone

6
SEX HORMONE BINDING GLOBULIN AND STEROID TRANSPORT *****

• Sex steroids circulate bound to sex hormone binding globulin (SHBG),


a glycoprotein produced by the liver

• Concentrations higher in females than males because of oestrogen


effect *

7
Free Albumin-bound SHBG-bound
Oestrogen 1% 30% 69%

Testosterone 1% 30% 69%


androstendione 7% 85% 8%
dihydrotestosterone 1% 71% 28%
DHEA 4% 88% 8%

The following increase SHBG concentration *

• Hyperthyroidism
• Pregnancy
• Oestrogen administration

The following decrease SHBG concentration *

• Corticosteroids
• Androgens
• Progestogens
• Growth hormone
• Insulin and insulin resistance
• IGF-I
• Increased weight - concentration inversely related to weight

8
Cortisol binding globulin

• Glycoprotein

• Binds cortisol, progesterone, deoxycorticosterone

• Progesterone: 2% unbound, 80% bound to albumin, 18% bound to


cortisol binding globulin and < 1% bound to SHBG *

9
OESTROGEN RECEPTORS

• Part of the steroid hormone super-family which includes the androgen


receptor, progesterone and glucocorticoid receptor, aldosterone
receptor, thyroid and retinoic acid receptor

• Nuclear transcription factor with binding domains for DNA and


oestrogens

• Two sub-types - alpha and beta - the alpha sub-type has a half life of
4-7h.

• Located in the nucleus even without ligand binding - the androgen,


mineralocorticoid and glucocorticoid receptors on the other hand
localise to the nucleus only after hormone binding

• The oestrogen receptor, however, does undergo nucleocytoplasmic


shuttling and is degraded in the cytoplasm

• Have similar binding affinities for oestradiol but different tissue


expression and distribution. Phytoestrogens, however, have a greater
affinity for the beta receptor

• Prior to ligand binding, the receptor is bound to a number of proteins


including Heat Shock Protein 90 - dissociation of HSP 90 leads to
activation of the receptor

• Following oestradiol binding, the receptor dimerises and undergoes


conformational changes with permit binding to oestrogen response
elements which regulate the transcription of oestrogen-sensitive genes

1
• Phosphorylation of the receptor and other peptides associated with it
is an important mechanism of regulation - cAMP and protein kinase A
pathways increase the transcriptional activity of the oestrogen receptor

• Activated by growth factors such as epidermal growth factor, insulin-


like growth factor-1 and TGF - alpha

• Oestradiol, tamoxifen and raloxifene are agonists in bone; tamoxifen


is an agonist in uterine tissue and tamoxifen and raloxifene are
antagonists in breast

PROGESTERONE RECEPTOR

• Expression is induced by oestrogen at the transcriptional level and


inhibited by progesterone at the transcriptional and translational level

• Two major forms - A & B

• The inactive receptor is bound to heat shock protein 90

• Progesterone binding leads to dimerisation with homodimers (AA) or


heterodimers (AB)

• The activated receptor binds DNA and regulates transcription of


progesterone responsive genes

• In most cell types, the B receptor is the positive regulator of


progesterone sensitive genes

• The A sub-type inhibits the activity of the B sub-type

2
ANDROGEN RECEPTOR

• Exists as a full length B form and a shorter A form

• Gene located on the X-chromosome - androgen insensitivity


syndrome is an X-linked disorder

• Similar to the progesterone, mineralocorticoid and glucocorticoid


receptor - most similar to the progesterone receptor

• Progestogens compete with androgens for the receptor and for the 5-
alpha reductase system - dihydroprogesterone which is produced
competes with testosterone and dihydrotestosterone

• Has a greater affinity for dihydrotestosterone

• Lower affinity for cyproterone acetate or spironolactone (~20%)


compared to testosterone

Androgens produce biological effects by:

1) Direct actions of testosterone - endocrine effects

2) Intracellular conversion to dihydrotestosterone

3) Intracellular conversion of testosterone to oestradiol (aromatization)

• Tissues derived from the Wollfian duct respond exclusively to


testosterone

• Hair follicles and derivatives of the urogenital sinus / tubercle require


conversion to dihydrotestosterone by 5-alpha reductase

• The hypothalamus actively converts androgens to oestrogens

3
4
ENDOCRINE CHANGES IN PREGNANCY

• MINERALOCORTICOIDS -increases the rate of maternal


ventilation. Oestrogen stimulates rennin and angiotensin secretion
with a resultant increase in aldosterone secretion. Renal secretion of
deoxycorticosterone also increases. Result is sodium and water
retention

• PITUITARY - 50% increase in size. Increased ACTH and prolactin


secretion. Decreased growth hormone secretion (HPL and placental
growth hormone have negative feed-back effect). Decreased FSH &
LH - suppressed by inhibin-A, oestrogen, progesterone and prolactin

• Increased total cortisol and thyroxine due to oestrogen stimulated


increase in binding proteins. Free cortisol and thyroxine may be
slightly increased. Thyroid gland increases in size by 50%

• Increased size of parathyroid gland and increased PTH secretion.


Also increased 1,2,5-dihydroxy D3 - increases calcium absorption and
mobilisation from bone. Calcium mobilisation is particularly increased
during lactation

HUMAN CHORIONIC GONADOTROPIN (HCG)

• Glycoprotein, with carbohydrates making up about one third of its


molecular weight. This accounts for the longer half life of HCG (24h)
compared to say LH (2h)

• Alpha chain similar to alpha chain of FSH, LH, TSH - therefore has
some intrinsic TSH activity

1
- Unique beta chain

• All human tissues make HCG but the placenta is unique in being
able to glycosylate it, increasing the half life and giving biological
activity. Sialic acid residues are essential for the longer half life

• HGC is produced mainly by the syncytiotrophoblast although gene


expression also occurs in cytotrophoblasts

• HCG is detectable in the 8 cell stage embryo and takes over from
LH in supporting the corpus luteum about 8 days after ovulation (1 day
after implantation)

• GnRH is synthesized by placental cells and the receptors are


present. In vitro, GnRH stimulates HCG production while endorphins
are inhibitors

• Inhibin restrains while activin enhances the GnRH-HCG system.


Progesterone inhibits HCG secretion

• Follistatin binds activin and inhibits its action on the GnRH-HCG


system

• Only known function is support of the corpus luteum, taking over


from LH on about the 8th day after ovulation, 1 day after implantation

• May stimulate androgen production from the early fetal testes and
may also regulate the function of the fetal zone of the adrenal cortex in
early pregnancy thereby influencing placental production of
oestrogens

2
• Maternal HCG ~100IU/L at the time of the first missed period and
peaks at ~100,000IU/L at ~10 weeks gestation

• Levels fall to about 10,000 - 20,000IU/L at 18-20 weeks and remain


at this level till term

• Mainly excreted by the kidneys - as a reduced fragment of the beta


sub-unit called the beta-core fragment

• Maternal, urinary, placental and amniotic fluid HCG levels at term


are higher in pregnancies with a female fetus

• Useful in the diagnosis and management of trophoblastic disease


and ectopic pregnancy

• HCG is produced by virtually all tissues - pulsatile release from the


pituitary gland together with LH - levels may be within the sensitivity of
current assays in post-menopausal women with very high LH levels.
Non-glycosylated therefore very short half life

OESTROGEN SECRETION

• Under fetal control and is a fundamental feto-maternal signalling


mechanism

• The placenta lacks 17-alpha hydroxylase and 17-20 desmolase


activity and cannot convert C21 products (progesterone /
pregnenolone) to C19 products (androstendione / DHEA)

• In early pregnancy, the placenta utilises maternal androgens for


oestrogen production

3
• By 20 weeks gestation, the majority of placental oestrogen
production is from fetal DHEA-Sulphate. The fetus rapidly sulphates
steroids, preventing biological activity.

• The placenta has an active sulphatase to remove sulphate groups

• Fetal DHEA-S is converted to oestradiol and oestrone. The placenta


cannot produce oestriol from DHEA-S

• Fetal DHEA-S is hydroxylated by the fetal liver to 16-alpha-hydroxy-


DHEA-S. This is then utilised by the placenta for OESTRIOL
production. Oestriol is first detectable at 9 weeks gestation when fetal
adrenal gland secretion of precursor begins.

• In the absence of a normal fetal adrenal gland, maternal oestrogen


concentrations are very low (for instance, anencephaly)

• OESTRIOL is the main placental oestrogen. Oestradiol and


oestrone are derived equally from maternal and fetal precursors

• Maternal oestraDIOL levels are higher than in the fetus

• Fetal oesTRIOL levels are higher than maternal levels

• Placental aromatisation of maternal androgens is so effective that


the fetus is protected from masculinisation. Extremely high androgen
levels or non-aromatisable analogues are required for fetal effects

4
PROGESTERONE SECRETION

• Produced by the corpus luteum until 10 weeks gestation -exclusively


until 7 weeks after which placental production begins

• At term, the placenta produces ~250mg/day

• Progesterone production by the placenta is independent of the


quantity of precursor available, utero-placental perfusion, fetal well-
being or even the presence of a live fetus

• Produced from maternal cholesterol (LDL); about 3% is derived from


maternal pregnenolone. Levels rise with increasing gestation age

• Human decidua and fetal membranes also produce progesterone,


mainly from pregnenolone sulphate

• Amniotic fluid progesterone concentration is maximal at 10-20


weeks gestation and fall gradually

• Myometrial progesterone concentrations are about 3x maternal


plasma levels in early pregnancy, and equal to plasma levels at term

• Levels of 17-alpha hydroxyprogesterone are high in early pregnancy


(corpus luteum), return to pre-pregnancy levels at about 10 weeks
(placenta has little 17-alpha hydroxylase activity) and increase after 32
weeks due to placental utilisation of fetal precursors

• Progesterone serves as a substrate for glucocorticoid and


mineralocorticoid production by the fetal adrenal gland, although fetal
serum LDL cholesterol is also utilised

5
• The fetal adrenal cortex lacks significant 3-beta-hydroxysteroid
dehydrogenase and isomerase activity and relies on placental
progesterone

• Maintains the decidua and stimulates prolactin secretion from the


decidua

• Inhibits myometrial contractions, prostaglandin production and


sensitivity to oxytocin

6
THE FETAL ADRENAL CORTEX

• Differentiated by 8-9 weeks gestation into a thick inner fetal zone and
a thin outer definitive zone which produces cortisol

• Is as big as the kidney by the end of the first trimester

• Fetal zone rapidly involutes after delivery

• The fetal zone produces large amounts of DHEA & DHEA-S - 3beta
hydroxysteroid dehydrogenase activity is suppressed

• DHEA-S is utilised by the placenta to produce oestradiol and


oestrone

• Early growth of the adrenal gland is independent of ACTH. After 15-


20 weeks, however, ACTH is required

• The trophic support of the fetal adrenal gland by ACTH is protected


by placental oestrogen: the placenta prevents maternal cortisol from
reaching the fetus by converting it to cortisone (11-beta-hydroxysteroid
dehydrogenase activity) - this activity is stimulated by oestrogen

• A relative deficiency of 11-beta-hydroxysteroid dehydrogenase type 1


(placental isoform) is associated with low birth weight

• The fetal adrenal gland is also under paracrine regulation by inhibin


and activin. Activin enhances ACTH stimulated steriodogenesis

• Oestrogen in high concentrations found in the fetus inhibits adrenal 3-


beta-hydroxysteroid dehydrogenase activity and in the presence of
ACTH enhances the production of DHEA

HUMAN PLACENTAL LACTOGEN

1
• Protein hormone secreted by the trophoblast - single polypeptide with
191 amino acids

• Similar structure to growth hormone

Placental HPL secretion is NOT influenced by somatostatin or growth


hormone releasing hormone

• Half life ~15 min

• Maternal levels correlate with fetal and placental weight

• Plasma concentrations rise with gestation age

• Very little HPL enters the fetal compartment

• Decreases insulin sensitivity (antagonist), stimulates insulin release,


stimulates lipolysis and decreases glucose utilisation by the mother.
Results in increased glucose and free fatty acid concentrations

GROWTH HORMONE RELEASING HORMONE (GHRH); GROWTH


HORMONE (GH) & SOMATOSTATIN

• GHRH and somatostatin are present in the placenta. The decidua


also produces somatostatin

• Somatostatin production DECREASES with increasing gestation

2
• Pregnancy GHRH and somatostatin do not contribute to maternal
levels

• Placental GH is DIFFERENT from pituitary GH

• After 15-20 weeks gestation, placental GH replaces pituitary GH and


by term, maternal pituitary GH is no longer detectable

• Placental GH is not present in fetal blood

• Maternal IGF-I levels increase in line with increasing placental GH


levels

• Placental GH is not regulated by GH releasing hormone but responds


to maternal glucose levels

• Placental GH stimulates gluconeogenesis and lypolysis

PROLACTIN

• Produced by the decidua, myometrium, fetal and maternal pituitary


glands

• Trophoblast and fetal membranes do not produce prolactin

• The endometrium requires the presence of progesterone to produce


prolactin while in the myometrium, prolactin is inhibitory

3
• Amniotic fluid prolactin is derived from the decidua, while fetal plasma
prolactin is from the fetal pituitary

• Maternal prolactin concentration increases with gestation - stimulation


of pituitary production by oestrogen

• Amniotic fluid prolactin concentration parallels maternal levels until 10


weeks, rise sharply to 20 weeks then decline until delivery

• Decidual prolactin is essential for fluid and electrolyte regulation in


the amniotic fluid and is unaffected by dopamine agonists

RENIN-ANGIOTENSIN SYSTEM

• Concentration of rennin substrate (angiotensinogen) increases 10


fold in early pregnancy - from ovarian stimulation by HCG

• Concentration of rennin remains unchanged

• Rennin substrate is also produced by chorionic tissue and is


concentrated in the amniotic fluid. Highest concentration is in the
gestation sac in early pregnancy

• Rennin and angiotensinogen are expressed by the chorion, amnion


and placenta

• Maternal rennin activity is increased 4 fold by mid-gestation -


compensatory response in the presence of vasodilators

4
RELAXIN

• Peptide hormone secreted by the corpus luteum, deciduas and


placenta under HCG stimulation. Similar structure to pro-insulin

• Concentrations peak in the first trimester

• Inhibits myometrial activity and relaxes pelvic ligaments. Softens the


cervix by increasing collagenase activity

ENDOCRINE CHANGES DURING PARTURITION

• Cortisol concentration in amniotic fluid rises dramatically between 34-


36 weeks gestation

• Correlates with pulmonary maturation

• Cord blood cortisol levels higher after spontaneous labour (with


vaginal delivery or caesarean section) compared to induced labour or
elective caesarean section

• Although maternal cortisol crosses the placenta readily, over 85% is


metabolised to cortisone

• The fetal liver has a limited capacity to convert cortisone to cortisol.


Adult liver and fetal lung converts cortisone to cortisol

• Fetal adrenal activity drives placental oestrogen production and this is


important in parturition. Increased oestrogen concentration from 34-35
weeks gestation

5
• Decreased placental oestrogen production (anencephaly, placental
sulphatase deficiency) associated with prolonged pregnancy

• Pregnancies associated with poor cortisol production (congenital


adrenal hyperplasia) are of normal duration

• Role of progesterone less clear in primates than in sheep


Increased oestrogen and a local decrease in progesterone may
regulate prostaglandin release

• Maternal and amniotic fluid prostaglandin concentrations increase in


association with labour and high doseaspirin is associated with
prolonged pregnancy

• CRH may play a role in parturition: produced by trophoblast, fetal


membranes and decidua

• Maternal and amniotic fluid CRH concentration increases with


gestation

• Maternal, but not amniotic fluid CRH increases with labour

• CRH binding protein is produced by trophoblast, decidua and fetal


membranes. Maternal and amniotic fluid levels fall prior to labour

• CRH stimulates prostaglandin release in the decidua, fetal


membranes and myometrium

• Increased CRH and decreased CRH binding protein occur in women


with threatened pre-term labour who deliver within 24h

6
• Cortisol, in the presence of progesterone, stimulates trophoblast CRH
synthesis. Progesterone is the only major inhibiting factor for CRH
production in placental tissue

• In the sheep, the CRH signal begins in the fetal brain but in humans,
it begins in the uterus

OXYTOCIN AND MYOMETRIAL RESPONSE

• Maternal oxytocin levels increase prior to the onset of parturition,


initially only at night

• Levels rise after the onset of labour, particularly in the second stage

• Produced in the amnion, chorion, decidua and pituitary

• Stimulates prostaglandin synthesis in the deciduas and myometrium

The concentration of oxytocin receptors is low in the non-pregnant uterus,


increases throughout gestation and doubles during labour

7
MENOPAUSE *****

• Point at which permanent cessation of menstruation occurs

• Caused by exhaustion of ovarian follicles - there is an accelerated loss of ovarian


follicles in the peri-menopausal period associated with decreased inhibin secretion*

Gonadotrophins

• This decrease in inhibin secretion means that post-menopausal HRT does not
suppress pituitary gonadotrophin secretion - FSH levels cannot therefore be used to
titrate oestrogen replacement *

• FSH levels rise > 20IU/L despite continued menstruation in the peri-menopausal
period but LH levels remain in the normal range *

• FSH and LH levels peak 1-3 years after the menopause, followed by a gradual
decline

Steroidogenesis

• Oestrogen levels do not begin to decline until a woman is within 12 months of the
menopause *

• Women experiencing the perimenopausal transition have higher serum oestradiol


levels - increased follicular response to the increase in FSH concentration

• Fertility is however not increased during this period as anovulation is prevalent *

• Oestrogen production by the ovaries stops after the menopause *

• Post-menopausal oestradiol concentration 10-20pg/ml, majority from peripheral


conversion of oestrone derived from androstendione. Percentage conversion of
androstendione to oestrone dependent on BMI *

• Almost every tissue has the potential to convert androgens to oestrogens by


aromatisation

• Circulating oestrone concentration higher than oestradiol concentration after the


menopause and proportion of oestrone therefore increases *

1
• Androgen production continues in the ovaries after the menopause - mainly
androstendione and testosterone. Androstendione levels after the menopause are 50%
those of pre-menopausal women - mostly of adrenal origin *

• DHEA & DHEA-S levels also fall by ~70% after the menopause

• Testosterone production declines by 25% after the menopause but the post-
menopausal ovary secretes more testosterone - stimulated by increased gonadotrophin
levels *

• Eventually, the ovarian stroma is exhausted and steroidigenesis stops

• Ther distal urethra is oestrogen sensitive and atrophy occurs after the menopause. *

2
LEPTIN

• Polypeptide hormone containing 167 amino acids and secreted


mainly by adipose tissue but is also produced in the ovary. It is a
member of the tumour necrosis factor group of cytokines. Leptin
secretion has a diurnal rhythm

• Circulates bound to a family of proteins and acts on CNS neurons


regulating eating behaviour and energy balance

• Coded by the ob gene in mice and the Lep gene in humans (located
on chromosome 7q33) - mutations in this gene result in obesity in
mice. However, only a very small minority of overweight humans have
a mutation in the Lep gene or leptin receptor

• Leptin binds to a cell membrane receptor which is related to the


cytokine receptor family

• Acts on hypothalamic neurons decreasing neuropeptide Y


expression and secretion. Neuropeptide Y stimulates eating,
decreases heat production and increases cortisol and insulin secretion

• Leptin expression is increased by insulin, glucocorticoids,


noradrenaline and food while circulating levels are reduced in weight-
related (under-weight) amenorrhoea, athletes and in delayed puberty

• Leptin levels are higher in females than in males and higher in post-
menopausal women compared to pre-menopausal women. As weight
increases, leptin levels increase three times as fast in women
compared to men. In girls, leptin levels fall with increasing Tanner
stage of puberty - leptin sensitivity increases at puberty

1
• In overweight individuals, the majority of leptin is unbound, while the
majority is bound in lean individuals

• A 10% reduction in body weight results in a 53% fall in leptin levels,


stimulating an effort to regain weight. Fasting and exercise decrease
leptin secretion and increase neuropeptide Y

• There is no increase in leptin levels in women with PCOS

2
Question 1: Human chorionic gonadotrophin

a. Binds to the leutenising hormone receptor


True False
b. Has a half life in blood of about 2h
True False
c. Is produced by the pre-implantation blastocyst
True False
d. May be produced in tissues other than trophoblast
True False

Question 2: The corpus luteum

a. Is made up of cells rich in smooth endoplasmic reticulum


True False
b. Is about 5cm in diameter
True False
c. Degenerates into the corpus albicans after 7 days
True False
d. Has a yellowish appearance
True False

Question 3: With respect to follicular development and ovulation

a. Leutenising hormone has a synergistic effect with follicle stimulating


hormone on the growth of granulosa cells
True False
b. A dominant follicle is selected by day 7 of a 28 day menstrual cycle
True False
c. Ovulation typically occurs on day 14 of a 28 day menstrual cycle
True False
d. The released oocyte is surrounded by granulosa cells which form a
layer called the zona pellucida
True False

1
Question 4: With respect to follicular development and ovulation

a. FSH stimulates the growth of 6-12 mature follicles per cycle


True False
b. Conversion of the primordial follicle to the primary follicle is dependent
on leutenising hormone
True False
c. The oocyte secretes follicular fluid into the follicular antrum
True False
d. The theca interna and externa are the inner and outer layers of
granulosa cells
True False

Question 5: With respect to oestrogens

a. Oestriol is converted to oestradiol by the liver


True False
b. Oestradiol circulate bound to plasma proteins
True False
c. Oestrogens are synthesised from cholesterol
True False
d. Oestrogens are secreted by the adrenal medulla
True False

Question 6: The corpus luteum

a. Is formed mainly from the cells of the theca interna


True False
b. Secretes progesterone
True False
c. Secretes oestrogen
True False
d. Is only formed if fertilisation occurs
True False

2
Question 7: During the sexual act in the male

a. Erection is caused by stimulation of the sympathetic nervous system


True False
b. Secretions from the urethral and bulbourethral glands is stimulated by
activation of the parasympathetic nervous system
True False
c. Ejaculation is under stimulation of the sympathetic nervous system
True False
d. Sympathetic impulses that stimulate ejaculation leave the spinal cord at
the level of S2,3&4
True False

Question 8: Leptin

a. Acts on the hypothalamus


True False
b. Stimulates neuropeptide Y expression
True False
c. Increases GnRH secretion
True False
d. Has 8 amino acids
True False

Question 9: Follicle stimulating hormone (FSH)

a. Has a receptor which is located within the cytosol


True False
b. Concentrations are elevated during childhood
True False
c. Concentrations are elevated during the menopause
True False
d. Secretion is inhibited by inhibin
True False

3
Question 10: The following hormones stimulate the proliferation and
branching of breast ducts

a. Prolactin
True False
b. Growth hormone
True False
c. Cortisol
True False
d. Progesterone
True False

Question 11: Oestrogen

a. Inhibits milk secretion by the breasts


True False
b. Stimulates osteoblastic activity
True False
c. Delays fusion of the epiphyses of long bones
True False
d. Stimulates fat deposition in sub-cutaneous tissue
True False

Question 12: Leutenising hormone (LH)

a. Stimulates progesterone secretion by the granulosa cells


True False
b. Induces the conversion of granulosa cells into lutein cells
True False
c. Production is stimulated by oestrogen during the pre-ovulatory phase
of the menstrual cycle
True False
d. Concentrations rise within a few days of the onset of menstruation
True False

Question 13: 17-beta-oestradiol

4
a. Is thrombogenic
True False
b. Does not bind to albumin
True False
c. Is active when administered orally
True False
d. Is formed in tissues other than the ovary
True False

Question 14: Human chorionic gonadotropin (HCG)

a. Stimulates oestrogen and progesterone secretion by the corpus


luteum
True False
b. Stimulates testosterone secretion by the testis
True False
c. Results in decreased thyroxin secretion in early pregnancy
True False
d. Stimulates production of dehydroepiandrosterone sulphate by the fetal
zone of the adrenal gland
True False

Question 15: With respect to follicular development and ovulation

a. Granulosa cells secrete oestrogen


True False
b. Granulosa cells secrete inhibin
True False
c. Granulosa cells secrete human chorionic gonadotropin
True False
d. Oestrogen reduces the number of FSH receptors on granulosa cells
True False

5
CORTICOTROPIN-RELEASING HORMONE

• Peptide with 41 amino acids

• Produced by the hypothalamus into the portal system

• Hypothalamic releasing hormones are produced by cells located in the


median eminence.

• CRH and vasopressin are produced in the paraventricular nucleus


GnRH is produced in the medial pre-optic area and the arcuate
nucleus

• CRH is produced by the placenta (cytotrophoblast), fetal membranes


and decidua

• Placental CRH production is inhibited by progesterone and stimulated


by glucocorticoids

• Levels in maternal blood and amniotic fluid increase with gestation age

• Stimulates prostaglandin release from the fetal membranes,


myometrium and decidua

• CRH binding protein is also produced by the trophoblast, fetal


membranes and decidua

• Binds to cell surface receptors on anterior pituitary corticotrophs and


stimulates ACTH release

1
• Also inhibits growth hormone release, suppresses fever, appetite and
sexual function and stimulates the sympathetic nervous
system ADRENOCORTICOTROPIC
HORMONE (ACTH)

• Polypeptide hormone with 39 amino acids

• Synthesised by anterior pituitary corticotrophs as a precursor - pre-pro-


opiomelanocortin.

• This gives rise to ACTH, beta-endorphin and melanocyte stimulating


hormone and corticotrophin-like peptide (CLIP) which are co-released ·

• Secretion from the anterior pituitary has circadian rhythm and this
regulates cortisol secretion.

• Concentration is lowest around 3am and increase to a maximum


between 6-8am·

• ACTH production inhibited by corticosteroids

• The placenta produces ACTH and CRH throughout pregnancy (peak at


term) which results in increased maternal free cortisol

• Cortisol levels in pregnancy are resistant to dexamethasone


suppression as a consequence and maternal ACTH levels are not
affected by corticosteroid administration

• CRH is produced by the cytotrophoblast while ACTH is produced by


the syncytiotrophoblast

2
• Maternal ACTH response to administered ACTH is blunted, indicating
high endogenous levels of CRH and ACTH

• Vasopressin stimulates pituitary ACTH release and this response is


increased during pregnancy

• Oxytocin is a potent stimulator of placental ACTH and CRH production


·

• Acts via a cell-surface receptor ·

• Stimulates the release of adrenal corticosteroids, androgens and


aldosterone ·

• Secretion inhibited by negative feed-back effect of cortisol but not


aldosterone

3
THYROTROPIN RELEASING HORMONE (TRH)

• Tri-peptide hormone produced in the median eminence of the


hypothalamus

• Stimulates TSH release from the anterior pituitary

• Stimulates prolactin production in the anterior pituitary

• Production is inhibited by thyroxine in a negative feed-back loop

THYROID STIMULATING HORMONE (TSH)

• Glycoprotein, alpha sub-unit shared with FSH, LH and HCG. Unique


beta sub-unit which is responsible for hormone activity. Produced by
the anterior pituitary

• Production increased by:

1) TRH - hypothalamus

• Production inhibited by:

1) Thyroxine - negative feed-back

2) Dopamine and somatostatin - hypothalamus

3) Growth hormone

4) Cortisol

1
ACTIONS

1) Increases protein synthesis in the thyroid gland, causing hypertrophy and


hyperplasia of follicular cells

2) Increases iodide trapping, iodination, coupling and endocytosis of colloid in


thyroid gland

3) Increases proteolysis of thyroglobulin

2
PROLACTIN

• Peptide hormone produced by anterior pituitary lactotrophs;


stimulates breast development and milk secretion. Acts by cell
surface receptor

• Synthesis is inhibited by dopamine from the hypothalamus*.


Disruption of the connections between the anterior pituitary and
the hypothalamus cause hyperprolactinaemia.

• Dopamine antagonists such as metoclopramide also cause


increased prolactin levels*

• Synthesis is stimulated by thyrotropin releasing hormone


from the hypothalamus *

• Plasma concentrations increase throughout pregnancy -


oestrogen induces hyperplasia and hypertrophy of lactotrophs *

• Secretion is stimulated by stress, trauma, sexual intercourse,


venepuncture, suckling*

ACTIONS

• Stimulates proliferation and branching of breast ducts

• Stimulates milk secretion*

• High concentrations inhibit hypothalamic production of


gonadotropin-releasing hormone

1
GROWTH HORMONE

• Polypeptide hormone (191 amino acids, molecular weight


~22kDa)* produced by anterior pituitary acidophil cells
(somatotrophs). Circulates bound to growth hormone binding
protein

Secretion is regulated by

• HYPOTHALAMUS - Growth hormone releasing hormone;


Somatostatin inhibits release

• Glucose and fatty acids - fall in concentration stimulated GH


release. Oral glucose load rapidly suppresses GH
release*.Secretion increased during prolonged fasting

• Amino acids (arginine especially) - increased concentration


stimulates GH release

• Fall in plasma free fatty acids increase GH secretion

• Secretion has circadian rhythm - increased secretion during


periods of deep sleep. Peak release during first 2h of deep
sleep. REM sleep inhibits GH release

• Released in discrete pulses - most frequent in adolescence

• Stress - trauma, surgery, fever, exercise, anxiety, pain, cold,


haemorrhage - stimulate release

• Thyroxine and cortisol stimulate secretion

• Somatomedines (insulin-like growth factors) - inhibit release

2
ACTIONS

• Stimulate growth - acts through stimulation of hepatic


production of insulin-like growth factors (somatomedines).
Insulin and carbohydrate essential for growth promoting effect
of GH. Not required for intra-uterine growth*

• Stimulates mitosis in the epiphyses but does not stimulate


epiphyseal fusion. Still has effects on bone after epiphyseal
fusion*

• Continues to be secreted and have effects after puberty*

• Stimulates production of insulin-like growth factors in the


liver*

• Anabolic - increased protein synthesis; increased amino acid


uptake, decreased plasma urea concentration and induces a
positive nitrogen balance*

• Increased fatty acid mobilisation from adipose tissue*

• Decreased glucose utilisation for energy with increased


plasma glucose concentration*

• Ketogenic and diabetogenic

• Opposes most insulin actions*

3
INSULIN-LIKE GROWTH FACTORS < (IGFs / Somatomedines)

• Peptides synthesised by the liver under GH stimulation

• Circulate bound to IGF-binding proteins

• Homologous to insulin. Have cell-surface receptors - IGF-1


receptor similar to insulin receptor while the IGF-2 receptor is
different

• Mediate the growth-promoting effects of GH; longer half-life


than GH. Have a direct effect on cartilage, bone and muscle
growth

• Low concentrations during fasting

• Negative-feed-back effect on GH secretion

• IGF production is inhibited by oestrogen and cortisol

4
5
OXYTOCIN *****

• Polypeptide hormone, 9 amino acids, produced mainly by the para-


ventricular nucleus of the hypothalamus and released by the posterior
pituitary. Also produced locally in the ovary and testis *

• Differs from ADH (arginine vasopressine) in only 2 amino acids

• Circulate largely unbound in plasma

• Kidneys, liver and brain are the main sites of clearance

• Half life ~1 min

• Secreted episodically by calcium-dependent exocytosis *

• Basal concentrations are similar in males and females

• Oestrogen increases the sensitivity of the myometrium to oxytocin,


while progesterone decreases sensitivity *

Secretion is stimulated by

• Suckling *

• Vaginal distension during intercourse

• Labour

• Secretion is inhibited by emotional stress *

ACTIONS

• Causes milk ejection (let down) by stimulating contraction of the


myoepithelial cells of the alveoli of the breast *

• Rhythmic contraction of uterine smooth muscle. High pharmacological


concentrations may cause tetanic contraction

• High concentrations cause fluid retention - homology with ADH *.

• Stimulate peristalsis *

1
ARGININE VASOPRESSIN (ANTIDIURETIC HORMONE) *****

• Polypeptide of 9 amino acids, produced by the supra-optic nucleus of


the hypothalamus and secreted by the pituitary gland *

• Similar structure to oxytocin *

Secretion is stimulated by

• Hypovolaemia and hypotension - 10% blood loss would stimulate ADH


secretion. Plays important role in arterial pressure regulation *

• Rise in plasma osmolarity - regulates sodium ion concentration of


extracellular fluid *

• Angiotensin

• Pain

• Emotional stress

• Nausea and vomiting

• Drugs such as morphine and nicotine

Secretion is inhibited by

• Decreased plasma osmolarity *

• Increased cardiac out-put *

• Atrial natruretic peptide

• Alcohol *

2
ACTIONS

• Increased water absorption from the renal tubules, in particular


increased permeability of the final third of the distal tubule and
collecting duct *

• Increased urine osmolarity and decreased urine flow *

• Stimulates sodium resorption and urea transport from the lumen into
the interstitial fluid in the medullary collecting ducts

• Potent vasoconstrictor ? acts on arteriolar smooth muscle of skin and


splanchnic circulation *

• Also causes bradycardia and decreased cardiac output ? physiological


effect on blood pressure is very small. However, plays important role
during haemorrhage and dehydration

• Has CRF-like activity and stimulates ACTH release

• Renal effects of ADH blocked by: hypokalaemia, hypercalcaemia,


lithium therapy

• Large doses of ADH cause smooth muscle contraction and can cause
hypertension and stimulate peristalsis

• In pregnancy plasma osmolality decreases from 285 to 275mosm/kg

In pregnancy both the plasma level of vasopressin and the release of vasopressin
following dehydration or water loading remain unchanged*

3
Question 1: Growth hormone

a. Increases the rate of glucose utilisation in the body


True False
b. Decreases amino acid uptake by cells
True False
c. Is a ketogenic hormone
True False
d. Is a diabetogenic hormone
True False

Question 2: Growth hormone

a. Secretion is inhibited by exercise


True False
b. Secretion is stimulated by an increase in plasma amino acid
concentration
True False
c. Secretion is stimulated by trauma
True False
d. Secretion is inhibited by a rise in plasma free fatty acid concentration
True False

Question 3: Prolactin

a. e) Secretion is stimulated by breast-feeding


True False
b. f) Plasma concentrations fall during the third trimester of pregnancy
True False
c. g) Secretion increases with physiological stress
True False
d. h) Plasma concentrations fall if the connections of the pituitary gland to
the hypothalamus are disrupted
True False

1
Question 4: Antidiuretic hormone

a. Secretion is stimulated by dehydration


True False
b. Causes a decrease in urine osmolarity
True False
c. Causes an increase in plasma osmolarity
True False
d. Decreases water excretion by the kidneys
True False

Question 5: Adrenocorticotropic hormone (ACTH)

a. Is secreted in conjunction with beta-endorphin


True False
b. Stimulates adrenal androgen secretion
True False
c. Has no effect on adrenal aldosterone secretion
True False
d. Binds to a receptor located within the cell nucleus
True False

Question 6: Adrenocorticotrophic hormone (ACTH)

a. Is produced by the cytotrophoblast


True False
b. Production by the placenta is stimulated by oxytocin
True False
c. Production by the placenta is inhibited by dexamethasone
True False
d. Production by the pituitary is inhibited by vasopressin
True False

2
Question 7: Arginine vasopressin

a. Controls water resorption by the kidneys


True False
b. Is an oligopeptide
True False
c. Release is increased by haemorrhage
True False
d. Plasma concentration is reduced in pregnancy
True False

Question 8: Arginine vasopressin

a. Reduces glomerullar filtration rate


True False
b. Controls water loss in the proximal convoluted tubule
True False
c. Is released in response to a rise in plasma osmolarity
True False
d. Is released in response to a fall in plasma volume
True False

Question 9: Oxytocin

a. j) Is produced locally by the ovary and testis


True False
b. k) In high concentrations causes diuresis (
True False
c. l) Production is stimulated by emotional stress
True False
d. g) Basal plasma levels are undetectable in males
True False

3
Question 10: Thyroid stimulating hormone

a. Secretion is stimulated by somatostatin from the hypothalamus


True False
b. Secretion is stimulated by cortisol
True False
c. Secretion is inhibited by growth hormone
True False
d. Inhibits protein synthesis in the thyroid gland
True False

Question 11: Oxytocin

a. Is produced mainly by the supra-optic nucleus of the hypothalamus


True False
b. Has 9 amino acids
True False
c. Is a glycoprotein
True False
d. Secretion is stimulated by suckling
True False

Question 12: Adrenocorticotrophic hormone (ACTH)

a. Production is governed by the hypothalamus


True False
b. Production is maximal about midnight
True False
c. Is present in the placenta
True False
d. Is increased in the maternal plasma in pregnancy
True False

4
Question 13: Corticotropin-releasing hormone

a. Is a glycoprotein
True False
b. Is secreted by the hypothalamus
True False
c. Inhibits growth hormone release
True False
d. Acts by binding to a cytosolic receptor
True False

Question 14: Oxytocin

a. Is released episodically
True False
b. Causes decreased renal tubular absorption of water
True False
c. Causes milk ejection
True False
d. Reduces intestinal peristalsis
True False

Question 15: Prolactin

a. Is produced by anterior pituitary lactotrophs


True False
b. Is a glycoprotein
True False
c. Has a similar structure to thyroid stimulating hormone
True False
d. Secretion is stimulated by oestrogen
True False

5
THYROXINE

• Produced by the follicular cells of the thyroid gland by the following


process:

1)Iodide trapping by follicular cells

2)Oxidation of iodine by peroxidase

3)Synthesis of thyroglobulin, a glycoprotein containing 140 tyrosine amino acids

4)Iodination of tyrosine (Organification of thyroglobulin): formation of mono-, then


di-iodothyrosine

5)Combination of di-iodothyrosine x2 -T4, mono- + di-iodothyrosine - T3

6)Storage in thyroid colloid -each thyroglobulin molecule contains ~5-6 T4 molecules


and one T3 molecule

7)Uptake of thyroglobulin by endocytosis, proteolytic cleavage and release of T3 and


T4

• The thyroid gland produces tetra-iodothyronine (T4) predominantly.


Virtually all T3 is formed from peripheral conversion of T4. T3 has 4
times the biological activity of T4*. Selenium is essential for the
peripheral conversion of T4 to T3 by 5-monodeiodinase

PROTEIN BINDING

• Highly protein bound to thyroxine binding globulin (70%), thyroxine


binding pre-albumin (~20%) and albumin (~10%). Only 0.03% of T4
and 0.3% of T3 is unbound*

• Half-life: T4 = 6 days, T3 = 1 day

1
• Does not cross the placenta in significant amounts. The fetal thyroid
produces thyroxine from about 12 weeks gestation.*

• Control of secretion - TSH from the anterior pituitary. Thyroxine


concentration falls during fasting*

ACTION

• Increased metabolic rate and stimulation of growth -has nuclear


receptor and mediates effects by stimulating DNA transcription. It takes
6-12 hours (T3) and 2-3 days (T4) for metabolic effects to manifest*

• Increased oxygen consumption with increased heat production in most


organs with the exception of the brain, spleen, testes, uterus and
anterior pituitary

• Increased protein synthesis at low concentrations - translation and


transcription. High thyroid hormone concentrations cause protein
catabolism

• Increased size and number of mitochondria

• Activation of Na-K ATPase

• Promote growth and development of the brain during fetal life and
childhood

• Stimulate linear growth of bone until puberty. Promote ossification of


bone and maturation of the epiphyseal growth regions *

• Stimulation of carbohydrate metabolism - intestinal glucose uptake and


glucose uptake and utilisation by tissues such as skeletal muscle and
adipose tissue; stimulate gluconeogenesis *

• Lipolytic action on fat stores and increase plasma levels of free fatty
acids. Also increase oxidation of fatty acids *

• Increased heart rate, myocardial contractility and cardiac out-put

• Increased pulse pressure with no change in mean arterial pressure -


systolic pressure increases and diastolic pressure falls

• Increased appetite, food intake, secretion of GI hormones

• L-thyroxine has about 5 times the potency of D-thyroxine*

2
THYROXINE BINDING GLOBULIN

• Glycoprotein synthesised by the liver*

• Each molecule binds one molecule of thyroxine

• Binds 70% of T3 and T4

• Other thyroxine binding proteins are thyroxine binding pre-albumin and


albumin

• Only 0.03% of T4 and 0.3% of T3 is free and biologically active*

THYROID HORMONES IN PREGNANCY

• Hepatic synthesis of thyroxin binding globulin is increased*

• Total levels of T4 and T3 are therefore increased

• Levels of FREE T4 may fall slightly in the second and third trimester

• TSH levels fall in the first trimester as HGC levels increase - HGC has
TSH-like action. TSH levels increase in the third trimester to normal
non-pregnant levels*

• Pregnancy is characterised by relative iodine deficiency

• There is active uptake of iodine by the placenta*

• Increased GFR resulte in a 2 fold increase in iodine excretion by the


kidney*

• Plasma iodine levels fall in pregnancy and iodine uptake by the thyroid
increases 3 fold

• Recommended daily intake of iodine outside pregnancy is 200


micrograms. Minimum dietary requirement ~75 micrograms / day

3
FETAL THYROID FUNCTION

• Embryonic thyroid gland derived from the endoderm of the buccal


cavity

• Thyroxine production begins at the end of the first trimester and adult
concentrations are reached at term*

• T3 levels increase after 28 weeks but remain below adult levels

• TSH is low before 18 weeks

• Thyroid hormones essential for optimal development of the CNS,


lungs, gut, liver and regulation of carbohydrate and lipid metabolism

• TSH does not cross the placenta*

• T4 & T3 cross the placenta very poorly and there is a marked materno-
fetal gradient

• Placenta permeable to TRH, iodides, thioamides and TSH receptor


antibodies*

• Anti-thyroid drugs such as carbimazole and propylthiouracil cross the


placenta and may cause fetal / neonatal hypothyroidism and goitre.*

4
SYNTHESIS OF ADRENAL STEROIDS *****

• From cholesterol - many of the enzymes are cytochrome P450


dependent

• Initial step is conversion of cholesterol to pregnenolone - rate-limiting


step

• Cortisol is not stored and increased secretion requires de-novo


synthesis

CORTISOL

• Synthesised from cholesterol by cytochrome P450 dependent enzymes

• Bound in plasma to cortisol-binding globulin (transcortin) ? only free


cortisol is biologically active

• Plasma concentrations peak in the morning and are lowest at night*

• Plasma concentration increases in pregnancy and with oestrogen


therapy*

• Does not cross the placenta in significant amounts. Synthetic


corticosteroids such as betamethasone and dexamethasone cross the
placenta

• Free cortisol is filtered by the kidneys and urinary free cortisol is an


index of cortisol secretion

1
• Metabolised by the liver, conjugated as glucuronides and excreted in
urine. Urinary 17-hydroxycorticosteroids are also an index of cortisol
secretion*

• Can be synthesised from exogenous cortisone by 11-beta-


hydroxydehydrogenase

• Receptor located within the cytoplasm or nucleus. Effects are mediated


through regulation of DNA transcription

REGULATION

• Secretion is under direct control of ACTH from the anterior pituitary

• ACTH secretion is under control of corticotrophin releasing hormone


(CRH) from the hypothalamus*

• Cortisol has negative feed-back effect on ACTH and CRH secretion*

• Anti-diuretic hormone also stimulates ACTH secretion and cortisol


feeds-back to inhibit ADH secretion

• STRESS - stimulates ACTH and then cortisol production - exercise,


starvation, surgery, sepsis, excessive heat or cold*

• Administration of exogenous glucocorticoids suppress ACTH


production and reduce plasma cortisol concentration

2
METABOLIC EFFECTS OF CORTICOSTEROIDS

GLUCOSE

• Stimulation of the conversion of protein to glucose and increased


glycogen storage

• Increased gluconeogenesis, increased plasma glucose concentration,


increased glucose uptake by the liver and glycogen synthesis. *

• The rise in plasma glucose induced by cortisol is only partially sensitive


to insulin.

• Glucose uptake and utilisation is inhibited in tissues in which glucose


uptake is insulin-sensitive, including skeletal muscle*

• Stimulates appetite

PROTEIN

• Reduced cellular protein content - decreased amino acid transport into


cells (except hepatocytes)

• Mobilisation of protein from muscle*

• Increased plasma animo acid concentration

• Increased liver protein, plasma protein synthesis by the liver and


plasma protein concentration

3
FAT

• Mobilisation of fatty acids (Lipolysis) from adipose tissue*

• Increased plasma free fatty acid concentration

• Deposition of fat in specific tissue causes typical moon facies and


buffalo hump seen in cortisol excess

INFLAMMATION

• Anti-inflammatory

• Stabilisation of lysosomal membranes

• Decreased capillary permeability and increases vascular tone

• Decreased fever

• Suppression of T-cell activity

• Decreased cytokine production

4
BONE

• Decreased bone formation

• Decreased synthesis of 1,25-dihydroxy D3

FETUS

Facilitate in-utero maturation of lungs, gut, CNS, retina and skin. Increased surfactant
production by fetal lung

ADRENOCORTICAL DEFICIENCY *****

• Addison's disease - rare

• Secondary to damage to the adrenal gland, autoimmune disease or


pituitary damage

• Progressive weakness, fatigue, weight loss, hypoglycaemia and


hyponatraemia, HYPERkalaemia, dehydration and HYPOtension
including postural hypotension

• Increased pigmentation of the skin, including palmar creases and


mucous membranes

• Amenorrhoea / impotence

Also associated with diarrhoea / constipation, depression, fever

5
ADRENAL GLAND

• Medulla - produces catecholamines and cortex - produces steroids

• Cortex has three distinct zones:

1) Zona glomerulosa - aldosterone

2) Zona fasciculata - cortisol and some androgens

3) Zona reticularis - androgens and some cortisol

ALDOSTERONE SECRETION

• Decreased renal blood flow stimulates renin secretion by the


juxtaglomerular cells of the kidney

• Renin converts angiotensinogen to angiotensin I

• Angiotensin I is converted to angiotensin II & III by angiotensin converting


enzyme

• Angiotensin II & III are potent vasoconstrictors and stimulate the synthesis
and secretion of aldosterone from the zona glomerulosa of the adrenal cortex

REGULATION OF ALDOSTERONE SECRETION

• Hyperkalaemia - a rise in plasma potassium is the most potent stimulus for


aldosterone secretion

• The renin - angiotensin system

• Total body sodium - decreased extracellular fluid volume and decreased


sodium concentration

• ACTH

• stimulates aldosterone release

• Aldosterone is metabolised in the liver and excreted in the kidneys as


glucuronide conjugates

1
EFFECTS OF ALDOSTERONE

• ~50% is bound to plasma proteins

• Interacts with receptor within the cytoplasm which is translocated to the


nucleus to stimulate transcription. Takes up to 45 minutes for sodium
absorption to be altered

• Increases sodium resorption in the distal renal tubules while increasing


potassium and proton excretion

• Sodium resorption results in water absorption and a rise in extracellular fluid


volume with little change in sodium concentration

• Also increases sodium absorption from the GI tract and resorption from the
sweat and salivary gland ducts

2
CALCIUM METABOLISM IN PREGNANCY*****

• Calcium metabolism is dramatically altered by pregnancy and


lactation.

• The normal fetal skeleton accumulates approximately 30g of calcium


by term.

• 80% of accretion occurs in the third trimester, at a rate of about 250-


300 mg/day

• Total serum calcium levels fall early in pregnancy, due to hemodilution


and the consequent decline in serum albumin.

• Ionized calcium phosphate levels remain normal throughout pregnancy

• PTH levels fall to 10-30% of the mean nonpregnant range in the first
trimester but increase again to the normal range by term.

• Serum calcitonin levels increase during gestation partly due to


extrathyroidal synthesis in the placenta and breast *

• Total and free 1,25-dihydroxyvitamin D levels increase 2-fold in the


first trimester, then remain constant until term.

1
VITAMIN D*****

• Steroid compound synthesised in the skin from 7-dehydrocholesterol


following ultraviolet irradiation - Cholecalciferol

• Fat-soluble vitamin, well absorbed from the GI tract

• Converted to 25-hydroxycholecalciferol (25-hydroxy D3) in the liver

• 25-Hydroxy D3 converted to 1,25-dihydroxy D3 in the kidneys - this


conversion is dependent on parathyroid hormone

• 1,25-Dihydroxy D3 is the most potent form of vitamin D and not stored.


The liver stores sufficient vitamin D3 to last for several months.

• In the absence of PTH, 25-hydroxy D3 is converted to 24,25-dihydroxy


D3 which has little vitamin D activity

Action of 1,25-dihydroxy D3

• Increases calcium absorption from the GI tract

• Increases phosphate absorption from the GI tract

• Stimulates bone resorption

PARATHYROID HORMONE *****

• Peptide hormone (84 amino acids) produced by the CHIEF cells of the
parathyroid gland as a pre-prohormone converted to a prohormone
then to PTH

• Acts via a cell surface receptor (like all peptide hormones)

• Secretion is stimulated by HYPO-calcaemia (normal plasma calcium =


2.2 - 2.6mmol/L)

• Pulsatile release with increased secretion at night

• Half life in blood ~5min

2
• Secretion stimulated by catecholamines and dopamine and inhibited by
1,25-dihydroxy D3

• The response of PTH to hypocalcaemia appears to require normal


magnesium concentrations -premature babies with hypomagnesaemia
may develop hypocalcaemia as a result of low PTH

ACTIONS

• Increases calcium and phosphate absorption from the GI tract through


1,25 dihydroxy D3, no direct effects

• Increases calcium and phosphate resorption from bone - an acute


phase caused by osteolysis and a delayed phase caused by activation
of osteoclasts

• Decreases renal calcium excretion - increased absorption in the distal


tubule and collecting ducts

• Increased magnesium and proton resorption in the kidneys

• Increases renal phosphate excretion (decreases phosphate resorption


in the proximal tubule)

• Decreased resorption of sodium and potassium in the kidneys

• Stimulates conversion of 25-hydroxy D3 to 1,25-dihydroxy D3 in the


kidneys

• Net effect is to increase plasma calcium and decrease phosphate


concentrations

3
CALCITONIN *****

• Peptide hormone (32 amino acids) produced by the parafollicular cells


of the THYROID gland

• Secretion is stimulated by a rise in plasma calcium (normal plasma


calcium = 2.2 - 2.6mmol/L)

• Inhibits osteolysis and osteoclast activity

• Stimulates bone formation

• Net effect is to lower plasma calcium concentration

• Increases excretion of calcium, phosphate, sodium, potassium and


magnesium by the kidneys

Calcitonin injections may suppress appetite, nausea & vomiting*

4
Question 1: The following are metabolic effects of corticosteroids

a. Increased amino acid uptake by skeletal muscle


True False
b. Increased amino acid uptake by hepatocytes
True False
c. Mobilisation of fatty acids from adipose tissue
True False
d. Decreased plasma free fatty acid concentration
True False

Question 2: Thyroxine

a. Stimulates growth hormone secretion


True False
b. Is not essential for normal brain development during fetal life
True False
c. Has metabolic effects within 3 hours of administration
True False
d. Causes a fall in plasma triglyceride concentration
True False

Question 3: Parathyroid hormone

a. Is produced by the oxyphil cells of the parathyroid gland


True False
b. Is released in response to increased plasma calcium concentration
True False
c. Increases calcium absorption from the GI tract
True False
d. Increases phosphate absorption from the renal tubules
True False

1
Question 4: Thyroid hormones

a. Increase oxygen consumption in most metabolically active tissue


True False
b. Decrease the rate of absorption of carbohydrates in the gut
True False
c. Increase circulating cholesterol concentration
True False
d. Are essential for skeletal maturation
True False

Question 5: Cortisol

a. Secretion can be estimated from urinary cortisol excretion


True False
b. Is metabolised by the liver
True False
c. Is converted to cortisone by 11-beta-hydroxydehydrogenase
True False
d. Secretion by the adrenal cortex is controlled by direct stimulation by
corticotrophin-releasing hormone
True False

Question 6: The following would cause a rise in plasma cortisol


concentrations

a. Acute hypoglycaemia
True False
b. ACTH
True False
c. Surgical operation
True False
d. Sepsis
True False

2
Question 7: The following are metabolic effects of corticosteroids

a. Stabilisation of lysosomal membranes


True False
b. Increased capillary permeability
True False
c. Suppression of T-cell activity
True False
d. Reduction of fever
True False

Question 8: The following are metabolic effects of corticosteroids

a. Induction of a positive nitrogen balance


True False
b. Decreased appetite
True False
c. Increased bone formation
True False
d. Decreased synthesis of 1,25-dihydroxy vitamin D3
True False

Question 9: Vitamin D

a. Increases calcium absorption from the GI tract


True False
b. Has a receptor located within the cell membrane
True False
c. Inhibits osteoclast activity
True False
d. Conversion of 25-hydroxyD3 to 1,25-dihydroxyD3 is inhibited by
parathyroid hormone
True False

3
Question 10: With respect to thyroid function

a. Thyroxine-binding globulin is found exclusively within the thyroid gland


True False
b. Triiodothyronine is present mainly in the free form in the circulation
True False
c. A fall in thyroxine-binding globulin causes hypothyroidism
True False
d. Thyroxine is a major precursor of triiodothyronine
True False

Question 11: Thyroxine

a. Is not produced by the fetus until the third trimester


True False
b. Crosses the placenta readily
True False
c. Is produced from two molecules of tyrosine
True False
d. Is secreted mainly in the form of tri-iodothyronine
True False

Question 12: Vitamin D

a. Is a steroid compound
True False
b. Is a fat soluble vitamin
True False
c. Is poorly absorbed from the GI tract
True False
d. Is synthesised in the skin from 7-dehydrocholesterol following
ultraviolet irradiation
True False

4
Question 13: With respect to the adrenal gland

a. The adrenal medulla produces glucocorticoids


True False
b. Aldosterone is produced by the zona glomerulosa
True False
c. Cortisol is produced by the zona fasciculata
True False
d. Androgens are produced by the zona fasciculata and zona reticularis
True False

Question 14: Calcitonin

a. Suppresses appetite
True False
b. Is produced mainly by the thymus gland
True False
c. Stimulates bone resorption
True False
d. Concentration increases in pregnancy
True False

Question 15: The following are metabolic effects of corticosteroids

a. Stimulation of gluconeogenesis
True False
b. Stimulation of protein synthesis in muscle tissue
True False
c. Increased glycogen content of the liver
True False
d. Increased glucose utilisation by cells
True False

5
ADRENAL MEDULLA

• Part of the sympathetic nervous system - contains chromaffin cells


derived from the neural creast

• Produces catecholamines (85% adrenaline, 15% noradrenaline) from


tyrosine in association with pro-opiomelanocortin and other
neuropeptides

• Innervated by cholinergic pre-ganglionic fibres of the sympathetic


nervous system

• Catecholamines are stored within granules in association with ATP and


a protein called chromogranin

CATECHOLAMINES

SYNTHESIS

• Tyrosine - to dihydroxyphenylalanine - to dopamine - to noradrenaline


to adrenaline

1
Production is stimulated by

1) Activation of the sympathetic nervous system

2) Cortisol

3) Hypoglycaemia, hypothermia, hypotension / hypovolaemia, anoxia,


exercise and excitement

• Metabolised by the liver and kidneys into vanillymandelic acid and


metanephrines and excreted in urine. Only 2-3% is excreted in urine
unchanged

ACTION

• Cell surface receptors - beta1,2&3; alpha 1&2

GLUCOSE

• Stimulate hepatic glycogenolysis and gluconeogenesis; increased


glucose utilisation

• Inhibit insulin secretion and insulin-stimulated glucose uptake by


skeletal muscle

• Stimulate glucagon secretion

• Net result is a rise in plasma glucose concentration

2
FAT

• Stimulate fatty acid mobilisation from adipose tissue with increased


fatty acid oxidation and ketone production

CARDIOVASCULAR

• Increased cardiac out-put (heart rate and contractility) - Beta 1

• Increased arteriolar dilatation - Beta 2

• Constriction of renal, GI and cutaneous arterioles with reduction in


perfusion - alpha 1

SMOOTH MUSCLE

• Relaxation of GI, Urinary and bronchial smooth muscle - Beta 2

• Constriction of sphincters - alpha 1

3
OTHERS

• Thermogenesis

• Increased potassium uptake into cells - Beta 2

• Decreased renal sodium excretion through increased rennin secretion


and stimulation of aldosterone release

• Increased sweating, dilatation of the pupils and platelet aggregation -


Alpha 2

4
ISLET OF LANGERHANS

• Endocrine component of pancreas

• Hormones secreted directly into the blood stream

• About 1 million per pancreas

• Alpha cells - 25%, produce GLUCAGON

• Beta cells - 60%, produce INSULIN

• Delta cells - 10%. Produce PANCREATIC POLYPEPTIDE

INSULIN SYNTHESIS

• Initially synthesizes as a single polypeptide chain - pre-prohormone

• Signal peptide is cleaved to yield pro-insulin

• Two disulphide bonds are formed followed by cleavage of the


connecting C-peptide *

• Insulin is packaged into secretory granules containing 6 insulin


molecules per Zn molecule

• Secreted by exocytosis. Equimolar concentrations of Insulin and C-


peptide are secreted *

INSULIN *****

• Polypeptide, A & B chains joined by two disulphide bonds *

• Half-life ~ 6 minutes *

• Not protein bound

• Acts via cell-surface insulin receptor with tyrosine kinase activity

• Production can be measured by measuring plasma C-peptide


concentration

• The most important degradative system for insulin is insulin degrading


enzyme which is a highly conserved metalloendopeptidase requiring
Zn++for its proteolytic action

1
• Protein disulfide isomerase and cathepsin D are also involved in insulin
metabolism.

• The liver and the kidney are the principal sites for insulin clearance*.
Very little insulin appears in urine

• Pig insulin differs from human insulin by one amino acid *

• Bovine insulin differs from human insulin by 3 amino acids

Insulin secretion is stimulated by

• Glucose *

• Amino acids, in particular arginine and lysine. These potentiate


glucose-stimulated insulin secretion *

• Gastro-intestinal hormones - Gastrin, Gastric inhibitory peptide,


Secretin, Cholecystokinin *

• Other hormones - cortisol, growth hormone, glucagon* (indirectly


through increased glucose), progesterone and oestrogen to a lesser
extent

• Fatty acids have little effect on insulin secretion

• Fasting insulin levels are higher in pregnancy compared to non-


pregnancy *

• When glucose is given orally, a greater insulin response is elicited than


when it is administered intravenously *

• Insulin secretion in response to a glucose load is bi-phasic: an early


rise (phase 1) - release of available insulin; late rise (phase 2) -
synthesis of new insulin. Plasma insulin peaks 30-60 minutes after a
meal

Sympathetic stimulation inhibits insulin secretion *

2
METABOLIC EFFECTS

LIVER

• Increased hepatic glucose uptake and glycogen deposition by:

1) Increased activity of glucokinase -phosphorylates glucose, trapping it within the


cell

2) Increased activity of phosphofructokinase -catalyses second step in glycogen


synthesis

3) Increases activity of glycogen synthetase

4) Inhibits phosphorylase - catalyses glycogen breakdown

• Inhibits hepatic gluconeogenesis *

• Stimulates fatty acid production from glucose *

SKELETAL MUSCLE

Increased glucose uptake (stimulates GLUT 4 transporter) and glycogen


synthesis *

BRAIN

No effect on glucose uptake and utilisation by the brain *

PROTEIN SYNTHESIS

Anabolic hormone - increased amino acid uptake and protein synthesis

Increased mRNA translation and DNA transcription *

3
FAT SYNTHESIS

• Increased fatty acid synthesis and fat deposition. Stimulates glucose


use (oxidation) in preference to fatty acid metabolism *

ELECTROLYTES

• Stimulates the translocation of K+, Mg2+ and phosphate from the


extracellular to the intracellular space and stimulates their resorption in
the renal tubules *

GLUCAGON *****

• Polypeptide hormone (29 amino acids) produced by the alpha cells of


the islet of Langerhans

• Not significantly protein-bound, acts by cell-surface receptor which


activates adenylate cyclase, resulting in increased intracellular cyclic
AMP

• Half-life ~ 6 mins *

Secretion is stimulated by

• Hypoglycaemia

• Amino acids - alanine and arginine (the only factors which stimulate
glucagon as well as insulin secretion)

• Exercise

• Hyperglycaemia and free fatty acids inhibit glucagon secretion

ACTIONS

4
• Increased hepatic glycogenolysis - activates glycogen phosphorylase

• Increased hepatic gluconeogenesis (synthesis of glucose from amino


acids)*

• Inhibits hepatic glycogen synthesis

• Increased fatty acid mobilisation and beta-oxidation -ketogenic *

• Results in increased plasma glucose, free fatty acids, keto-acids but


decreased plasma amino acids *

• Minimal effects on peripheral adipose tissue or skeletal muscle

SOMATOSTATIN

• Peptide hormone containing 14 or 28 amino acids

• Produced by the hypothalamus and the delta cells of the islet of


Langerhans

• Inhibits growth hormone release

Pancreatic somatostatin production is stimulated by:

1) Increased glucose

2) Increased amino acids

3)Increased fatty acids

4)Hormones released from the upper GI tract

Pancreatic somatostatin has the following effects

• Inhibition of insulin and glucagon secretion

• Decreased motility of the stomach, duodenum and gall bladder

• Decreased secretion and absorption from the GI tract

5
Question 1: Glucagon

a. Secretion is stimulated by the amino acids alanine and arginine


True False
b. Secretion is stimulated by exercise
True False
c. Is a ketogenic hormone
True False
d. Acts by increasing intracellular cGMP
True False

Question 2: Catecholamines

a. Secretion is inhibited by hypoglycaemia


True False
b. Secretion is stimulated by hypothermia
True False
c. Function through receptors located in the cytoplasm
True False
d. Stimulate hepatic glycogen synthesis
True False

Question 3: Catecholamines

a. Stimulate glycogenolysis in skeletal muscle


True False
b. Stimulate hepatic gluconeogenesis
True False
c. Produce a rise in plasma glucose concentration
True False
d. Stimulate insulin secretion by the beta cells of the pancreas
True False

1
Question 4: Insulin

a. Decreases protein catabolism


True False
b. Stimulates glucose oxidation
True False
c. Inhibits lypolysis
True False
d. Stimulates potassium uptake by cells
True False

Question 5: Catecholamines

a. Cause constriction of GI sphincters by stimulation of beta-adrenergic


receptors
True False
b. Reduce blood flow to the kidneys
True False
c. Increase sodium excretion by the kidneys
True False
d. Reduce blood flow to the GI tract
True False

Question 6: The following stimulate insulin secretion

a. Gastric inhibitory peptide


True False
b. Growth hormone
True False
c. Free fatty acids
True False
d. Starvation
True False

2
Question 7: Glucagon

a. Is produced by the delta cells of the Islets of Langerhans


True False
b. Is a polypeptide hormone
True False
c. Secretion is stimulated by high glucose concentration
True False
d. Secretion is stimulated by free fatty acids
True False

Question 8: Insulin secretion is stimulated by

a. Arginine
True False
b. Glucagon
True False
c. Somatostatin
True False
d. Noradrenaline
True False

Question 9: Insulin

a. Production can be measured by measuring plasma C-peptide


True False
b. Increases glucose uptake by the liver
True False
c. Decreases glycogen synthesis by the liver
True False
d. Increases the activity of hepatic glucokinase
True False

3
Question 10: Insulin

a. Increases the activity of phosphorylase in the liver


True False
b. Decreases the activity of glycogen synthetase
True False
c. Increases the activity of phosphofructokinase
True False
d. Stimulates hepatic gluconeogenesis
True False

Question 11: Human insulin

a. Produced endogenously has a half life of about 30 minutes in the


circulation
True False
b. Is degraded in the kidney
True False
c. Facilitates glucose uptake in the brain
True False
d. Is present in lower concentrations in fasting pregnant women
compared to fasting non-pregnant controls
True False

Question 12: The adrenal medulla

a. Contains chromaffin cells


True False
b. Produces glucocorticoids
True False
c. Is innervated by cholinergic pre-ganglionic fibres
True False
d. Is innervated by the sympathetic nervous system
True False

4
Question 13: Glucagon

a. Has a half life of 5-10 minutes in the circulation


True False
b. Secretion is stimulated by cortisol
True False
c. Promotes protein breakdown
True False
d. Promotes fat synthesis
True False

Question 14: Insulin

a. Decreases fat deposition


True False
b. Increases protein synthesis and storage
True False
c. Stimulates amino acid uptake into cells
True False
d. Decreases mRNA translation
True False

Question 15: Insulin

a. Is a glycoprotein
True False
b. Is bound to serum albumin
True False
c. Has a half life of about 30 minutes within the circulation
True False
d. Has a receptor located in the cell membrane
True False

5
Question 1: The oestrogen receptor

a. Is localised to the nucleus only following ligand binding


True False
b. Does not undergo nucleocytoplasmic shuttling
True False
c. Binds heat shock protein 90 on activation
True False
d. Is regulated by phosphorylation
True False

Question 2: Tamoxifen

a. Has anti-oestrogen effects on the vaginal epithelium


True False
b. Has oestrogenic effects on bone
True False
c. Increases anti-thrombin III levels
True False
d. Decreases LDL cholesterol levels
True False

Question 3: The fetal adrenal cortex

a. Is not functional until after 12-15 weeks gestation


True False
b. Grows during the first trimester under the influence of maternal ACTH
True False
c. Grows in late pregnancy under the influence of fetal pituitary ACTH
True False
d. During intra-uterine life is independent of pituitary ACTH
True False

1
Question 4: The fetal adrenal cortex

a. Has an inner fetal zone which produces mainly cortisol


True False
b. Is rich in 3-beta hydroxysteroid dehydrogenase enzymes
True False
c. Produces large amounts of oestriol
True False
d. Produces mainly DHEA & DHEA-S
True False

Question 5: The androgen receptor

a. Has a greater affinity for cyproterone acetate than testosterone


True False
b. Has a greater affinity for spironolactone than dihydrotestosterone
True False
c. Localises to the nucleus only after ligand binding
True False
d. Has A & B isoforms
True False

Question 6: Human placental lactogen

a. Concentration in maternal blood is directly related to the functional


mass of the placenta
True False
b. Has a half life in blood of about 1h
True False
c. Increases mobilisation of maternal free fatty acids
True False
d. Has the same concentration in maternal blood as in fetal blood at term
True False

2
Question 7: During the synthesis of steroid hormones

a. The conversion of cholesterol to pregnenolone occurs in the smooth


endoplasmic reticulum
True False
b. The transfer of cholesterol from the outer to the inner mitochondrial
membrane is the rate-limiting step
True False
c. Trophic hormones from the pituitary gland increase cholesterol uptake
True False
d. The majority of cholesterol used is from circulating plasma HDL
True False

Question 8: During the synthesis of steroid hormones

a. Testosterone is converted to oestrogens by 17-beta-hydroxysteroid


dehydrogenase
True False
b. Conversion of androgens to oestrogens occurs in the mitochondria
True False
c. The reactions converting pregnenolone to progesterone occur in the
rough endoplasmic reticulum
True False
d. Androstendione is converted to testosterone by 21-hydroxylase
True False

Question 9: Sex hormone binding globulin concentrations are increased


by

a. Insulin
True False
b. Growth hormone
True False
c. Progesterone
True False
d. Cortisol
True False

3
Question 10: The fetal adrenal cortex

a. Activity is regulated by inhibin and activin


True False
b. Cannot utilise cholesterol directly to produce sex steroids
True False
c. Converts LDL cholesterol to pregnenolone-sulphate
True False
d. Produces androstendione but not testosterone
True False

Question 11: With respect to the renin-angiotensin system in pregnancy

a. Angiotensinogen is produced by the ovary


True False
b. Angiotensinogen concentrations decrease in early pregnancy
True False
c. Renin is produced by the fetal membranes
True False
d. Maternal renin activity decreases during the first half of pregnancy
True False

Question 12: Tamoxifen

a. Is structurally related to diethylstilbestrol


True False
b. Has a higher affinity for the oestrogen receptor than oestrogen
True False
c. Kills breast cancer cells
True False
d. Increases FSH levels in post-menopausal women
True False

4
Question 13: Atrial natriuretic peptide

a. Is a smooth muscle relaxant


True False
b. Levels increase during labour
True False
c. Levels decrease in the third trimester
True False
d. Is secreted by the placenta
True False

Question 14: Human chorionic gonadotrophin (HCG)

a. Production is stimulated by progesterone


True False
b. Production is stimulated by inhibin
True False
c. Production is inhibited by GnRH
True False
d. Has a half life of 2h
True False

Question 15: Human chorionic gonadotrophin (HCG)

a. Produced by the placenta is non-glycosylated


True False
b. Inhibits androgen production by the fetal testes
True False
c. Takes over the function of supporting the corpus luteum from LH at 12
weeks gestation
True False
d. Is produced mainly by the cytotrophoblasts
True False

5
Question 1: Human chorionic gonadotrophin

a. Binds to the leutenising hormone receptor


True False
b. Has a half life in blood of about 2h
True False
c. Is produced by the pre-implantation blastocyst
True False
d. May be produced in tissues other than trophoblast
True False

Question 2: The corpus luteum

a. Is made up of cells rich in smooth endoplasmic reticulum


True False
b. Is about 5cm in diameter
True False
c. Degenerates into the corpus albicans after 7 days
True False
d. Has a yellowish appearance
True False

Question 3: With respect to follicular development and ovulation

a. Leutenising hormone has a synergistic effect with follicle stimulating


hormone on the growth of granulosa cells
True False
b. A dominant follicle is selected by day 7 of a 28 day menstrual cycle
True False
c. Ovulation typically occurs on day 14 of a 28 day menstrual cycle
True False
d. The released oocyte is surrounded by granulosa cells which form a
layer called the zona pellucida
True False

1
Question 4: With respect to follicular development and ovulation

a. FSH stimulates the growth of 6-12 mature follicles per cycle


True False
b. Conversion of the primordial follicle to the primary follicle is dependent
on leutenising hormone
True False
c. The oocyte secretes follicular fluid into the follicular antrum
True False
d. The theca interna and externa are the inner and outer layers of
granulosa cells
True False

Question 5: With respect to oestrogens

a. Oestriol is converted to oestradiol by the liver


True False
b. Oestradiol circulate bound to plasma proteins
True False
c. Oestrogens are synthesised from cholesterol
True False
d. Oestrogens are secreted by the adrenal medulla
True False

Question 6: The corpus luteum

a. Is formed mainly from the cells of the theca interna


True False
b. Secretes progesterone
True False
c. Secretes oestrogen
True False
d. Is only formed if fertilisation occurs
True False

2
Question 7: During the sexual act in the male

a. Erection is caused by stimulation of the sympathetic nervous system


True False
b. Secretions from the urethral and bulbourethral glands is stimulated by
activation of the parasympathetic nervous system
True False
c. Ejaculation is under stimulation of the sympathetic nervous system
True False
d. Sympathetic impulses that stimulate ejaculation leave the spinal cord at
the level of S2,3&4
True False

Question 8: Leptin

a. Acts on the hypothalamus


True False
b. Stimulates neuropeptide Y expression
True False
c. Increases GnRH secretion
True False
d. Has 8 amino acids
True False

Question 9: Follicle stimulating hormone (FSH)

a. Has a receptor which is located within the cytosol


True False
b. Concentrations are elevated during childhood
True False
c. Concentrations are elevated during the menopause
True False
d. Secretion is inhibited by inhibin
True False

3
Question 10: The following hormones stimulate the proliferation and
branching of breast ducts

a. Prolactin
True False
b. Growth hormone
True False
c. Cortisol
True False
d. Progesterone
True False

Question 11: Oestrogen

a. Inhibits milk secretion by the breasts


True False
b. Stimulates osteoblastic activity
True False
c. Delays fusion of the epiphyses of long bones
True False
d. Stimulates fat deposition in sub-cutaneous tissue
True False

Question 12: Leutenising hormone (LH)

a. Stimulates progesterone secretion by the granulosa cells


True False
b. Induces the conversion of granulosa cells into lutein cells
True False
c. Production is stimulated by oestrogen during the pre-ovulatory phase
of the menstrual cycle
True False
d. Concentrations rise within a few days of the onset of menstruation
True False

Question 13: 17-beta-oestradiol

4
a. Is thrombogenic
True False
b. Does not bind to albumin
True False
c. Is active when administered orally
True False
d. Is formed in tissues other than the ovary
True False

Question 14: Human chorionic gonadotropin (HCG)

a. Stimulates oestrogen and progesterone secretion by the corpus


luteum
True False
b. Stimulates testosterone secretion by the testis
True False
c. Results in decreased thyroxin secretion in early pregnancy
True False
d. Stimulates production of dehydroepiandrosterone sulphate by the fetal
zone of the adrenal gland
True False

Question 15: With respect to follicular development and ovulation

a. Granulosa cells secrete oestrogen


True False
b. Granulosa cells secrete inhibin
True False
c. Granulosa cells secrete human chorionic gonadotropin
True False
d. Oestrogen reduces the number of FSH receptors on granulosa cells
True False

5
..

Question 1: Growth hormone

a. Increases the rate of glucose utilisation in the body


True False
b. Decreases amino acid uptake by cells
True False
c. Is a ketogenic hormone
True False
d. Is a diabetogenic hormone
True False

Question 2: Growth hormone

a. Secretion is inhibited by exercise


True False
b. Secretion is stimulated by an increase in plasma amino acid
concentration
True False
c. Secretion is stimulated by trauma
True False
d. Secretion is inhibited by a rise in plasma free fatty acid concentration
True False

Question 3: Prolactin

a. e) Secretion is stimulated by breast-feeding


True False
b. f) Plasma concentrations fall during the third trimester of pregnancy
True False
c. g) Secretion increases with physiological stress
True False
d. h) Plasma concentrations fall if the connections of the pituitary gland to
the hypothalamus are disrupted
True False

1
Question 4: Antidiuretic hormone

a. Secretion is stimulated by dehydration


True False
b. Causes a decrease in urine osmolarity
True False
c. Causes an increase in plasma osmolarity
True False
d. Decreases water excretion by the kidneys
True False

Question 5: Adrenocorticotropic hormone (ACTH)

a. Is secreted in conjunction with beta-endorphin


True False
b. Stimulates adrenal androgen secretion
True False
c. Has no effect on adrenal aldosterone secretion
True False
d. Binds to a receptor located within the cell nucleus
True False

Question 6: Adrenocorticotrophic hormone (ACTH)

a. Is produced by the cytotrophoblast


True False
b. Production by the placenta is stimulated by oxytocin
True False
c. Production by the placenta is inhibited by dexamethasone
True False
d. Production by the pituitary is inhibited by vasopressin
True False

2
Question 7: Arginine vasopressin

a. Controls water resorption by the kidneys


True False
b. Is an oligopeptide
True False
c. Release is increased by haemorrhage
True False
d. Plasma concentration is reduced in pregnancy
True False

Question 8: Arginine vasopressin

a. Reduces glomerullar filtration rate


True False
b. Controls water loss in the proximal convoluted tubule
True False
c. Is released in response to a rise in plasma osmolarity
True False
d. Is released in response to a fall in plasma volume
True False

Question 9: Oxytocin

a. j) Is produced locally by the ovary and testis


True False
b. k) In high concentrations causes diuresis (
True False
c. l) Production is stimulated by emotional stress
True False
d. g) Basal plasma levels are undetectable in males
True False

3
Question 10: Thyroid stimulating hormone

a. Secretion is stimulated by somatostatin from the hypothalamus


True False
b. Secretion is stimulated by cortisol
True False
c. Secretion is inhibited by growth hormone
True False
d. Inhibits protein synthesis in the thyroid gland
True False

Question 11: Oxytocin

a. Is produced mainly by the supra-optic nucleus of the hypothalamus


True False
b. Has 9 amino acids
True False
c. Is a glycoprotein
True False
d. Secretion is stimulated by suckling
True False

Question 12: Adrenocorticotrophic hormone (ACTH)

a. Production is governed by the hypothalamus


True False
b. Production is maximal about midnight
True False
c. Is present in the placenta
True False
d. Is increased in the maternal plasma in pregnancy
True False

4
Question 13: Corticotropin-releasing hormone

a. Is a glycoprotein
True False
b. Is secreted by the hypothalamus
True False
c. Inhibits growth hormone release
True False
d. Acts by binding to a cytosolic receptor
True False

Question 14: Oxytocin

a. Is released episodically
True False
b. Causes decreased renal tubular absorption of water
True False
c. Causes milk ejection
True False
d. Reduces intestinal peristalsis
True False

Question 15: Prolactin

a. Is produced by anterior pituitary lactotrophs


True False
b. Is a glycoprotein
True False
c. Has a similar structure to thyroid stimulating hormone
True False
d. Secretion is stimulated by oestrogen
True False

5
Question 1: The following are metabolic effects of corticosteroids

a. Increased amino acid uptake by skeletal muscle


True False
b. Increased amino acid uptake by hepatocytes
True False
c. Mobilisation of fatty acids from adipose tissue
True False
d. Decreased plasma free fatty acid concentration
True False

Question 2: Thyroxine

a. Stimulates growth hormone secretion


True False
b. Is not essential for normal brain development during fetal life
True False
c. Has metabolic effects within 3 hours of administration
True False
d. Causes a fall in plasma triglyceride concentration
True False

Question 3: Parathyroid hormone

a. Is produced by the oxyphil cells of the parathyroid gland


True False
b. Is released in response to increased plasma calcium concentration
True False
c. Increases calcium absorption from the GI tract
True False
d. Increases phosphate absorption from the renal tubules
True False

1
Question 4: Thyroid hormones

a. Increase oxygen consumption in most metabolically active tissue


True False
b. Decrease the rate of absorption of carbohydrates in the gut
True False
c. Increase circulating cholesterol concentration
True False
d. Are essential for skeletal maturation
True False

Question 5: Cortisol

a. Secretion can be estimated from urinary cortisol excretion


True False
b. Is metabolised by the liver
True False
c. Is converted to cortisone by 11-beta-hydroxydehydrogenase
True False
d. Secretion by the adrenal cortex is controlled by direct stimulation by
corticotrophin-releasing hormone
True False

Question 6: The following would cause a rise in plasma cortisol


concentrations

a. Acute hypoglycaemia
True False
b. ACTH
True False
c. Surgical operation
True False
d. Sepsis
True False

2
Question 7: The following are metabolic effects of corticosteroids

a. Stabilisation of lysosomal membranes


True False
b. Increased capillary permeability
True False
c. Suppression of T-cell activity
True False
d. Reduction of fever
True False

Question 8: The following are metabolic effects of corticosteroids

a. Induction of a positive nitrogen balance


True False
b. Decreased appetite
True False
c. Increased bone formation
True False
d. Decreased synthesis of 1,25-dihydroxy vitamin D3
True False

Question 9: Vitamin D

a. Increases calcium absorption from the GI tract


True False
b. Has a receptor located within the cell membrane
True False
c. Inhibits osteoclast activity
True False
d. Conversion of 25-hydroxyD3 to 1,25-dihydroxyD3 is inhibited by
parathyroid hormone
True False

3
Question 10: With respect to thyroid function

a. Thyroxine-binding globulin is found exclusively within the thyroid gland


True False
b. Triiodothyronine is present mainly in the free form in the circulation
True False
c. A fall in thyroxine-binding globulin causes hypothyroidism
True False
d. Thyroxine is a major precursor of triiodothyronine
True False

Question 11: Thyroxine

a. Is not produced by the fetus until the third trimester


True False
b. Crosses the placenta readily
True False
c. Is produced from two molecules of tyrosine
True False
d. Is secreted mainly in the form of tri-iodothyronine
True False

Question 12: Vitamin D

a. Is a steroid compound
True False
b. Is a fat soluble vitamin
True False
c. Is poorly absorbed from the GI tract
True False
d. Is synthesised in the skin from 7-dehydrocholesterol following
ultraviolet irradiation
True False

4
Question 13: With respect to the adrenal gland

a. The adrenal medulla produces glucocorticoids


True False
b. Aldosterone is produced by the zona glomerulosa
True False
c. Cortisol is produced by the zona fasciculata
True False
d. Androgens are produced by the zona fasciculata and zona reticularis
True False

Question 14: Calcitonin

a. Suppresses appetite
True False
b. Is produced mainly by the thymus gland
True False
c. Stimulates bone resorption
True False
d. Concentration increases in pregnancy
True False

Question 15: The following are metabolic effects of corticosteroids

a. Stimulation of gluconeogenesis
True False
b. Stimulation of protein synthesis in muscle tissue
True False
c. Increased glycogen content of the liver
True False
d. Increased glucose utilisation by cells
True False

5
Question 1: Glucagon

a. Secretion is stimulated by the amino acids alanine and arginine


True False
b. Secretion is stimulated by exercise
True False
c. Is a ketogenic hormone
True False
d. Acts by increasing intracellular cGMP
True False

Question 2: Catecholamines

a. Secretion is inhibited by hypoglycaemia


True False
b. Secretion is stimulated by hypothermia
True False
c. Function through receptors located in the cytoplasm
True False
d. Stimulate hepatic glycogen synthesis
True False

Question 3: Catecholamines

a. Stimulate glycogenolysis in skeletal muscle


True False
b. Stimulate hepatic gluconeogenesis
True False
c. Produce a rise in plasma glucose concentration
True False
d. Stimulate insulin secretion by the beta cells of the pancreas
True False

1
Question 4: Insulin

a. Decreases protein catabolism


True False
b. Stimulates glucose oxidation
True False
c. Inhibits lypolysis
True False
d. Stimulates potassium uptake by cells
True False

Question 5: Catecholamines

a. Cause constriction of GI sphincters by stimulation of beta-adrenergic


receptors
True False
b. Reduce blood flow to the kidneys
True False
c. Increase sodium excretion by the kidneys
True False
d. Reduce blood flow to the GI tract
True False

Question 6: The following stimulate insulin secretion

a. Gastric inhibitory peptide


True False
b. Growth hormone
True False
c. Free fatty acids
True False
d. Starvation
True False

2
Question 7: Glucagon

a. Is produced by the delta cells of the Islets of Langerhans


True False
b. Is a polypeptide hormone
True False
c. Secretion is stimulated by high glucose concentration
True False
d. Secretion is stimulated by free fatty acids
True False

Question 8: Insulin secretion is stimulated by

a. Arginine
True False
b. Glucagon
True False
c. Somatostatin
True False
d. Noradrenaline
True False

Question 9: Insulin

a. Production can be measured by measuring plasma C-peptide


True False
b. Increases glucose uptake by the liver
True False
c. Decreases glycogen synthesis by the liver
True False
d. Increases the activity of hepatic glucokinase
True False

3
Question 10: Insulin

a. Increases the activity of phosphorylase in the liver


True False
b. Decreases the activity of glycogen synthetase
True False
c. Increases the activity of phosphofructokinase
True False
d. Stimulates hepatic gluconeogenesis
True False

Question 11: Human insulin

a. Produced endogenously has a half life of about 30 minutes in the


circulation
True False
b. Is degraded in the kidney
True False
c. Facilitates glucose uptake in the brain
True False
d. Is present in lower concentrations in fasting pregnant women
compared to fasting non-pregnant controls
True False

Question 12: The adrenal medulla

a. Contains chromaffin cells


True False
b. Produces glucocorticoids
True False
c. Is innervated by cholinergic pre-ganglionic fibres
True False
d. Is innervated by the sympathetic nervous system
True False

4
Question 13: Glucagon

a. Has a half life of 5-10 minutes in the circulation


True False
b. Secretion is stimulated by cortisol
True False
c. Promotes protein breakdown
True False
d. Promotes fat synthesis
True False

Question 14: Insulin

a. Decreases fat deposition


True False
b. Increases protein synthesis and storage
True False
c. Stimulates amino acid uptake into cells
True False
d. Decreases mRNA translation
True False

Question 15: Insulin

a. Is a glycoprotein
True False
b. Is bound to serum albumin
True False
c. Has a half life of about 30 minutes within the circulation
True False
d. Has a receptor located in the cell membrane
True False

5
Question 1: The oestrogen receptor

a. Is localised to the nucleus only following ligand binding


True False
b. Does not undergo nucleocytoplasmic shuttling
True False
c. Binds heat shock protein 90 on activation
True False
d. Is regulated by phosphorylation
True False

Question 2: Tamoxifen

a. Has anti-oestrogen effects on the vaginal epithelium


True False
b. Has oestrogenic effects on bone
True False
c. Increases anti-thrombin III levels
True False
d. Decreases LDL cholesterol levels
True False

Question 3: The fetal adrenal cortex

a. Is not functional until after 12-15 weeks gestation


True False
b. Grows during the first trimester under the influence of maternal ACTH
True False
c. Grows in late pregnancy under the influence of fetal pituitary ACTH
True False
d. During intra-uterine life is independent of pituitary ACTH
True False

1
Question 4: The fetal adrenal cortex

a. Has an inner fetal zone which produces mainly cortisol


True False
b. Is rich in 3-beta hydroxysteroid dehydrogenase enzymes
True False
c. Produces large amounts of oestriol
True False
d. Produces mainly DHEA & DHEA-S
True False

Question 5: The androgen receptor

a. Has a greater affinity for cyproterone acetate than testosterone


True False
b. Has a greater affinity for spironolactone than dihydrotestosterone
True False
c. Localises to the nucleus only after ligand binding
True False
d. Has A & B isoforms
True False

Question 6: Human placental lactogen

a. Concentration in maternal blood is directly related to the functional


mass of the placenta
True False
b. Has a half life in blood of about 1h
True False
c. Increases mobilisation of maternal free fatty acids
True False
d. Has the same concentration in maternal blood as in fetal blood at term
True False

2
Question 7: During the synthesis of steroid hormones

a. The conversion of cholesterol to pregnenolone occurs in the smooth


endoplasmic reticulum
True False
b. The transfer of cholesterol from the outer to the inner mitochondrial
membrane is the rate-limiting step
True False
c. Trophic hormones from the pituitary gland increase cholesterol uptake
True False
d. The majority of cholesterol used is from circulating plasma HDL
True False

Question 8: During the synthesis of steroid hormones

a. Testosterone is converted to oestrogens by 17-beta-hydroxysteroid


dehydrogenase
True False
b. Conversion of androgens to oestrogens occurs in the mitochondria
True False
c. The reactions converting pregnenolone to progesterone occur in the
rough endoplasmic reticulum
True False
d. Androstendione is converted to testosterone by 21-hydroxylase
True False

Question 9: Sex hormone binding globulin concentrations are increased


by

a. Insulin
True False
b. Growth hormone
True False
c. Progesterone
True False
d. Cortisol
True False

3
Question 10: The fetal adrenal cortex

a. Activity is regulated by inhibin and activin


True False
b. Cannot utilise cholesterol directly to produce sex steroids
True False
c. Converts LDL cholesterol to pregnenolone-sulphate
True False
d. Produces androstendione but not testosterone
True False

Question 11: With respect to the renin-angiotensin system in pregnancy

a. Angiotensinogen is produced by the ovary


True False
b. Angiotensinogen concentrations decrease in early pregnancy
True False
c. Renin is produced by the fetal membranes
True False
d. Maternal renin activity decreases during the first half of pregnancy
True False

Question 12: Tamoxifen

a. Is structurally related to diethylstilbestrol


True False
b. Has a higher affinity for the oestrogen receptor than oestrogen
True False
c. Kills breast cancer cells
True False
d. Increases FSH levels in post-menopausal women
True False

4
Question 13: Atrial natriuretic peptide

a. Is a smooth muscle relaxant


True False
b. Levels increase during labour
True False
c. Levels decrease in the third trimester
True False
d. Is secreted by the placenta
True False

Question 14: Human chorionic gonadotrophin (HCG)

a. Production is stimulated by progesterone


True False
b. Production is stimulated by inhibin
True False
c. Production is inhibited by GnRH
True False
d. Has a half life of 2h
True False

Question 15: Human chorionic gonadotrophin (HCG)

a. Produced by the placenta is non-glycosylated


True False
b. Inhibits androgen production by the fetal testes
True False
c. Takes over the function of supporting the corpus luteum from LH at 12
weeks gestation
True False
d. Is produced mainly by the cytotrophoblasts
True False

5
1

1 Adrenocorticotrophic hormone:
A Production is governed by the hypothalamus. T
B Production is maximal about midnight. F
C Is present in the placenta. T
D Is increased in the maternal plasma in pregnancy. T
E Secretion is inhibited by glucocorticoids. T

2 The hypothalamus is the site of synthesis of:


A Oxytocin. T
B Thyrotrophin-releasing hormone. T
C α- melanocyte-stimulating hormone F
D Luteinising hormone. F
E Gonadaotrophin-releasing hormone. T

3 Concerning the hypothalamopituitary axis:


A Secretion of luteinising hormone is independent of F
gonadotrophin-releasing hormone.
B Gonadotrophin releasing hormone neurons are absent in Kallman T
syndrome.
C Oestradiol has an acute positive feedback effect on T
gonadotrophin secretion.
D Oestradiol has a chronic negative feedback action on T
gonadotrophin secretion.
E The preovulatory rise in gonadotrophin-releasing hormone T
secretion is due to an increase in gonadotrophin releasing
hormone pulse frequency.

4 Corticotrophin releasing hormone:


A Is a polypeptide. T
B Is released from the median eminence of the hypothalamus. T
C Acts on the basophil cells of the anterior pituitary. F
D Release has a circadian variation. T
E Release is increased by painful stimuli. T

5 Ganadotrophin-releasing hormone:
A Is adimeric glycoprotein. F
B Is synthesised in the hypothalamus. T
C Stimulates synthesis of follicle-stimulating hormone. T
D Secretion is increased following castration. T
E Pulsatile administration inhabits ovarian steroid production. F

6 Deficient adrenocortical function causes an increase in:


A Blood pressure. F
B Blood glucose. F
C Skin pigmentation. T
2

D Serum sodium. F
E Plasma lipids. T

7 Adrenal androgens:
A Are synthesised in the zona glomerulosa of the adrenal cortex. F
B Are secreted in excessive amounts in the presence of 11β- T
hydroxylase deficiency.
C Stimulate protein synthesis. T
D Consist mainly of testosterone. F
E Are secreted in increased amounts in response to rise in T
adrenocorticotrophic hormone.

8 Recognized features of congenital adrenal hyperplasia in the


female are:
A Acute hypotension. F
B Enlargement of the clitoris. T
C Abnormal karyotype. F
D Hirsutism. T
E Absent uterus. F

9 In congenital adrenal cortical hyperplasia:


A The commonest deficiency is C 18hydroxylase. F
B Plasma cortisol concentration is rised. F
C Urinary excretion of 17-oxysteroids is elevated. T
D Dexamethasone will suppress urinary excretion of 17- T
oxysteroids.
E There are no virilising effects. F

10 In congenital adrenal hyperplasia:


A The commonest cause is a deficiency of 21-hydroxylase. T
B Plasma cortisol concentration is increased. F
C There may be excessive secretion of 17α-hydroxyprogesterone. T
D Sodium retention is characteristic. F
E Blood catecholamine concentrations are increased. F

11 Streroid hormones:
A All contains 20 carbon atoms. F
B Can be produced by structures of urogenital ridge origin. T
C Are mostly activated in the liver. F
D Are predominantly excreted unchanged in the urine. F
E Mainly circulate unbound to carrier proteins. F

12 The following substances are steroids:


A Aldoesterone. T
B Follicle-stimulating hormone F
C Vitamin D. T
D Inhibin. F
E Thyroxine. F
3

13 The following hormones bind to receptors on the cell


membrane:
A Corticosterone. F
B Adrenaline. T
C Luteinising hormone. T
D Oestradiol. F
E Gonadatrophin-releasing hormone. T

14 The following statements concerning the formation of


hormones are correct:
A ACTH is derived from pro-opiomelanocortin. T
B Oestrogens are derived from androgens. T
C Prolactin is derived from dopamine. F
D Melatonin is derived from serotonin. T
E Angiotensin II is derived from rennin. F

15 The release of catecholamines from the adrenal medulla


increases:
A During sleep in healthy individuals. F
B When the nerves to the adrenal gland are stimulated. T
C Following an increase in blood sugar. F
D Immediately following a myocardial infarction. T
E During acute haemorrhage. T

16 The release of catecholamines from the adrenal medulla


increases:
A During sleep in healthy individuals. F
B When the nerves to the adrenal gland are stimulated. T
C When the blood sugar rises. F
D Following a myocardial infarction. T
E In the presence of a phaeochromocytoma. T

17 Adrenaline (epinephrine):
A Stimulates myometrial contractions. F
B Exert its action by alpha receptors only. F
C Constricts the pupils. F
D Causes glycogenolysis. T
E Inhabits the mobilization of free fatty acids. F

18 In hypopituitarism:
A Clinical features of deficiency are usually absent until about 70% T
of the gland has been destroyed.
B Thyroid-stimulation hormone is usually the first hormone to be F
affected.
C Aldosterone secretion is normal. T
D Orthostatic hypotension is common. T
E Hypoglycaemia occurs on fasting. T
4

19 Pituitary gonadatrophin:
A Release is dependent upon hypothalamic function. T
B Secretion increases during pregnancy. F
C Blood levels are raised during lactational amenorrhoea. F
D Release in the puerperium is enhanced by bromocriptine. F
E Release is inhibited by oxytocin. F

20 Pulsatile secretion of pituitary luteinising hormone:


A Increase in amplitude during puberty. T
B Occurs in the first year of life. F
C Ceases at the menopause. F
D Is controlled by continuous secretion of gonadotrophin-releasing T
hormone.
E Is modified by the level of circulating oestrogen. F

21 Concerning thyroid function:


A Oestrogen increases the production of thyroxine-binding T
globulin.
B More than 98% of circulating thyroxine is bound to thyroxine- F
binding globulin.
C Thyrotrophin-releaseing hormone is a decapeptide. T
D Thyroid-stimulating hormone levels are increased in primary T
hypothyroidism.
E Thyroid-stimulating hormone is a glycoprotein. T

22 Concerning thyroid function:


A Thyroid-binding globulin is found entirely within the thyroid F
gland.
B Triiodothyronine is mainly in the free form in the circulation. F
C Triiodothyronine acts more rapidly than thyroxine. T
D A fall in thyroid-binding globulin causes hypothyroidism. F
E Thyroxine is a major precursor of Triiodothyronine. T

23 Thyroxine:
A Exist in the free state in thyroid gland. F
B Is more than 99% protein-bound in plasma. T
C Is released by the fetal thyroid. T
D In the unbound state in plasma is approximately equal in F
concentration to unbound Triiodothyronine.
E Is actively transported across the placenta. F

24 Thyroid hormones:
A Increase oxygen consumption in most metabolically active T
tissues.
B In the circulation are less than 90% bound to protein. F
C Decrease the rate of absorption of carbohydrate from the gut. F
D Increase the circulating cholesterol concentrations. F
E Are essential for skeletal maturation. F
5

25 Concerning thyroid hormones:


A Triiodothyronine is converted in the tissues to thyroxine. F
B The circulating concentration of thyroid binding globulin T
increases in pregnancy.
C Triiodothyronine acts more rapidly than thyroxine. T
D Starvation causes plasma triiodothyronine concentrations to rise. F
E D-thyroxine is more active than L-thyroxine. T

26 Iodine:
A Requirements are unchanged by pregnancy. F
B Uptake by the thyroid gland is increased by thyroid-stimulating T
hormone.
C Is excreted by the kidney. T
D Is bound to tyrosine in the thyroid gland. T
E May inhibit thyroxine synthesis. T

27 During normal pregnancy:


A Plasma thyroid-binding globulin concentration increases. T
B Plasma total thyroxine concentration falls. F
C Plasma TSH concentration increases. T
D Triiodthyronine readily crosses the placenta to the fetous. F
E Fetal thyroid function is largely dependant upon the function of F
the maternal thyroid.

28 Serum concentrations of the following increase during


pregnancy:
A Sex hormone-binding globulin. T
B Prolactin. T
C Total thyroxine. T
D Follicle-stimulating hormone. F
E Corticotrophin-releasing hormone. T

29 Parathyroid hormone:
A Is a polypeptide. T
B Increases bone resorption. T
C Decreases phosphate excretion in the urine. F
D Secretion is diminished by an increase in serum ionized calcium T
concentration.
E Decreases the formation of 1,25-dihydroxycholecalciferol. F

30 Parathyroid hormone:
A Decreases the renal excretion of phosphate. F
B Increases calcium resorption from bone. T
C Depresses pituitary activity. F
D Concentrations in blood are raised when the calcium level falls. T
E Increases renal tubular reabsorption of calcium. T
6

31 Parathyroid hormone:
A Increase bone resorption. T
B Concentrations are increase in pregnancy. T
C Reduces phosphate excretion in urine. F
D Increases the formation of 1,25-dihydroxycholecalciferol. T
E Stimulates osteoblasts. F

32 Calcitonin:
A Synthesised in the parathyroid glands. F
B Is a decapeptide. F
C Secretion is increased at serum calcium levels below 1.5 mmol/L F
(6.1mg/100mL).
D Inhibits bone resorption. T
E Increases renal tubular excretion of calcium. T

33 Calcitonin:
A Lower the basal metabolic rate. T
B Concentration is increased in pregnancy. T
C Is released when the blood phosphate level rises. F
D Is produced in the thyroid gland. T
E Release is stimulated by high calcium ion levels. T

34 Calcitonin:
A Increases the basal metabolic rate. F
B Increases the blood calcium level. F
C Is a steroid hormone. F
D Is produced in the parathyroid glands. F
E Release is stimulated by low calcium ion levels. F

35 Calcitonin:
A Suppresses appetite. T
B Is produced mainly in the thymus. F
C Lowers plasma calcium concentrations. T
D Secretion is inhibited by gastrin. F
E Inhibits bone resorption. T

36 In a healthy adult, serum calcium:


A Constitutes 15% of total body calcium. F
B Is not involved in the extrinsic system of blood coagulation. F
C Concentration will be lowered by calcitonin. T
D Concentration normally falls after menopause. F
E Is approximately 90% protein-bound. F

37 The secretion of growth hormone:


A Occurs in the hypothalamus. F
B Ceases when the adult state is reached. F
7

C Is decreased during stress. F


D Is increased during fasting. T
E Is increased with exercise. T

38 The secretion of growth hormone:


A Occurs in the hypothalamus. F
B Decreased during sleep. F
C Is decreased during stress. F
D Is decreased with hypoglycaemia. F
E Opposes the action of insulin. F

39 Concerning growth hormone:


A Plasma levels are reduced by glucose infusion. T
B Maternal plasma levels are directly related to fetal growth. F
C It is active on bone only until the epiphyses fuse. F
D Its secretion is controlled by the hypothalamus. T
E Increased activity produces a positive nitrogen balance. T

40 Growth hormone:
A Is a protein. T
B Has a molecular weight of 2000 daltons. F
C Secretion is stimulated by hyperglycaemia. F
D Has growth-promoting effects mediated through insuline-like T
growth factors.
E Is synthesised in the hypothalamus. T

41 Growth hormone:
A Promotes protein synthesis. T
B Faciliatates the hepatic synthesis of somatomedin C. T
C Promotes insulin-mediated uptake of glucose. F
D Increases circulating free fatty acids. T
E Stimulates epiphyseal fusion. F

42 The secretion of growth hormone:


A Occurs in the hypothalamus. F
B Ceases when adult state is reached. F
C Is decreased during stress. F
D Is increased during fasting. T
E Is increased with exercise. T

43 The actions of growth hormone include :


A Promotion of protein synthesis. T
B Facilitation of hepatic synthesis of somatonedin C (insulin-like T
growth factor).
C Promotion of the insulin-mediated uptake of glucose. F
D Stimulation of lipolysis. T
E Stimulation of the growth spurt at the onset of puberty. F
8

44 Concerning growth hormone:


A Plasma levels are reduced by glucose infusion. T
B Maternal plasma levels are directly related to fetal growth. F
C It is active on bone only until the epiphyses fuse. F
D It is secretion is controlled by the hypothalamus. T
E Increased activity produces a positive nitrogen balance. T

45 Successful lactation is:


A Maintained by oestrogens. F
B Maintained by progesterone. F
C Initiated by a prolactin surge. T
D Maintained by human placental lactogen. F
E Inhibited by dopamine. T

46 Prolactin:
A Release is stimulated by thyrotrophin-releasing hormone. T
B Plasma levels are raised in the first trimester. T
C Release is increased by suckling. T
D May be produced by the decidua. F
E Release is inhibited by metaclopramide. F

47 Prolactin:
A Is secreted by the hypothalamus. F
B Plasma levels are raised in the first trimester of pregnancy. T
C Is identical to placental lactogen. F
D Controls milk ejection. F
E Release is inhibited by dopamine. T

48 Prolactin:
A Release is stimulated by thyrotropin-releasing hormone. T
B Plasma levels are raised in the first trimester of pregnancy. T
C Is identical to human placental lactogen. F
D Control milk ejection. F
E Release is inhibited by metoclopramide. F

49 During human lactation:


A Oxytocin increases mammary duct pressure. T
B Oestrogen promotes the milk producing effects of prolactin on F
the breast.
C Human placental lactogen is essential for milk synthesis. F
D Prolactin stimulates gonadotrophin release. F
E Suckling enhances prolactin release. T

50 In human lactation:
A Oestrogens promote development of the breast lobules. F
B Oestrogens promote the milk-producing effect of prolactin on the F
breast.
C Human placental lactogen is essential for milk synthesis. F
9

D Prolactin stimulates gonadotrophin release. F


E Oxytocin causes milk ejection. T

51 Release of prolactin from the anterior pituitary gland is


enhanced by:
A Dopamine. F
B Thyrotrophin-releasing hormone. T
C Oestrogens. F
D Chlorpromazine. T
E Growth hormone. F

52 Human placental lactogen:


A Is a single-chain polypeptide. F
B Reaches the same concentration in fetal and maternal blood at F
term.
C May be secreted by the deciduas. F
D Is detectable only after the 25th week of pregnancy. F
E Is an insulin antagonist. T

53 Human placental lactogen:


A Concentration in maternal plasma is directly proportional to the F
functional mass of placenta.
B Has a half-life in blood of less than one hour. T
C Is a steroid hormone. F
D Increases the mobilization of maternal free fatty acids. T
E Reaches the same concentration in fetal blood as in maternal F
blood at term.

54 Luteinising hormone:
A Is required for normal corpus luteum survival. T
B Has a half-life in the circulation of 30 hours. F
C Is released in pulses. T
D In the male stimulates testosterone production. T
E Plasma concentrations are increased in postmenopausal women. T

55 Luteinising hormone:
A Is a glycoprotein. T
B Has a molecular weight of approximately 4000 daltons. F
C Stimulates androgen production by ovary. F
D Concentrations in the circulation are high in girls before puberty. F
E Concentrations in the circulation are raised in untreated adult T
with Turner syndrome.

56 Follicle-stimulating hormone:
A Binds to receptors on the cell membrane. T
B Promotes the expression of luteinising hormone receptors. T
C Is responsible for degeneration of the corpus luteum. F
D Is a steroid hormone. F
10

E Is synthesized in the hypothalamus. F

57 Plasma levels of follicle stimulating hormone are elevated in:


A Adults with Klinfelter syndrome. T
B Women taking oral contraceptive preparations. T
C Postmenopausal women. T
D Adults with Turner syndrome (45X0). T
E Pregnancy. F

58 Serum concentrations of follicle-stimulating hormone are:


A Increased in Kleinfelter syndrome. T
B Decreased in primary hypogonadism. T
C Decreased following the menopause. F
D Increased with oestrogen therapy. F
E Increased following male castration. T

59 Oxytocin:
A Is released episodically. F
B Causes decreased renal tubular reabsorption of water. F
C Is responsible for milk ejection. T
D Reduces intestinal peristalsis. F
E Inhibits prolactin secretion. F

60 In the human, Oxytocin:


A Promote milk synthesis in the breast. F
B Release is inhibited by alcohol. T
C Action on the uterus is enhanced by oestrogens. F
D Release is inhibited by dopamin. F
E In excess causes dehydration. F

61 The following agents cause myometrial contractions:


A Magnesium sulphate. F
B Nifidipine. F
C Progesterone. F
D Salbutamol. F
E Sodium nitroprusside. T

63 Rennin:
A Is secreted by the zona glomerulosa of the adrenal cortex. F
B Is a proteolytic enzyme. T
C Is secreted at an increased rate if the renal perfusion pressure T
falls.
D Acts upon circulating angiotensinogen. T
E Is released in response to an increase in extracellular fluid F
volume.

64 In the rennin-angiotensin system:


A Decreased renal blood flow causes increased rennin production. T
11

B Rennin is secreted by the juxta-glumerular cells of kidney. T


C Rennin causes production of angiotensin I. T
D Angiotensin I is a decapeptide. F
E Angiotensin II suppresses aldosterone secretion. F

65 In the rennin-angiotensin system:


A Rennin is an enzyme. T
B Angiotensin II is converted to angiotensin I. F
C Angiotensinogen is globulin. T
D Rennin release is inhibited by sodium restriction. T
E Rennin is present in amniotic fluid. T

66 Concerning the rennin-angiotensin system:


A Rennin is secreted by the proximal tubule. F
B Rennin is responsible for the conversion of angiotensin I to F
angiotensin II.
C Angiotensin II is a potent presser agent. T
D The presser effect of angiotensin II is suppressed in normal T
pregnancy.
E Angiotensin II increases the secretion of aldosterone from the F
adrenal cortex.

67 Aldosterone:
A Reduces sodium reabsorption in the proximal convoluted tubule. F
B Reduces sodium absorption in the descending loop of Henle. F
C Increases sodium absorption in the distal convoluted tubule. T
D Increase potassium loss from the tubule. T
E Increases sodium absorption in the collecting tubule. T

68 Aldasterone:
A Secretion is increased by a low potassium intake. F
B Production is reduced in normal pregnancy. F
C Secretion is entirely regulated by the rennin-angiotensin system. F
D Is a principle mineralcorticoid secreted by the adrenal gland. F
E In the kidney, acts on the distal convoluted tubule. T

69 Aldosterone secretion is increased:


A On standing. T
B Following haemorrhage. T
C During pregnancy. T
D By hypocalcaemia. F
E On a low sodium diet. T

62 Antidiuretic hormone:
A Is normally synthesised in the hypothalamus. T
B Is secreted by the posterior pituitary gland. T
C Exerts its effect by reducing the glomerular filtration rate. F
D Is an oligapeptide. F
12

E Secretion is increased when plasma osmolality falls. F

70 Argentine vasopressin:
A Reduces the glomerular filtration rate. F
B Controls water loss in the proximal renal tubule. F
C Is synthesized by the posterior pituitary gland. F
D Is released in response to a rise in plasma osmolality. T
E Is released in response to a fall in circulating plasma volume. T

71 Argenin vasopressin:
A Is produced in the hypothalamus. T
B Is a polypeptides. T
C Is structurally similar to prolactin. F
D Controls water reabsorption by kidney. T
E Decreases glomerular filtration. F

72 Vasopressin:
A Is a nanopeptide. T
B Is synthesised in the posterior pituitary gland. F
C Release is increased when plasma osmolality rises. T
D Release is increased by haemorrhage. T
E Plasma concentration is reduced in pregnancy. F

73 Human chorionic gonadotrophin:


A Is a glycoprotein. T
B Secretion peaks at 20 week of gestation. F
C Has intrinsic anti-thyroid activity. F
D Is synthesised by the corpus luteum of pregnancy. F
E Binds to luteinising hormone receptors. T

74 Human chorionic gonadotrophin:


A Is not produced by the decidua. T
B Is biochemically indistinguishable from luteinising hormone. F
C Is active if given to nonpregnant women. T
D Production rises steadily throughout pregnancy. F
E Has no influence upon the production of oestrogens by the T
placenta.

75 Human chorionic gonadotrophin:


A Has a half-life in blood of two hours. F
B Contains a β chain indistinguishable from that of human F
luteinising hormone.
C Is produced by the preimplantation blastocyst. F
D Is responsible for the maintenance of the corpus luteum in early T
pregnancy.
E May be produced in tissues other than trophoblast. F
13

76 Human chorionic gonadotrophin:


A Is a steroid hormone. F
B Is partly metabolized in the kidney. F
C Has an α-subunit which is antigenically similar to β-subunit of F
luteinising hormone.
D Reaches peak concentrations at the 17th week of pregnancy. F
E Is secreted in trophoblastic diseases. T

77 Serum concentration of the following increase during


pregnancy:
A Sex hormone-binding globulin. T
B Prolactin. T
C Total thyroxine. T
D Follicle-stimulating hormone. F
E 17α-hydroxyprogesterone. T

78 During pregnancy, uterine deciduas synthesized:


A Human chorionic gonadotrophin. F
B Prostaglandin E2. T
C Progesterone. F
D Prolactin. T
E Oxytocin. T

79 Products of human decidua include:


A α-fetoprotein. F
B Prolactin. T
C Human chorionic gonadatrophin. F
D Human placental lactogen. F
E Prostaglandin F2α. T

80 Trophoblast:
A Develops from the blastocyst. T
B Gives rise to the fetal blood vessels in the placenta. T
C Enters the maternal circulation during normal pregnancy. T
D Replaces endothelium of pregnant spiral arterioles. T
E Is genetically identical to decidua. F

81 The following cell types are present in the human corpus


luteum:
A Endothelial cells. T
B Macrophages. T
C Pericytes. T
D Fibroblasts. T
E Granulose cells. T

82 The corpus luteum of pregnancy produces:


A Relaxin. T
B Progesterone. T
14

C 17α-hydroxyprogesterone. F
D Human chorionic gonadotrophin. F
E Oestradiol. T

83 Concerning puberty in the female:


A The development of secondary sexual characteristics is preceded F
by increased production of adrenal androgens.
B Puberty is associated with the nocturnal release of luteinising T
hormone.
C The ovaries are not sensitive to gonadotrophins before puberty. F
D Appearance of pupic hair usually precedes menarche. T
E Appearance of pupic hair precedes the onset of breast F
development.

84 Concerning the human ovary:


A Aromatase catalyses the conversion of testosterone to 17β- T
oestradiol.
B Relaxin is synthesised in the corpus luteum. T
C Inhabin stimulates follicle-stimulating hormone release from the F
pituitary gland.
D Progesterone decreases the sensitivity of myometrial cells to T
oxytocin.
E Cholesterol is converted to androstenedione in granulose cells. T

85 Concerning ovarian function:


A Progesterone is the major steroid of the developing follicle. F
B Granulose cells secrete oestradiol. T
C Oestradiol is derived from androgen precursors. T
D Insulin-like growth factor (IGF)-1 is not secreted by the ovary. T
E Circulating inhibin concentrations are a marker of granulose cell F
function.

86 Concerning ovarian function :


A Progesterone is the major steroid of the developing follicle. F
B Granulose cells secrete oestradiol. T
C Oestradiol is derived from androgen precursors. T
D Insulin-like growth factor (somatomedin C) is not secreted by the F
ovary.
E Circulating inhibin concentrations are a marker of granulose cell F
function.

87 Ovaries secrete:
A Progesterone. T
B Androstenedione. T
C Testosterone. T
D 17β-oestradiol. T
E Aldosterone. F

88 Inhibin:
15

A Is structurally identical to relaxin. F


B Is released in pulses. T
C Is a steroid. F
D Is produced by the ovarian follicle. T
E Inhibit the release of follicle-stimulating hormone. T

89 During the normal ovarian cycle:


A The principle oestrogen secreted is 17-Oestradiol. T
B The most potent oestrogen is oestriol. F
C Oestrogen production is maximal by about the 8th day of the F
cycle.
D Oestrogens decrease the secretion of follicle-stimulating T
hormone.
E Oestrogens are synthesized primarily by the ovarian stroma. F

90 Progesterone:
A Is synthesised by trophoblast. T
B Increase myometrial activity. F
C Is predominantly excreted in the urine as pregnanetriol. F
D Binds to cortisol-binding globulin in the circulation. T
E Is synthesised from cholesterol. T

91 Progesterone:
A Is a C -21 steroid. T
B Is synthesised by the ovary before ovulation. T
C Increases ventilation. T
D Rises basal body temperature. T
E Binds to corticotrophin-binding globulin. T

92 Concerning sex hormones:


A The ovary secretes androstenedione. T
B The ovary secretes testosterone. T
C The ovary secretes dihydrotestosterone. T
D Sex hormone-binding globulin (SHBG) concentrations are higher T
in women than in men.
E Androgens bound to protein have high biological activity. F

93 Sex hormone-binding globulin:


A Levels are increased in pregnancy. T
B Is the main binding protein for progesterone. F
C Levels are decreased during oestrogen therapy. F
D Is the main binding protein for aldosterone. F
E Has a greater affinity than albumin for testosterone. F

94 The physiological action of oestradiol depends upon:


A Metabolism to a more potent substance. F
B Binding to an intracellular receptor. T
16

C Alteration of gene expression. T


D Active transport of the hormone into cells. F
E Cyclic AMP production. F

95 In the human testis:


A Meiosis occurs between the early and late spermatid phases. F
B One spermatogonium always forms 8 sprmatides. F
C The fluid in the seminiferous tubules contains a high F
concentration of protein.
D The process of spermatogenesis takes 34 days. F
E Inhibin is produced by primary spermatocytes. F

96 The interstitial (leydig) cells of testis:


A Secrete seminal fluid. F
B Are stimulated by luteinising hormone. T
C Secrete androgen binding protein. F
D Secrete fructose. F
E Produce testosterone. T

97 Concerning testicular hormone:


A Testosterone reduces plasma luteinising hormone concentrations. T
B Inhibin stimulates luteinising hormone production. F
C Oestrogens are formed in the testis. T
D Testosterone is converted to hydrotestosterone by 5α-reductase. T
E Testosterone in plasma is predominantly bound to albumin. F

98 In the human testis:


A The main site of testosterone synthesis is the sertoli cell. F
B Testosterone synthesis is stimulated by follicle stimulating F
hormone (FSH).
C The predominant androgen product is androstenedione. F
D The Leydig cells synthesize testosterone. T
E Inhibin is synthesized by the sertoli cell. T

99 The following statements about testicular hormones are true:


A Testosterone reduces plasma luteinising hormone levels. T
B Inhibin increases plasma follicle-stimulating hormone levels. F
C Oestrogens are formed in the testis. T
D Testosterone is excreted in urine as 17-ketosteroids. F
E Testosterone in plasma is partly bound to albumin. T

100 Concerning sex hormone:


A The ovary secretes androstenedione. T
B The ovary secretes testosterone. T
C The ovary secretes dihydroxycholecalciferol. F
D Sex hormone-binding globulin (SHBG) concentrations are higher T
in women than men.
E Androgens bound to protein have high biological activity. F
17

101 Hirsutism in women is characteristically associated with:


A Testicular feminization. F
B Turner syndrome. F
C Polycystic ovary syndrome. T
D Arrhenoblastoma. T
E Hypopituitarism. F

102 Hirsutism is associated with:


A Testicular feminization syndrome. F
B Turner syndrome. F
C Addison's disease. T
D Sertoli-leydig cell tumors. F
E Congenital adrenal hyperplasia. T

103 Testosterone:
A Is produced only in the gonads. F
B Is mainly excreted unchanged in the urine. F
C Stimulates secretion of luteinising hormone. F
D Circulates in plasma mainly in the free form. F
E Stimulates growth of the prostate gland. T

104 Testosterone in the human male:


A Depresses pituitary secretion of luteinising hormone. T
B Promotes growth of scalp hair. F
C Promotes union of long bone epiphyses. F
D Is a more potent androgen than dihydrotestosterone. F
E Is secreted maximally in the evening. T

105 In the human male, dihydrotestesterone:


A Is a precursor of testosterone. T
B Has one-tenth of the potency of testosterone. F
C Is responsible for involution of the Mullerian system. F
D Is responsible for the development of the male external genitalia. T
E Binds to an intracellular receptor. T

106 Oestradiol-17β:
A Is synthesized by aromatisation of testosterone. T
B Vasodilates the uterine artery. T
C Suppresses uterine activity by upregulating the oxytocin F
receptor.
D Promotes secondary sexual hair growth in females. F
E Is thrombogenic. T

107 Oestradiol-17β:
A Is synthesized by aromatisation of testosterone. T
B Can be administered orally. T
C Suppresses uterine activity by upregulating the oxytocin F
18

receptor.
D Promotes secondary sexual hair growth in females. F
E Is thrombogenic. T

108 After the menopause:


A The plasma concentration of follicle stimulating hormone T
increases.
B The plasma progesterone concentration increases. F
C Oestrone is the oestrogen found in highest concentration in the T
plasma.
D The plasma testosterone concentration doubles. F
E The plasma prolactin concentration increases. F

109 Menopause:
A Is due to failure of the endometrium to respond to oestrogens. F
B Is associated with cessation of steroidogenesis within the ovary. F
C Is associated with a fall in circulating luteinising hormone. F
D Is preceded by a period of enhanced fertility. F
E Results in atrophy of the epithelium of the distal urethra. T
Comment :
110 Concerning androgens in normal premenopausal women:
A 95% of circulating testosterone is derived from peripheral F
conversion of androstenedione.
B The ovaries and adrenals contribute equally to circulating F
androgen concentrations.
C DHEAS (dehydroepiandrosterone sulphate) is derived almost T
exclusively from adrenal glands.
D About50% of circulating testosterone is bound to sex hormone- F
binding globulin (SHBG).
E Testosterone promotes the synthesis of SHBG. F

111 In a woman of reproductive age, serum concentrations of the


following hormones exhibit a recognized pattern of diurnal
variation:
A Progesterone. F
B Melatonin. T
C Cortisol. T
D Oestrone. F
E Follicle-stimulating hormone. F

112 Insulin secretion is stimulated by:


A Gastrin. T
B Noradrenaline (norepinephrine). F
C Somatostatin. F
D Glucagons. T
E Arginine. T

113 Insulin secretion:


A Is partly controlled by the direct action of blood glucose upon the T
19

pancreas.
B Affect the rate of entry of glucose into the β-cells of the F
pancreatic islets.
C Is stimulated by adrenaline. F
D Is stimulated by arginine. T
E Is stimulated more effectively by glucose administered F
interavenously than by glucose administered orally.

114 Concerning insulin:


A The half-life of endogenous insulin in the circulation is 30 F
minutes.
B The kidney is a major site of insulin degradation. T
C It facilitates glucose uptake by the brain. F
D Fasting concentrations are lower in pregnant women at term than T
they are in nonpregnant women.
E It is formed when C-peptide is separated from proinsulin. T

115 Human insulin:


A Is composed of two chains of amino acids. T
B Differs from pig insulin by one amino acid. T
C Facilitates glucose uptake by red blood cells. F
D Increases protein synthesis in the liver. T
E Increase triglyceride deposition in adipose tissue. T

116 Actions of insulin include stimulation of:


A Glycogenolysis by the liver. F
B Cellular uptake of amino acids. T
C Entry of glucose into neurons. F
D Entry of glucose into adipose tissue. T
E Cellular uptake of potassium. T

117 Glucagon promotes:


A Hepatics gluconeogenesis. T
B Glucose uptake by muscle. F
C Glycogen synthesis by muscle. T
D Breakdown of protein. T
E Synthesis of fat. T

118 Therapeutic indications for the use of synthetic progestagens


include:
A Induction of abortion. F
B Contraception. T
C Metastatic endometrial carcinoma. T
D Suppression of lactation. F
E Endometriosis. T

119 The pineal gland:


A Lies anteriorly to the 3rd ventricle. F
20

B Is innervated by the parasympasthetic nervous system. F


C Produces melatonin. T
D Increases in size at puberty. F
E Activity is related to day length. T

hormone Type Amino M.W. chain Half life


acid
GH Protein 191 22 000 single 20-30 min
Prolactin Protein 198 23 000 single 15 min
Inhibin Protein 32 000 double

hormone Type Amino M.W. chain Half life


acid
LH glycoprotein 204 30 000 double 1 hour
FSH glycoprotein 204 30 000 double 3 hours
TSH glycoprotein 201 28 000
HCG glycoprotein 30 38 400 double 5 hours
Thyroglobulin glycoprotein 140 660 000

hormone Type Amino M.W. chain Half life


acid
ACTH Polypeptide 39 4 500 5-15 min
PTH Polypeptide 84 9 500 5 min
Calcitonin Polypeptide 32 3 500
Insulin Polypeptide 51 5 800 double 5 min
Glucagon Polypeptide 29 3 500 single 3-6 min
CRH Polypeptide 41
GHRH Polypeptide 40
21

hormone Type Amino M.W. chain Half life


acid
Oxytocin Nanopeptide 9 1-4 min
Aldosterone 20 min
Vasopressin Nanopeptide 9
Somatostatin Cyclic peptide 14 1640
1

1 Adrenocorticotrophic hormone: 83 √
A Production is governed by the hypothalamus. T T T
B Production is maximal about midnight. F F F
C Is present in the placenta. T T T
D Is increased in the maternal plasma in pregnancy. T T T
E Secretion is inhibited by glucocorticoids. T T T

2 The hypothalamus is the site of synthesis of: √


A Oxytocin. T T T
B Thyrotrophin-releasing hormone. T T T
C α- melanocyte-stimulating hormone F F F
D Luteinising hormone. F F F
E Gonadaotrophin-releasing hormone. T T T

3 Concerning the hypothalamopituitary axis: √


A Secretion of luteinising hormone is independent of F F F
gonadotrophin-releasing hormone.
B Gonadotrophin releasing hormone neurons are absent in Kallman T T T
syndrome.
C Oestradiol has an acute positive feedback effect on T T T
gonadotrophin secretion.
D Oestradiol has a chronic negative feedback action on T T T
gonadotrophin secretion.
E The preovulatory rise in gonadotrophin-releasing hormone T T T
secretion is due to an increase in gonadotrophin releasing
hormone pulse frequency.

4 Corticotrophin releasing hormone: √


A Is a polypeptide. T T T
B Is released from the median eminence of the hypothalamus. T T T
C Acts on the basophil cells of the anterior pituitary. F F F
D Release has a circadian variation. T T T
E Release is increased by painful stimuli. T T T

5 Ganadotrophin-releasing hormone: √
A Is adimeric glycoprotein. F F F
B Is synthesised in the hypothalamus. T T T
C Stimulates synthesis of follicle-stimulating hormone. T T T
D Secretion is increased following castration. T T T
E Pulsatile administration inhabits ovarian steroid production. T * F F

6 Deficient adrenocortical function causes an increase in: √


A Blood pressure. F F F
B Blood glucose. F F F
C Skin pigmentation. T T T
2

D Serum sodium. F F F
E Plasma lipids. T T T

7 Adrenal androgens: √
A Are synthesised in the zona glomerulosa of the adrenal cortex. F F F
B Are secreted in excessive amounts in the presence of 11β- T T T
hydroxylase deficiency.
C Stimulate protein synthesis. T T T
D Consist mainly of testosterone. F F F
E Are secreted in increased amounts in response to rise in T T T
adrenocorticotrophic hormone.

8 Recognized features of congenital adrenal hyperplasia in the √


female are:
A Acute hypotension. F F F
B Enlargement of the clitoris. T T T
C Abnormal karyotype. F F F
D Hirsutism. T T T
E Absent uterus. F F F

9 In congenital adrenal cortical hyperplasia: √


A The commonest deficiency is C 18hydroxylase. F F F
B Plasma cortisol concentration is rised. F F F
C Urinary excretion of 17-oxysteroids is elevated. T T T
D Dexamethasone will suppress urinary excretion of 17- T T T
oxysteroids.
E There are no virilising effects. F F F

10 In congenital adrenal hyperplasia: √


A The commonest cause is a deficiency of 21-hydroxylase. T T T
B Plasma cortisol concentration is increased. F F F
C There may be excessive secretion of 17α-hydroxyprogesterone. T T T
D Sodium retention is characteristic. F F F
E Blood catecholamine concentrations are increased. F F F

11 Streroid hormones: √
A All contains 20 carbon atoms. F F F
B Can be produced by structures of urogenital ridge origin. T T T
C Are mostly activated in the liver. F F F
D Are predominantly excreted unchanged in the urine. F F F
E Mainly circulate unbound to carrier proteins. F F F

12 The following substances are steroids: √


A Aldoesterone. T T T
B Follicle-stimulating hormone F F F
C Vitamin D. T T T
D Inhibin. F F F
E Thyroxine. F F F
3

13 The following hormones bind to receptors on the cell √


membrane:
A Corticosterone. F F F
B Adrenaline. T T T
C Luteinising hormone. T T T
D Oestradiol. F F F
E Gonadatrophin-releasing hormone. T T T

14 The following statements concerning the formation of √


hormones are correct:
A ACTH is derived from pro-opiomelanocortin. T T T
B Oestrogens are derived from androgens. T T T
C Prolactin is derived from dopamine. F F F
D Melatonin is derived from serotonin. F * T T
E Angiotensin II is derived from rennin. F F F

15 The release of catecholamines from the adrenal medulla √


increases:
A During sleep in healthy individuals. F F F
B When the nerves to the adrenal gland are stimulated. T T T
C Following an increase in blood sugar. F F F
D Immediately following a myocardial infarction. T T T
E During acute haemorrhage. T T T

16 The release of catecholamines from the adrenal medulla √


increases:
A During sleep in healthy individuals. F F F
B When the nerves to the adrenal gland are stimulated. T T T
C When the blood sugar rises. F F F
D Following a myocardial infarction. T T T
E In the presence of a phaeochromocytoma. T T T

17 Adrenaline (epinephrine): √
A Stimulates myometrial contractions. F F F
B Exert its action by alpha receptors only. F F F
C Constricts the pupils. T * F F
D Causes glycogenolysis. T T T
E Inhabits the mobilization of free fatty acids. F F F

18 In hypopituitarism: √
A Clinical features of deficiency are usually absent until about 70% T T T
of the gland has been destroyed.
B Thyroid-stimulation hormone is usually the first hormone to be T * F F
affected.
C Aldosterone secretion is normal. T T T
D Orthostatic hypotension is common. T T T
E Hypoglycaemia occurs on fasting. T T T
4

19 Pituitary gonadatrophin:
A Release is dependent upon hypothalamic function. T T T
B Secretion increases during pregnancy. F F F
C Blood levels are raised during lactational amenorrhoea. F F F
D Release in the puerperium is enhanced by bromocriptine. F * F * T
E Release is inhibited by oxytocin. F F F

20 Pulsatile secretion of pituitary luteinising hormone:


A Increase in amplitude during puberty. T T T
B Occurs in the first year of life. T F * T
C Ceases at the menopause. F F F
D Is controlled by continuous secretion of gonadotrophin-releasing T * T * F
hormone.
E Is modified by the level of circulating oestrogen. F * F * T

21 Concerning thyroid function: √


A Oestrogen increases the production of thyroxine-binding T T T
globulin.
B More than 98% of circulating thyroxine is bound to thyroxine- T * F F
binding globulin.
C Thyrotrophin-releaseing hormone is a decapeptide. F T * F
D Thyroid-stimulating hormone levels are increased in primary T T T
hypothyroidism.
E Thyroid-stimulating hormone is a glycoprotein. T T T

22 Concerning thyroid function:


A Thyroid-binding globulin is found entirely within the thyroid T F F
gland.
B Triiodothyronine is mainly in the free form in the circulation. F F F
C Triiodothyronine acts more rapidly than thyroxine. T T T
D A fall in thyroid-binding globulin causes hypothyroidism. F F F
E Thyroxine is a major precursor of Triiodothyronine. T T T

23 Thyroxine: √
A Exist in the free state in thyroid gland. F F F
B Is more than 99% protein-bound in plasma. T T T
C Is released by the fetal thyroid. T T T
D In the unbound state in plasma is approximately equal in F F F
concentration to unbound Triiodothyronine.
E Is actively transported across the placenta. T F F

24 Thyroid hormones: √
A Increase oxygen consumption in most metabolically active T T T
tissues.
B In the circulation are less than 90% bound to protein. F F F
C Decrease the rate of absorption of carbohydrate from the gut. F F F
D Increase the circulating cholesterol concentrations. T * F F
E Are essential for skeletal maturation. F * F * T
5

25 Concerning thyroid hormones:


A Triiodothyronine is converted in the tissues to thyroxine. F F F
B The circulating concentration of thyroid binding globulin T T T
increases in pregnancy.
C Triiodothyronine acts more rapidly than thyroxine. T T T
D Starvation causes plasma triiodothyronine concentrations to rise. T * F F
E D-thyroxine is more active than L-thyroxine. F T F

26 Iodine: √
A Requirements are unchanged by pregnancy. F F F
B Uptake by the thyroid gland is increased by thyroid-stimulating T T T
hormone.
C Is excreted by the kidney. T T T
D Is bound to tyrosine in the thyroid gland. T T T
E May inhibit thyroxine synthesis. T T T

27 During normal pregnancy: √


A Plasma thyroid-binding globulin concentration increases. T T T
B Plasma total thyroxine concentration falls. F F F
C Plasma TSH concentration increases. T T T
D Triiodthyronine readily crosses the placenta to the fetous. T * F F
E Fetal thyroid function is largely dependant upon the function of F F F
the maternal thyroid.

28 Serum concentrations of the following increase during √


pregnancy:
A Sex hormone-binding globulin. T T T
B Prolactin. T T T
C Total thyroxine. T T T
D Follicle-stimulating hormone. F F F
E Corticotrophin-releasing hormone. F * T T

29 Parathyroid hormone: √
A Is a polypeptide. T T T
B Increases bone resorption. T T T
C Decreases phosphate excretion in the urine. F F F
D Secretion is diminished by an increase in serum ionized calcium T T T
concentration.
E Decreases the formation of 1,25-dihydroxycholecalciferol. F F F

30 Parathyroid hormone: √
A Decreases the renal excretion of phosphate. F F F
B Increases calcium resorption from bone. T T T
C Depresses pituitary activity. T * F F
D Concentrations in blood are raised when the calcium level falls. F * T T
E Increases renal tubular reabsorption of calcium. F * T T
6

31 Parathyroid hormone: √
A Increase bone resorption. T T T
B Concentrations are increase in pregnancy. T T T
C Reduces phosphate excretion in urine. T * F F
D Increases the formation of 1,25-dihydroxycholecalciferol. T T T
E Stimulates osteoblasts. F F F

32 Calcitonin: √
A Synthesised in the parathyroid glands. F F F
B Is a decapeptide. F F F
C Secretion is increased at serum calcium levels below 1.5 mmol/L F F F
(6.1mg/100mL).
D Inhibits bone resorption. T T T
E Increases renal tubular excretion of calcium. T T T

33 Calcitonin:
A Lower the basal metabolic rate. T T T
B Concentration is increased in pregnancy. T T T
C Is released when the blood phosphate level rises. T F
D Is produced in the thyroid gland. T T T
E Release is stimulated by high calcium ion levels. T T T

34 Calcitonin: √
A Increases the basal metabolic rate. F F F
B Increases the blood calcium level. F F F
C Is a steroid hormone. F F F
D Is produced in the parathyroid glands. F F F
E Release is stimulated by low calcium ion levels. F F F

35 Calcitonin: √
A Suppresses appetite. T T T
B Is produced mainly in the thymus. T * F F
C Lowers plasma calcium concentrations. T T T
D Secretion is inhibited by gastrin. T * F F
E Inhibits bone resorption. T T T

36 In a healthy adult, serum calcium: √


A Constitutes 15% of total body calcium. F F F
B Is not involved in the extrinsic system of blood coagulation. T * F F
C Concentration will be lowered by calcitonin. T T T
D Concentration normally falls after menopause. F F F
E Is approximately 90% protein-bound. T * F F

37 The secretion of growth hormone: √


A Occurs in the hypothalamus. F F F
B Ceases when the adult state is reached. F F F
7

C Is decreased during stress. T * F F


D Is increased during fasting. T T T
E Is increased with exercise. T T T

38 The secretion of growth hormone: √


A Occurs in the hypothalamus. F F F
B Decreased during sleep. F F F
C Is decreased during stress. F F F
D Is decreased with hypoglycaemia. F F F
E Opposes the action of insulin. T F * T

39 Concerning growth hormone: √


A Plasma levels are reduced by glucose infusion. T T T
B Maternal plasma levels are directly related to fetal growth. F F F
C It is active on bone only until the epiphyses fuse. F F F
D Its secretion is controlled by the hypothalamus. T T T
E Increased activity produces a positive nitrogen balance. T T T

40 Growth hormone: √
A Is a protein. F * T T
B Has a molecular weight of 2000 daltons. F F F
C Secretion is stimulated by hyperglycaemia. F F F
D Has growth-promoting effects mediated through insuline-like T T T
growth factors.
E Is synthesised in the hypothalamus. F T * F

41 Growth hormone: √
A Promotes protein synthesis. T T T
B Faciliatates the hepatic synthesis of somatomedin C. T T T
C Promotes insulin-mediated uptake of glucose. F F F
D Increases circulating free fatty acids. T T T
E Stimulates epiphyseal fusion. F F F

42 The secretion of growth hormone: √


A Occurs in the hypothalamus. F F F
B Ceases when adult state is reached. F F F
C Is decreased during stress. F F F
D Is increased during fasting. T T T
E Is increased with exercise. T T T

43 The actions of growth hormone include : √


A Promotion of protein synthesis. T T T
B Facilitation of hepatic synthesis of somatonedin C (insulin-like T T T
growth factor).
C Promotion of the insulin-mediated uptake of glucose. F F F
D Stimulation of lipolysis. T T T
E Stimulation of the growth spurt at the onset of puberty. F * F * T
8

44 Concerning growth hormone: √


A Plasma levels are reduced by glucose infusion. T T T
B Maternal plasma levels are directly related to fetal growth. F F F
C It is active on bone only until the epiphyses fuse. F F F
D It is secretion is controlled by the hypothalamus. T T T
E Increased activity produces a positive nitrogen balance. T T T

45 Successful lactation is: √


A Maintained by oestrogens. F F F
B Maintained by progesterone. F F F
C Initiated by a prolactin surge. T T
D Maintained by human placental lactogen. F F F
E Inhibited by dopamine. T T T

46 Prolactin: √
A Release is stimulated by thyrotrophin-releasing hormone. T T T
B Plasma levels are raised in the first trimester. T T T
C Release is increased by suckling. T T T
D May be produced by the decidua. T F * T
E Release is inhibited by metaclopramide. F F F

47 Prolactin: √
A Is secreted by the hypothalamus. F F F
B Plasma levels are raised in the first trimester of pregnancy. T T * T
C Is identical to placental lactogen. F F F
D Controls milk ejection. F F F
E Release is inhibited by dopamine. T T T

48 Prolactin: √
A Release is stimulated by thyrotropin-releasing hormone. T T T
B Plasma levels are raised in the first trimester of pregnancy. T T T
C Is identical to human placental lactogen. F F F
D Control milk ejection. F F F
E Release is inhibited by metoclopramide. F F F

49 During human lactation: √


A Oxytocin increases mammary duct pressure. T T T
B Oestrogen promotes the milk producing effects of prolactin on T * F F
the breast.
C Human placental lactogen is essential for milk synthesis. F F F
D Prolactin stimulates gonadotrophin release. F F F
E Suckling enhances prolactin release. T T T

50 In human lactation: √
A Oestrogens promote development of the breast lobules. F F F
B Oestrogens promote the milk-producing effect of prolactin on the T * F F
breast.
C Human placental lactogen is essential for milk synthesis. F F F
9

D Prolactin stimulates gonadotrophin release. F F F


E Oxytocin causes milk ejection. T T T

51 Release of prolactin from the anterior pituitary gland is √


enhanced by:
A Dopamine. F F F
B Thyrotrophin-releasing hormone. T T T
C Oestrogens. T * F F
D Chlorpromazine. T T T
E Growth hormone. F F F

52 Human placental lactogen:


A Is a single-chain polypeptide. T F * T
B Reaches the same concentration in fetal and maternal blood at F F F
term.
C May be secreted by the deciduas. F F F
D Is detectable only after the 25th week of pregnancy. F F F
E Is an insulin antagonist. T T T

53 Human placental lactogen:


A Concentration in maternal plasma is directly proportional to the F F * T
functional mass of placenta.
B Has a half-life in blood of less than one hour. T T T
C Is a steroid hormone. F F F
D Increases the mobilization of maternal free fatty acids. T T T
E Reaches the same concentration in fetal blood as in maternal F F F
blood at term.

54 Luteinising hormone: √
A Is required for normal corpus luteum survival. T T T
B Has a half-life in the circulation of 30 hours. F F F
C Is released in pulses. T T T
D In the male stimulates testosterone production. T T T
E Plasma concentrations are increased in postmenopausal women. T T T

55 Luteinising hormone: √
A Is a glycoprotein. T T T
B Has a molecular weight of approximately 4000 daltons. F F F
C Stimulates androgen production by ovary. T F * T
D Concentrations in the circulation are high in girls before puberty. T * F F
E Concentrations in the circulation are raised in untreated adult T T T
with Turner syndrome.

56 Follicle-stimulating hormone: √
A Binds to receptors on the cell membrane. T T T
B Promotes the expression of luteinising hormone receptors. T T T
C Is responsible for degeneration of the corpus luteum. F F F
D Is a steroid hormone. F F F
10

E Is synthesized in the hypothalamus. F F F

57 Plasma levels of follicle stimulating hormone are elevated in:


A Adults with Klinfelter syndrome. T T T
B Women taking oral contraceptive preparations. F T * F
C Postmenopausal women. T T T
D Adults with Turner syndrome (45X0). T T T
E Pregnancy. F F F

58 Serum concentrations of follicle-stimulating hormone are:


A Increased in Kleinfelter syndrome. T T T
B Decreased in primary hypogonadism. F * T T
C Decreased following the menopause. F F F
D Increased with oestrogen therapy. T * F F
E Increased following male castration. T T T

59 Oxytocin: √
A Is released episodically. T F * T
B Causes decreased renal tubular reabsorption of water. T * F F
C Is responsible for milk ejection. T T T
D Reduces intestinal peristalsis. F F F
E Inhibits prolactin secretion. F F F

60 In the human, Oxytocin: √


A Promote milk synthesis in the breast. F F F
B Release is inhibited by alcohol. T T T
C Action on the uterus is enhanced by oestrogens. T F * T
D Release is inhibited by dopamin. T * F F
E In excess causes dehydration. F F F

61 The following agents cause myometrial contractions:


A Magnesium sulphate. F F
B Nifidipine. F F F
C Progesterone. F F F
D Salbutamol. F F F
E Sodium nitroprusside. F T

63 Rennin: √
A Is secreted by the zona glomerulosa of the adrenal cortex. F F F
B Is a proteolytic enzyme. F * T T
C Is secreted at an increased rate if the renal perfusion pressure F * T T
falls.
D Acts upon circulating angiotensinogen. T T T
E Is released in response to an increase in extracellular fluid F F F
volume.

64 In the rennin-angiotensin system: √


A Decreased renal blood flow causes increased rennin production. T T T
11

B Rennin is secreted by the juxta-glumerular cells of kidney. T T T


C Rennin causes production of angiotensin I. T T T
D Angiotensin I is a decapeptide. T F * T
E Angiotensin II suppresses aldosterone secretion. F F F

65 In the rennin-angiotensin system: √


A Rennin is an enzyme. F * T T
B Angiotensin II is converted to angiotensin I. F F F
C Angiotensinogen is globulin. F * T T
D Rennin release is inhibited by sodium restriction. T * T * F
E Rennin is present in amniotic fluid. T T T

66 Concerning the rennin-angiotensin system: √


A Rennin is secreted by the proximal tubule. F F F
B Rennin is responsible for the conversion of angiotensin I to F F F
angiotensin II.
C Angiotensin II is a potent presser agent. T T T
D The presser effect of angiotensin II is suppressed in normal T T T
pregnancy.
E Angiotensin II increases the secretion of aldosterone from the T F * T
adrenal cortex.

67 Aldosterone: √
A Reduces sodium reabsorption in the proximal convoluted tubule. F F F
B Reduces sodium absorption in the descending loop of Henle. F F F
C Increases sodium absorption in the distal convoluted tubule. F * T T
D Increase potassium loss from the tubule. T T T
E Increases sodium absorption in the collecting tubule. T T T

68 Aldasterone: √
A Secretion is increased by a low potassium intake. F F F
B Production is reduced in normal pregnancy. F F F
C Secretion is entirely regulated by the rennin-angiotensin system. F F F
D Is a principle mineralcorticoid secreted by the adrenal gland. T F * T
E In the kidney, acts on the distal convoluted tubule. F * T T

69 Aldosterone secretion is increased: √


A On standing. F * T T
B Following haemorrhage. T T T
C During pregnancy. T T T
D By hypocalcaemia. F F F
E On a low sodium diet. T T T

62 Antidiuretic hormone: √
A Is normally synthesised in the hypothalamus. T T T
B Is secreted by the posterior pituitary gland. T T T
C Exerts its effect by reducing the glomerular filtration rate. F F F
D Is an oligapeptide. T * F F
12

E Secretion is increased when plasma osmolality falls. T * F F

70 Argentine vasopressin: √
A Reduces the glomerular filtration rate. F * F * T
B Controls water loss in the proximal renal tubule. F F F
C Is synthesized by the posterior pituitary gland. F F F
D Is released in response to a rise in plasma osmolality. T T T
E Is released in response to a fall in circulating plasma volume. T T T

71 Argenin vasopressin: √
A Is produced in the hypothalamus. T T T
B Is a polypeptides. T T T
C Is structurally similar to prolactin. F F F
D Controls water reabsorption by kidney. T T T
E Decreases glomerular filtration. F * F F

72 Vasopressin: √
A Is a nanopeptide. T T T
B Is synthesised in the posterior pituitary gland. F F F
C Release is increased when plasma osmolality rises. T T T
D Release is increased by haemorrhage. T T T
E Plasma concentration is reduced in pregnancy. F F F

73 Human chorionic gonadotrophin: √


A Is a glycoprotein. T T T
B Secretion peaks at 20 week of gestation. F F F
C Has intrinsic anti-thyroid activity. F F F
D Is synthesised by the corpus luteum of pregnancy. F F F
E Binds to luteinising hormone receptors. T T T

74 Human chorionic gonadotrophin: √


A Is not produced by the decidua. T T T
B Is biochemically indistinguishable from luteinising hormone. F F F
C Is active if given to nonpregnant women. T T T
D Production rises steadily throughout pregnancy. F F F
E Has no influence upon the production of oestrogens by the F T F
placenta.

75 Human chorionic gonadotrophin: √


A Has a half-life in blood of two hours. F F F
B Contains a β chain indistinguishable from that of human F F F
luteinising hormone.
C Is produced by the preimplantation blastocyst. T * F F
D Is responsible for the maintenance of the corpus luteum in early T T T
pregnancy.
E May be produced in tissues other than trophoblast. T F * T
13

76 Human chorionic gonadotrophin: √


A Is a steroid hormone. F F F
B Is partly metabolized in the kidney. F F F
C Has an α-subunit which is antigenically similar to β-subunit of F F F
luteinising hormone.
D Reaches peak concentrations at the 17th week of pregnancy. F F F
E Is secreted in trophoblastic diseases. T T T

77 Serum concentration of the following increase during √


pregnancy:
A Sex hormone-binding globulin. T T T
B Prolactin. T T T
C Total thyroxine. T T T
D Follicle-stimulating hormone. F F F
E 17α-hydroxyprogesterone. F * T T

78 During pregnancy, uterine deciduas synthesized: √


A Human chorionic gonadotrophin. F F F
B Prostaglandin E2. T T T
C Progesterone. T * F F
D Prolactin. T T T
E Oxytocin. T T T

79 Products of human decidua include: √


A α-fetoprotein. F F F
B Prolactin. T T T
C Human chorionic gonadatrophin. F F F
D Human placental lactogen. F F F
E Prostaglandin F2α. T T T

80 Trophoblast: √
A Develops from the blastocyst. T T T
B Gives rise to the fetal blood vessels in the placenta. T T T
C Enters the maternal circulation during normal pregnancy. T T T
D Replaces endothelium of pregnant spiral arterioles. T T T
E Is genetically identical to decidua. F F F

81 The following cell types are present in the human corpus √


luteum:
A Endothelial cells. T T T
B Macrophages. T T T
C Pericytes. T T T
D Fibroblasts. F * T T
E Granulose cells. T T T

82 The corpus luteum of pregnancy produces: √


A Relaxin. T T T
B Progesterone. T T T
14

C 17α-hydroxyprogesterone. T * F F
D Human chorionic gonadotrophin. F F F
E Oestradiol. T T T

83 Concerning puberty in the female: √


A The development of secondary sexual characteristics is preceded F * F T
by increased production of adrenal androgens.
B Puberty is associated with the nocturnal release of luteinising T T T
hormone.
C The ovaries are not sensitive to gonadotrophins before puberty. F F F
D Appearance of pupic hair usually precedes menarche. T T T
E Appearance of pupic hair precedes the onset of breast F F F
development.

84 Concerning the human ovary: √


A Aromatase catalyses the conversion of testosterone to 17β- T T T
oestradiol.
B Relaxin is synthesised in the corpus luteum. T T T
C Inhabin stimulates follicle-stimulating hormone release from the F F F
pituitary gland.
D Progesterone decreases the sensitivity of myometrial cells to T T T
oxytocin.
E Cholesterol is converted to androstenedione in granulose cells. F T * F

85 Concerning ovarian function: √


A Progesterone is the major steroid of the developing follicle. F F F
B Granulose cells secrete oestradiol. T T T
C Oestradiol is derived from androgen precursors. T T T
D Insulin-like growth factor (IGF)-1 is not secreted by the ovary. T * T * F
E Circulating inhibin concentrations are a marker of granulose cell F * F T
function.

86 Concerning ovarian function : √


A Progesterone is the major steroid of the developing follicle. F F F
B Granulose cells secrete oestradiol. T T T
C Oestradiol is derived from androgen precursors. T T T
D Insulin-like growth factor (somatomedin C) is not secreted by the T * F F
ovary.
E Circulating inhibin concentrations are a marker of granulose cell F * F * T
function.

87 Ovaries secrete: √
A Progesterone. T T T
B Androstenedione. T T T
C Testosterone. T T T
D 17β-oestradiol. T T T
E Aldosterone. F F F

88 Inhibin: √
15

A Is structurally identical to relaxin. F F F


B Is released in pulses. F * T T
C Is a steroid. F F F
D Is produced by the ovarian follicle. T T T
E Inhibit the release of follicle-stimulating hormone. T T T

89 During the normal ovarian cycle: √


A The principle oestrogen secreted is 17-Oestradiol. T T T
B The most potent oestrogen is oestriol. F F F
C Oestrogen production is maximal by about the 8th day of the F F F
cycle.
D Oestrogens decrease the secretion of follicle-stimulating T T T
hormone.
E Oestrogens are synthesized primarily by the ovarian stroma. T * F F

90 Progesterone: √
A Is synthesised by trophoblast. F * T T
B Increase myometrial activity. F F F
C Is predominantly excreted in the urine as pregnanetriol. T * F F
D Binds to cortisol-binding globulin in the circulation. T T T
E Is synthesised from cholesterol. T T T

91 Progesterone: √
A Is a C -21 steroid. F * T T
B Is synthesised by the ovary before ovulation. T T T
C Increases ventilation. T T T
D Rises basal body temperature. T T T
E Binds to corticotrophin-binding globulin. T T T

92 Concerning sex hormones: √


A The ovary secretes androstenedione. T T T
B The ovary secretes testosterone. T T T
C The ovary secretes dihydrotestosterone. T T T
D Sex hormone-binding globulin (SHBG) concentrations are higher F * T T
in women than in men.
E Androgens bound to protein have high biological activity. F F F

93 Sex hormone-binding globulin: √


A Levels are increased in pregnancy. T T T
B Is the main binding protein for progesterone. F F F
C Levels are decreased during oestrogen therapy. T * F F
D Is the main binding protein for aldosterone. F F F
E Has a greater affinity than albumin for testosterone. F * F * T

94 The physiological action of oestradiol depends upon: √


A Metabolism to a more potent substance. F F F
B Binding to an intracellular receptor. T T T
16

C Alteration of gene expression. T T T


D Active transport of the hormone into cells. F F F
E Cyclic AMP production. F F F

95 In the human testis: √


A Meiosis occurs between the early and late spermatid phases. F F F
B One spermatogonium always forms 8 sprmatides. F F F
C The fluid in the seminiferous tubules contains a high F F F
concentration of protein.
D The process of spermatogenesis takes 34 days. F F F
E Inhibin is produced by primary spermatocytes. T * F F

96 The interstitial (leydig) cells of testis: √


A Secrete seminal fluid. F F F
B Are stimulated by luteinising hormone. T T T
C Secrete androgen binding protein. F F F
D Secrete fructose. F F F
E Produce testosterone. T T T

97 Concerning testicular hormone: √


A Testosterone reduces plasma luteinising hormone concentrations. F * T T
B Inhibin stimulates luteinising hormone production. F F F
C Oestrogens are formed in the testis. T T T
D Testosterone is converted to hydrotestosterone by 5α-reductase. F * T T
E Testosterone in plasma is predominantly bound to albumin. F F F

98 In the human testis: √


A The main site of testosterone synthesis is the sertoli cell. F F F
B Testosterone synthesis is stimulated by follicle stimulating F F F
hormone (FSH).
C The predominant androgen product is androstenedione. F F F
D The Leydig cells synthesize testosterone. T T T
E Inhibin is synthesized by the sertoli cell. T T T

99 The following statements about testicular hormones are true: √


A Testosterone reduces plasma luteinising hormone levels. F * T T
B Inhibin increases plasma follicle-stimulating hormone levels. F F F
C Oestrogens are formed in the testis. T T T
D Testosterone is excreted in urine as 17-ketosteroids. T F * T
E Testosterone in plasma is partly bound to albumin. T T T

100 Concerning sex hormone: √


A The ovary secretes androstenedione. T T T
B The ovary secretes testosterone. T T T
C The ovary secretes dihydroxycholecalciferol. F F F
D Sex hormone-binding globulin (SHBG) concentrations are higher F * T T
in women than men.
E Androgens bound to protein have high biological activity. F F F
17

101 Hirsutism in women is characteristically associated with: √


A Testicular feminization. T F F
B Turner syndrome. F F F
C Polycystic ovary syndrome. T T T
D Arrhenoblastoma. T T T
E Hypopituitarism. F F F

102 Hirsutism is associated with: √


A Testicular feminization syndrome. T * F F
B Turner syndrome. F F F
C Addison's disease. F * T T
D Sertoli-leydig cell tumors. F F F
E Congenital adrenal hyperplasia. T T T

103 Testosterone: √
A Is produced only in the gonads. F F F
B Is mainly excreted unchanged in the urine. F F F
C Stimulates secretion of luteinising hormone. F F F
D Circulates in plasma mainly in the free form. F F F
E Stimulates growth of the prostate gland. T T T

104 Testosterone in the human male: √


A Depresses pituitary secretion of luteinising hormone. T T T
B Promotes growth of scalp hair. F F F
C Promotes union of long bone epiphyses. F F F
D Is a more potent androgen than dihydrotestosterone. F F F
E Is secreted maximally in the evening. F * T T

105 In the human male, dihydrotestesterone: √


A Is a precursor of testosterone. F * T T
B Has one-tenth of the potency of testosterone. F F F
C Is responsible for involution of the Mullerian system. T * F F
D Is responsible for the development of the male external genitalia. T T T
E Binds to an intracellular receptor. T T T

106 Oestradiol-17β: √
A Is synthesized by aromatisation of testosterone. T T T
B Vasodilates the uterine artery. F * T T
C Suppresses uterine activity by upregulating the oxytocin receptor. T F F
D Promotes secondary sexual hair growth in females. F F F
E Is thrombogenic. T T T

107 Oestradiol-17β: √
A Is synthesized by aromatisation of testosterone. T T T
B Can be administered orally. T T T
C Suppresses uterine activity by upregulating the oxytocin receptor. T * F F
D Promotes secondary sexual hair growth in females. F F F
18

E Is thrombogenic. T T T

108 After the menopause: √


A The plasma concentration of follicle stimulating hormone T T T
increases.
B The plasma progesterone concentration increases. F F F
C Oestrone is the oestrogen found in highest concentration in the T T T
plasma.
D The plasma testosterone concentration doubles. F F F
E The plasma prolactin concentration increases. F F F

109 Menopause: √
A Is due to failure of the endometrium to respond to oestrogens. F F F
B Is associated with cessation of steroidogenesis within the ovary. T F * T
C Is associated with a fall in circulating luteinising hormone. F F F
D Is preceded by a period of enhanced fertility. F F F
E Results in atrophy of the epithelium of the distal urethra. T T T
Comment :
110 Concerning androgens in normal premenopausal women: √
A 95% of circulating testosterone is derived from peripheral F F F
conversion of androstenedione.
B The ovaries and adrenals contribute equally to circulating T * F F
androgen concentrations.
C DHEAS (dehydroepiandrosterone sulphate) is derived almost F * T T
exclusively from adrenal glands.
D About50% of circulating testosterone is bound to sex hormone- F F F
binding globulin (SHBG).
E Testosterone promotes the synthesis of SHBG. T * F F

111 In a woman of reproductive age, serum concentrations of the √


following hormones exhibit a recognized pattern of diurnal
variation:
A Progesterone. F F F
B Melatonin. T T T
C Cortisol. T T T
D Oestrone. F F F
E Follicle-stimulating hormone. F F F

112 Insulin secretion is stimulated by: √


A Gastrin. T T T
B Noradrenaline (norepinephrine). F F F
C Somatostatin. F F F
D Glucagons. T T T
E Arginine. T T T

113 Insulin secretion: √


A Is partly controlled by the direct action of blood glucose upon the F * T T
pancreas.
B Affect the rate of entry of glucose into the β-cells of the F F F
19

pancreatic islets.
C Is stimulated by adrenaline. F F F
D Is stimulated by arginine. T T T
E Is stimulated more effectively by glucose administered F F F
interavenously than by glucose administered orally.

114 Concerning insulin: √


A The half-life of endogenous insulin in the circulation is 30 F F F
minutes.
B The kidney is a major site of insulin degradation. F * T T
C It facilitates glucose uptake by the brain. F F F
D Fasting concentrations are lower in pregnant women at term than T * T F
they are in nonpregnant women.
E It is formed when C-peptide is separated from proinsulin. T T T

115 Human insulin: √


A Is composed of two chains of amino acids. T T T
B Differs from pig insulin by one amino acid. T T T
C Facilitates glucose uptake by red blood cells. F F F
D Increases protein synthesis in the liver. T T T
E Increase triglyceride deposition in adipose tissue. T T T

116 Actions of insulin include stimulation of: √


A Glycogenolysis by the liver. F F F
B Cellular uptake of amino acids. T T T
C Entry of glucose into neurons. F F F
D Entry of glucose into adipose tissue. T T T
E Cellular uptake of potassium. T T T

117 Glucagon promotes: √


A Hepatics gluconeogenesis. T T T
B Glucose uptake by muscle. F F F
C Glycogen synthesis by muscle. F T * F
D Breakdown of protein. T T T
E Synthesis of fat. F T * F

118 Therapeutic indications for the use of synthetic progestagens √


include:
A Induction of abortion. F F F
B Contraception. T T T
C Metastatic endometrial carcinoma. T T T
D Suppression of lactation. F F F
E Endometriosis. T T T

119 The pineal gland: √


A Lies anteriorly to the 3rd ventricle. F F F
B Is innervated by the parasympasthetic nervous system. T * F F
C Produces melatonin. T T T
20

D Increases in size at puberty. F F F


E Activity is related to day length. T T T

hormone Type Amino M.W. chain Half life


acid
GH Protein 191 22 000 single 20-30 min
Prolactin Protein 198 23 000 single 15 min
Inhibin Protein 32 000 double

hormone Type Amino M.W. chain Half life


acid
LH glycoprotein 204 30 000 double 1 hour
FSH glycoprotein 204 30 000 double 3 hours
TSH glycoprotein 201 28 000
HCG glycoprotein 30 38 400 double 5 hours
Thyroglobulin glycoprotein 140 660 000

hormone Type Amino M.W. chain Half life


acid
ACTH Polypeptide 39 4 500 5-15 min
PTH Polypeptide 84 9 500 5 min
Calcitonin Polypeptide 32 3 500
Insulin Polypeptide 51 5 800 double 5 min
Glucagon Polypeptide 29 3 500 single 3-6 min
CRH Polypeptide 41
GHRH Polypeptide 40

hormone Type Amino M.W. chain Half life


acid
21

Oxytocin Nanopeptide 9 1-4 min


Aldosterone 20 min
Vasopressin Nanopeptide 9
Somatostatin Cyclic peptide 14 1640
1

1 Adrenocorticotrophic hormone: √ 26
A Production is governed by the hypothalamus. T T
B Production is maximal about midnight. F F
C Is present in the placenta. T T
D Is increased in the maternal plasma in pregnancy. T T
E Secretion is inhibited by glucocorticoids. T T
Comment
Production maximal early morning 4a.m. to 10a.m.
Comments 2:
The production of ACTH is governed through the secretion of CRF from
the hypothalamic nuclei. It governs cortisol secretion where cortisol is
secreted maximally in the morning and concentrations are at a nadir at
midnight. ACTH can be expressed in numerous tissues besides the
pituitary and include the placenta. ACTH concentrations rise in stress,
disease and in pregnancy. Through negative feedback, glucocorticoids
(not mineralocorticoids - aldosterone) switch off ACTH production.
2 The hypothalamus is the site of synthesis of: √
A Oxytocin. T T
B Thyrotrophin-releasing hormone. T T
C α- melanocyte-stimulating hormone F F
D Luteinising hormone. F F
E Gonadaotrophin-releasing hormone. T T

3 Concerning the hypothalamopituitary axis: √


A Secretion of luteinising hormone is independent of gonadotrophin- F F
releasing hormone.
B Gonadotrophin releasing hormone neurons are absent in Kallman T T
syndrome.
C Oestradiol has an acute positive feedback effect on gonadotrophin T T
secretion.
D Oestradiol has a chronic negative feedback action on gonadotrophin T T
secretion.
E The preovulatory rise in gonadotrophin-releasing hormone secretion is T T
due to an increase in gonadotrophin releasing hormone pulse frequency.

4 Corticotrophin releasing hormone: √


A Is a polypeptide. T T
B Is released from the median eminence of the hypothalamus. T T
C Acts on the basophil cells of the anterior pituitary. F F
D Release has a circadian variation. T T
E Release is increased by painful stimuli. T T

5 Ganadotrophin-releasing hormone: √
A Is adimeric glycoprotein. F F
B Is synthesised in the hypothalamus. T T
2

C Stimulates synthesis of follicle-stimulating hormone. T T


D Secretion is increased following castration. T T
E Pulsatile administration inhabits ovarian steroid production. F F
Comment:
GONADOTROPIN RELEASING HORMONE
· Decapeptide produced by the arcuate and preoptic nuclei of the
hypothalamus and stored in the median eminence. Synthesised as
a preprohormone
· Pulsatile release under influence of higher centres: 8-10 pulses
per day in the adult male; pulsatility in the female is dependent on
the menstrual cycle
· Acts via a cell-surfece receptor
· Stimulates LH and FSH release from the anterior pituitary
· Sustained high levels cause down-regulation of pituitary GnRH
receptors and androgen / oestrogen production by the gonad
becomes suppressed.
6 Deficient adrenocortical function causes an increase in: √
A Blood pressure. F F
B Blood glucose. F F
C Skin pigmentation. T T
D Serum sodium. F F
E Plasma lipids. T T
Comment
Adrenocortical deficiency causes hypotention due to loss of sodium.
Hypoglycaemia
Skin pigmentation due to excessive ACTH.
Decrease sodium due to absence of aldosterone.
7 Adrenal androgens: √
A Are synthesised in the zona glomerulosa of the adrenal cortex. F F
B Are secreted in excessive amounts in the presence of 11β-hydroxylase T T
deficiency.
C Stimulate protein synthesis. T T
D Consist mainly of testosterone. F F
E Are secreted in increased amounts in response to rise in T T
adrenocorticotrophic hormone.
Comment
Androgens synthesis in the zona reticularis of the adrenal cortex.
Adrenal androgens consist mainly of DHEA.

8 Recognized features of congenital adrenal hyperplasia in the female √


are:
A Acute hypotension. F F
B Enlargement of the clitoris. T T
C Abnormal karyotype. F F
D Hirsutism. T T
E Absent uterus. F F
Comment
It causes chronic hypotension.
The excess androgens result in precocious puberty in a male or
3

virilization in a female (child or adult). In a male adult, the effects are


masked by normal male characteristics.

9 In congenital adrenal cortical hyperplasia: √


A The commonest deficiency is C 18hydroxylase. F F
B Plasma cortisol concentration is rised. F F
C Urinary excretion of 17-oxysteroids is elevated. T T
D Dexamethasone will suppress urinary excretion of 17-oxysteroids. T T
E There are no virilising effects. F F

10 In congenital adrenal hyperplasia: √


A The commonest cause is a deficiency of 21-hydroxylase. T T
B Plasma cortisol concentration is increased. F F
C There may be excessive secretion of 17α-hydroxyprogesterone. T T
D Sodium retention is characteristic. F F
E Blood catecholamine concentrations are increased. F F

11 Streroid hormones: √
A All contains 20 carbon atoms. F F
B Can be produced by structures of urogenital ridge origin. T T
C Are mostly activated in the liver. F F
D Are predominantly excreted unchanged in the urine. F F
E Mainly circulate unbound to carrier proteins. F F

12 The following substances are steroids: √


A Aldoesterone. T T
B Follicle-stimulating hormone F F
C Vitamin D. T T
D Inhibin. F F
E Thyroxine. F F

13 The following hormones bind to receptors on the cell membrane: √


A Corticosterone. F F
B Adrenaline. T T
C Luteinising hormone. T T
D Oestradiol. F F
E Gonadatrophin-releasing hormone. T T

14 The following statements concerning the formation of hormones are √


correct:
A ACTH is derived from pro-opiomelanocortin. T T
B Oestrogens are derived from androgens. T T
C Prolactin is derived from dopamine. F F
D Melatonin is derived from serotonin. T T
E Angiotensin II is derived from rennin. F F

15 The release of catecholamines from the adrenal medulla increases: √


A During sleep in healthy individuals. F F
4

B When the nerves to the adrenal gland are stimulated. T T


C Following an increase in blood sugar. F F
D Immediately following a myocardial infarction. T T
E During acute haemorrhage. T T
Comments 2:
Catecholamine release is generated by stress (ie waking), sympathetic
stimulation (flight, fright, fight response), during hypoglycaemia (as
response to stressful stimuli and counteraction of catecholamines in
recruiting glucose), during illness (MI, sepsis) or hypotensive episode.
16 The release of catecholamines from the adrenal medulla increases: √
A During sleep in healthy individuals. F F
B When the nerves to the adrenal gland are stimulated. T T
C When the blood sugar rises. F F
D Following a myocardial infarction. T T
E In the presence of a phaeochromocytoma. T T

17 Adrenaline (epinephrine): √
A Stimulates myometrial contractions. F F
B Exert its action by alpha receptors only. F F
C Constricts the pupils. F F
D Causes glycogenolysis. T T
E Inhabits the mobilization of free fatty acids. F F
Comment
Adrenaline relaxes myometrium via β-receptor. Exert action by α- and β.
18 In hypopituitarism: √
A Clinical features of deficiency are usually absent until about 70% of the T T
gland has been destroyed.
B Thyroid-stimulation hormone is usually the first hormone to be affected. F F
C Aldosterone secretion is normal. T T
D Orthostatic hypotension is common. T T
E Hypoglycaemia occurs on fasting. T T

19 Pituitary gonadatrophin:
A Release is dependent upon hypothalamic function. T T
B Secretion increases during pregnancy. F F
C Blood levels are raised during lactational amenorrhoea. F F
D Release in the puerperium is enhanced by bromocriptine. T F *
E Release is inhibited by oxytocin. F F

20 Pulsatile secretion of pituitary luteinising hormone:


A Increase in amplitude during puberty. T T
B Occurs in the first year of life. T F *
C Ceases at the menopause. F F
D Is controlled by continuous secretion of gonadotrophin-releasing F T *
hormone.
E Is modified by the level of circulating oestrogen. T F *

21 Concerning thyroid function: √


A Oestrogen increases the production of thyroxine-binding globulin. T T
5

B More than 98% of circulating thyroxine is bound to thyroxine-binding F F


globulin.
C Thyrotrophin-releaseing hormone is a decapeptide. F T *
D Thyroid-stimulating hormone levels are increased in primary T T
hypothyroidism.
E Thyroid-stimulating hormone is a glycoprotein. T T
Comment:
 Highly protein bound to thyroxine binding globulin (70%),
thyroxine binding pre-albumin (~20%) and albumin (~10%).
Only 0.03% of T4 and 0.3% of T3 is unbound*.
THYROTROPIN RELEASING HORMONE (TRH)

 Tri-peptide hormone produced in the median eminence


of the hypothalamus.
22 Concerning thyroid function:
A Thyroid-binding globulin is found entirely within the thyroid gland. F F
B Triiodothyronine is mainly in the free form in the circulation. F F
C Triiodothyronine acts more rapidly than thyroxine. T T
D A fall in thyroid-binding globulin causes hypothyroidism. F F
E Thyroxine is a major precursor of Triiodothyronine. T T
Comment:
Hepatic synthesis of thyroxin binding globulin is increased duing
pregnancy.*
23 Thyroxine: √
A Exist in the free state in thyroid gland. F F
B Is more than 99% protein-bound in plasma. T T
C Is released by the fetal thyroid. T T
D In the unbound state in plasma is approximately equal in concentration to F F
unbound Triiodothyronine.
E Is actively transported across the placenta. F F
Comment
Transport across the placenta is by diffusion.
24 Thyroid hormones: √
A Increase oxygen consumption in most metabolically active tissues. T T
B In the circulation are less than 90% bound to protein. F F
C Decrease the rate of absorption of carbohydrate from the gut. F F
D Increase the circulating cholesterol concentrations. F F
E Are essential for skeletal maturation. T F *
Comment:
 Increased oxygen consumption with increased heat
production in most organs with the exception of the brain,
spleen, testes, uterus and anterior pituitary.
25 Concerning thyroid hormones:
A Triiodothyronine is converted in the tissues to thyroxine. F F
B The circulating concentration of thyroid binding globulin increases in T T
pregnancy.
C Triiodothyronine acts more rapidly than thyroxine. T T
D Starvation causes plasma triiodothyronine concentrations to rise. F F
E D-thyroxine is more active than L-thyroxine. F T
6

Comment
Thyroxine is converted in the tissue to triiodothyronine.
Starvation increase rT.
L-thyroxine has about 5 times the potency of D-thyroxine*
26 Iodine: √
A Requirements are unchanged by pregnancy. F F
B Uptake by the thyroid gland is increased by thyroid-stimulating hormone. T T
C Is excreted by the kidney. T T
D Is bound to tyrosine in the thyroid gland. T T
E May inhibit thyroxine synthesis. T T

27 During normal pregnancy: √


A Plasma thyroid-binding globulin concentration increases. T T
B Plasma total thyroxine concentration falls. F F
C Plasma TSH concentration increases. F T
D Triiodthyronine readily crosses the placenta to the fetous. F F
E Fetal thyroid function is largely dependant upon the function of the F F
maternal thyroid.
Comment:
 TSH levels fall in the first trimester as HGC levels increase -
HGC has TSH-like action. TSH levels increase in the third
trimester to normal non-pregnant levels.*
28 Serum concentrations of the following increase during pregnancy: √
A Sex hormone-binding globulin. T T
B Prolactin. T T
C Total thyroxine. T T
D Follicle-stimulating hormone. F F
E Corticotrophin-releasing hormone. T T

29 Parathyroid hormone: √
A Is a polypeptide. T T
B Increases bone resorption. T T
C Decreases phosphate excretion in the urine. F F
D Secretion is diminished by an increase in serum ionized calcium T T
concentration.
E Decreases the formation of 1,25-dihydroxycholecalciferol. F F

30 Parathyroid hormone: √
A Decreases the renal excretion of phosphate. F F
B Increases calcium resorption from bone. T T
C Depresses pituitary activity. F F
D Concentrations in blood are raised when the calcium level falls. T T
E Increases renal tubular reabsorption of calcium. T T

31 Parathyroid hormone: √
A Increase bone resorption. T T
B Concentrations are increase in pregnancy. T T
C Reduces phosphate excretion in urine. F F
D Increases the formation of 1,25-dihydroxycholecalciferol. T T
7

E Stimulates osteoblasts. F F
Comment
During pregnancy and lactation there is increase demand for calcium
and parathyroid gland hypertrophy and become more vascular and
active.

32 Calcitonin: √
A Synthesised in the parathyroid glands. F F
B Is a decapeptide. F F
C Secretion is increased at serum calcium levels below 1.5 mmol/L F F
(6.1mg/100mL).
D Inhibits bone resorption. T T
E Increases renal tubular excretion of calcium. T T
Comment:
CALCITONIN *****
 Peptide hormone (32 amino acids) produced by the
parafollicular cells of the THYROID gland.
 Secretion is stimulated by a rise in plasma calcium (normal
plasma calcium = 2.2 - 2.6mmol/L).
33 Calcitonin:
A Lower the basal metabolic rate. T T
B Concentration is increased in pregnancy. T T
C Is released when the blood phosphate level rises. F F
D Is produced in the thyroid gland. T T
E Release is stimulated by high calcium ion levels. T T

34 Calcitonin: √
A Increases the basal metabolic rate. F F
B Increases the blood calcium level. F F
C Is a steroid hormone. F F
D Is produced in the parathyroid glands. F F
E Release is stimulated by low calcium ion levels. F F
Comment
Calcitonin decreases appetite, so it reduces the BMR. It decreases blood
calcium.
Stimulated by increased calcium levels, gastrin, secretin, CCK,
glucagons, oestrogen, dopamine and β- agonist.

35 Calcitonin: √
A Suppresses appetite. T T
B Is produced mainly in the thymus. F F
C Lowers plasma calcium concentrations. T T
D Secretion is inhibited by gastrin. F F
E Inhibits bone resorption. T T

36 In a healthy adult, serum calcium: √


A Constitutes 15% of total body calcium. F F
B Is not involved in the extrinsic system of blood coagulation. F F
C Concentration will be lowered by calcitonin. T T
8

D Concentration normally falls after menopause. F F


E Is approximately 90% protein-bound. F F
Comment
Total body calcium in an adult man about 1200g (30000 mmol), 99% of it
is found in bone while only 1% is found in soft tissues and body fluid.
The plasma calcium level ranges between 9 and 10.5mg\dl (2.1 and 2.6
mmol\l). About 50% is found in un-ionised complexes, mainly bound to
plasma albumin. The rest is found in ionized form (1.00-1.3 mmol\l).
37 The secretion of growth hormone: √
A Occurs in the hypothalamus. F F
B Ceases when the adult state is reached. F F
C Is decreased during stress. F F
D Is increased during fasting. T T
E Is increased with exercise. T T
Comment:

GROWTH HORMONE

 Polypeptide hormone (191 amino acids, molecular weight


~22kDa)* produced by anterior pituitary acidophil cells
(somatotrophs). Circulates bound to growth hormone binding
protein.
38 The secretion of growth hormone: √
A Occurs in the hypothalamus. F F
B Decreased during sleep. F F
C Is decreased during stress. F F
D Is decreased with hypoglycaemia. F F
E Opposes the action of insulin. T F *
Comment:
Secretion increase during sleep but decreases during REM sleep.
39 Concerning growth hormone: √
A Plasma levels are reduced by glucose infusion. T T
B Maternal plasma levels are directly related to fetal growth. F F
C It is active on bone only until the epiphyses fuse. F F
D Its secretion is controlled by the hypothalamus. T T
E Increased activity produces a positive nitrogen balance. T T
Comment:

GROWTH HORMONE

 Polypeptide hormone (191 amino acids, molecular


weight ~22kDa)* produced by anterior pituitary
acidophil cells (somatotrophs). Circulates bound to
growth hormone binding protein

Secretion is regulated by

 HYPOTHALAMUS - Growth hormone releasing


9

hormone; Somatostatin inhibits release

 Glucose and fatty acids - fall in concentration


stimulated GH release. Oral glucose load rapidly
suppresses GH release*.Secretion increased during
prolonged fasting

 Amino acids (arginine especially) - increased


concentration stimulates GH release

 Fall in plasma free fatty acids increase GH


secretion

 Secretion has circadian rhythm - increased


secretion during periods of deep sleep. Peak release
during first 2h of deep sleep. REM sleep inhibits GH
release

 Released in discrete pulses - most frequent in


adolescence

 Stress - trauma, surgery, fever, exercise, anxiety,


pain, cold, haemorrhage - stimulate release

 Thyroxine and cortisol stimulate secretion

 Somatomedines (insulin-like growth factors) -


inhibit release

ACTIONS

 Stimulate growth - acts through stimulation of


hepatic production of insulin-like growth factors
(somatomedines). Insulin and carbohydrate
essential for growth promoting effect of GH. Not
required for intra-uterine growth*

 Stimulates mitosis in the epiphyses but does not


stimulate epiphyseal fusion. Still has effects on bone
after epiphyseal fusion*

 Continues to be secreted and have effects after


puberty*

 Stimulates production of insulin-like growth factors


in the liver*

 Anabolic - increased protein synthesis; increased


amino acid uptake, decreased plasma urea
10

concentration and induces a positive nitrogen


balance*

 Increased fatty acid mobilisation from adipose


tissue*

 Decreased glucose utilisation for energy with


increased plasma glucose concentration*

 Ketogenic and diabetogenic

 Opposes most insulin actions*


40 Growth hormone: √
A Is a protein. T T
B Has a molecular weight of 2000 daltons. F F
C Secretion is stimulated by hyperglycaemia. F F
D Has growth-promoting effects mediated through insuline-like growth T T
factors.
E Is synthesised in the hypothalamus. F T *
Comment
In the hypothalamus synthesis of GHRH. But GH synthesis from anterior
pituitary.
41 Growth hormone: √
A Promotes protein synthesis. T T
B Faciliatates the hepatic synthesis of somatomedin C. T T
C Promotes insulin-mediated uptake of glucose. F F
D Increases circulating free fatty acids. T T
E Stimulates epiphyseal fusion. F F
Comment:

 Stimulates mitosis in the epiphyses but does not


stimulate epiphyseal fusion. Still has effects on bone
after epiphyseal fusion*
42 The secretion of growth hormone: √
A Occurs in the hypothalamus. F F
B Ceases when adult state is reached. F F
C Is decreased during stress. F F
D Is increased during fasting. T T
E Is increased with exercise. T T

43 The actions of growth hormone include : √


A Promotion of protein synthesis. T T
B Facilitation of hepatic synthesis of somatonedin C (insulin-like growth T T
factor).
C Promotion of the insulin-mediated uptake of glucose. F F
D Stimulation of lipolysis. T T
E Stimulation of the growth spurt at the onset of puberty. T F *

44 Concerning growth hormone: √


11

A Plasma levels are reduced by glucose infusion. T T


B Maternal plasma levels are directly related to fetal growth. F F
C It is active on bone only until the epiphyses fuse. F F
D It is secretion is controlled by the hypothalamus. T T
E Increased activity produces a positive nitrogen balance. T T

45 Successful lactation is: √


A Maintained by oestrogens. F F
B Maintained by progesterone. F F
C Initiated by a prolactin surge. T
D Maintained by human placental lactogen. F F
E Inhibited by dopamine. T T

46 Prolactin: √
A Release is stimulated by thyrotrophin-releasing hormone. T T
B Plasma levels are raised in the first trimester. T T
C Release is increased by suckling. T T
D May be produced by the decidua. T F *
E Release is inhibited by metaclopramide. F F

47 Prolactin: √
A Is secreted by the hypothalamus. F F
B Plasma levels are raised in the first trimester of pregnancy. T T
C Is identical to placental lactogen. F F
D Controls milk ejection. F F
E Release is inhibited by dopamine. T T
Comment
Prolactin similar to HPL and GH but not identical.
48 Prolactin: √
A Release is stimulated by thyrotropin-releasing hormone. T T
B Plasma levels are raised in the first trimester of pregnancy. T T
C Is identical to human placental lactogen. F F
D Control milk ejection. F F
E Release is inhibited by metoclopramide. F F

49 During human lactation: √


A Oxytocin increases mammary duct pressure. T T
B Oestrogen promotes the milk producing effects of prolactin on the breast. F F
C Human placental lactogen is essential for milk synthesis. F F
D Prolactin stimulates gonadotrophin release. F F
E Suckling enhances prolactin release. T T

50 In human lactation: √
A Oestrogens promote development of the breast lobules. F F
B Oestrogens promote the milk-producing effect of prolactin on the breast. F F
C Human placental lactogen is essential for milk synthesis. F F
D Prolactin stimulates gonadotrophin release. F F
E Oxytocin causes milk ejection. T T
12

51 Release of prolactin from the anterior pituitary gland is enhanced by: √


A Dopamine. F F
B Thyrotrophin-releasing hormone. T T
C Oestrogens. F F
D Chlorpromazine. T T
E Growth hormone. F F

52 Human placental lactogen:


A Is a single-chain polypeptide. T F *
B Reaches the same concentration in fetal and maternal blood at term. F F
C May be secreted by the deciduas. F F
D Is detectable only after the 25th week of pregnancy. F F
E Is an insulin antagonist. T T
Comment:
HUMAN PLACENTAL LACTOGEN

 Protein hormone secreted by the trophoblast - single


polypeptide with 191 amino acids

 Similar structure to growth hormone


Placental HPL secretion is NOT influenced by somatostatin
or growth hormone releasing hormone

 Half life ~15 min

 Maternal levels correlate with fetal and placental weight

 Plasma concentrations rise with gestation age

 Very little HPL enters the fetal compartment.

 Decreases insulin sensitivity (antagonist), stimulates


insulin release, stimulates lipolysis and decreases glucose
utilisation by the mother. Results in increased glucose and
free fatty acid concentrations.
53 Human placental lactogen:
A Concentration in maternal plasma is directly proportional to the functional T F *
mass of placenta.
B Has a half-life in blood of less than one hour. T T
C Is a steroid hormone. F F
D Increases the mobilization of maternal free fatty acids. T T
E Reaches the same concentration in fetal blood as in maternal blood at F F
term.

54 Luteinising hormone: √
A Is required for normal corpus luteum survival. T T
B Has a half-life in the circulation of 30 hours. F F
C Is released in pulses. T T
13

D In the male stimulates testosterone production. T T


E Plasma concentrations are increased in postmenopausal women. T T
Comment:

LEUTENISING HORMONE (LH)


· Glycoprotein produced by the anterior pituitary. Protein hormones
are not significantly protein bound
· Acts via a receptor on the cell membrane
· Has an alpha and a beta subunit
· Concentrations are low in childhood, increase during puberty and
are elevated after the menopause
· Concentrations begin to rise within a few days of the onset of
menstruation but fall during the latter part of the follicular phase
(negative feed-back from rising oestrogen concentrations)
· There is a pre-ovulatory LH peak occurring about 18h before
ovulation. Without the LH surge, ovulation does not occur
· Stimulates the synthesis of androstendione and testosterone by
the theca cells which are converted to oestrogen by the granulosa
cells
· Stimulates progesterone production by the granulosa cells and
later the conversion of granulosa into lutein cells
· FSH and oestrogen induce the expression of LH receptors by
granulosa cells.
Secretion is regulated by:
1) GnRH from the anterior pituitary
2) Oestrogen from the ovary - initially inhibits LH secretion but
induces the pre-ovulatory surge
3) Inhibin - mainly inhibits FSH, but also LH secretion
4) Progesterone - in high concentrations inhibit LH secretion.
55 Luteinising hormone: √
A Is a glycoprotein. T T
B Has a molecular weight of approximately 4000 daltons. F F
C Stimulates androgen production by ovary. T F *
D Concentrations in the circulation are high in girls before puberty. F F
E Concentrations in the circulation are raised in untreated adult with Turner T T
syndrome.

56 Follicle-stimulating hormone: √
A Binds to receptors on the cell membrane. T T
B Promotes the expression of luteinising hormone receptors. T T
C Is responsible for degeneration of the corpus luteum. F F
D Is a steroid hormone. F F
E Is synthesized in the hypothalamus. F F
Comment:
FOLLICLE-STIMULATING HORMONE (FSH)
· Gonadotropin - produced by the anterior pituitary gland.
Glycoprotein
· Has an alpha and a beta sub-unit
· Functions via a receptor on the cell membrane
14

· Main function is the stimulation of the growth of 6-12 primary


follicles per month
· Concentartions are low during childhood, increase during puberty
and are elevated after the menopause
· Concentrations begin to rise within a few days of the onset of
menstruation and then fall as the oestrogen levels rise. There is a
pre-ovulatory surge in FSH concentration
· Stimulates granulosa cell proliferation and also the ovarian stroma
to form the theca interna and externa of the vesicular follicle.
Production is regulated by:
1) GnRH: from the hypothalamus - stimulates FSH production
2) Oestrogen from the ovary - negative feed-back to inhibit pituitary
FSH production
3) Inhibin: from the ovary - inhibits FSH production
4) Activin: from the ovary - stimulates FSH production
5) High levels of progesterone inhibit FSH production.
57 Plasma levels of follicle stimulating hormone are elevated in:
A Adults with Klinfelter syndrome. T T
B Women taking oral contraceptive preparations. F T *
C Postmenopausal women. T T
D Adults with Turner syndrome (45X0). T T
E Pregnancy. F F

58 Serum concentrations of follicle-stimulating hormone are:


A Increased in Kleinfelter syndrome. T T
B Decreased in primary hypogonadism. T T
C Decreased following the menopause. F F
D Increased with oestrogen therapy. F F
E Increased following male castration. T T
Comment
FSH increases after the menopause, Turner syndrome, premature ovarian
failure.
And decreases with oestrogen therapy, compined oral contraceptive pills
and pregnancy.
59 Oxytocin: √
A Is released episodically. T F *
B Causes decreased renal tubular reabsorption of water. F F
C Is responsible for milk ejection. T T
D Reduces intestinal peristalsis. F F
E Inhibits prolactin secretion. F F
Comment:
OXYTOCIN *****
 Polypeptide hormone, 9 amino acids, produced mainly by
the para-ventricular nucleus of the hypothalamus and
released by the posterior pituitary. Also produced locally in
the ovary and testis *
 Differs from ADH (arginine vasopressine) in only 2 amino
acids
 Circulate largely unbound in plasma
15

 Kidneys, liver and brain are the main sites of clearance


 Half life ~1 min
 Secreted episodically by calcium-dependent exocytosis *
 Basal concentrations are similar in males and females
 Oestrogen increases the sensitivity of the myometrium to
oxytocin, while progesterone decreases sensitivity *
Secretion is stimulated by
 Suckling *
 Vaginal distension during intercourse
 Labour
 Secretion is inhibited by emotional stress *
ACTIONS
 Causes milk ejection (let down) by stimulating contraction of
the myoepithelial cells of the alveoli of the breast *
 Rhythmic contraction of uterine smooth muscle. High
pharmacological concentrations may cause tetanic
contraction
 High concentrations cause fluid retention - homology with
ADH *.
 Stimulate peristalsis *.
60 In the human, Oxytocin: √
A Promote milk synthesis in the breast. F F
B Release is inhibited by alcohol. T T
C Action on the uterus is enhanced by oestrogens. T F *
D Release is inhibited by dopamin. F F
E In excess causes dehydration. F F

61 The following agents cause myometrial contractions:


A Magnesium sulphate. F F
B Nifidipine. F F
C Progesterone. F F
D Salbutamol. F F
E Sodium nitroprusside. T

63 Rennin: √
A Is secreted by the zona glomerulosa of the adrenal cortex. F F
B Is a proteolytic enzyme. T T
C Is secreted at an increased rate if the renal perfusion pressure falls. T T
D Acts upon circulating angiotensinogen. T T
E Is released in response to an increase in extracellular fluid volume. F F

64 In the rennin-angiotensin system: √


A Decreased renal blood flow causes increased rennin production. T T
B Rennin is secreted by the juxta-glumerular cells of kidney. T T
C Rennin causes production of angiotensin I. T T
D Angiotensin I is a decapeptide. T F *
E Angiotensin II suppresses aldosterone secretion. F F

65 In the rennin-angiotensin system: √


16

A Rennin is an enzyme. T T
B Angiotensin II is converted to angiotensin I. F F
C Angiotensinogen is globulin. T T
D Rennin release is inhibited by sodium restriction. F T *
E Rennin is present in amniotic fluid. T T

66 Concerning the rennin-angiotensin system: √


A Rennin is secreted by the proximal tubule. F F
B Rennin is responsible for the conversion of angiotensin I to angiotensin II. F F
C Angiotensin II is a potent presser agent. T T
D The presser effect of angiotensin II is suppressed in normal pregnancy. T T
E Angiotensin II increases the secretion of aldosterone from the adrenal T F *
cortex.

67 Aldosterone: √
A Reduces sodium reabsorption in the proximal convoluted tubule. F F
B Reduces sodium absorption in the descending loop of Henle. F F
C Increases sodium absorption in the distal convoluted tubule. T T
D Increase potassium loss from the tubule. T T
E Increases sodium absorption in the collecting tubule. T T
Comment:
ADRENAL GLAND

• Medulla - produces catecholamines and cortex - produces


steroids
• Cortex has three distinct zones:
1) Zona glomerulosa - aldosterone
2) Zona fasciculata - cortisol and some
androgens
3) Zona reticularis - androgens and some cortisol

ALDOSTERONE SECRETION
• Decreased renal blood flow stimulates renin secretion by the
juxtaglomerular cells of the kidney
• Renin converts angiotensinogen to angiotensin I
• Angiotensin I is converted to angiotensin II & III by angiotensin
converting enzyme
• Angiotensin II & III are potent vasoconstrictors and stimulate the
synthesis and secretion of aldosterone from the zona glomerulosa
of the adrenal cortex

REGULATION OF ALDOSTERONE SECRETION


• Hyperkalaemia - a rise in plasma potassium is the most potent
stimulus for aldosterone secretion
• The renin - angiotensin system
• Total body sodium - decreased extracellular fluid volume and
decreased sodium concentration
• ACTH
• stimulates aldosterone release
17

• Aldosterone is metabolised in the liver and excreted in the


kidneys as glucuronide conjugates.

EFFECTS OF ALDOSTERONE
• ~50% is bound to plasma proteins
• Interacts with receptor within the cytoplasm which is translocated
to the nucleus to stimulate transcription. Takes up to 45 minutes for
sodium absorption to be altered
• Increases sodium resorption in the distal renal tubules while
increasing potassium and proton excretion
• Sodium resorption results in water absorption and a rise in
extracellular fluid volume with little change in sodium concentration
• Also increases sodium absorption from the GI tract and
resorption from the sweat and salivary gland ducts.
68 Aldasterone: √
A Secretion is increased by a low potassium intake. F F
B Production is reduced in normal pregnancy. F F
C Secretion is entirely regulated by the rennin-angiotensin system. F F
D Is a principle mineralcorticoid secreted by the adrenal gland. T F *
E In the kidney, acts on the distal convoluted tubule. T T
Comment:
REGULATION OF ALDOSTERONE SECRETION
• Hyperkalaemia - a rise in plasma potassium is the most potent
stimulus for aldosterone secretion.
69 Aldosterone secretion is increased: √
A On standing. T T
B Following haemorrhage. T T
C During pregnancy. T T
D By hypocalcaemia. F F
E On a low sodium diet. T T

62 Antidiuretic hormone: √
A Is normally synthesised in the hypothalamus. T T
B Is secreted by the posterior pituitary gland. T T
C Exerts its effect by reducing the glomerular filtration rate. F F
D Is an oligapeptide. F F
E Secretion is increased when plasma osmolality falls. F F
Comment:
ARGININE VASOPRESSIN (ANTIDIURETIC HORMONE) *****
 Polypeptide of 9 amino acids, produced by the supra-optic
nucleus of the hypothalamus and secreted by the pituitary
gland *
 Similar structure to oxytocin *
Secretion is stimulated by
 Hypovolaemia and hypotension - 10% blood loss would
stimulate ADH secretion. Plays important role in arterial
pressure regulation *
 Rise in plasma osmolarity - regulates sodium ion
concentration of extracellular fluid *
18

 Angiotensin
 Pain
 Emotional stress
 Nausea and vomiting
 Drugs such as morphine and nicotine
Secretion is inhibited by
 Decreased plasma osmolarity *
 Increased cardiac out-put *
 Atrial natruretic peptide
 Alcohol *
ACTIONS
 Increased water absorption from the renal tubules, in
particular increased permeability of the final third of the
distal tubule and collecting duct *
 Increased urine osmolarity and decreased urine flow *
 Stimulates sodium resorption and urea transport from the
lumen into the interstitial fluid in the medullary collecting
ducts
 Potent vasoconstrictor acts on arteriolar smooth muscle of
skin and splanchnic circulation *
 Also causes bradycardia and decreased cardiac output
physiological effect on blood pressure is very small.
However, plays important role during haemorrhage and
dehydration
 Has CRF-like activity and stimulates ACTH release
 Renal effects of ADH blocked by: hypokalaemia,
hypercalcaemia, lithium therapy
 Large doses of ADH cause smooth muscle contraction and
can cause hypertension and stimulate peristalsis
 In pregnancy plasma osmolality decreases from 285 to
275mosm/kg

In pregnancy both the plasma level of vasopressin and the release of


vasopressin following dehydration or water loading remain unchanged*.

70 Argentine vasopressin: √
A Reduces the glomerular filtration rate. F F
B Controls water loss in the proximal renal tubule. F F
C Is synthesized by the posterior pituitary gland. F F
D Is released in response to a rise in plasma osmolality. T T
E Is released in response to a fall in circulating plasma volume. T T

71 Argenin vasopressin: √
A Is produced in the hypothalamus. T T
B Is a polypeptides. T T
C Is structurally similar to prolactin. F F
D Controls water reabsorption by kidney. T T
E Decreases glomerular filtration. F F
Comment:
19

 Similar structure to oxytocin *. But oxytocin Differs from ADH


(arginine vasopressine) in only 2 amino acids
72 Vasopressin: √
A Is a nanopeptide. T T
B Is synthesised in the posterior pituitary gland. F F
C Release is increased when plasma osmolality rises. T T
D Release is increased by haemorrhage. T T
E Plasma concentration is reduced in pregnancy. F F
Comment:
ARGININE VASOPRESSIN (ANTIDIURETIC HORMONE) *****
 Polypeptide of 9 amino acids, produced by the supra-optic
nucleus of the hypothalamus and secreted by the pituitary
gland *
In pregnancy both the plasma level of vasopressin and the release of
vasopressin following dehydration or water loading remain unchanged*.
73 Human chorionic gonadotrophin: √
A Is a glycoprotein. T T
B Secretion peaks at 20 week of gestation. F F
C Has intrinsic anti-thyroid activity. F F
D Is synthesised by the corpus luteum of pregnancy. F F
E Binds to luteinising hormone receptors. T T
Comments 2:
Human chorionic gonadotrophin (hCG) is a heterodimeric glycoprotein
hormone consisting of an alpha- and a beta-subunit. The hormone is
produced by the outer layer of the blastocyst and placenta & binds to LH
receptors. HCG has TSH like activity and Secretion peaks at 11-12 weeks
gestation. This hormone prolongs the lifespan of the corpus luteum as
well as stimulates production of progesterone.
Comment:
HCG has same activity as TSH. Supports the corpus luteum. Secreted by
syncytiotrophoblast.
HUMAN CHORIONIC GONADOTROPIN (HCG)

 Glycoprotein, with carbohydrates making up about one


third of its molecular weight. This accounts for the longer
half life of HCG (24h) compared to say LH (2h)

 Alpha chain similar to alpha chain of FSH, LH, TSH -


therefore has some intrinsic TSH activity- Unique beta
chain

 All human tissues make HCG but the placenta is unique


in being able to glycosylate it, increasing the half life and
giving biological activity. Sialic acid residues are essential
for the longer half life

 HGC is produced mainly by the syncytiotrophoblast


although gene expression also occurs in cytotrophoblasts
20

 HCG is detectable in the 8 cell stage embryo and takes


over from LH in supporting the corpus luteum about 8 days
after ovulation (1 day after implantation)

 GnRH is synthesized by placental cells and the receptors


are present. In vitro, GnRH stimulates HCG production
while endorphins are inhibitors

 Inhibin restrains while activin enhances the GnRH-HCG


system. Progesterone inhibits HCG secretion

 Follistatin binds activin and inhibits its action on the


GnRH-HCG system

 Only known function is support of the corpus luteum,


taking over from LH on about the 8th day after ovulation, 1
day after implantation

 May stimulate androgen production from the early fetal


testes and may also regulate the function of the fetal zone
of the adrenal cortex in early pregnancy thereby influencing
placental production of oestrogens.

 Maternal HCG ~100IU/L at the time of the first missed


period and peaks at ~100,000IU/L at ~10 weeks gestation

 Levels fall to about 10,000 - 20,000IU/L at 18-20 weeks


and remain at this level till term

 Mainly excreted by the kidneys - as a reduced fragment


of the beta sub-unit called the beta-core fragment

 Maternal, urinary, placental and amniotic fluid HCG levels


at term are higher in pregnancies with a female fetus

 Useful in the diagnosis and management of trophoblastic


disease and ectopic pregnancy

 HCG is produced by virtually all tissues - pulsatile release


from the pituitary gland together with LH - levels may be
within the sensitivity of current assays in post-menopausal
women with very high LH levels. Non-glycosylated therefore
very short half life.
74 Human chorionic gonadotrophin: √
A Is not produced by the decidua. T T
B Is biochemically indistinguishable from luteinising hormone. F F
C Is active if given to nonpregnant women. T T
D Production rises steadily throughout pregnancy. F F
E Has no influence upon the production of oestrogens by the placenta. F T
21

Comment
HCG is produced by syncytiotrophoblast.
Structurally similar to LH.
α-chain similar to all glycoproteins(LH, FSH, TSH, HCG).
β-chain similar to LH, but β of HCG contains 30 amino acid.
May stimulate androgen production from the early fetal testes and
may also regulate the function of the fetal zone of the adrenal
cortex in early pregnancy thereby influencing placental production
of oestrogens.
75 Human chorionic gonadotrophin: √
A Has a half-life in blood of two hours. F F
B Contains a β chain indistinguishable from that of human luteinising F F
hormone.
C Is produced by the preimplantation blastocyst. F F
D Is responsible for the maintenance of the corpus luteum in early T T
pregnancy.
E May be produced in tissues other than trophoblast. T F *
Comment:
HUMAN CHORIONIC GONADOTROPIN (HCG)

 Glycoprotein, with carbohydrates making up about one


third of its molecular weight. This accounts for the longer
half life of HCG (24h) compared to say LH (2h).

Alpha chain similar to alpha chain of FSH, LH, TSH - therefore


has some intrinsic TSH activity - Unique beta chain.

 HGC is produced mainly by the syncytiotrophoblast


although gene expression also occurs in cytotrophoblasts

 HCG is detectable in the 8 cell stage embryo and takes


over from LH in supporting the corpus luteum about 8 days
after ovulation (1 day after implantation).
76 Human chorionic gonadotrophin: √
A Is a steroid hormone. F F
B Is partly metabolized in the kidney. F F
C Has an α-subunit which is antigenically similar to β-subunit of luteinising F F
hormone.
D Reaches peak concentrations at the 17th week of pregnancy. F F
E Is secreted in trophoblastic diseases. T T

77 Serum concentration of the following increase during pregnancy: √


A Sex hormone-binding globulin. T T
B Prolactin. T T
C Total thyroxine. T T
D Follicle-stimulating hormone. F F
E 17α-hydroxyprogesterone. T T
Comment
SHBG levels are decreased by testosterone and increased by oestradiol.
22

Levels are therefore highest in females during pregnancy and lowest in


male after puberty.
Synthesis of SHBG is stimulated by: oestrogen and thyroxine, and
inhibited by: insulin, GH, androgenic drugs and progestagens.
Concentration are lower in obese subjects.
78 During pregnancy, uterine deciduas synthesized: √
A Human chorionic gonadotrophin. F F
B Prostaglandin E2. T T
C Progesterone. F F
D Prolactin. T T
E Oxytocin. T T

79 Products of human decidua include: √


A α-fetoprotein. F F
B Prolactin. T T
C Human chorionic gonadatrophin. F F
D Human placental lactogen. F F
E Prostaglandin F2α. T T

80 Trophoblast: √
A Develops from the blastocyst. T T
B Gives rise to the fetal blood vessels in the placenta. T T
C Enters the maternal circulation during normal pregnancy. T T
D Replaces endothelium of pregnant spiral arterioles. T T
E Is genetically identical to decidua. F F

81 The following cell types are present in the human corpus luteum: √
A Endothelial cells. T T
B Macrophages. T T
C Pericytes. T T
D Fibroblasts. T T
E Granulose cells. T T

82 The corpus luteum of pregnancy produces: √


A Relaxin. T T
B Progesterone. T T
C 17α-hydroxyprogesterone. F F
D Human chorionic gonadotrophin. F F
E Oestradiol. T T

83 Concerning puberty in the female: √


A The development of secondary sexual characteristics is preceded by T F
increased production of adrenal androgens.
B Puberty is associated with the nocturnal release of luteinising hormone. T T
C The ovaries are not sensitive to gonadotrophins before puberty. F F
D Appearance of pupic hair usually precedes menarche. T T
E Appearance of pupic hair precedes the onset of breast development. F F
Comment
23

Usually breast growth and the growth spurt occur first, followed by the
appearance of pubic, then axillary hair and then menstruation.
84 Concerning the human ovary: √
A Aromatase catalyses the conversion of testosterone to 17β-oestradiol. T T
B Relaxin is synthesised in the corpus luteum. T T
C Inhabin stimulates follicle-stimulating hormone release from the pituitary F F
gland.
D Progesterone decreases the sensitivity of myometrial cells to oxytocin. T T
E Cholesterol is converted to androstenedione in granulose cells. F T *

85 Concerning ovarian function: √


A Progesterone is the major steroid of the developing follicle. F F
B Granulose cells secrete oestradiol. T T
C Oestradiol is derived from androgen precursors. T T
D Insulin-like growth factor (IGF)-1 is not secreted by the ovary. T T
E Circulating inhibin concentrations are a marker of granulose cell function. F F
Comments 2:
Oestrogen is the major hormonal product of the follicular cells (which
also secrete inhibin) of the developing follicle whereas, progesterone is
the product of the corpus luteum. The granulosa cells also secrete
oestradiol and inhibin. Oestrogen is derived through aromatisation of
testosterone. The liver under the influence of Growth Hormone mainly
secretes IGF-1.
Comments 2:
Progesterone is mainly produced in the luteal phase and oestradiol is the
major steroid in the developing follicle. Normally, thecal cell androgens
produced in response to LH diffuse to granulosa cells, where granulosa
cell aromatase converts these androgens to estrogens. Somatomedin-C) –
is secreted by the pituitary. Inhibin is a glycoprotein hormone produced
and secreted by the granulosa and theca cells of the ovary with evidence
that inhibin levels correlate to ovarian reserve incl granulose cell function.
86 Concerning ovarian function : √
A Progesterone is the major steroid of the developing follicle. F F
B Granulose cells secrete oestradiol. T T
C Oestradiol is derived from androgen precursors. T T
D Insulin-like growth factor (somatomedin C) is not secreted by the ovary. T F
E Circulating inhibin concentrations are a marker of granulose cell function. F F

87 Ovaries secrete: √
A Progesterone. T T
B Androstenedione. T T
C Testosterone. T T
D 17β-oestradiol. T T
E Aldosterone. F F
Comment:

 The antral follicle secretes increasing amounts of steroids -


mainly 17-beta oestradiol and oestrone (granulosa cells -
produced de novo or by aromatization of androgens from
24

thecal cells). The main androgens produced are


testosterone and androstendione (thecal cells) *.

88 Inhibin: √
A Is structurally identical to relaxin. F F
B Is released in pulses. T T
C Is a steroid. F F
D Is produced by the ovarian follicle. T T
E Inhibit the release of follicle-stimulating hormone. T T
Comment:

Released in a pulsatile fashion with periodicity of 60-70


minutes.

Secretion by the granulosa cells is stimulated by FSH. Inhibin in


turn inhibits pituitary FSH production.

Comments 2:
Inhibin and relaxin are structurally dissimilar. Ovarian inhibin
exerts a pulsatile release of gonadotrophins. Inhibin is a
glycoprotein hormone produced and secreted by the granulosa and
theca cells of the ovary. It also inhibits FSH secretion.
89 During the normal ovarian cycle: √
A The principle oestrogen secreted is 17-Oestradiol. T T
B The most potent oestrogen is oestriol. F F
C Oestrogen production is maximal by about the 8th day of the cycle. F F
D Oestrogens decrease the secretion of follicle-stimulating hormone. T T
E Oestrogens are synthesized primarily by the ovarian stroma. F F
Comment
Oestrogen production is maximal preovulatary 1 day before LH surge.
The maximum secretion of progesterone from the corpus luteum occurs 8
days after LH surge.

90 Progesterone: √
A Is synthesised by trophoblast. T T
B Increase myometrial activity. F F
C Is predominantly excreted in the urine as pregnanetriol. F F
D Binds to cortisol-binding globulin in the circulation. T T
E Is synthesised from cholesterol. T T
Comment
Pregnanediol glucuronide is the major metabolite of secreted
progesterone. pregnanetriol is a metabolite of adrenocortical hormones.
Progesterone= 80% is bound to albumin.
18% to cortisol binding globulin.
2% is free.
91 Progesterone: √
A Is a C -21 steroid. T T
B Is synthesised by the ovary before ovulation. T T
C Increases ventilation. T T
25

D Rises basal body temperature. T T


E Binds to corticotrophin-binding globulin. T T

92 Concerning sex hormones: √


A The ovary secretes androstenedione. T T
B The ovary secretes testosterone. T T
C The ovary secretes dihydrotestosterone. T T
D Sex hormone-binding globulin (SHBG) concentrations are higher in T T
women than in men.
E Androgens bound to protein have high biological activity. F F
Comment:
SHBG levels are decreased by testosterone and increased by oestradiol.
Levels are therefore highest in females during pregnancy and lowest in
male after puberty.
Synthesis of SHBG is stimulated by: oestrogen and thyroxine, and
inhibited by: insulin, GH, androgenic drugs and progestagens.
Concentration are lower in obese subjects
93 Sex hormone-binding globulin: √
A Levels are increased in pregnancy. T T
B Is the main binding protein for progesterone. F F
C Levels are decreased during oestrogen therapy. F F
D Is the main binding protein for aldosterone. F F
E Has a greater affinity than albumin for testosterone. T F *
Comment
Progesterone= 80% is bound to albumin.
18% to cortisol binding globulin.
2% is free.
Steroid binding profiles:
Hormone free% SHBG% CBG% albumin %
Oestradiol 1.8 37.3 0.1 60.8
Oestrone 3.6 16.3 0.1 80.1
Progesterone 2.4 0.6 17.7 79.3
Testosterone 1.4 66 2.3 30.4
Androstenedione 7.5 6.6 1.4 84.5
Cortisol 3.8 0.2 89.7 6.3
SHBG has a high affinity low capacity, albumin has a low affinity but
high capacity.
94 The physiological action of oestradiol depends upon: √
A Metabolism to a more potent substance. F F
B Binding to an intracellular receptor. T T
C Alteration of gene expression. T T
D Active transport of the hormone into cells. F F
E Cyclic AMP production. F F

95 In the human testis: √


A Meiosis occurs between the early and late spermatid phases. F F
B One spermatogonium always forms 8 sprmatides. F F
C The fluid in the seminiferous tubules contains a high concentration of F F
protein.
26

D The process of spermatogenesis takes 34 days. F F


E Inhibin is produced by primary spermatocytes. F F
Comment
Meiosis occurs between primary spermatocyte and early spermatids.
One spermatogonium forms 16 spermatides.
Spermatocyte takes 74 days.
Seminiferous tubules contain low protein and low glucose contains.
Inhibin is produced from sertoli cells.
96 The interstitial (leydig) cells of testis: √
A Secrete seminal fluid. F F
B Are stimulated by luteinising hormone. T T
C Secrete androgen binding protein. F F
D Secrete fructose. F F
E Produce testosterone. T T
Comment
Androgen binding protein is secreted by sertoli cells. Fructose secreted in
the seminal fluid from seminal vesicles.
Comments 2:
The testes are responsible for producing sperm and male hormones
(mainly testosterone) that regulate reproductive organ development. In the
adult testis, Leydig cells express aromatase (P450arom) and actively
synthesise oestradiol. 17-alpha-hydroxyprogesterone is secreted by the
adrenal gland. Prolactin is secreted by the anterior pituitary gland. In the
testes, LH binds to receptors on Leydig cells, stimulating synthesis and
secretion of testosterone.
97 Concerning testicular hormone: √
A Testosterone reduces plasma luteinising hormone concentrations. T T
B Inhibin stimulates luteinising hormone production. F F
C Oestrogens are formed in the testis. T T
D Testosterone is converted to hydrotestosterone by 5α-reductase. T T
E Testosterone in plasma is predominantly bound to albumin. F F
Comment
Testosterone is the principal hormone of the testis, is a C19 steroid, it has
an inhibitory feedback effect on LH.
Extragonadal and extraadrenal aromatization of circulating testosterone
and androstenedione.the remainder comes from the testis.
Some of the oestradiol comes from the leydig cells, but some is produced
by aromatization of androgens in sertoli cells.
Testosterone in plasma is predominantly bound to sex hormone binding
globulin.
98 In the human testis: √
A The main site of testosterone synthesis is the sertoli cell. F F
B Testosterone synthesis is stimulated by follicle stimulating hormone F F
(FSH).
C The predominant androgen product is androstenedione. F F
D The Leydig cells synthesize testosterone. T T
E Inhibin is synthesized by the sertoli cell. T T

99 The following statements about testicular hormones are true: √


27

A Testosterone reduces plasma luteinising hormone levels. T T


B Inhibin increases plasma follicle-stimulating hormone levels. F F
C Oestrogens are formed in the testis. T T
D Testosterone is excreted in urine as 17-ketosteroids. F F
E Testosterone in plasma is partly bound to albumin. T T
Comment:
Binding of testosterone to plasma protein:
Male % Femal %
Free 2 1
Albumin-bound 38 19
SHBG-bound 60 80

Testosterone is metabolized to 17-ketosteroids in the liver, and after


transformation to sulfate or glucuronide, is excreted in the urine.

Most testosterone circulates in the bloodstream bound either to sex


hormone-binding globulin (SHBG) or to albumin; only 2-3% is unbound
in its functionally active form.
100 Concerning sex hormone: √
A The ovary secretes androstenedione. T T
B The ovary secretes testosterone. T T
C The ovary secretes dihydroxycholecalciferol. F F
D Sex hormone-binding globulin (SHBG) concentrations are higher in T T
women than men.
E Androgens bound to protein have high biological activity. F F

101 Hirsutism in women is characteristically associated with: √


A Testicular feminization. F F
B Turner syndrome. F F
C Polycystic ovary syndrome. T T
D Arrhenoblastoma. T T
E Hypopituitarism. F F

102 Hirsutism is associated with: √


A Testicular feminization syndrome. F F
B Turner syndrome. F F
C Addison's disease. T T
D Sertoli-leydig cell tumors. F F
E Congenital adrenal hyperplasia. T T

103 Testosterone: √
A Is produced only in the gonads. F F
B Is mainly excreted unchanged in the urine. F F
C Stimulates secretion of luteinising hormone. F F
D Circulates in plasma mainly in the free form. F F
E Stimulates growth of the prostate gland. T T
Comment
Testosterone in most tissues binds very weakly to the receptor and must
28

first be converted to DHT by the enzyme 5a-reductase. DHT is 100 times


more potent. This convertion is essential in hair follicles and in the
development of the urogenital sinus and urogenital tubercle into male
external genitalia and prostate.
104 Testosterone in the human male: √
A Depresses pituitary secretion of luteinising hormone. T T
B Promotes growth of scalp hair. F F
C Promotes union of long bone epiphyses. F F
D Is a more potent androgen than dihydrotestosterone. F F
E Is secreted maximally in the evening. T T

105 In the human male, dihydrotestesterone: √


A Is a precursor of testosterone. T T
B Has one-tenth of the potency of testosterone. F F
C Is responsible for involution of the Mullerian system. F F
D Is responsible for the development of the male external genitalia. T T
E Binds to an intracellular receptor. T T

106 Oestradiol-17β: √
A Is synthesized by aromatisation of testosterone. T T
B Vasodilates the uterine artery. T T
C Suppresses uterine activity by upregulating the oxytocin receptor. F F
D Promotes secondary sexual hair growth in females. F F
E Is thrombogenic. T T
Comment
In female the secondary sex characteristics are due to oestrogen: they
consist of the development of breast. And deposition of body fat. But the
features such as the growth spurt, the growth of axillary and pubic hair
and the development of libido are due to adrenal androgens (mainly
DHEA).
107 Oestradiol-17β: √
A Is synthesized by aromatisation of testosterone. T T
B Can be administered orally. T T
C Suppresses uterine activity by upregulating the oxytocin receptor. F F
D Promotes secondary sexual hair growth in females. F F
E Is thrombogenic. T T

108 After the menopause: √


A The plasma concentration of follicle stimulating hormone increases. T T
B The plasma progesterone concentration increases. F F
C Oestrone is the oestrogen found in highest concentration in the plasma. T T
D The plasma testosterone concentration doubles. F F
E The plasma prolactin concentration increases. F F
Comment

MENOPAUSE
· Average age 50.8 years
· Caused by ‘burn-out’ of the ovarian follicles - insufficient to
produce enough oestrogen to suppress FSH secretion or to induce
29

LH surge
· Oestrogen and progesterone levels fall
· FSH & LH concentrations are elevated which suppress GnRH
secretion.
109 Menopause: √
A Is due to failure of the endometrium to respond to oestrogens. F F
B Is associated with cessation of steroidogenesis within the ovary. T F *
C Is associated with a fall in circulating luteinising hormone. F F
D Is preceded by a period of enhanced fertility. F F
E Results in atrophy of the epithelium of the distal urethra. T T
Comment :
110 Concerning androgens in normal premenopausal women: √
A 95% of circulating testosterone is derived from peripheral conversion of F F
androstenedione.
B The ovaries and adrenals contribute equally to circulating androgen F F
concentrations.
C DHEAS (dehydroepiandrosterone sulphate) is derived almost exclusively T T
from adrenal glands.
D About50% of circulating testosterone is bound to sex hormone-binding F F
globulin (SHBG).
E Testosterone promotes the synthesis of SHBG. F F
Comment:
Source of androgen in women:
Steroid ovary% adrenal% peripheral convertion%
Testosterone 20 30 50
Androstenedione 30 60 10
DHEAS - 80
Dihydrotestosterone 100
----------------------------------------------------------------------------------------
Binding of testosterone to plasma protein:
Male % Femal %
Free 2 1
Albumin-bound 38 19
SHBG-bound 60 80
--------------------------------------------------------------------------------------
SHBG levels are decreased by testosterone and increased by oestradiol.
Levels are therefore highest in females during pregnancy and lowest in
male after puberty.
Synthesis of SHBG is stimulated by: oestrogen and thyroxine, and
inhibited by: insulin, GH, androgenic drugs and progestagens.
Concentration are lower in obese subjects.
111 In a woman of reproductive age, serum concentrations of the √
following hormones exhibit a recognized pattern of diurnal variation:
A Progesterone. F F
B Melatonin. T T
C Cortisol. T T
D Oestrone. F F
E Follicle-stimulating hormone. F F
30

112 Insulin secretion is stimulated by: √


A Gastrin. T T
B Noradrenaline (norepinephrine). F F
C Somatostatin. F F
D Glucagons. T T
E Arginine. T T
Comment:
Insulin secretion is stimulated by
 Glucose *
 Amino acids, in particular arginine and lysine. These
potentiate glucose-stimulated insulin secretion *
 Gastro-intestinal hormones - Gastrin, Gastric inhibitory
peptide, Secretin, Cholecystokinin *
 Other hormones - cortisol, growth hormone,
glucagon* (indirectly through increased glucose),
progesterone and oestrogen to a lesser extent
 Fatty acids have little effect on insulin secretion
 Fasting insulin levels are higher in pregnancy compared to
non-pregnancy *
 When glucose is given orally, a greater insulin response is
elicited than when it is administered intravenously *
 Insulin secretion in response to a glucose load is bi-phasic:
an early rise (phase 1) - release of available insulin; late rise
(phase 2) - synthesis of new insulin. Plasma insulin peaks
30-60 minutes after a meal
Sympathetic stimulation inhibits insulin secretion *.
Comments 2:
Gastrin stimulates gastric acid secretion and proliferation of gastric
mucosa and also stimulates insulin release. Somatostatin's effects are all
inhibitory including that of insulin release. Glucagon stimulates an
increase in blood sugar levels, thus opposing the action of insulin. The
increase in glucose would cause a counteractive increase in insulin.
Insulin secretion is influenced by neurotransmitters interacting with islet
cell receptors, particularly those that bind norepinephrine.
113 Insulin secretion: √
A Is partly controlled by the direct action of blood glucose upon the T T
pancreas.
B Affect the rate of entry of glucose into the β-cells of the pancreatic islets. F F
C Is stimulated by adrenaline. F F
D Is stimulated by arginine. T T
E Is stimulated more effectively by glucose administered interavenously F F
than by glucose administered orally.
Comment:
 When glucose is given orally, a greater insulin response is
elicited than when it is administered intravenously *
 No effect on glucose uptake and utilisation by the brain *
114 Concerning insulin: √
A The half-life of endogenous insulin in the circulation is 30 minutes. F F
B The kidney is a major site of insulin degradation. T T
31

C It facilitates glucose uptake by the brain. F F


D Fasting concentrations are lower in pregnant women at term than they are F T
in nonpregnant women.
E It is formed when C-peptide is separated from proinsulin. T T
Comment:
 Polypeptide, A & B chains joined by two disulphide bonds *
 Half-life ~ 6 minutes *
 The liver and the kidney are the principal sites for insulin
clearance*. Very little insulin appears in urine.
115 Human insulin: √
A Is composed of two chains of amino acids. T T
B Differs from pig insulin by one amino acid. T T
C Facilitates glucose uptake by red blood cells. F F
D Increases protein synthesis in the liver. T T
E Increase triglyceride deposition in adipose tissue. T T

116 Actions of insulin include stimulation of: √


A Glycogenolysis by the liver. F F
B Cellular uptake of amino acids. T T
C Entry of glucose into neurons. F F
D Entry of glucose into adipose tissue. T T
E Cellular uptake of potassium. T T
Comments 2:
Insulin is anabolic in nature and it predominantly inhibits glycogenolysis
in the liver (though it also stimulates glycogenolysis) and stimulates
glucose uptake in adipose tissues and cellular uptake of amino acids
(protein synthesis). Some tissues do not require insulin for efficient uptake
of glucose: important examples are brain and the liver. This is because
these cells don't use GLUT4 (a hexose transporter for facillitaed diffusion
of glucose) for importing glucose, but rather, another transporter that is
not insulin-dependent. Insulin is used for the treatment of hyperkalaemia
and hence 'drives' K+ back into cells.
117 Glucagon promotes: √
A Hepatics gluconeogenesis. T T
B Glucose uptake by muscle. F F
C Glycogen synthesis by muscle. F T *
D Breakdown of protein. T T
E Synthesis of fat. F T *
Comment:
GLUCAGON *****

 Polypeptide hormone (29 amino acids) produced by the


alpha cells of the islet of Langerhans
 Not significantly protein-bound, acts by cell-surface receptor
which activates adenylate cyclase, resulting in increased
intracellular cyclic AMP
 Half-life ~ 6 mins *
Secretion is stimulated by
 Hypoglycaemia
32

 Amino acids - alanine and arginine (the only factors which


stimulate glucagon as well as insulin secretion)
 Exercise
 Hyperglycaemia and free fatty acids inhibit glucagon
secretion
ACTIONS
 Increased hepatic glycogenolysis - activates glycogen
phosphorylase
 Increased hepatic gluconeogenesis (synthesis of glucose
from amino acids)*
 Inhibits hepatic glycogen synthesis
 Increased fatty acid mobilisation and beta-oxidation -
ketogenic *
 Results in increased plasma glucose, free fatty acids, keto-
acids but decreased plasma amino acids *
 Minimal effects on peripheral adipose tissue or skeletal
muscle
Comments 2:
Glucagon causes the recruitment of glucose from hepatic stores and has a
positive inotropic effect on the heart. Paradoxically stimulates insulin
release
118 Therapeutic indications for the use of synthetic progestagens include: √
A Induction of abortion. F F
B Contraception. T T
C Metastatic endometrial carcinoma. T T
D Suppression of lactation. F F
E Endometriosis. T T
Comment:

THERAPEUTIC USE OF PROGESTOGENS *****

 Contraception - combined oral contraceptive pill /


progestogen-only pill / progestogen implants and injectables
/ progestogen containing intra-uterine system
 Treatment of endometriosis
 Treatment of menorrhagia - less effective compared to
tranexamic acid or mefenamic acid
 Hormone replacement therapy in women with a uterus
 Endometrial support in women undergoing IVF

Treatment of endometrial cancer and pre-malignant endometrial


lesions (endometrial hyperplasia). Surgery +/- radiotherapy are the
mainstay treatment for endometrial cancer.
119 The pineal gland: √
A Lies anteriorly to the 3rd ventricle. F F
B Is innervated by the parasympasthetic nervous system. F F
C Produces melatonin. T T
D Increases in size at puberty. F F
E Activity is related to day length. T T
33

Comment
Pineal gland lies posterior to the 3rd ventricle.
Innervated by postganglionic sympathetic nerves (nerve canarii).
Decreases in size at puberty.

hormone Type Amino M.W. chain Half life


acid
GH Protein 191 22 000 single 20-30 min
Prolactin Protein 198 23 000 single 15 min
Inhibin Protein 32 000 double

hormone Type Amino M.W. chain Half life


acid
LH glycoprotein 204 30 000 double 1 hour
FSH glycoprotein 204 30 000 double 3 hours
TSH glycoprotein 201 28 000
HCG glycoprotein 30 38 400 double 5 hours
Thyroglobulin glycoprotein 140 660 000

hormone Type Amino M.W. chain Half life


acid
ACTH Polypeptide 39 4 500 5-15 min
PTH Polypeptide 84 9 500 5 min
Calcitonin Polypeptide 32 3 500
Insulin Polypeptide 51 5 800 double 5 min
Glucagon Polypeptide 29 3 500 single 3-6 min
CRH Polypeptide 41
GHRH Polypeptide 40

hormone Type Amino M.W. chain Half life


34

acid
Oxytocin Nanopeptide 9 1-4 min
Aldosterone 20 min
Vasopressin Nanopeptide 9
Somatostatin Cyclic peptide 14 1640
KARYOTYPING *****

• Utilises peripheral blood lymphocytes and can still be performed on samples


received in the lab >24h after sampling. Can also be performed on chorionic villi,
fetal cells from amniotic fluid and fetal blood following fetal blood sampling *
• Results available within 48-72h from fetal blood and chorionic villi but in 2-3
weeks from amniotic fluid *
• Cell division is arrested in metaphase by the addition of colchicines which prevents
spindle formation *
• Chromosomes are typically stained with Giemsa (G-banding). The banding pattern
of each chromosome is characteristic with the dark bands containing mainly
INACTIVE genes *
• Other banding patterns can be obtained using quinacrine (Q banding, visualised by
ultraviolet light), C-banding to show highly repetitive micro-satellite repeat DNA at
the centromeres
• Banding allows the identification of missing or additional chromosomal material of
at least 4,000kb

CHROMOSOME HETEROMORPHISMS
• These are variations in the amount of DNA present in particular chromosomes from
different individuals caused by variations in the size of repetitive DNA - they are
inherited rather than acquired
• The Y chromosome shows the greatest variation while the X chromosome shows
the least variation
• There are 4 main groups of heteromorphisms

1) The size of Yq (long arm of Y chromosome) - 10% of normal males have a


Yq which is obviously shorter or longer than usual
2) Size of centromeric heterochromatin
3) Satellite polymorphisms - variations in the size of the satellite in acrocentric
chromosomes (13-15, 21, 22). Metacentric chromosomes do not have
satellites
4) Fragile sites *- constriction sites other than the centromere which are
prone to breakage. There are at least 80 common fragile sites. Most are not
associated with any clinical syndrome except the fragile site at Xq27.3 which
is associated with learning disability

DNA STRUCTURE *****

• Nucleic acids (DNA or RNA) are made up of a chain of nucleotides


• Each nucleotide contains a nitrogenous base, a sugar molecule and a phosphate
group *
• The base is either a purine (adenine and guanine) or a pyrimidine (cytosine,
thymine and uracil) *
• Thymine occurs in DNA only while uracil occurs only in RNA *
• The sugar molecule is either ribose (RNA) or deoxyribose (DNA) and they are held
together by 3-5- phosphodiester bonds *
• The DNA molecule is made up of two chains of nucleotides held together by
hydrogen bonds between the bases
• A purine in one chain is always paired to a pyrimidine in the other chain: A-T; G-
C*
• The chain end terminated by the 5' carbon of the sugar molecule is the 5' end and
this always lies opposite the 3' end of the other strand. The two chains are arranged in
a double helix

DNA REPLICATION *****

• Occurs during the S (synthesis) phase of the cell cycle *


• The double helix is separated by the enzyme DNA helicase at different sites on the
DNA molecule *
• DNA synthesis is undertaken by DNA polymerase in the 5' to 3' direction
• The leading strand is synthesised as a continuous chain while the lagging strand is
synthesised in pieces called Okazaki fragments and then joined by DNA ligase
• Only one chain of the double helix is newly synthesised and DNA replication is
therefore semiconservative *
• Transcription is the synthesis of mRNA from DNA

CHROMOSOME STRUCTURE

• Primary coiling of DNA into double helix


• Secondary coiling of double helix around spherical histone beads to form
nucleosomes
• Tertiary coiling of nucleosomes to form chromatin fibres
• Chromatine fibres form loops on a scaffold of non-histone acidic protein
• Chromatin fibres are wound into a tight coil to form chromosome
• All chromosomes have a centromere, the site of attachment of the spindle apparatus
during cell division
• METACENTRIC - have the centromere in the middle (1,3,16,19,20) *
• ACROCENTRIC - have the centromere close to one end (13,14,15,21, 22) -these
are the chromosomes usually involved in balanced translocations *
• SUB-METACENTRIC - have centromere in-between
• Chromosomes have a short (p = petit) and a long (q) arm

HISTONES *****

• Basic nuclear protein around which the DNA molecule is tightly bound
• Makes up about 50% of the mass of a chromosome
• Rich in lysine or arginine (~25%, these are basic amino acids)
• The genes coding for histone protein are clustered and tandemly repeated 30-40
times in humans
AUTOSOMAL DOMINANT INHERITANCE *****

• The trait / condition is expressed in heterozygous


individuals *

• Males and females affected in equal numbers *

• Vertical pedigree pattern - does not skip


generations *

• Males and females can transmit the condition to their


male / female off-spring *

• Unaffected individuals cannot transmit the disease


(the exception is in conditions with variable or non-
penetrance - where individuals with the mutant allele
have a normal phenotype but can transmit the
condition) *

• Show variable expressivity - both in terms of severity


of the condition and age of onset *

• Affected individuals have a 1 in 2 risk of their


offspring being affected *

• There is a paternal age effect with respect to the


incidence of new mutations *

• Examples include: otosclerosis, von Willebrand


disease, familial hypercholesterolaemia, Huntington’s
disease, Tuberous sclerosis, Myotonic dystrophy,
Neurofibromatosis, Hereditary spherocytosis, familial
polyposis coli *

AUTOSOMAL RECESSIVE INHERITANCE *****

• The trait / condition is only expressed in homozygous


individuals *

• Males and females are affected equally *

• Horizontal pedigree pattern *


• Both males and females can transmit the trait to their
male / female offspring *

• If both parents are carriers, 1 in 4 offspring will have


the disease, 1 in 4 will be normal homozygotes and 1 in
2 will be normal carriers *

• The offspring of an affected individual must either


have the disease or be carriers as there is no normal
allele *

• Variable penetrance and variable expressivity are


features of autosomal dominant inheritance *

• There is an increased risk of autosomal recessive


conditions in the offspring of consanguineous
marriages *

• Examples include: Cystic fibrosis, sickle cell disease,


phenylketonuria, adrenogenital syndrome,
mucopolysaccharidoses, spinal muscular atrophy

X-LINKED RECESSIVE INHERITANCE *****

• Males are affected predominantly *

• Male-male transmission never occurs. Unaffected


males never transmit the condition *

• Affected males transmit the allele to their daughters


who become heterozygous carriers and are almost
always unaffected. Lyonisation (random inactivation of
one X chromosome) may result in some females having
mild symptoms *

• Knight’s move pedigree pattern *

• The severity of the disease is uniform across


generations *
• A woman with an affected son and an affected
brother is an obligate carrier as the possibility of two
independent new mutations is extremely small. A
woman with one affected son is not an obligate carrier *

• Examples include: red-green colour blindness,


Duchenne / Becker muscular dystrophy, Haemophilia A
/ B, X-linked agammaglobulinaemia *

Females can be affected by X-linked recessive conditions due


to *****

• Atypical lyonisation - random inactivation of


(predominantly) the normal X chromosome *

• X-autosome translocation under which


circumstances the normal X chromosome is
preferentially inactivated (otherwise an autosomal
monosomy would result) *

• Turner syndrome (45X) *

• A new mutation on the second X chromosome *

X-LINKED DOMINANT INHERITANCE *****

• Males and females are affected


• The disease is uniformly severe in males but the
degree of severity is variable in females because of
lyonisation
• Male-to-male transmission does not occur. All
daughters of an affected male will be affected. 1 in 2
sons of an affected female will be affected
• Main example is Vitamin D resistant rickets

LYONISATION *****

• Random inactivation of one of the X chromosomes in


females - occurs 15-16 days after conception. In
interphase cells, the inactive X-chromosome appears
as a darkly staining mass of chromatin - the sex
chromatin or Barr body *
• Inactivation is dependent on the activity of the XIST
(X inactivation specific transcript) which is expressed
only from the inactive chromosome and results in
differential methylation of this chromosome
• The X chromosome is, however, not totally
inactivated (otherwise all females will have Turner’s
syndrome and 47XXY or 47XXX individuals will be
normal) *
• In female carriers of an X-autosome balanced
translocation, the normal X-chromosome is
preferentially inactivated (otherwise an autosomal
monosomy would result and this will be incompatible
with live birth)
• Normal males have no Barr bodies. 47XXY males
have one Barr body and 47XXX females have two Barr
bodies *
• Random inactivation of predominantly the normal X
chromosome can result in a female being affected by
an X-linked recessive disorder *
NON-DYSJUNCTION *****

• Failure of the chromosomal bivalents to segregate to the two


daughter cells during anaphase - can occur during meiosis I & II
or during mitosis *
• Occurring during meiosis I - the gamete receives both
homologues of the chromosome pair *
• Occurring during meiosis II - the gamete receives two copies
of one of the homologues of the chromosome pair *
• The majority of autosomal trisomies result from non-
dysjunction during one of the maternal meiotic division. The
other daughter gamete inherits an autosomal monosomy -
always incompatible with survival to term *
• Increased risk of non-dysjunction with increasing maternal
age *

RECIPROCAL TRANSLOCATIONS *****

• Result from breakage of at least two metacentric


chromosomes (centromere in the middle) with exchange of
genetic material *
• Long arms of chromosomes 11 and 22 more commonly
involved *
• Incidence in general population ~1:500
• The total number of chromosomes is maintained at 46 *
• Individual with balanced reciprocal translocation
phenotypically normal. Segregation at meiosis may result in
off-spring inheriting an unbalanced chromosomal
complement, resulting in an increased risk of miscarriage or
congenital anomalies *
• Risk of abnormal off-spring dependent on the particular
chromosomal rearrangement but lies between 1-10% *

ROBERTSONIAN TRANSLOCATION *****

• Results from breakage of two acrocentric chromosomes


(13,14,15,21,22) at or close to the centromere with fusion of
the long arms and (usually) a loss of the short arms *
• Total number of chromosomes reduced to 45. Short arms
usually contain only ribosomal RNA genes with multiple copies
on other acrocentric chromosomes
• Incidence 1:1,000
• 13q14q translocations most common
• Increased risk of miscarriage / fetal anomaly in carriers of
balanced Robertsonian translocations *
• Female carriers of 13q21q or 14q21q Robertsonian
translocations have a 10% risk of having a baby with Down
syndrome while male carriers have a 1-3% risk *
• 21q21q balanced translocation carriers have a 100% risk of
a live birth with Down syndrome (monosomy 21 which is the
other possibility is not compatible with live birth) *

MOSAICISM *****

• The presence of two or more cell lines within an organ /


individual with different genetic constitution but derived from the
same zygote *
• The presence of different cell lines derived from different
zygotes is termed CHIMERISM - and may occur, for instance,
as a result of twin-twin transfusion syndrome *
• Mosaicism usually results from mitotic non-dysjunction in an
early stage embryo *
• 1-2% of all cases of Down syndrome are mosaics *
PRE-IMPLANTATION GENETIC DIAGNOSIS (PGD) *****

This topic has been tested on a regular basis since 2003

• This requires IVF with biopsy of the embryo at the 6-10 cell stage. The indications are as an
alternative to pre-natal diagnosis in a couple known to be at risk of transmitting a genetic
diaorder. It can also be used as a primary tool in couples undergoing IVF treatment.

BIOPSY FOR PRE-IMPLANTATION DIAGNOSIS

Uses the following cells:

• Polar body from oocyte or zygote

• Blastomeres from cleavage stage embryos

• Trophectoderm cells from blastocysts - however, up to 60% of human embryos arrest


in culture and do not reach the blastocyst stage. Blastocyst biopsy gives more cells for
diagnosis but less time as the blastocyst would have to be transferred into the uterus
as soon as possible

• Most centres use cleavage stage embryo biopsy. There are no reports of blastocyst
biopsy / use of inner cell mass cells.

CLEAVAGE STAGE BIOPSY

• Undertaken at the 8-10 cell stage, 3 days after fertilisation

• 1-2 blastomeres are aspirated after drilling of the zona pellucida

• Zona drilling may be performed using acid Tyrodes solution, laser or by partial zona
dissection

• Compaction of the embryo begins to occur after the 8 cell stage with the formation of
inter-cellular junctions - this is Ca2+ / Mg2+ dependent. Biopsy is therefore facilitated
by using Ca2+ / Mg2+ free culture medium

POLAR BODY BIOPSY

• Removal of both polar bodies is required for accurate diagnosis. Can be performed
simultaneously or sequentially

• Second polar body is only released after fertilization

• Only maternal chromosomes are examined

• Used in the diagnosis of common aneuploidies in IVF

Pre-implantation genetic diagnosis is currently possible in the following conditions. The number of
conditions is however constantly increasing.

a) Cystic fibrosis

b) Tay Sachs disease

c) Beta-thalassaemia
d) Congenital adrenal hyperplasia

e) Spinal muscular atrophy

f) Marfan?s syndrome

g) Osteogenesis imparfecta

h) Fragile X syndrome

i) Duchenne muscular dystrophy

j) Huntington?s chorea

k) Myotonic dystrophy

l) Charcot-Marie-Tooth disease

m) Familial adenomatous polyposis coli

n) Retinitis pigmentosum

• Pre-implantation diagnosis uses a single cell for diagnosis. This is made possible by
the use of PCR technology, amplifying fragments of DNA thousands of times. FISH
uses fluorescent tagged DNA probes that bind to specific DNA sequences. Restriction
fragment length polymorphisms can be used to identify abnormal gene sequences

PCR PROBLEMS

• Contamination by DNA from the environment, laboratory staff and other cells including
sperm and cumulus cells - for this reason, ICSI(Intra-Cytoplasmic Sperm Injection) is
used prior to pre-implantation genetic diagnosis

• Allele drop-out: This is the preferential amplification of one allele only and is particularly
important in autosomal dominant conditions. If the mutated allele drops out, the
embryo would be erroneously diagnosed as normal.

MOSAICISM AND PRE-IMPLANTATION DISGNOSIS

• Up to 50% of human pre-implantation embryos show mosaicism, the commonest


abnormalities being haploid and tetraploid nuclei

• It is now recognised that 1-2 cells biopsied from the blastocyst are not necessarily
representative of the rest of the embryo

• Mosaicism does not affect sexing as female cells would have to appear in a male
embryo (or vice versa)

RESTRICTION FRAGMENT LENGTH POLYMORPHISM *****


• Polymorphisms are inherited differences found among the individuals in a population.

• Restriction enzymes are used to cut DNA at precise points producing a collection of
DNA fragments of precisely defined length.

• These can be separated by electrophoresis, with the smaller fragments migrating


farther than the larger fragments.

• Fragments can be visualized with a "probe" -a molecule of single-stranded DNA that is


complementary to a run of nucleotides in one or more of the restriction fragments and
is radioactive or fluorescent
• RFLPs used for screening human DNA for the presence of potentially deleterious
genes (for instance, in PGD) or providing evidence to establish the innocence of, or a
probability of the guilt of, a crime suspect by DNA-fingerprinting
RAPID PRE-NATAL DIAGNOSIS: TECHNIQUES
The availability of rapid diagnostic techniques has led the UK National Screening Committee to
recommend that screening programmes for Down’s syndrome need no longer include karyotyping

Karyotype is typically available after 2-3 weeks while results of rapid diagnostic tests can be available
within 24-48h. However, rapid tests only analyse specific chromosomes, typically 13, 18, 21 and X & Y if
required

Fluorescence in-situ hybridisation (FISH)

• Uses non-dividing (interphase) cells in uncultured samples

• Fluorescently labelled chromosome-specific DNA sequences are used to identify chromosome


copy number

• Three chromosomes can be detected at the same time (three flurochromes available - red,
blue and green)

• Commercial kits are available for FISH and this is therefore a relatively inexpensive technique

• Designed to detect specific aneuploidies and will not detect other abnormalities or structural
chromosomal defects

• False positive rate less than 1 in 30,000 cases

• False negative rate less than 1 in 4000 cases

• Misdiagnosis may be due to:

1) Maternal cell contamination - this will not be detectable if the fetus is female.

2) Structural chromosome anomalies

3) DNA polymorphism causing cross-hybridisation of DNA probe with other DNA locations

4) Mosaicism - this may only be detected if a sufficient cells (at least 100) are used for analysis

Polymerase Chain Reaction (PCR)

• Uses quantitative fluorescence PCR (QF-PCR)

• The number of copies of a particular chromosome is determined using DNA analysis

• Relies on the analysis of non-coding regions of DNA which show a wide variation in size
between different individuals

• These regions are amplified by PCR and separated on a gel according to size

• If three copies are present of different sizes, this is easily detectable bi differences in migration
on the gel

• If two copies are present of identical size, this will also be detectable because the total
quantity of DNA can also be determined

• QF-PCR requires semi-automated equipment which may not be available in many


cytogenetics labs and the system has to be validated in every centre. However, once
established, costs compare favourably with FISH

• Maternal cell contamination and mosaicism can be detected


• Failure of DNA amplification occurs in about 0.1% of samples

• Will not detect abnormalities in other chromosomes (apart from those analysed) or structural
anomalies

• QF-PCR is not reliable when used to analyse a single cell and FISH is the preferred method
for pre-implantation genetic diagnosis

Comparative Genomic Hybridisation

• This technique is not currently in clinical use

• DNA from sample is labelled with one flurochrome (say red) and compared with a normal
reference DNA labelled green

• The two DNA samples are added to a slide containing multiple metaphases from a normal
male for hybridisation

• Computer aided analysis is used to determine the red : green ratio for every chromosome and
therefore determine the copy number

• This allows analysis of all chromosomes at once, unlike FISH or QF-PCR

• The procedure is however, labour intensive

• Recent development of genomic microarrays (cloned DNA segments from different


chromosomes are fixed onto a slide and used instead of metaphase chromosomes) may
simplify the procedure and improve its clinical usefulness
DIZYGOTIC TWINS *****

· Two oocytes, each fertilised by a single sperm

· Share as many genetic / physical features as brothers / sisters

· Always dichorionic and diamniotic

· Increased incidence in Afro-carribeans, older women, positive family history


and after use of fertility drugs / IVF

MONOZYGOTIC TWINS *****

· Result from the fertilisation of one oocyte by one sperm, the resulting zygote
splitting into two during the early stages of development

· Incidence ~1:300 - constant worldwide

· Are genetically identical and therefore of the same sex

· 70% are monochorionic and diamniotic and 30% are dichrionic and
diamniotic. Monochorionic monoamniotic twins are rare

· Twin pregnancies are associated with an increased risk of congenital


anomalies (monozygotic twins in particular) but there is no increased risk of
aneuploidy
Genetics :. Basic principles :. Module 1: 60 Random questions for MCQ Test
Question 1: The following are recognised problems when the polymerase chain reaction is used
for pre-implantation genetic diagnosis

a. Allele drop-out

True False
b. Contamination by sperm DNA

True False
c. Contamination by DNA from laboratory personnel

True False
d. Confined placental mosaicism

True False

Question 2: During the cell cycle

a. DNA replication occurs during the G1 phase

True False
b. The G2 phase occurs before the M phase

True False
c. Interphase includes the G1 and G2 phases but not the S phase

True False
d. The nucleus in the G2 phase contains twice the diploid amount of DNA

True False

Question 3: The following conditions are inherited as described

a. Congenital adrenal hyperplasia ? autosomal recessive

True False
b. Hereditary spherocytosis ? X-linked recessive

True False
c. Achondroplasia ? autosomal dominant

True False
d. Alport syndrome ? X-linked recessive

True False

Question 4: During karyotyping

a. Chromosomes can be stained with quinacrine and examined under ultra-violet light

True False
b. In G-banding, the dark bands contain mainly active genes

True False
c. The banding pattern of each chromosome is characteristic

True False
d. G-banding is a sensitive method of detecting individual missing genes

True False

Question 5: During karyotyping


a. G-banding is a sensitive method of detecting polymorphisms

True False
b. G-banding can detect point mutations

True False
c. G-banding can detect chromosomal translocations

True False
d. G-banding is used to demonstrate highly repetitive micro-satellite repeat DNA at the
centromeres

True False

Question 6: Dizygotic twins

a. May be identical

True False
b. May be monochorionic

True False
c. May be monoamniotic

True False
d. Occur more frequently in older women

True False

Question 7: Cleavage stage embryo biopsy for pre-implantation genetic diagnosis

a. Requires drilling of the zona pellucida

True False
b. Is by aspiration of 3-4 blastomeres

True False
c. Is facilitated by using culture medium supplemented with calcium and magnesium

True False
d. Is undertaken at the 32 cell stage

True False

Question 8: With respect to Robertsonian translocations

a. Female carriers of 14q21q Robertsonian translocations have a 50% risk of having a baby with
Down syndrome

True False
b. Male carriers of 14q21q Robertsonian translocations have a 2% risk of having a baby with
Down syndrome

True False
c. Female carriers of 21q21 Robertsonian translocation have a 100% risk of a live birth with Down
syndrome

True False
d. The risk of having a baby with Down syndrome as a result of a 14q21q translocation increases
with increasing maternal age

True False

Question 9: X-chromosome inactivation (Lyonisation)


a. Results in the complete inactivation of one X chromosome

True False
b. Results in the formation of two Barr bodies in 47XXX individuals

True False
c. Occurs at 15-16 days after conception

True False
d. May result in a female being affected by an X-linked recessive disorder

True False

Question 10: The following can be used for pre-implantation genetic diagnosis

a. Polar body from the oocyte

True False
b. Polar body from the zygote

True False
c. Blastomeres from cleavage stage embryos

True False
d. Trophectoderm cells from the blastocyst

True False

Question 11: Robertsonian translocations

a. Involve acrocentric chromosomes

True False
b. Result in a reduction of the total number of chromosomes to 45

True False
c. Involving chromosomes 13 and 14 are most common

True False
d. Do not increase the risk of miscarriage in carriers of balanced Robertsonian translocations

True False

Question 12: The following conditions are inherited as described

a. Alpert syndrome ? autosomal recessive

True False
b. Cystic fibrosis - autosomal recessive

True False
c. Cystinuria ? autosomal dominant

True False
d. Androgen insensitivity syndrome ? autosomal recessive

True False

Question 13: During karyotyping

a. Peripheral blood polymorphs are used

True False
b. Samples must be delivered to the lab within 12h for successful karyotype
True False
c. Cell division is arrested during prophase by the addition of colchicines

True False
d. Chromosomes are characteristically stained with haematoxylline and eosine

True False

Question 14: Chromosome heteromorphism

a. Refers to the presence of variation in the DNA content of a particular chromosome in different
individuals

True False
b. Is caused by loss of small amounts of DNA during an individual?s life

True False
c. The X chromosome shows the greatest variation

True False
d. The Y chromosome shows the least variation

True False

Question 15: X-chromosome inactivation (Lyonisation)

a. Results in the preferential inactivation of the paternally derived X-chromosome

True False
b. Results in the formation of the Barr body

True False
c. In famale carriers of a balanced X-autosome translocation, results in the preferential
inactivation of the abnormal X chromosome

True False
d. Results in the formation of two Barr bodies in 47XXY individuals

True False
HYPERCHOLESTEROLAEMIA (FAMILIAL) *****

• Autosomal dominant

• Onset in 30s - 40s with xanthelasma, xanthomata and corneal arcus and ischaemic heart disease

• Population frequency 1:500

MALIGNANT HYPERPYREXIA *****

• Autosomal dominant

• Asymptomatic until exposed to succinylcholine or halothane during general anaesthesia,


producing muscle rigidity, rise in body temperature and high serum creatinine

• 60% mortality per spisode

MARFAN SYNDROME *****

• Autosomal dominant

• 25% caused by new mutation

• Associated with lens subluxation, aortic aneurysm, arachnodactyly, long limbs and joint laxity

• Average life-span 40-50 years


MUSCULAR DYSTROPHY

Facioscapulohumeral

• Autosomal dominant

• Associated with moderate - severe disability - 20% wheelchair bound by age 40

• Proximal muscle waskness, peri-orbital and peri-oral muscle weakness

Myotonic Dystrophy *****

• Autosomal dominant

• Unstable length mutation in the number of CTG repeats

• Correlation between the length of the repeats and severity of the disease

• The length of the repeat may change during transmission, altering the severity of the disease from
one generation to the next

• Neonatal disease does not occur if the father has the disease. With maternal disease, there is a
20% risk of neonatal death or severe neonatal hypotonia and mental handicap

• Clinical features include frontal balding, distal muscle and sterno-mastoid weakness, gonadal
atrophy in males, difficulty relaxing a clenched fist, cardiac conduction defects and retinopathy
• Pre-natal diagnosis by direct DNA analysis

OSTEOGENESIS IMPERFECTA

• Genetic defect in collagen metabolism

• Type I - autosomal dominant, associated with recurrent fractures, blue sclera, otosclerosis leading
to conduction deafness. Normal life span

• Type II - variable inheritance with a 3% recurrence risk - causes perinatal death with muntiple
fractures at birth

OTOSCLEROSIS

• Autosomal dominant with variable penetrance

• Progressive hearing loss in middle age - treated by surgery. Commoner in Caucasians

Von WILLEBRAND DISEASE *****

• Mostly autosomal dominant although some variants may be autosomal recessive

• Qualitative or quantitative defect in von Willebrand factor (vWF)

• vWF promotes platelet adhesion to sub-endothelium and binds and stalilises factor VIII

• Factor VIII activity may be reduced


• Prolonged bleeding time and reduced platelet aggregation with ristocetin

• Infection, stress, inflammation, physical exercise and recent surgery and pregnancy cause raised
vWF and factor VIII levels
You will be expected to know

• The mode of inheritance of the different conditions

• The characteristics of the pedigree

• Method of pre-natal diagnosis if available

• Very basic features of the disorder, for instance, age at presentation

• You will be required to know details of conditions which have effects on obstetric
or gynaecological care, for instance, inherited bleeding disorders.
Genetics :. Autosomal dominant conditions :. Module 2: 60 Random questions for MCQ Test
Question 1: Acute intermittent porphyria

a. Is an autosomal recessive trait

True False
b. Can be precipitated by oestrogen administration

True False
c. Can be precipitated by sulphonamide administration

True False
d. Pre-natal diagnosis is possible by amniocentesis

True False

Question 2: Achondroplasia

a. Is inherited as an autosomal dominant condition with variable penetrance

True False
b. Is associated with a point mutation in the gene for fibroblast growth factor receptor 3

True False
c. Is associated with a normal IQ and life span

True False
d. Is associated with an increased risk of spinal cord compression

True False

Question 3: Alpert syndrome

a. Is an autosomal recessive condition

True False
b. Is associated with nephritis

True False
c. Is associated with chronic renal failure

True False
d. Is associated with a normal IQ

True False

Question 4: With respect to Epidermolysis bullosa

a. The disorder is inherited as an X-linked recessive condition

True False
b. Pre-natal diagnosis by amniocentesis is possible

True False
c. Pre-natal diagnosis by CVS is possible

True False
d. Fetal skin biopsy is required for pre-natal diagnosis

True False

Question 5: The following are characteristic of osteogenesis Imperfecta

a. Type I ? autosomal recessive inheritance


True False
b. Type I ? normal life span

True False
c. Type I ? recurrent fractures, blue sclera and conductive deafness

True False
d. Type II ? associated with perinatal death

True False

Question 6: Marfan syndrome

a. Is an autosomal recessive disorder

True False
b. Carries a normal life span

True False
c. Is associated with aortic aneurysms

True False
d. Is caused by a new mutation in 1% of cases

True False

Question 7: Acute intermittent porphyria

a. Can be precipitated by infection

True False
b. Patients typically improve following alcohol consumption

True False
c. Is caused by a mutation in the cytochrome P450 gene

True False
d. Family members can be screened by biochemical analysis

True False

Question 8: Von Willebrand disease

a. Is inherited as an autosomal dominant condition

True False
b. Is associated with increased platelet aggregation

True False
c. Is ssociated with increased platelet aggregation with ristocetin

True False
d. Causes prolonged bleeding time

True False

Question 9: Hereditary spherocytosis

a. Is associated with reduced mean cell haemoglobin concentration

True False
b. Is associated with a normal life span

True False
c. Is particularly common in people of African origin

True False
d. Is associated with splenomegaly

True False

Question 10: With respect to Von Willebrand disease

a. Fetal sexing by ultrasound is useful in prenatal diagnosis

True False
b. There is a quantitative or a qualitative defect in von Willebrand factor

True False
c. Factor VIII concentration may be reduced

True False
d. Surgery is associated with increased von Willebrand factor concentration

True False

Question 11: The following are characteristic features of autosomal dominant inheritance

a. Males and females affected in equal numbers

True False
b. Unaffected individuals cannot transmit the condition

True False
c. Males can transmit the condition to their sons

True False
d. If the mother is affected and the father is unaffected, there is a 1 in 4 chance of the off-spring
being affected

True False

Question 12: Hereditary spherocytosis

a. Is an X-linked recessive condition

True False
b. Is associated with a conjugated hyperbilirubinaemia

True False
c. Is associated with increased osmotic fragility of red cells

True False
d. Can be treated by spleenectomy

True False

Question 13: The following are characteristic features of autosomal dominant inheritance

a. The disease is expressed only in homozygous individuals

True False
b. There is a vertical pedigree pattern

True False
c. There is a maternal age effect on new mutations

True False
d. Variable expressivity within a family
True False

Question 14: Huntington disease

a. Is an autosomal dominant condition

True False
b. Typically presents between the ages of 30-50 years

True False
c. Is associated with psychiatric manifestations

True False
d. Pre-natal diagnosis is not possible

True False

Question 15: The following are characteristic features of myotonic dystrophy

a. Autosomal dominant inheritance

True False
b. Frontal balding and proximal muscle weakness

True False
c. Difficulty opening a clenched fist

True False
d. Gonadal atrophy in males

True False
ALPHA-1 ANTI-TRYPSIN DEFICIENCY *****

• Autosomal recessive
• Carrier frequency 3% in Caucasians
• Z-allele is a point mutation which interferes with the
release of anti-trypsin from hepatocytes
• Diagnosis made by protease inhibitor activity assay or
DNA analysis (CVS or amnio)
• Associated with juvenile cirrhosis, emphysema

CONGENITAL ADRENAL HYPERPLASIA *****

• Autosomal recessive
• 95% due to 21-hydroxylase deficiency
• Incidence 1:17,000 (UK), 1:500 Yupik Eskimos

Clinical *****

• Neonatal vomiting and shock from salt-losing


• Ambiguous genitalia virilisation of female fetus
• Precocious puberty in male
• Primary / secondary amenorrhoea with hirsutism and
virilisation in late-onset type
• Elevated urinary ketosteroids and pregnanetriol
• Elevated plasma 17-hydroxyprogesterone and ACTH
• Normal life-span and fertility if promptly identified and
treated with steroid replacement

Pre-Natal Diagnosis *****

• 17-hydroxyprogesterone concentration in amniotic fluid


or DNA analysis from CVS or amniocentesis
• Maternal administration of dexamethasone to prevent
virilisation of affected female fetus

CYSTINURIA

• Autosomal recessive
• Defect in gene for transporter of cationic and neutral
amino acids (cystine, lysine, arginine and ornithine)
• Associated with recurrent urinary calculi. IQ is unaffected

FREIDREICH’S ATAXIA

• Autosomal recessive
• Commonest inherited cause of cerebellar ataxia
• Ataxia, pes cavus, loss of deep tendon reflexes and
extensor plantar response
• Wheelchair bound by age 45
• Pre-natal diagnosis possible by DNA analysis (CVS or
amniocentesis)

GALACTOSAEMIA *****

• Autosomal recessive
• Mutation in galactose-1-phosphate uridyl transferase
gene

Clinical Features

• Neonatal vomiting, failure to thrive, jaundice,


hepatomegaly, cataract
• Galactosuria
• Absent red cell galactose-1-phosphate uridyl transferase
activity
• Treated by exclusion of milk products from diet for life.
Delay in treatment (over 1month of age) results in learning
disability
• Prenatal diagnosis by DNA analysis (CVS or
amniocentesis), GLAT activity in chorionic villi or
amniocytes or amniotic fluid galacitol levels

GAUCHER DISEASE *****

• Autosomal recessive
• Mutation in beta-galactosidase gene
• Type I (chronic adult type) common in Ashkenazi Jews
with carrier frequency of 1:10 and presents with bone pain
and splenomegaly
• Infantile form presents with progressive neurological
deterioration and hepato-splenomegaly
• Prenatal diagnosis possible by DNA analysis (CVS or
amniocentesis) or beta-galactosidase assay

HAEMOCHROMATOSIS *****

• Autosomal recessive
• Increased serum iron, ferritin, transferrin saturation and
increased iron in liver biopsy
• Long term complications include diabetes mellitus,
cardiomyopathy, arthropathy and cirrhosis
• Repeated venesection reduces iron overload
MUCOPOLYSACCHARIDOSES *****

• There are a total of 7 types


• All are autosomal recessive except type II (Hunter syndrome) which
is X-linked recessive. Type I = Hurler syndrome. *
• Type IV is associated with normal intelligence.
• Pre-natal diagnosis possible by biochemical analysis of
glycosaminoglycans in amniotic fluid or by enzyme assay in cultured
amniocytes or chorionic villi

PHENYLKETONURIA *****

• Autosomal recessive
• Deficiency of phenylalanine hydroxylase
• Promptly treated with a low phenylalanine diet is associated with
normal life span although some degree of learning disability may be
present
• After childhood (6-8years), the brain is less susceptible to the harmful
effects of high phenylalanine levels
• Risk of fetal structural abnormalities including microcephaly if
treatment is not re-introduced in an affected woman before conception
• Causes high blood and urine phenylalanine levels
• Neonatal screening using the Guthrie test

SICKLE CELL DISEASE *****

• Autosomal recessive
• Point mutation in beta-globin gene resulting in substitution of valine
for glutamic acid at position 6
• Fetal haemoglobin does not have a beta chain therefore fetal /
neonatal anaemia do not occur
• If both parents are carriers, 25% of off-springs will be normal, 25%
will have the disease and 50% will be carriers
• Severe chronic haemolytic anaemia with recurrent episodes of
infarction in spleen, lungs and bone. Increased susceptibility to
pneumococcal and salmonella infections

SUCCINYLCHOLINE SENSITIVITY *****

• Autosomal recessive
• Mutation in cholinesterase 1 gene resulting in cholinesterase
deficiency
• Affected patients are asymptomatic until exposed to suxamethonium
during induction of general anaesthesia

TAY-SACHS DISEASE *****

• Autosomal recessive
• Mutation in beta-N-acetylhexosaminidase A gene
• Highest frequency in Ashkenazi Jews with carrier frequency of 1:30
• 90% have cherry-red macular spot
• Pre-natal diagnosis by direct DNA analysis (CVS or amniocentesis)
or by assay of beta-N-acetylhexosaminidase A activity in chorionic villi or
amniocytes
• Death usually by the age of 3-4 years.
You will be expected to know

• The mode of inheritance of the different conditions

• The characteristics of the pedigree

• Method of pre-natal diagnosis if available

• Very basic features of the disorder, for instance, age at presentation

• You will be required to know details of conditions which have effects on


obstetric or gynaecological care, for instance, inherited bleeding disorders.
Genetics :. Autosomal recessive conditions :. Module 3: 60 Random questions for MCQ Test
Question 1: Congenital adrenal hyperplasia

a. Is most commonly due to 11-hydroxylase deficiency

True False
b. Is most common in Eskimos

True False
c. Is associated with male precocious puberty

True False
d. Maternal administration of dexamethasone will prevent virilisation of an affected female fetus

True False

Question 2: The following are typical features of galactosaemia

a. Increased red cell galactose-1-phosphate uridyl transferase activity

True False
b. Failure to thrive

True False
c. Normal IQ even in the absence of treatment

True False
d. Hepatomegaly

True False

Question 3: Cystinuria

a. Is an autosomal dominant condition

True False
b. Is associated with learning disability

True False
c. Is associated with an increased risk of gall stones

True False
d. Is associated with increased urinary excretion of cystine, lysine, arginine and ornithine

True False

Question 4: Cystic fibrosis

a. Is an autosomal recessive disorder

True False
b. Is always caused by a mutation in the delta-F508 gene

True False
c. Is associated with congenital absence of the vas in males

True False
d. Is associated with congenital absence of the fallopian tubes in females

True False

Question 5: With respect to Cystic fibrosis

a. The carrier frequency of 1:250 in Caucasians


True False
b. Prenatal diagnosis is possible by amniocentesis

True False
c. Pre-natal diagnosis is not possible by CVS

True False
d. The sodium concentration is typically < 60mM in sweat

True False

Question 6: Congenital adrenal hyperplasia

a. Is an X-linked recessive disorder

True False
b. Can be diagnosed pre-natally by chorionic villus sampling

True False
c. Can be diagnosed pre-natally by amniocentesis

True False
d. Is associated with reduced IQ

True False

Question 7: Galactosaemia

a. Is an autosomal dominant disorder

True False
b. Is associated with premature ovarian failure

True False
c. Is treated by exclusion of milk products from the diet until adulthood and during pregnancy

True False
d. Causes learning disability if treatment is instituted after 1 month of age

True False

Question 8: The following are typical features of galactosaemia

a. Glycosuria

True False
b. Galactosuria

True False
c. Cataract

True False
d. Neonatal jaundice

True False

Question 9: Sickle cell disease

a. Is an autosomal recessive disorder

True False
b. Shows an abnormal haemoglobin band on electrophoresis

True False
c. Is associated with increased levels of fetal haemoglobin in affected adults

True False
d. Causes neonatal anaemia

True False

Question 10: The following are characteristic features of Wilson disease (hepatolenticular
degeneration)

a. X-linked recessive inheritance

True False
b. Basal ganglia degeneration

True False
c. Psychiatric manifestations

True False
d. Kayser-Fleischer rings

True False

Question 11: The following are characteristic of haemochromatosis

a. Autosomal dominant inheritance

True False
b. Increased serum iron

True False
c. Decreased serum transferrin saturation

True False
d. Diabetes mellitus

True False

Question 12: Alpha-1-antitrypsin deficiency

a. Is inherited as an autosomal dominant condition

True False
b. Has a carrier frequency of 3% in Caucasians

True False
c. Pre-natal diagnosis by chorionic villus sampling is possible

True False
d. Pre-natal diagnosis cannot be performed using amniotic fluid

True False

Question 13: The following are characteristic features of autosomal recessive inheritance

a. Males and females are equally affected

True False
b. There is a vertical pedigree pattern

True False
c. Affected individuals are infertile

True False
d. All the off-spring of an affected individual will either have the condition or be carriers
True False

Question 14: The following are characteristic features of Wilson disease (hepatolenticular
degeneration)

a. Elevated serum ceruloplasmin

True False
b. Increased hepatic copper deposition

True False
c. Onset in infancy

True False
d. Treatment with D-penicillamine

True False

Question 15: The following are characteristic features of autosomal recessive inheritance

a. The offspring of a carrier and a normal homozygous partner cannot be affected

True False
b. There is an increased risk in consanguineous marriages

True False
c. There is variable expressivity within a family

True False
d. There is variable penetrance within a pedigree

True False
ALPORT SYNDROME

• X-linked recessive; occurs in 1:5000 males


• Mutation in gene encoding for glomerullar basement
membrane collagen
• Pre-natal diagnosis by DNA analysis
• Female carriers asymptomatic but have microscopic
haematuria, 30% risk of hypertension and 5-10% risk of chronic
renal failure
• Associated with nephritis, proteinuria, and sensori-neural
deafness

ANDROGEN INSENSITIVITY SYNDROME *****

• X-linked recessive *
• Mutation in androgen receptor gene
• Female phenotype, normal breast development, absent uterus
with blind-ending vagina; paucity of pubic and axillary hair *
• Testes found in abdomen or inguinal canal *
• Increased risk of gonadoblastoma *
• Infertility is universal
• Karyotype 46XY
• Risk of inguinal hernia increased *

COLOUR BLINDNESS *****

• Clolour vision dependent on the products of three genes for


blue, red and green colour vision
• Red-green colour blindness X-linked recessive. * Defects
occur in 8% of male Caucasians, less common in Africans /
Orientals

FABRY DISEASE

• X-linked recessive
• Mutation in alpha galactosidase A gene resulting in alpha
galactosidase A deficiency in plasma, leucocytes and skin
fibroblasts
• Presents in childhood in affected males with limb pains and
skin lesions
• Pre-natal diagnosis possible by fetal sexing and DNA analysis

GLUCOSE-6-PHOSPHATE DEHYDROGENASE DEFICIENCY *****

• X-linked recessive *
• Deficiency of erythrocyte glucose-6-phosphate
dehydrogenase

Clinical Features
• May be asymptomatic
• Neonatal jaundice
• Chronic haemolytic anaemia
• Haemolysis induced by eating fava beans, sulphonamides,
primaquine and infection
• Female carriers may suffer haemolytic crises

HAEMOPHILIA *****

• A - factor VIII deficiency *


• B - factor IX deficiency *
• Both are X-linked recessive and clinically indistinguishable *
• Spontaneous bleeding into joints / soft tissue or following
trauma
• Due to lyonisation, some female carriers may have low levels
to cause increased risk of bleeding after surgery *
• Treated by factor VIII or IX replacement
• Male fetus inherits Y chromosome from father and therefore
cannot be affected if the father has the disease. 50% risk of male
fetus being affected if the mother is a carrier (inherits one of 2 X
chromosomes). *

LESCH-NYHAN SYNDROME

• X-linked recessive
• Deficiency in red cell hypoxanthine-guanine phosphoribosyl
transferase
• Onset in infancy with spasticity, learning disability and chorea-
athetosis
• Pre-natal diagnosis by direct DNA analysis

MUSCULAR DYSTROPHY *****

DUCHENNE *****

• X-linked recessive
• Null mutation resulting in absence of dystrophin on muscle
biopsy
• Onset in childhood (by age of 5) with proximal muscle
weakness, calf pseudo-hypertrophy; mild learning disability in
25%, majority wheelchair bound by age 12 with death in 20s.

BECKER *****

• X-linked recessive
• Reduced dystrophin on muscle biopsy
• Onset in late childhood, calf pseudo-hypertrophy. Wheelchair
bound by age 25, life-span may be normal - better prognosis
than Duchenne
• Pre-natal diagnosis for Duchenne / Becker muscular
dystrophy by direct DNA analysis (CVS / Amnio)
• Carriers have elevated serum creatinine and can be used for
screening
• In the absence of a family history, 2/3 of mothers of boys with
Duchenne / Becker muscular dystrophy will be carriers
Genetics :. X-linked conditions :. Module 4: 60 Random questions for MCQ Test
Question 1: Androgen insensitivity syndrome

a. Is an autosomal recessive trait

True False
b. Is characterised by a female phenotype

True False
c. Is associated with secondary amenorrhoea

True False
d. Is characterised by a normal uterus and the presence of testes in the abdomen or inguinal
canal

True False

Question 2: With respect to colour blindness

a. There is reduced visual acuity

True False
b. Red-green colour blindness is an autosomal recessive disorder

True False
c. Red-green colour defects affects 0.1% of male Caucasians

True False
d. Red-green colour blindness is more common in Africans

True False

Question 3: With respect to the muscular dystrophies

a. Duchenne muscular dystrophy is an X-linked recessive disorder

True False
b. Becker muscular dystrophy is an autosomal recessive disorder

True False
c. Duchenne muscular dystrophy is associated with learning disability

True False
d. Becker muscular dystrophy carries a worse prognosis than Duchenne muscular dystrophy

True False

Question 4: The following are characteristic features of haemophilia

a. Haemophilia A - factor VIII deficiency

True False
b. Haemophilia B - factor XI deficiency

True False
c. Haemophilia A - X-linked recessive

True False
d. Haemophilia B - autosomal recessive

True False

Question 5: The following are characteristic features of X-linked recessive inheritance


a. Females affected more often than males

True False
b. Vertical pedigree pattern

True False
c. Affected males transmit the trait to their daughters

True False
d. Affected males transmit the trait to their sons

True False

Question 6: With respect to the muscular dystrophies

a. Dystrophin can be detected on muscle biopsy in Duchenne muscular dystrophy

True False
b. Dystrophin is not detectable on muscle biopsy in Becker muscular dystrophy

True False
c. Pre-natal diagnosis is possible in Duchenne and Becker muscular dystrophy by direct DNA
analysis of chorionic villi

True False
d. If there is no family history of muscular dystrophy, 10% of mothers of boys with the disease will
be carriers

True False

Question 7: Androgen insensitivity syndrome

a. Is associated with an increased risk of gonadoblastoma

True False
b. There is normal pubic and axillary hair development

True False
c. Infertility is universal

True False
d. Is associated with an increased risk of inguinal hernia

True False

Question 8: With respect to the muscular dystrophies

a. Facioscapulohumeral muscular dystrophy is an X-linked recessive disorder

True False
b. Carriers of muscular dystrophy have normal serum creatinine concentrations

True False
c. 10% of children with Duchenne muscular dystrophy will be wheelchair bound by the age of 12

True False
d. Duchenne muscular dystrophy typically presents with distal muscle weakness

True False

Question 9: The following features are characteristic of glucose-6-phosphate dehydrogenase


deficiency

a. X-linked recessive inheritance


True False
b. Haemolysis induced by eating fava beans

True False
c. Haemolysis induced by sulphonamides

True False
d. Haemolysis induced by infection

True False

Question 10: With respect to Alport syndrome

a. Pre-natal diagnosis can be performed using chorionic villus sampling

True False
b. Pre-natal diagnosis can be performed by amniocentesis

True False
c. There is a 5-10% risk of chronic renal failure in female carriers

True False
d. There is a 1% risk of chronic renal failure in males

True False

Question 11: The following are characteristic features of haemophilia

a. Haemophilia B - clinically more severe disease than haemophilia A

True False
b. Haemophilia A - prenatal diagnosis by CVS is contraindicated

True False
c. Haemophilia B - 50% risk of male fetus being affected if the mother is a carrier

True False
d. Haemophilia A - 50% risk of male fetus being affected if the father has the disease

True False

Question 12: Alport syndrome

a. Affects males only

True False
b. Is an autosomal dominant condition

True False
c. Is associated with nephritis and proteinuria

True False
d. Is associated with sensori-neural deafness

True False

Question 13: A female may be affected by Duchenne muscular dystrophy under the following
circumstances

a. Atypical lyonisation

True False
b. Her father is affected and her mother is a carrier
True False
c. Her father has the disease and there is a new mutation in the maternally derived X
chromosome

True False
d. She carrier an X-autosome translocation

True False

Question 14: The following are characteristic features of X-linked recessive inheritance

a. There is variable penetrance within a pedigree

True False
b. Knight?s move pedigree pattern

True False
c. A woman with a affected child is an obligate carrier

True False
d. A woman with an affected son and an affected brother is an obligate carrier

True False

Question 15: With respect to X-linked dominant conditions

a. Females are more severely affected than males

True False
b. Females are affected less often than males

True False
c. All daughters of an affected father are affected

True False
d. All sons of an affected woman will be affected

True False
PRADER-WILLI SYNDROME


Characterised by 15q deletion in paternally derived
chromosome

The majority of deletions occur de-novo but 2% are associated
with parental structural chromosomal anomaly

In 25% of cases, there is no deletion but maternal
uniparental disomy of chromosome 15 with consequent lack of
critical paternally derived genes

CLINICAL FEATURES

Neonatal hypotonia with poor suckling


Flat face
Hypoplastic external genitalia
Almond-shaped palpebral fissures
Small hands and feet
Learning disability
Over-eating in childhood with obesity

ANGELMAN SYNDROME

Characterised by 15q deletion in maternally derived


chromosome ; 5% associated with paternal uniparental disomy

CLINICAL FEATURES

Developmental delay. Poor speech


Jerky movements
Paroxysms of inappropriate laughter
Abnormal EEG

FRAGILE X SYNDROME *****

Fragile site at Xq27.3 under culture conditions with thymidine


or deoxycytidine deprivation
Associated with increased length of the CGG trinucleotide
repeat at this locus
The length of the CGG repeat is variable (median 30, range
6-52). The premutation is associated with 60-200 repeats while
in the full mutation, there are over 230 repeats
Males / females with the premutation are normal. The
length of the premutation is however unstable and expansion at
meiosis to the full mutation is possible (risk of expansion
increases with increasing length of the premutation)
Females with the full mutation may be normal or have mild
or moderate learning disability. Prediction of degree of mental
impairment is not possible pre-natally
Males with the full mutation have learning disability with
enlarged testes after puberty (50%)
Pre-natal diagnosis by DNA analysis (CVS or
amniocentesis)
DOWNS SYNDROME *****

Trisomy 21
95% due to non-dysjunction in the FIRST meiotic division;
1-2% mosaics, 3% unbalanced Robertsonian translocation
Extra chromosome derived from the mother in 80-85% of
cases
Associated with increasing maternal age
Associated with low AFP, low unconjugated oestriol and
increased HCG in maternal serum and increased nuchal
thickening

CLINICAL FEATURES *****

1) GENERAL

Neonatal hypotonia
Learning disability
Short stature
Increased risk of Alzheimer's, hypothyroidism, acute
leukaemia, epilepsy and atlanto-axial instability

2) CRANIO-FACIAL

Brushfield spots
Brachycephaly
Epicanthic folds
Protruding tongue
Upward sloping palpebral fissures
Strabismus
Nystagmus; congenital cataract

3) LIMBS

Single palmar crease


Fifth finger clinodactyly

TYPICAL CONGENITAL ANOMALIES *****

CARDIAC - atrio-ventricular canal defects


Duodenal atresia, also anal atresia

RECURRENCE RISK *****

No translocation - age related risk + 0.34% at term (0.42%


mid-trimester)
Mother carrier of t(14;21) - 15% recurrence risk
Father carrier of t(14;21) - 1% recurrence risk
Mother or father carrier of t(21;21) - 100% recurrence risk
KLINEFELTER SYNDROME *****

• 47,XXY
• Incidence 1:1000 males; risk increases with increased
maternal age
• Extra X chromosome is maternal in 50-60% of cases;
recurrence risk close to population risk

Clinical Features *****

• Diagnosis typically made following infertility investigation -


commonest cause of hypogonadism and infertility in males

• Poorly developed secondary sexual characteristics, small


testicles, gynaecomastia, increased height (elongated limbs with)

• Low testosterone levels

• Gynaecomastia with increased risk of breast cancer

• Associated with increased risk of diabetes mellitus,


osteoporosis, scoliosis and emphysema

• IQ 10-20 points lower than siblings

• Infertility is universal except in mosaics

• Testosterone replacement at puberty / adulthood to promote


development of secondary sexual characteristics

TURNER SYNDROME *****

• 45X, incidence 1:5000 female births; 99% of fetuses with 45X


spontaneously miscarry

• 50% have 45X, 25% are mosaics, 17% have an


isochromosome of Xq. Other abnormalities include ring X
chromosome and Xp deletions. 4% have 45X/46XY mosaicism
which is associated with an increased risk of gonadoblastoma in
dysgenetic gonads

• In 80% of cases, only maternal X is present

• Recurrence risk close to population risk

Clinical Features *****

• Short stature with broad chest and widely spaced nipples; low
hair line with webbed neck
• Wide carrying angle and short 4th metacarpals and hypoplasia
of the nails

• Peripheral lymphoedema

• Coarctation of the aorta and also atrial septal defects

• Primary amenorrhoea although menstruation may occur


especially in mosaics; absence of adolescent growth spurt and
poorly developed secondary sexual characteristics

• Increased risk of systemic hypertension, Hashimoto’s


thyroiditis, Crohn’s disease, GI bleeds

• Intelligence and life-span are normal


TRISOMY 13 *****

• Incidence 1:5,000 live births


• Maternal age association
• Only 10% survive the first year
• Recurrence risk 100% in a t(13;13) carrier
• Typical features include holoproencephaly, cleft lip and palate,
post-axial polydactyly, scalp defects

TRISOMY 18 (EDWARDS SYNDROME) *****

• Incidence 1:3000 live births; 1:2 male:female ratio; maternal


age effect

• 95% due to maternal non-disjunction

• Associated with low maternal serum AFP, HCG, uE3

• 10% survive to 1 year of age

Clinical Features

• IUGR

• Cranio-facial: prominent occiput, low set ears, small mouth


and jaw

• Limbs: clenched fist with overlapping index and little finger,


rocker-bottom feet

• Cardiac, GI and neural tube defects

TRIPLOIDY *****

• 69 chromosomes, typically 69XXY or 69XXX. Partial molar


pregnancy *

• Extra chromosomes paternally derived in most cases (double


fertilisation; diploid sperm) *

• Occurs in 2% of conceptions, most of which miscarry


spontaneously *

• Associated with a large placenta, IUGR with a small head but


a much smaller trunk and hydropic placental villi *
Genetics :. Chromosomal anomalies :. Module 5: 60 Random questions for MCQ Test
Question 1: Turner syndrome

a. Is associated with short stature

True False
b. Is typically associated with secondary amenorrhoea

True False
c. Has a 45XX karyotype

True False
d. Is associated with widely spaced nipples

True False

Question 2: With respect to Down syndrome

a. 95% of cases are due to non-dysjunction during the second meiotic division

True False
b. The extra chromosome is maternally derived in 99% of cases

True False
c. Mosaicism accounts for 5% of cases

True False
d. The recurrence risk in a 20 year old with one baby with Down syndrome is 5%

True False

Question 3: The following malformations are typically associated with Down syndrome

a. Atrio-ventricular canal defects

True False
b. Coarctation of the aorta

True False
c. Duodenal atresia

True False
d. Neural tube defect

True False

Question 4: Klinefelter syndrome

a. Is associated with a 46,XXY karyotype

True False
b. Is associated with anosmia

True False
c. Testosterone levels are within the normal adult male range

True False
d. Is associated with small testicles

True False

Question 5: The following are characteristic of Angelman syndrome

a. Abnormal ECG
True False
b. Poor speech

True False
c. Inappropriate paroxysms of laughter

True False
d. Obesity

True False

Question 6: With respect to Klinefelter syndrome

a. Testosterone replacement in infancy is recognised treatment

True False
b. 1% are mosaics

True False
c. The extra X chromosome is always maternal in origin

True False
d. There is a 1% recurrence risk

True False

Question 7: Trisomy 18

a. Occurs in 1:30,000 live births

True False
b. Is commoner by males at birth

True False
c. Is associated with low maternal serum AFP

True False
d. Is associated with low maternal serum HCG

True False

Question 8: The following are characteristic of Angelman syndrome

a. 15q deletion in paternally derived chromosome

True False
b. Paternal uniparental disomy of chromosome 15

True False
c. Normal IQ

True False
d. Neonatal hypotonia

True False

Question 9: Down syndrome

a. Is associated with increased maternal serum AFP

True False
b. Is associated with increased maternal serum HCG

True False
c. Is associated with increased maternal serum unconjugated oestriol

True False
d. Is associated with decreased nuchal thickening

True False

Question 10: With respect to the fragile X syndrome

a. Affected males have enlarged testes at birth

True False
b. The fragile site is induced by thymidine deprivation in culture

True False
c. The fragile site is induced by deoxycytidine deprivation in culture

True False
d. There is a reduction in the length of the CGG trinucleotide repeat at Xq27.3

True False

Question 11: The following are characteristic of Prader-Willi syndrome

a. Failure to thrive

True False
b. Learning disability

True False
c. Triploidy

True False
d. Deletion in paternally derived chromosome 15q

True False

Question 12: The following cranio-facial abnormalities are typical of Down syndrome

a. Brachycephaly

True False
b. Epicanthic folds

True False
c. Protruding tongue

True False
d. Small low set ears

True False

Question 13: Triploidy

a. Is associated with a 47XXX karyotype

True False
b. Occurs in 2% of conceptions

True False
c. Is associated with a large placenta

True False
d. The extra chromosomes are maternally derived in 95% of cases
True False

Question 14: Trisomy 18

a. Is associated with low maternal unconjugated oestriol

True False
b. Is associated with a 10% survival to 1 year of age

True False
c. 95% is associated with paternal non-disjunction

True False
d. Is typically associated with atrio-ventricular canal defects

True False

Question 15: With respect to Turner syndrome

a. Development of secondary sexual characteristics is normal

True False
b. With 45X karyotype is associated with an increased risk of ovarian cancer in the streak ovaries

True False
c. Life span is normal

True False
d. The uterus is typically absent

True False
1

1 The following disorders have an X-linked pattern of inheritance: √


A Glucose-6-phosphate dehydrogenase deficiency. T
B Kleinfelter syndrome. F
C Adrenogenital syndrome. F
D Heamophilia B. T
E Familial hypercholesterolaemia. F
Comment:
Adrenogenital syndrome results from autosomal recessive enzymatic
defects.
Familial hypercholesterolaemia is autosomal dominant.
Klinefelters syndrome is a chromosomal abnormality XYY.
Comments 2:
The disorders associated with X linked inheritance include both
Haemophilias, A and B, G6PD deficiency, Duchenne's dystrophy and
vitamin D resistant rickets. FH is autosomal recessive as is
adrenogenital syndrome. Klinefelter's is a chromosomal abnormality
XYY.
2 The following are X-linked disorders: √
A Myotonic dystrophy. F
B Duchenne muscular dystrophy. T
C Haemophilia A. T
D Huntington's disease. F
E Fragile X syndrome. T
Comment:
X-linked recessive disorders are: haemophilia, chrismas disease,
Duchennes muscular dystrophy, G-6-Ph dehydrogenase deficiency,
blue\green colour blindness, Wisckott-aldrich syndrome and Hunter
syndrome.
X-linked dominant are: rickets, incontinentia pigmenti and Xg blood
group.
Due to the novel nature of the fragile X mutation, inheritance is less
straightforward than in classic Mendelian traits, but the boy acquires
the FMR gen from his mothers (carrier) X chromosome.
Comments 2:
X-linked disorders include Haemophilias, Duchenne Muscular
dystrophy (recessive) and vitamin D resistant rickets (dominant).
Myotonic dystrophy and Huntingdon's are autosomal dominant. Due to
the novel nature of the fragile X mutation, inheritance is less
2

straightforward than in classic Mendelian traits, but the boy acquires


the FMR gene from his mother's (carrier) X chromosome.
3 The following genetic disorders are inherited as autosomal √
recessives:
A Duchenne muscular dystrophy. F
B Huntingdon's chorea. F
C Tay-sachs disease. T
D Retinoblastoma. F
E Achondroplasia. F

4 The following are autosomal recessive diseases: √


A Neurofibromatosis. F
B Cystic fibrosis. T
C Phenylketonuria. T
D Polyposis coli. F
E Sickle cell anaemia. T

5 The following are inherited as autosomal recessive conditions:


A Tuberous sclerosis. F
B Phenylketonuria. T
C Achondroplasia. F
D Sickle cell anemia. T
E Von Gierke's disease. T

6 Genes on sex chromosome are responsible for inheritance of: √


A Glucose-6-phosphate dehydrogenase deficiency. T
B Defective colour vision. T
C Hairy ear rims. T
D Homocystinuria. F
E Hurler syndrome. F

7 The following genes and chromosomes are correctly paired: √


A HLA: chromosome 6. T
B Clotting factor VIII: X-chromosome. T
C Glucose-6-phosphate dehydrogenase: X-chromosome. T
D Testis determining factor: X-chromosome. F
E Xg blood group: chromosome 1. F
Comment:
Testis determining factor and Y chromosome.
Xg blood group and X chromosome.
8 The following statements relate to familial diseases: √
A Achondroplasia is a dominant trait. T
B Babies with down syndrome usually have 46 chromosome. F
3

C Congenital pyloric stenosis is commoner in babies of affected parents T


than in general population.
D All the daughters of a female carrier of red green colour blindness are F
themselves carriers.
E Heamophilia occurs in all the sons of an affected father. F

9 In an X-linked pedigree: √
A None of the sons of an affected male will be affected. T
B Half of the daughters of an affected male will not carry the gene. F
C Half of the sons of carrier females will be affected. T
D Females are never affected. F
E All the daughters of a carrier female will themselves be carriers. F

10 When a man has haemophilia: √


A 50% of his daughters would not expect to be carriers. F
B 25% of his sons would be expected to be affected. F
C Good medical control of his blood deficiency reduces the risk of the F
condition in his children.
D His newborn child is likely to require an urgent blood transfusion. F
E His sister has a 50% probability of being a carrier. T

11 Concerning inheritable diseases: √


A Huntingdon's chorea is transmitted by a dominant gene. T
B Phenylketonuria is transmitted by a recessive gene. T
C Haemophilia is an autosomal dominant condition. F
D Von Willenbrand's disease is a sex –linked condition. F
E Cystic fibrosis is transmitted by an x-linked receive gene. F

12 The following conditions are hereditary: √


A Polyposis coli. T
B Retinoblastoma. T
C Xeroderma pigmentosa. T
D Burkitts lymphoma. T
E Osteosarcoma. T

13 Concerning chromosomal errors: √


A Structural variations may not have a phenotypic consequence. T
B within a Trisomy indicates the presence of three haploid components T
cell.
C Nondisjunction during mitosis can contribute to mosaicism. T
D A trisomic parent can produce normal gametes. T
E Triploidy is rarely detected in neonates at term. T
Comment:
4

Mosaics characterized by a 45X\46XX karyotype 15% tend to have


milder phenotypic manifestations of turner syndrome and may be even
fertile, in 5% of patients; the mosaic karyotype is 45X\46XY in which
case an original male zygote was subsequently modified by a mitotic
nondisjunction.
14 The following are structural aberrations of chromosomes:
A Deletions. T
B Inversions. T
C Aneuploidy. F
D Polyploidy. F
E Translocations. T

15 The following are structural abnormalities of chromosomes:


A Deletions. T
B Duplications. T
C Aneuploidy. F
D Polyploidy. F
E Translocations. T
Comment:
Structural abnormalities occur when breaks occur in chromosomes.
Include: deletion, duplication, inversion, translocation and
isochromosome formation.
16 In diseases determined by dominant genes: √
A A sibling of an affected person will always be affected. F
B Homozygous affected parents will always produce affected offspring. T
C The parents of affected persons have an increased rate of F
consanguineous marriage.
D Normal children of affected parents have normal offspring. T
E New cases can arise by spontaneous mutation. F
Comment:
Consanguinity increases the risk of all kinds of genetically determined
disease. In autsomal dominant condition there is no skipping a
generation, the disease has a clear pedigree.
An equal ratio of affected males and females.
The condition may only become apparent after 30 years of age.
17 Chromosome analysis: √
A Can be performed more quickly from chorionic villous samples than T
from amniotic fluid samples.
B Is based entirely on assessment of chromosome size. F
C Is carried out at the metaphase stage of mitosis. T
D Of a classical hydatidiform mole is usually 46xx. F
E Of the fetus will be abnormal in about 80% of cases of exomphalos. F
Comment:
5

The fetus is abnormal in about 50% of exomphalos cases.


18 Chromosomes: √
A Are found in the same number in all mammalian cells. F
B Can be analyzed more quickly from a blood sample than from an T
amniotic fluid sample.
C Can be reliably identified by their lengths. F
D The Y chromosome is larger than the X chromosome. F
E DNA content is doubled during the S (synthesis) phase of the cell T
cycle.
Comment:
Leucocytes analysis takes 2-3 days, whereas amniotic fluid takes 2-3
weeks.
Y chromosome is smaller than X chromosome.
Chromosomes can be identified by banding.
19 In the female: √
A Only the X chromosome of maternal origin is active. F
B The Barr body is sex chromatin. T
C About 80% of polymorphonuclear leucocytes have a drum stick of F
chromatin.
D An extra X chromosome is associated with two Barr bodies. T
E An extra X chromosome is associated with below average intelligence. T

Comment:
One of the X chromosomes in the female undergoes inactivation and
this may be the maternal or paternal X chromosome. The Barr body
seen in all nucleated cells is the inactivated X chromosome and one is
seen with the XX genotype. Two Barr bodies are seen with the XXX
genotype. Triple X syndrome is associated with a below average
intelligence.
About 5-15% of PMNs have drum stick of chromatin.
20 In the neonate, the appearance of the external genitalia may not
correspond with the genotype in the presence:
A Adrenogenital syndrome. T
B Testicular feminization syndrome. T
C Renal agenesis (potter's syndrome). F
D Trisomy21. F
E Severe hypospadias. T

21 In the human:
A Haploid cells contain homologous pairs of chromosomes. F
B Polyploidy cells contain extra sets of chromosomes. T
C Monosomic cells contain an extra chromosome. F
D Haploid cells contain 22 autosomes. T
6

E Chromosomes divide during the S phase of the cell cycle. F

22 In the human, a haploid number of chromosomes is found in:


A Red blood cells. F
B Blastocysts. F
C Primary oocytes. F
D The first polar body. T
E Spermatozoa. T

23 Mitochondrial DNA: √
A Is located in the nucleus. F
B Inheritance is patrilineal. F
C Is present in two copies per cell. F
D Mutation causes cystic fibrosis. F
E Is involved in the control of oxidative phosphorylation. T
Comment:
The DNA is located in the mitochondria.
It has 10 copies of DNA.
Inheritance is matrilineal.
24 In DNA: √
A Acodon is a sequence of three bases. T
B All codons have an identified function. F
C There is a greater variety of amino acids than there are different F
codons.
D Replication can be initiated at several different points along a T
chromosome.
E Complementary pairing precedes messenger RNA synthesis. T
Comment:
A codon is a sequence of three base pairs with a further sequence of
opposite base pairs on tRNA representing the anticodon. These codons
code for all amino acids but also code for extra or nonsense sequences
which may encode termination sequences.
25 Concerning the genetic control of protein synthesis: √
A Mature messenger RNA contains introns. F
B A codon has 3-base sequences. T
C Each amino acid has a single codon. F
D Transfer RNA has anticodon recognition sites. T
E Each transfer RNA carries a specific amino acid. T

26 Ribonucleic acid (RNA): √


A Contains deoxyribose. F
B Is composed of two nucleotide units. F
C Is the main constituent of chromosomes. F
7

D Is the main constituent of ribosomes. T


E Is required during protein synthesis. T

27 Messenger RNA (mRNA ): √


A Synthesis is dependent on RNA polymerase II. T
B Is an exact copy of sense DNA. F
C Contains exons. T
D Is measured by western analysis. F
E Translation occurs in the nucleus. F

28 Messenger ribonucleic acid (mRNA): √


A Is a double-stranded polymer. F
B Is transcribed from DNA in the nucleus. T
C Is not present in reticulocytes. F
D Contains thymine. F
E Is not present in oocytes. F

29 During the normal cell cycle: √


A The principal phase of deoxyribonucleic acid (DNA) synthesis is G1. F
B A tetraploid quantity of DNA is present at the end of G2. T
C G2 is the post-mitotic resting phase. F
D Cells are generally sensitive to anti-metabolites in the S phase. T
E The DNA is completely replicated several times. F

30 Steps involved in the identification of restriction fragment length √


polymorphisms (RFLP) include:
A Western blotting. F
B Restriction enzyme digestion. T
C Southern blotting. T
D Agarose gel electrophoresis. T
E Thin layer chromatography. F
8

1 Concerning radiation physics: √


A An electron has a greater mass than a proton. F
B A positron has the same charge as an electron. F
C A proton has appositive charge. T
D A neutron has almost the same mass as proton. T
E The hydrogen nucleus is a neutron. F
Comment:
The atom is composed of a nucleus containing the positively charged
protons and neutral neutrons of roughly equal mass, orbited by the
smaller negatively charged electron. A positron is an elementary
particle of roughly equal size to an electron but positively charged.
Hydrogen atomic weight 1 is composed of a proton in its nucleus.
2 In radiotherapy: √
A One Gray is equivalent to one joule per kilogram. T
B The skin usually receives a greater dose of radiation than the T
underlining tissues.
C The major effect of radiation energy is to damage the cytoplasm of the F
cell.
D Cells in tissues which are hypoxic are more vulnerable to radiation. F
E Radiation-induced change in tissues may take 6 weeks to develop. T
Comment:
The higher the energy of the initial beam, the deeper the area of
maximum energy build. This fact is used to spare sensitive superficial
structure like skin.
3 Concerning radiotherapy: √
A A gray (100 rads) is a unit of energy absorption. T
B Liver parenchyma is more radiosensitive than intestinal epithelium. F
C Cells with a slow reproductive capacity are usually the most F
radiosensitive.
D The concentration of intracellular oxygen is inversely proportional to F
the susceptibility of the cell to radiation damage.
E Well-differentiated tumors show higher radiosensitive than anaplastic F
tumors.
Comment:
Seminomas and lymphoreticular tumors are very sensitive to
9

radiotherapy, sarcomas are usually most resistant.


Less differentiated tumors are the most sensitive.
Poorly oxygenated areas are less sensitive.
Recurrent tumors less sensitive than the original tumors.
Some squamous cell carcinomas are sensitive (skin, servix), others are
not (lung).
The cells most sensitive to radiation are those of rapid turnover (e.g.
bone marrow, intestinal epithelium, cancer cells).
4 The natural decay of radioactive isotopes results in the emission: √
A Alpha particles T
B Gamma rays T
C Neutron rays. F
D Proton beams F
E Beta particles. T

5 Radiation damage to tissue : √


A Is greatest in tissue with a high mitotic index. T
B Is enhanced in the presence of reduced oxygen content. F
C May cause a non specific inflammatory response. T
D Does not cause neoplsia. F
E May cause chromosomal non-disjunction. T

6 In experimental condition, ultrasound may produce biological √


effects on tissues by the following means:
A Acceleration of cell division. F
B Heat generation. T
C Cavitations. T
D Duplication of chromosome numbers. F
E Microstreaming. T
Comment:
There are two principle bio effects of ultrasound: thermal and
mechanical. Thermal is created through the impact of acoustic energy
upon tissue. Mechanical bio effects include cavitation through
particulate streaming associated with the violent agitation of particles
within the medium.
7 the ultrasound energy used in a real-time machine for diagnostic √
imaging:
A Is pulsed. T
B Has a velocity measured in meters per second. T
C Has a velocity which is the same in all human tissues. F
D Has a frequency measured in decibels. F
E Is entirely dissipated within the tissues. F
Comment:
10

The velocity is not the same on tissues; it depends on the density


tissue.
The frequency is measured in MHZ.
The energy is not entirely dissipated some of it is reflected back to the
transducer.
8 Doppler ultrasound: √
A Is used to monitor fetal breathing. T
B Is used in fetal heart rate monitors. T
C Can be used to measure blood velocity in the fetus. T
D Measures proton relaxation times. F
E Requires injection of contrast agents. F

9 A woman who has rhesus (Rh ) genotype of Cde/CDe: √


A Could develop anti-C antibodies. F
B Could safely be transfused with D positive blood. T
C May develop anti-E antibodies. T
D Should receive anti–D immunoglobulin after giving birth to a Rh (D) F
positive infant.
E Is Rh negative. F
11

Autosomal dominant Autosomal recessive


Achondroplasia Cystic fibrosis
Osteogenesis imperfecta Phenylketonurea
Von Willebrand Tay Sacks disease
Huntingtons chorea Sickle cell anemia
Marphan syndrome Thalassamia
Multiple neurofibromatosis 21-Hydroxylase deficiency
Multiple polyposis Galactosaemia
Multiple exostoses Glycogen storage disease
Multiple telangiectasia Hurler syndrome
Myotonia congenita Homosystinuria
Dystrophia myotonica Congenital adrenal hyperplasia
Tuberous sclerosis Mucopolysaccharidosis
Peutz syndrome Xerodremia pigmentosa
Familial hypercholestrerolaemia
Adult polycystic kidney
Retinoblastoma

X linked dominant X linked recessive


Vitamin D resistant rickets Haemophilia
Incontinenta pigmenti Christmas disease
Xg blood group Duchinnes muscular dystrophy
G-6-P dehydrogenase deficiency
Blue\green colour blindness
Wisckott-Aldrich syndrome
Ichthyosis
Hunters syndrome
ACUTE PHASE PROTEINS
• Plasma proteins which show a rise in concentration in response to early mediators of the inflammatory
response to infection or tissue injury

• Proteins that increase dramatically in concentration include:

1)C-reactive protein

2)Mannose binding protein

3)Alpha-1 acid glycoprotein

4)Serum amyloid P component

• Proteins that show a moderate increase in concentration include

1)Fibrinogen

2)Fibronectin

3)Ceruloplasmin

4)Haptoglobin

5)Angiotensin

6)Alpha-1 proteinase inhibitor

7)Alpha-1 antichymotrypsin

C-REACTIVE PROTEIN

• Acute phase protein

• Pentamer with units non-covalently arranged around a calcium-binding cavity

• Binding to microbes is calcium-dependent

• Results in activation of complement via the classical pathway. This results in C3b deposition on
microbes and opsonisation

CYTOKINES

• Low molecular weight secreted proteins that mediate immunity, cell migration, growth, differentiation
and repair

• Both mRNA and proteins have a short half life, enabling tight regulation

• Act in a paracrine or autocrine manner

• Effects mediated by cell surface receptors which undergo ligand-induced sub-unit association to
enable signal transduction

• Cytokines are pleiotropic - exhibit multiple effects on a variety of cell types with considerable overlap
and redundancy with respect to individual functions

TUMOUR NECROSIS FACTOR-ALPHA

• Secreted by macrophages, monocytes, NK cells

CAUSES:

• Endothelial cell activation


• Neutrophil activation

• Fever

• Synthesis of acute phase proteins by the liver

• Apoptosis

• Catabolism and cachexia

INTERFERON-GAMMA

• Secreted by NK cells and T-lymphocytes

• Activates macrophages, inhibits viral replication and stimulates some antibody responses

INTERLEUKIN-12

• Secreted by macrophages and dendritic cells

• Activates NK cell activity and stimulates interferon-gamma secretion by NK cells

• Stimulates the differentiation of nave T-cells into the effector cells of the adaptive immune system

INTERLEUKIN-1

• Secreted by macrophages, monocytes, NK cells and endothelia cells

• Causes endothelial cell activation

• Induces fever

• Stimulates synthesis of acute phase proteins in the liver


POLYMORPHONUCLEAR NEUTROPHILS *****

• Phagocytic cells - dominant white cell in the blood stream, making up 40-75% of total leukocyte
count *

• Non-dividing

• Have a single multi-lobed nucleus

• Granules do not stain with either acidic or basic dyes (hence neutrophils)

• Contain primary granules anti-microbial effectors such as myeloperoxidase and cathepsin G

• Contain secondary granules with lactoferrin, lysozyme and alkaline phosphatase

• Glycogen granules enable function under anaerobic conditions

• Count is increased by: Bacterial infections, trauma (including surgery), infarction, burns,
haemorrhage, disseminated malignancy and corticosteroid therapy

• Count is decreased by: viral infections, TB, brucellosis, typhoid, drugs such as carbimazole
and sulphonamides, severe septicaemia, hyper-splenism, B12 and folate deficiency

MONOCYTES *****

• Phagocytic cells that make up 2-10% of total leukocyte count

• Are phagocytic within the circulation *

• Differentiate into macrophages, including Kupffer cells and microglia within connective tissue
MACROPHAGES *****

• Mononuclear phagocytic cells derived from circulating monocytes *

• Present in all connective tissue but are particularly concentrated in the lungs forming alveolar
macrophages, liver (Kupffer cells), spleen, kidneys (glomerular mesangial cells), brain
(microglial cells) and bone (osteoclasts)*

• Long-lived in tissues

• Effective against intracellular organisms

• Recognise and phagocytose apoptotic cells without initiating an inflammatory response

• Activated macrophages secrete interleukin-12 which activates NK cells. Activated NK cells in


turn secrete interferon-gamma which activates macrophages. Macrophages also secrete TNF-
alpha and interleukin-1

• Activated macrophages generate reactive oxygen species such as superoxide, hydroxyl radical
and nitric oxide which are essential for killing microbes

• Fibroblast growth factor, angiogenic factors and metalloproteinases are secreted and play a
role in the remodelling of infected tissues

NATURAL KILLER (NK) CELLS *****

• NK cells a group of cytolytic lymphocytes, distinct from B-lymphocytes and T-lymphocytes, that
participate in both innate immunity and adaptive immunity*

• Lack B-cell receptors (surface immunoglobulins) and T-cell receptors (TCR / CD3) *

• Express receptors for the Fc portion of some IgG antibodies *


• Large granular lymphocytes - make up 10% of lymphocytes in blood and peripheral lymphoid
tissues and about 2% of lymphocytes in peripheral blood *

• Specialized to kill certain types of target cells, especially cells infected with virus or malignant
cells *

Features of NK Cells

• NK cells do not express surface immunoglobulins or TCR/CD3 on their surface. The surface
markers that best characterize NK cells are CD2+,CD3-, CD4-, and CD56+ *

• NK receptors are polymorphic so that the repertoire of NK receptors differs from person to
person.

• The first NK cell receptor, called the killer-activating receptor recognises stress-induced
molecules. This interaction sends a positive signal which enables the NK cell to kill the cell to
which it has bound unless the second receptor cancels that signal.

• This second receptor, called the killer-inhibitory receptor (KIR), recognises MHC-I molecules
which are usually present on all nucleated human cells.

• If MHC-I molecules are expressed on the cell, the killer-inhibitory receptor sends a negative
signal that overrides the kill signal and prevents the NK cell from killing that cell

• This ensures that NK cells are not activated by normal host cells

• Viruses often suppress class I MHC expression in cells they infect, the virus-infected cell
becomes susceptible to killing by NK cells.

• Malignant cells have reduced or no class I MHC expression and are susceptible to killing by NK
cells.

• NK cells kill cells to which antibody molecules have attached through a process
called antibody-dependent cytotoxicity

• The killing is done by the exocytosis of granules containing perforin and granzymes.
• Pre-programmed to recognise their targets and have no need to develop into a clone of
identical cells.

• Secrete cytokines such as IFN-γ and TNF-α *

BASOPHILS / MAST CELLS

• These blood cells give rise to tissue mast cells

• Mast cells have large basophilic granules that contain histamine and other substances that
induce vasodilatation, increase vascular permeability, increase secretions and smooth muscle
constriction.

• Mast cell degranulation is triggered by activated components of complement and by antigen


cross-linking of IgE bound to the mast cell surface.

• Excessive activation of mast cells leads to symptoms that characterise allergic diseases such
as hay fever and asthma.

• Generalised mast cell activation results in widespread blood vessel dilation; this can result in
a catastrophic drop in blood pressure - an effect termed anaphylactic shock.

• Of the phagocytes only macrophages and Langerhans cells retain an intact nucleus after
leaving the marrow and have the potential to divide in the periphery.

• Chromatin (chromosomal material) is lost both from eosinophils and neutrophils during the end
stages of their maturation in the marrow.

• This is reflected in the polymorphic nuclei of these cells; they are sometimes known as
polymorphs.

• Both eosinophils and neutrophils have a short life-span after leaving the marrow (see table
below)
All three main types of phagocyte have the capacity to kill microbes they ingest or
adhere to.

• Liver macrophages (Kupffer cells) are perfused principally by the portal blood supply, which
drains the intestines and the spleen.

• Splenic macrophages play a major role in taking up and destroying effete blood cells.

INNATE RESPONSE IN PREGNANCY *****

• Maternal deciduas infiltrated with macrophages and NK cells especially in the first trimester

• Increased number of monocytes and granulocytes in peripheral maternal blood from the first
trimester

• Circulating monocytes and granulocytes are activated with respect to some functions

• Increased concentration of acute phase proteins and activation of the clotting system

• Number of circulating NK cells reduced and their function is suppresse

COMPLEMENT SYSTEM *****

• Range of circulating and membrane-associated proteins that are important for host defence

• Many components are proteolytic enzymes *


• The most abundant and central component is C3

• By convention, a bar over any component denotes enzymatic activity

• Following cleavage of a component, the larger product has the suffix 'b' and the smaller 'a'

• The alternative pathway is activated when complement proteins are activated on microbial
surfaces - this is part of the innate immune system and cannot be controlled as there are no
complement-regulatory proteins on microbes *

• The classical pathway is activated by the binding of antibodies to microbes or other antigens -
part of the adaptive immune system *

• The lectin pathway is activated by the binding of a plasma protein (mannose-binding lectin) to
surface glycoproteins of microbes. Mannose-binding lectin activates components of the
classical pathway but is a component of the innate immune system as it is activated in the
absence of antibodies *

• The complement system plays 3 roles in host defence

1) Opsonisation - C3b coats microbes promoting phagocytosis through C3b receptors


on phagocytes *

2) Chemo-attraction - components of the complement system act as chemo-


attractants for monocytes and neutrophils *

3) Microbial killing - complement activation culminates in the formation of the


membrane attack complex which forms holes in microbes causing death by osmotic
influx of water and ions *

GRAFT REJECTION *****

HYPERACUTE REJECTION

• Occurs within minutes of transplantation


• Characterised by thrombosis of graft vessels and ischaemic necrosis of the graft

• Antibody mediated response caused by pre-existing antibodies that may have developed as a
result of previous blood transfusion - therefore more likely to occur in those with previous
transfusion

• Uncommon problem as donor and recipient are cross-matched

• Corneal and cartilage transplants are not rejected as these are avascular structures and
therefore do not sensitize the recipient

ACUTE REJECTION

• Occurs within 10 days (early) or after 10 days (late)

• Early acute rejection is a cell mediated hypersensitivity reaction (T-cell) although antibody
mediated hypersensitivity also contributes

• Late acute rejection may be a result of break-through of immuno-suppression and is


characterised by the deposition of complement and immunoglobulins in arterioles and
capillaries

• Preventable by immuno-suppressive therapy

CHRONIC TRANSPLANT REJECTION

• Occurs over several months / years and manifests as a gradual loss of graft function

• Characterised by fibrosis and narrowing of graft vessels (graft arteriosclerosis)

• T-cell mediated response with the production of cytokines which induce fibrosis and
proliferation of vascular smooth muscle
GRAFT VERSUS HOST REACTION

• Occurs when competent T-cells are transplanted from a donor to a recipient who is unable to
reject them. The cells survive, recognise the host's antigens and mount an immunological
reaction

• May follow bone marrow transplant or occur as a result of T-cells present in transplanted
organs

• Causes fever, haemolytic anaemia, splenomegaly, weight loss, rash, diarrhoea

The reaction is greater the greater the HLA difference between the host and the donor.
May be fatal

HYPERSENSITIVITY REACTIONS

IMMEDIATE (TYPE I) HYPERSENSITIVITY *****

• Rapid IgE and mast cell dependent vascular and smooth muscle reaction that is usually
followed by an inflammatory reaction in an individual who has been previously exposed to an
antigen (sensitized) *

• Antigen binds to IgE on mast cells, cross-linking receptors and inducing mast cell
degranulation *

• Released mast cell granules contain histamine, heparin, neutral proteases and cytokines
typical of TH2 responses - IL-3 and IL-4 *

• Cyclo-oxygenase and lipoxygenase are also activated with the production of vasodilator
prostaglandins, leukotrienes and thromboxanes *

• In the skin is characterised by a wheal and flare reaction *

• The result is increased capillary permeability, smooth muscle contraction and this manifests as
wheezing, oedema, hypotension - this is the acute phase and occurs within minutes *
• The latent phase peaks after 3-5 hours and is caused by cytokine mediated inflammatory
infiltration by neutrophils and eosinophils - this is responsible for tissue damage that occurs
after repeated episodes of immediate hypersensitivity *

• Typical disease processes include asthma, hay fever, drug and insect bite induced
anaphylaxis, penicillin allergy, food allergy, atopic dermatitis *

TYPE II (ANTIBODY MEDIATED CYTOTOXIC)


HYPERSENSITIVITY *****

• Caused by antibodies binding to cells or tissues and initiating damage by opsonisation or


activation of inflammatory cells - typically IgG. Tissue damage is caused by phagocytic and
non-phagocytic cells *

• Antibodies may be allo-immune (for instance in transfusion reactions) or auto-immune


(myasthenia gravis) *

• Disorders include *

1) Transfusion reaction and Rhesus disease

2) Auto-immune haemolytic anaemia

3) Auto-immune thrombocytopaenia purpura

4) Myasthenia gravis, Grave’s and Hashimoto’s disease

5) Acute rheumatic fever - anti-streptococcal antibodies cross-react with myocardial


antigen causing myocarditis and arthritis

6) Pernicious anaemia

7) Drug-induced haemolytic anaemia

8) Goodpasture's syndrome - deposition of antibodies and complement to glomerular


and lung basement membrane

IMMUNE COMPLEX MEDIATED (TYPE III) HYPERSENSITIVITY *****


• Caused by immune complex deposition in the blood vessels or tissues *

• These complexes activate macrophages, activate complement and cause the extracellular
release of neutrophil granules and tissue damage *

• The inflammatory response is characterised by the Arthus reaction which is characterised by a


neutrophilic infiltration and immunoglobulin + complement deposition *

• The inciting antigen may be 'self' or 'non-self' from infectious organisms

• Disease processes include *

1) Polyarteritis nodosa and SLE

2) Farmer's lung - occurring in patients sensitized to thermophilic actinomycetes;


pigeon-fancier's disease - these disorders are generally known as extrinsic allergic
alveolitis

3) Post-streptococcal glomerulonephritis

4) Jarisch-Herxheimer reaction

DELAYED CELL MEDIATED (TYPE IV) HYPERSENSITIVITY *****

• Occurs in allergic reactions to bacteria, fungi and viruses, contact dermatitis and in the
rejection of transplanted tissues *

• Typically occurs in the Mantoux reaction to tuberculin injected in the skin of a sensitized
individual - characterised by erythema and induration *

• Reaction occurs after several hours and peaks 24-48h after contact *

• The early phase is characterised by a perivascular cuffing with mononuclear cells, and then
polymorphs which later leave. The late phase is characterised by a lymphocyte and monocyte-
macrophage infiltration *
• T-lymphocyte dependent, and TH1 dependent in most cases, resulting in activation of
macrophages. TH2 responses do however occur, for instance in the hepatic lesions in
schistosomiasis *

• Antigen interacts with primed T-cells with tissue damage resulting from inappropriate cell
mediated immune reactions

• The chronic inflammatory response leads to the formation of granulomas, which may be
caseating in the case of TB *

Occurs in Crohn's disease, psoriasis, celiac disease, TB, leprosy, contact dermatitis,
tissue damage following measles and herpes infection; schistosomiasis *

LYMPHOCYTE MATURATION AND THE GENERATION OF


DIVERSITY*****

• B-lymphocyte maturation occurs in the bone marrow while T-lymphocyte maturation occurs in
the thymus

• The proliferation of pro-B and pro-T cells is stimulated by interleukin-7 secreted by the bone
marrow or thymus

T-CELL RECEPTOR *****

• Only recognises peptide antigens displayed by MHC molecules. Does not recognise lipid,
polysaccharide or nucleic acid antigens *

• Recognises the linear structure of peptides *

• Each receptor has an alpha and a beta chain each with a variable and a constant domain
homologous to those on immunoglobulins

• The variable domain has three hyper-variable regions

• Membrane-bound proteins - are not secreted *


• About 10% of T-cells express receptors with gamma and delta chains and recognise lipid and
other non-peptide antigens displayed by molecules other than classical MHC molecules - these
are abundant in epithelial surfaces *

LYMPHOCYTES *****

• Make up 20-45% of total leukocyte count

• Only immune cells with specific receptors for antigens

• Count is increased in viral infections, toxoplasmosis, brucellosis, and whooping cough

• Count is decreased in uraemia, Legionnaires disease, corticosteroid treatment, chemotherapy


and radiotherapy and HIV infection

• Characterised by their differential expression of CD (cluster of differentiation) antigens

• Following stimulation by antigen, some lymphocytes differentiate into memory cells which are
capable of mounting a rapid response if the same antigen is later encountered

• Most lymphocytes are located in secondary lymphoid tissues - lymph nodes, the white pulp of
the spleen, the gut- and bronchial-associated lymphoid tissues.

• The part of the B cell receptor that binds antigen is surface immunoglobulin*

• Subsequent B cell proliferation and maturation generates plasma cells that secrete this
immunoglobulin *

• In contrast to B cell receptors, T cell receptors do not bind free antigen. The antigen must be
processed into small peptides by antigen presenting cells and these antigen derived peptides
are displayed on the antigen presenting cell surface in the clefts of molecules called major
histocompatibility molecules (MHC) *

• There are three main types of T cells *


1) T helper cells that facilitate B cell responses to antigen

2) T inflammatory cells that facilitate macrophages in the eradication of intracellular


infection

3) Cytotoxic T cells that recognise and destroy virus infected cells.

B-LYMPHOCYTES *****

• Mature in the bone marrow. Develop into antibody producing plasma cells. Only immune cells
capable of antibody production *

• Have membrane-bound immunoglobulins which act as antigen receptors *

• Plasma cells migrate to the bone marrow where they produce small quantities of antibodies
for long periods of time after the initial infection has been cleared

• B-cell receptors bind soluble antigens

• The bound antigen molecules are engulfed into the B cell by receptor-mediated
endocytosis. *

• The antigen is digested into fragments which are then displayed at the cell surface nestled
inside a class II histocompatibility molecule. *

• Helper T cells with complementary TCRs bind the B cell and secretelymphokines that *

1) Stimulate the B cell to enter the cell cycle and develop, by repeated mitosis, into
aclone of cells with identical BCRs;

2) Switch from synthesizing their BCRs as integral membrane proteins to a soluble


version;

3) Differentiate into plasma cells that secrete immunoglobulins.


T-LYMPHOCYTES *****

• Mature in the thymus gland *

• Can only recognise peptide antigens that are bound to MHC molecules on specialised antigen-
presenting cells - MHC-restricted *

• CD4 positive T-cells are helper T-cells - help B cells produce antibodies and help phagocytes
destroy ingested organisms *

• CD8 positive T-cells are cytotoxic T-cells - capable of killing host cells infected with intracellular
organisms *

• Na-ve T-cells have antigen receptors but do not perform the functions required to eliminate the
antigen

CD8+ T cells

• CD8+ T cells bind that are part of class I histocompatibility molecules. Almost all the cells of
the body express class I molecules

• The best understood CD8+ T cells are cytotoxic T lymphocytes - secrete molecules that
destroy the cell to which they have bound.

• The role of the CD8+ T cells is to monitor all the cells of the body, ready to destroy any that
express foreign antigen fragments in their class I molecules.

CD4+ T cells

• CD4+ T cells bind epitopes that are part of class II histocompatibility molecules. Only
specialized antigen-presenting cells express class II molecules

• Essential for both the cell mediated and antibody-mediated branches of the immune system
• Bind to antigen presented by antigen-presenting cells and releaselymphokines that attract
other cells to the area.

• Bind to antigen presented by B cells, resulting in the development of clonesof plasma


cells secreting antibodies against the antigenic material.

ANTIGEN PRESENTING CELLS *****

• All nucleated cells can present antigens derived from microbes in the cytoplasm to cytotoxic T-
cells *

• Professional antigen-presenting cells include dendritic cells (Langerhans cells are epidermal
dendritic cells) and macrophages *

• B-cells ingest protein antigens and present them to helper T-cells *

Langerhans cells

• Professional antigen presenting cells

• Found in all tissues.

• Can be activated by local tissue injury

• This activation process induces Langerhans cells to migrate to secondary lymphoid tissues
where they present to T lymphocytes antigen derived peptide/MHC complexes.

LYMPH NODES *****

• Capsulated structures which drain lymphatics


• Have a cortex and a medulla with sinuses

• Lymph drains from the efferent lymphatics into the sub-capsular sinus and then through the
medullary sinuses into the efferent lymph vessels

• B-cells are concentrated in the cortex where they form follicles. Un-stimulated B-cells form
primary follicles while secondary follicles are formed following antigenic stimulation. Secondary
follicles have a germinal centre which is the site of B-cell maturation and the generation of B-
cell memory

• T-cells are concentrated in the para-cortical area

• Naive T-cells leave the circulation to enter the lymph nodes through specialised post-capillary
venules called high endothelial venules

SPLEEN *****

• Has a capsule, red pulp and white pulp

• The red pulp is the red cell area - the spleenic capillaries are very porous and red cells leave
the capillaries to enter the red pulp. They then squeeze through the trabeculae which are lined
by a vast number of macrophages to re-enter the circulation at the venous end

• The white pulp contains follicles (B-cell area) and peri-arteriolar lymphoid sheaths (T-cell area)

• Old red cells and platelets are removed from the circulation in the spleen

Immune cells & distribution *****


Number/litre in blood Life-span after
of adults leaving bone
marrow

Distribution
Cell type

trace numbers ~30 days Mainly in tissues

Basophils /

Mast cells

2-8x109/l 5-7 hr in blood Blood & tissues

Neutrophils
0.04-0.4x109/l 14-18 hr in blood blood and tissues

Eosinophils

0.2-0.8x109/l Days to weeks Most are in tissues -


spleen, liver and
lungs

Monocytes/

macrophages
5-14 days Mostly in tissues and
T zones of secondary
lymphoid tissues

Langerhans
cells

~0.2-1.0x109/l Blood spleen and


sites of inflammation

NK cells

0.04-2.5x109/l 1 day ? several years Most T cells are in


secondary lymphoid
tissues or epithelia,

T
lymphocytes

<5% are in the blood


0.03-0.7x109/l 2 days to several Most B cells are in
weeks; memory secondary lymphoid
clones up to years tissues<5% are in
the blood
B
lymphocytes

PASSIVE IMMUNOTHERAPY *****

• The use of immunoglobulins to treat or prevent disease. Antibodies may be from human or
animal sources or may be genetically engineered

• Does not confer long term immunity

• Associated with serum sickness especially when antibodies are from an animal source

• Antibodies are used to treat:


1) Tetanus and diphtheria

2) Botulism, snake and scorpion bites

3) Hepatitis B

4) Auto-immune disorders including immune thrombocytopaenia

NEONATAL PASSIVE IMMUNITY

• Maternal IgG is actively transported by the placenta into the fetus. IgA is not taken up by the
placenta *

• Secreted IgA in breast milk is not absorbed but confers mucosal immunity in the GI tract *

VACCINATION *****

Types of Vaccines

• Killed organisms

• Live-attenuated organisms - replicate in the host. Provide a larger 'dose' and have the
advantage of inducing immunity at the natural site of infection

• Sub-unit vaccines - make use of the most antigenic subunit of the organism. These can be
produced by genetic engineering

• Toxoid - bacterial exotoxins are first detoxified by treatment with formaldehyde


BACTERIAL

Live Attenuated

• BCG - tuberculosis

Inactivated

• Cholera vaccine

• Pertussis vaccine

• Typhoid vaccine

Sub-unit

• H. influenzae

• N. meningitides

• S. pneumoniae

Toxoid

• Tetanus

• Diphtheria
VIRUSES

Live Attenuated

• MMR - measles, mumps, rubella

• Polio (Sabin)

• Varicella zoster - not in general use

• Adenovirus

• Yellow fever

Inactivated

• Polio (Salk)

• Influenza

• Rabies

• Hepatitis A

Sub-unit

• Hepatitis B
• Influenza

THE MAJOR HISTOCOMPATIBILITY COMPLEX

• Made up of membrane bound proteins on antigen presenting cells that display peptide antigens
for recognition by T-cells *

• Play a central role in graft rejection *

• Human MHC molecules are called human leukocyte antigens (HLA) *

• MHC genes are highly polymorphic - there are many different alleles present in different
individuals. No two individuals in an outbred population have identical MHC genes or
molecules (except identical twins) *

• Human MHC located on chromosome 6 *

• Variations in MHC genes are not due to DNA recombinations

• There are two sets of highly polymorphic HLA genes called class I and class II genes

• The MHC also contains several non-polymorphic genes, some involved in antigen presentation
and others with unknown functions

• Inherited in an autosomal co-dominant fashion *

• Class I molecules are expressed by all nucleated cells while class II molecules are expressed
by professional antigen-presenting cells only (dendritic cells, macrophages and B-
lymphocytes) *

• MHC molecules cannot discriminate between self and non-self peptides

• MHC molecules cannot present non-peptide antigens


MHC CLASS I *****

• Three polymorphic class I genes: HLA-A, HLA-B and HLA-C *

• Each person inherits one set from each parent, therefore each cell can express 6 different
class I molecules *

• Each class I molecule is made up of an alpha chain non-covalently attached to beta-2-


microglobulin *

• CD8 positive T cells can therefore only respond to peptide antigens displayed by class I MHC
molecules

MHC CLASS II *****

• Three polymorphic genes: HLA-DR, HLA-DQ, HLA-DP *

• Each person inherits one set from each parent, therefore each cell can express 6 different
class II molecules *

• Each class II molecule is made up of an alpha and a beta chain *

• The peptide binding site for class II molecules can bind peptides with 10-30 amino acids

• The beta2 domain on the beta chain of class II molecules is invariant and contains the binding
site for the T-cell receptor CD4

• CD4 positive cells can therefore only recognise peptide antigens displayed by MHC class II
molecules
POLYMORPHONUCLEAR NEUTROPHILS *****

• Phagocytic cells - dominant white cell in the blood stream, making up 40-75% of total leukocyte
count

• Non-dividing

• Have a single multi-lobed nucleus

• Granules do not stain with either acidic or basic dyes (hence neutrophils)

• Contain primary granules anti-microbial effectors such as myeloperoxidase and cathepsin G

• Contain secondary granules with lactoferrin, lysozyme and alkaline phosphatase

• Glycogen granules enable function under anaerobic conditions

• Count is increased by: Bacterial infections, trauma (including surgery), infarction, burns,
haemorrhage, disseminated malignancy and corticosteroid therapy

• Count is decreased by: viral infections, TB, brucellosis, typhoid, drugs such as carbimazole
and sulphonamides, severe septicaemia, hyper-splenism, B12 and folate deficiency

MONOCYTES *****

• Phagocytic cells that make up 2-10% of total leukocyte count

• Are phagocytic within the circulation

• Differentiate into macrophages, including Kupffer cells and microglia within connective tissue
MACROPHAGES *****

• Mononuclear phagocytic cells derived from circulating monocytes

• Present in all connective tissue but are particularly concentrated in the lungs forming alveolar
macrophages, liver (Kupffer cells), spleen, kidneys (glomerular mesangial cells), brain
(microglial cells) and bone (osteoclasts)

• Long-lived in tissues

• Effective against intracellular organisms

• Recognise and phagocytose apoptotic cells without initiating an inflammatory response

• Activated macrophages secrete interleukin-12 which activates NK cells. Activated NK cells in


turn secrete interferon-gamma which activates macrophages. Macrophages also secrete TNF-
alpha and interleukin-1

• Activated macrophages generate reactive oxygen species such as superoxide, hydroxyl radical
and nitric oxide which are essential for killing microbes

• Fibroblast growth factor, angiogenic factors and metalloproteinases are secreted and play a
role in the remodelling of infected tissues

NATURAL KILLER (NK) CELLS *****

• NK cells a group of cytolytic lymphocytes, distinct from B-lymphocytes and T-lymphocytes, that
participate in both innate immunity and adaptive immunity*

• Lack B-cell receptors (surface immunoglobulins) and T-cell receptors (TCR / CD3) *

• Express receptors for the Fc portion of some IgG antibodies *


• Large granular lymphocytes - make up 10% of lymphocytes in blood and peripheral lymphoid
tissues and about 2% of lymphocytes in peripheral blood *

• Specialized to kill certain types of target cells, especially cells infected with virus or malignant
cells *

Features of NK Cells

• NK cells do not express surface immunoglobulins or TCR/CD3 on their surface. The surface
markers that best characterize NK cells are CD2+,CD3-, CD4-, and CD56+ *

• NK receptors are polymorphic so that the repertoire of NK receptors differs from person to
person.

• The first NK cell receptor, called the killer-activating receptor recognises stress-induced
molecules. This interaction sends a positive signal which enables the NK cell to kill the cell to
which it has bound unless the second receptor cancels that signal.

• This second receptor, called the killer-inhibitory receptor (KIR), recognises MHC-I molecules
which are usually present on all nucleated human cells.

• If MHC-I molecules are expressed on the cell, the killer-inhibitory receptor sends a negative
signal that overrides the kill signal and prevents the NK cell from killing that cell

• This ensures that NK cells are not activated by normal host cells

• Viruses often suppress class I MHC expression in cells they infect; the virus-infected cell
becomes susceptible to killing by NK cells.

• Malignant cells have reduced or no class I MHC expression and are susceptible to killing by NK
cells.

• NK cells kill cells to which antibody molecules have attached through a process
called antibody-dependent cytotoxicity

• The killing is done by the exocytosis of granules containing perforin and granzymes.
• Pre-programmed to recognise their targets and have no need to develop into a clone of
identical cells.

• Secrete cytokines such as IFN-γ and TNF-α *

NKT Cells

• T cells with an αβ TCR. However, they also express some of the cell-surface molecules of NK
cells - hence their name.

• Differ from most T cells in the sharply limited diversity of their αβ TCRs and these respond to
glycolipid antigens presented by a cell-surface molecule designated CD1d (rather than to
peptide antigens presented by MHC molecules).

• Secrete IFN-γ and IL-4.

INNATE RESPONSE IN PREGNANCY *****

• Maternal deciduas infiltrated with macrophages and NK cells especially in the first trimester

• Increased number of monocytes and granulocytes in peripheral maternal blood from the first
trimester

• Circulating monocytes and granulocytes are activated with respect to some functions

• Increased concentration of acute phase proteins and activation of the clotting system

• Number of circulating NK cells reduced and their function is suppresse


PHAGOCYTOSIS - KILLING MECHANISM *****

• Phagocytes have a range of mechanisms by which microbes are killed once within the cell.
These can be divided into oxygen-dependent and independent mechanisms

• OXYGEN DEPENDENT MECHANISMS - mainly involve the generation of highly reactive free
radicals:

• Superoxide (this is removed by superoxide dismutase)

• Hydroxyl radical

• Nitric oxide and its superoxide derivative peroxynitrite

OXYGEN INDEPENDENT MECHANISMSA - involve becteriostatic and


bacteriocidal chemicals such as lysozyme, lactoferrin, cathepsin G and a variety of
proteolytic enzymes which digest the killed microbe
COMPLEMENT SYSTEM *****

• Range of circulating and membrane-associated proteins that are important for host defence

• Many components are proteolytic enzymes *

• The most abundant and central component is C3

• By convention, a bar over any component denotes enzymatic activity

• Following cleavage of a component, the larger product has the suffix 'b' and the smaller 'a'

• The alternative pathway is activated when complement proteins are activated on microbial
surfaces - this is part of the innate immune system and cannot be controlled as there are no
complement-regulatory proteins on microbes *

• The classical pathway is activated by the binding of antibodies to microbes or other antigens -
part of the adaptive immune system *

• The lectin pathway is activated by the binding of a plasma protein (mannose-binding lectin) to
surface glycoproteins of microbes. Mannose-binding lectin activates components of the
classical pathway but is a component of the innate immune system as it is activated in the
absence of antibodies *

• The complement system plays 3 roles in host defence

1) Opsonisation - C3b coats microbes promoting phagocytosis through C3b receptors


on phagocytes *

2) Chemo-attraction - components of the complement system act as chemo-


attractants for monocytes and neutrophils *

3) Microbial killing - complement activation culminates in the formation of the


membrane attack complex which forms holes in microbes causing death by osmotic
influx of water and ions *
AnAAAQuestion 1: Natural killer cells

a. Are specialised neutrophils

True False
b. Recognise virally infected cells

True False
c. Make up 50% of lymphocytes

True False
d. Express immunoglobulins

True False

Question 2: The following chemicals are produced by phagocytes as part of the killing
mechanism

a. Hydrogen peroxide

True False
b. Superoxide dismutase

True False
c. Superoxide radical

True False
d. Hydroxyl radical

True False

Question 3: With respect to the complement system

a. The lectin pathway is a component of the innate immune system

True False
b. The lectin pathway is activated by the binding of mannose-binding lectin to the surface
glycoprotein on microbes

True False
c. Phagocytes express receptors for C3b

True False
d. C3b causes opsonisation of bacteria

True False

Question 4: Macrophages

a. Are derived from neutrophils

True False
b. Are present in the circulation

True False
c. Are multinucleated cells

True False
d. Are concentrated in the lungs

True False

Question 5: Polymorphonuclear neutrophils

a. Count is increased following tissue infarction


True False
b. Count is decreased after major surgery

True False
c. Count is increased in vitamin B12 deficiency

True False
d. Count is increased in disseminated malignancy

True False

Question 6: Polymorphonuclear neutrophils

a. Differentiate into monocytes

True False
b. Are peroxidase positive

True False
c. Are alkaline phosphatase positive

True False
d. Contain glycogen

True False

Question 7: With respect to the complement system

a. By convention, on cleavage of any component, the larger product is given the suffix ?a?

True False
b. The alternative pathway is a component of the innate immune system

True False
c. The alternative pathway is activated following antibody binding to microbes

True False
d. The alternative pathway can be controlled by complement regulatory proteins

True False

Question 8: Cytokines

a. Are high molecular weight proteins

True False
b. Have a short half life

True False
c. Act via receptors located within the cell nucleus

True False
d. Have multiple effects on a variety of cell types

True False

Question 9: The following chemicals are produced by phagocytes as part of the killing
mechanism

a. Nitric oxide

True False
b. Peroxynitrite
True False
c. Lysozyme

True False
d. Lactoferrin

True False

Question 10: Polymorphonuclear neutrophils

a. Stain with acid dyes

True False
b. Stain with basic dyes

True False
c. Divide by mitosis within the circulation

True False
d. Are multi-nucleated

True False

Question 11: Interferon-gamma

a. Is secreted by macrophages

True False
b. Is secreted by NK cells

True False
c. Activates macrophages

True False
d. Inhibits viral replication

True False

Question 12: Natural killer cells

a. Secrete perforin

True False
b. Secrete interferon-gamma

True False
c. Can be activated by uninfected normal host cells

True False
d. Are activated by MHC-negative virally infected cells

True False

Question 13: The following mechanisms are used by the bacteria described to escape
destruction by the innate immune system

a. Staphylococci ? production of catalase which breaks down reactive oxygen species

True False
b. Staphylococci ? capsular polysaccharide inhibits phagocytosis

True False
c. Neisseria meningitides ? resistance to complement activation
True False
d. Pseudomonas ? resistance to anti-microbial action of peptide antibiotics

True False

Question 14: The following are acute phase proteins

a. Serum amyloid P component

True False
b. Insulin-like growth factor

True False
c. Fibrinogen

True False
d. Ceruloplasmin

True False

Question 15: C-reactive protein

a. Is an acute phase protein

True False
b. Is a dimer

True False
c. Binding to microbes is calcium-independent

True False
d. Binding to microbes activates complement via the alternative pathway
Question 1: Plasma cells

a. Are increased in myeloma

True False
b. Are characteristic of acute infection

True False
c. Are phagocytic

True False
d. Synthesise antibodies

True False

Question 2: IgM

a. Fixes complement by the alternative pathway

True False
b. Binds to mast cells

True False
c. Is synthesised by plasma cells

True False
d. Is smaller than IgE

True False

Question 3: The human major histocompatibility complex genes

a. Are located on chromosome 11

True False
b. Are composed of humal leukocyte antigen genes

True False
c. Will be identical in dizygotic twins

True False
d. Code for blood group antigens

True False

Question 4: Concerning antibodies in pregnancy

a. IgG crosses the placenta

True False
b. The 4 classes of IgG all cross the placenta readily

True False
c. The concentration of IgG in fetal blood at term is 10x that in maternal blood

True False
d. IgM cannot be produced by the fetus

True False

Question 5: Natural killer cells

a. Are related to B-lymphocytes

True False
b. Express the T-cell receptor on their surface

True False
c. Express immunoglobulins on their surface

True False
d. Are CD 56 positive

True False

Question 6: Antibodies

a. Are produced by the fetus before 12 weeks gestation

True False
b. Have an average molecular weight of 100kDa

True False
c. Of the Rhesus type are genetically transmitted

True False
d. Are produced by ribosomes of plasma cells

True False

Question 7: In anaphylactic shock in humans

a. IgE is the mediator

True False
b. There is degranulation of mast cells

True False
c. Complement is required

True False
d. Histamine release occurs

True False

Question 8: T-lymphocytes

a. Differentiate in the thymus

True False
b. Are involved in both cell mediated and humoral immune responses

True False
c. Are phagocytic

True False
d. Produce antibodies

True False

Question 9: The biological effects of complement in the human include

a. Opsonisation

True False
b. Cell membrane lysis

True False
c. Participation in the blood coagulation system
True False
d. Promotion of sperm motility

True False

Question 10: Lymphocytes

a. Form about 2% of the total white cell count

True False
b. Have a life-span of about 30 days

True False
c. Do not participate in cell mediated immunity

True False
d. Can differentiate into plasma cells

True False

Question 11: Concerning antibodies in pregnancy

a. Maternal IgM is responsible for Rhesus disease

True False
b. The IgA concentration in cord blood is higher than in maternal blood

True False
c. IgE readily crosses the placenta

True False
d. Fetal IgM is monomeric

True False

Question 12: IgM

a. Has a molecular weight of 150kDa

True False
b. Contains J chains

True False
c. Is a dimmer in external secretions

True False
d. Contains an additional peptide produced by epithelial cells

True False

Question 13: B-lymphocytes

a. Produce tumour necrosis factor

True False
b. Do not produce antibodies

True False
c. Play a role in delayed hypersensitivity

True False
d. Produce IgE
True False

Question 14: Type III hypersensitivity is characterised by

a. Damage limited to a particular cell type

True False
b. Increased vascular permeability

True False
c. Micro-thrombus formation

True False
d. Involvement of IgE antibodies

True False

Question 15: Mast cells

a. Form 3% of total circulating white cells

True False
b. Release histamine on degranulation

True False
c. Contain heparin

True False
d. Have a specific affinity for IgA antibodies

True False
IMMUNOGLOBULINS *****

• Made up of 4 polypeptide chains - two identical light chains and two


identical heavy chains assembled in a Y-shaped molecule. All are
glycosylated

• Each light chain is attached to one heavy chain, and the heavy chains
are attached to each other by disulfide bonds

• Each light chain has one variable and one constant domain

• Each heavy chain has one variable and 3-4 constant domains

• Each variable domain has three hyper-variable regions

• The fragment of the antibody containing the whole of the light chain
attached to the variable and the first constant domain of the heavy
chain is the Fab region (Fragment antigen binding)

• The Fc (Fragment crystalline) region is made up of the remaining


constant heavy chain domains

• The hinge region is located between the Fab and Fc regions which is
flexible and enables the two Fab regions to move and bind epitopes
that are separated by a variable distance

• Each antibody molecule therefore has two Fab (two antigen binding
sites) and one Fc fragment

• There are two different types of light chain - kappa and lambda

• There are five different types of heavy chain - mu (IgM), delta (IgD),
gamma (IgG), epsilon (IgE) and alpha (IgA) *

• Antibodies of the same specificity but containing different heavy chains


are said to belong to different classes or isotypes - the class of
antibody is named according to the heavy chain as above

• Fetal liver is a major site of development of the human immune system.

• Pre-B and B-lymphocytes are present in the human fetal liver at 12


weeks in a random distribution and increase with gestation *

• Most fetal liver cells are pre-B but mature B-cells are also present.
Functional assays and transplantation experiments indicate that these
B-cells are functional and can transfer immunologic memory *

IgA *****
• Two subtypes (1&2), produced by epithelial plasma cells and dimerises
intracellularly - 4 antigen binding sites.

• Plasma IgA is mainly monomeric *

• 10-15% of total immunoglobulins *

• Half life in serum = 6 days. Monomer has molecular weight of 160kDa *

• Activates complement via the alternative (not classical) pathway.

• Does not cross the placenta and does not sensitize mast cells *

• Not absorbed from the GI tract in neonates (IgG is absorbed) *

• IgA is the major class of Ig in secretions - tears, saliva, colostrum,


mucus. Since it is found in secretions secretory IgA is important in local
(mucosal) immunity *

• IgA found in secretions is dimeric, linked by two additional chains. One


of these is the J chain (from join) - a polypeptide of molecular mass 1,5
kD, rich with cysteine and structurally completely different from other
immunoglobulin chains. This chain is formed in the antibodies secreting
cells. Another polypeptide (1,5 kD) - secretory chain, is produced by
the epithelial cells *

• It is also possible to find trimeric and even tetrameric IgA.

IgD *****

• One type only, main function is as receptor on naive B-cells *

• Makes up to 1% of serum immunoglobulin, is not secreted in epithelia


and does not activate complement or sensitize mast cells *

• Does not cross the placenta *

IgE *****

• One type only, monomer. Important in immediate hypersensitivity


reactions including anaphylaxis *

• Half-life in serum = 2 days.


• Molecular weight 190kDa, makes up 0.002% of serum
immunoglobulins *

• Sensitizes mast cells and binds to macrophages and polymorphs *

• Does not activate complement *

• Raised in parasitic infections and responsible for atopic allergy *

IgG *****

• Four subtypes (1-4), monomer *

• Half life in serum = 23 days. IgG 3 has a shorter half-life (7 days)

• Crosses the placenta - all subtypes are actively transported.


Concentration in cord blood slightly higher than that in maternal blood
at term (mean fetal : maternal concentration ratio = 1.27) *

• Pre-term neonates have lower concentrations *

• Causes Rhesus disease *

• IgG declines exponentially in the neonate, reaching its nadir between


2-4 months of life, when it starts to rise again owing to antibody
synthesis by the infant

• Feed-back inhibition of B-cells, activates antibody-dependent cell


mediated cytotoxicity, activates complement *

• Particularly abundant in extra-vascular fluid where it opsonises


microbes.

• Molecular weight 150kDa, makes up 75% of immunoglobulins *

IgM *****

• One type, pentamer - ten antigen binding sites *

• Half-life in serum = 5 days.

• Activates complement and serves as naive B-cell receptor *

• First immunoglobulin to be synthesised during B-cell maturation *

• Molecular weight 970kDa, makes up 5-10% of immunoglobulins *


• Does NOT bind to mast cells, neutrophils or macrophages *

• Does not cross the placenta - the presence of IgM in fetal blood
indicates fetal infection and fetal antibody production *
ANTIGEN RECOGNITION BY B-LYMPHOCYTES *****

• Makes use of membrane-bound antibodies as antigen receptors *

• Antigens are recognised in their native conformation with no requirement for processing or
presentation by specialised antigen presenting cells

• Peptide and non-peptide antigens (including lipids, polysaccharides, nucleic acids are
recognised *

B-CELL ACTIVATION AND ANTIBODY SYNTHESIS *****

• B-cell responses to antigen are either T-cell dependent or T-cell independent *

• The initial response to antigen is called the primary response, has a latent period of 5-10
days and is characterised by the production of small amounts of mainly IgM. *

• The response to subsequent encounters with a previously encountered antigen is the


secondary response, has a latent period of 1-3 days and is characterised by the production of
large quantities of antibody, with an increase in IgG production *

• T-cell dependent responses are induced by protein antigens and the secondary response
shows affinity maturation (production of antibodies with higher affinity for the antigen) and
immunoglobulin heavy chain class switching. Memory cells are generated

• T-cell independent responses are induced by polysaccharide, lipid antigens and other small
chemicals. These responses generally do not show affinity maturation or class switching and
memory cells are generally not produced

• B-cell stimulation begins with the cross-linking of antigen receptors (IgD or IgM) by antigen
binding *. This induces:

1) Cell division - clonal expansion


2) Increased expression of cytokine receptors

3) Migration out of lymphoid follicles

4) Secretion of low levels of IgM

• B-cells ingest process and display peptide antigens in association with MHC class II *

• CD4 positive T cells that recognise antigen displayed by B-cells are induced to express the
ligand for CD40 (CD40L) which interacts with CD40 on B-cells. The T-cells then secrete
cytokines (interferon gamma for TH1 cells and interleukin-4 for TH2 cells) - IFN-gamma
stimulates B-cells to secrete IgG (TH1 antibody) while IL-4 stimulates the production of IgE
(TH2 antibody) - CLASS SWITCHING. This does not occur in the absence of CD40 or CD40L *

• Affinity maturation occurs in the germinal centres of the lymphoid follicles and is caused by
somatic hyper-mutation of the immunoglobulin gene in dividing B-cells and the production of
antibodies with different affinities. Cells that bind strongly to antigen on follicular dendritic cells
are rescued, while the rest undergo apoptosis. As the infection subsides, there is less antigen
and therefore only B-cells producing higher affinity antibodies are selected*
Question 1: With respect to the structure of antibodies

a. The Fc fragment includes part of the light chain

True False
b. Each antibody molecule has two identical Fab regions

True False
c. Each antibody molecule has two Fc regions

True False
d. The hinge region is located between the Fab and the Fc regions

True False

Question 2: IgD

a. Makes up 10% of serum immunoglobulins

True False
b. Is glycosylated

True False
c. Activates complement

True False
d. Sensitizes mast cells

True False

Question 3: IgE

a. Is responsible for contact dermatitis

True False
b. Binds to macrophages and polymorphs

True False
c. Is increased in parasitic infections

True False
d. Makes up 10% of serum immunoglobulins

True False

Question 4: IgG

a. Is glycosylated

True False
b. Has a molecular weight of 150kDa

True False
c. Makes up 30% of total serum immunoglobulins

True False
d. Is not present in extra-vascular fluid

True False

Question 5: IgM

a. Has a molecular weight of 150kDa

True False
b. Sensitizes mast cells

True False
c. Is not glycosylated

True False
d. Binds to macrophages and polymorphs

True False

Question 6: During T-cell dependent B-cell activation

a. Affinity maturation occurs in the germinal centres of lymphoid follicles

True False
b. Affinity maturation is caused by somatic hypermutation of immunoglobulin genes

True False
c. IgE secretion is stimulated by interferon gamma

True False
d. IgG secretion is stimulated by interleukin-4

True False

Question 7: Antibodies

a. Serve as B-cell antigen receptors

True False
b. May be membrane-bound

True False
c. Are present in breast milk

True False
d. Do not cross the placenta

True False

Question 8: Primary antigen stimulation of B-cells results in

a. Activation of cell division

True False
b. Increased expression of cytokine receptors

True False
c. Migration out of lymphoid follicles

True False
d. Secretion of large quantities of IgG

True False

Question 9: The following antigens induce T-cell dependent B-cell antibody production

a. Peptide antigens

True False
b. Polysaccharide antigens

True False
c. Lipid antigens
True False
d. Nucleic acid antigens

True False

Question 10: Antibodies

a. Are alpha-globulins

True False
b. Have receptors on macrophages

True False
c. Do not recognise lipid antigens

True False
d. Do not recognise nucleic acid antigens

True False

Question 11: IgG

a. Opsonises bacteria

True False
b. Is important in the passive immunity of the neonate

True False
c. Activates antibody-dependent cell mediated cytotoxicity

True False
d. Serves as antigen receptor on B-cells

True False

Question 12: With respect to the structure of antibodies

a. Each light chain has one constant domain

True False
b. Each heavy chain has three variable domains

True False
c. Each variable domain on the heavy or light chain has three hyper-variable regions

True False
d. The Fab fragment consists of the entire light chain attached to the variable and first constant
domain of the heavy chain

True False

Question 13: IgE

a. Crosses the placenta

True False
b. Activates complement via the classical pathway

True False
c. Sensitizes mast cells

True False
d. Is responsible for atopic allergy
True False

Question 14: IgM

a. Fixes complement by the alternative pathway

True False
b. Binds to mast cells

True False
c. Is synthesised by plasma cells

True False
d. Is smaller than IgE

True False

Question 15: With respect to the structure of antibodies

a. There are five different types of light chains

True False
b. There are two different types of heavy chains

True False
c. Antibodies of identical specificity but containing different heavy chains are said to belong to
different isotypes or classes

True False
d. IgA has five sub-types

True False
ANTIGEN PROCESSING AND PRESENTATION

EXTRACELLULAR ANTIGENS

• Taken up by professional antigen presenting cells and degraded in intracellular phagocytic


vesicles

• Class II MHC molecules are synthesised in the endoplasmic reticulum

• Newly synthesised class II molecules have a peptide called the class II invariant chain peptide
(CLIP) tightly bound to the peptide binding cleft

• When exocytic vesicles containing newly synthesised class II molecules fuse with endosomal
vesicles containing degraded peptides, CLIP is removed and the peptide binding site is free to
bind peptides derived from degraded extracellular antigens

• Only class II molecules that display a peptide are expressed on the cell surface. Molecules
without peptides are unstable and degraded by proteolysis

• Both self and foreign peptides are displayed

CYTOSOLIC ANTIGENS

• Derived from microbes within the cytosol and other 'self' cytosolic proteins

• Degradation of these proteins occurs in organelles called proteasomes, generating peptides

• Synthesis of class I MHC molecules occurs in the endoplasmic reticulum

• Peptides are transported from the proteasomes into the endoplasmic reticulum by the
transporter associated with antigen processing (TAP). The normal route of transport of
peptides is from the endoplasmic reticulum into the cytosol following protein synthesis.

• As the peptides enter the ER, they are bound by class I molecules (class II molecules cannot
bind peptides here because they contain CLIP)
• Only class I molecules containing peptide are displayed on the cell surface. Molecules without
peptide are degraded. Self and non-self peptides are displayed.

TH1 AND TH2 CELLS *****

• Subsets of helper T cells (TH) - CD4 positive

• Differentiation into TH1 cells is stimulated by interleukin-12 secreted by antigen presenting


cells

• The main TH1 cytokine is interferon-gamma: activates macrophages, stimulates production of


IgG which promotes phagocytosis of microbes, stimulates MHC class II expression on antigen
presenting cells

• Other TH1 cytokines include TNF-alpha and IL-2

• The main TH2 cytokine is interleukin-4: stimulates the production of IgE. Interleukin-5 activates
eosinophils while other cytokines including IL-4 inhibit macrophage activity. TH2 cells therefore
stimulate phagocyte-independent, eosinophil mediated immunity - particularly important against
helminthic parasites
T-CELL BINDING TO ANTIGEN-PRESENTING CELLS

• The binding of T-cells to antigens is of low affinity and needs to be stabilised to enable cell
activation

• Stability is provided by cell adhesion molecules, of which integrins are the most important
group.

• The major T-cell integrin involved in binding to antigen presenting cells is the lymphocyte
function-associated antigen-1 (LFA-1). Its ligand is the intercellular cell adhesion molecule-1
(ICAM-1) which is expressed by antigen presenting cells

• LFA-1 on nagative T-cells has a low affinity for ICAM-1. Chemokines produced by the innate
immune system and antigen recognition by T-cells increases the affinity of LFA-1 for ICAM-1

• Integrins also play a role in T-cell migration to sites of infection

LYMPHOCYTE MATURATION AND THE GENERATION OF


DIVERSITY*****

• B-lymphocyte maturation occurs in the bone marrow while T-lymphocyte maturation occurs in
the thymus

• The proliferation of pro-B and pro-T cells is stimulated by interleukin-7 secreted by the bone
marrow or thymus

• There is a need to generate receptors that can recognise all the possible antigens that the cells
may encounter

• Diversity is generated by somatic recombination of genes that code for the variable regions of
the receptors

• All antigen receptor gene loci have contain several hundred variable region (V) genes and a
few constant region (C) genes - this applies to the immunoglobulin light and heavy chain and
the T-cell receptor alpha and beta chain
• Between the variable and constant genes are several stretches of nucleotides called joining (J)
and diversity (D) segments. All antigen receptor gene loci contain J segments but only the
immunoglobulin heavy chain and the T-cell receptor beta loci have D segments

• Diversity of antigen receptors is therefore generated by the use of different V, D & J gene
segments in different clones of lymphocytes (Combinatorial diversity). In addition, further
diversity is generated by nucleotide changes at the junctions of V, D & J genes (junctional
diversity)

• Positive selection occurs only during T-cell maturation. As T-cells only recognise antigens
displayed by MHC, cells that do not recognise MHC molecules in the thymus die by apoptosis.
These cells would otherwise be useless

• Negative selection occurs during both B and T cell maturation ?- cells expressing receptors
that bind strongly to antigens within the thymus (MHC associated) or in the bone marrow either
undergo apoptosis or further maturation is stopped. These cells would otherwise react strongly
to –self- antigens. In B-cells, recombination may be re-activated to generate a new light chain -
receptor editing

• Allelic exclusion occurs during both B-and T-cell maturation: once a receptor has been
synthesised, further recombination at un-recombined loci is shut-off, so that a receptor is
generated from one of the inherited parental alleles only

T-CELL RECEPTOR *****

• Only recognises peptide antigens displayed by MHC molecules. Does not recognise lipid,
polysaccharide or nucleic acid antigens *

• Recognises the linear structure of peptides *

• Each receptor has an alpha and a beta chain each with a variable and a constant domain
homologous to those on immunoglobulins

• The variable domain has three hyper-variable regions

• Membrane-bound proteins - are not secreted *


About 10% of T-cells express receptors with gamma and delta chains and recognise
lipid and other non-peptide antigens displayed by molecules other than classical MHC
molecules - these are abundant in epithelial surfaces *
LYMPHOCYTES *****

• Make up 20-45% of total leukocyte count

• Only immune cells with specific receptors for antigens

• Count is increased in viral infections, toxoplasmosis, brucellosis, and


whooping cough

• Count is decreased in uraemia, Legionnaires disease, corticosteroid


treatment, chemotherapy and radiotherapy and HIV infection

• Produced from bone marrow stem cells

• Characterised by their differential expression of CD (cluster of


differentiation) antigens

• Following stimulation by antigen, some lymphocytes differentiate into


memory cells which are capable of mounting a rapid response if the
same antigen is later encountered

• Most lymphocytes are located in secondary lymphoid tissues - lymph


nodes, the white pulp of the spleen, the gut- and bronchial-associated
lymphoid tissues.

• The part of the B cell receptor that binds antigen is surface


immunoglobulin*

• Subsequent B cell proliferation and maturation generates plasma cells


that secrete this immunoglobulin *

• In contrast to B cell receptors, T cell receptors do not bind free antigen.


The antigen must be processed into small peptides by antigen
presenting cells and these antigen derived peptides are displayed on
the antigen presenting cell surface in the clefts of molecules called
major histocompatibility molecules (MHC) *

• There are three main types of T cells *

1) T helper cells that facilitate B cell responses to antigen

2) T inflammatory cells that facilitate macrophages in the eradication of intracellular


infection

3) Cytotoxic T cells that recognise and destroy virus infected cells.

B-LYMPHOCYTES *****
• Mature in the bone marrow. Develop into antibody producing
plasma cells. Only immune cells capable of antibody production *

• Have membrane-bound immunoglobulins which act as antigen


receptors*

• Plasma cells migrate to the bone marrow where they produce


small quantities of antibodies for long periods of time after the initial
infection has been cleared

• B-cell receptors bind soluble antigens

• The bound antigen molecules are engulfed into the B cell by


receptor-mediated endocytosis. *

• The antigen is digested into fragments which are then displayed at


the cell surface nestled inside a class II histocompatibility molecule. *

• Helper T cells with complementary TCRs bind the B cell and


secrete lymphokines that *

1) Stimulate the B cell to enter the cell cycle and develop, by repeated mitosis, into
aclone of cells with identical BCRs;

2) Switch from synthesizing their BCRs as integral membrane proteins to a soluble


version;

3) Differentiate into plasma cells that secrete immunoglobulins.

T-LYMPHOCYTES *****

• Mature in the thymus gland *

• Can only recognise peptide antigens that are bound to MHC molecules
on specialised antigen-presenting cells - MHC-restricted *

• CD4 positive T-cells are helper T-cells - help B cells produce


antibodies and help phagocytes destroy ingested organisms *

• CD8 positive T-cells are cytotoxic T-cells - capable of killing host cells
infected with intracellular organisms *

• Nagative T-cells have antigen receptors but do not perform the


functions required to eliminate the antigen
CD8+ T cells

• CD8+ T cells bind that are part of class I histocompatibility


molecules. Almost all the cells of the body express class I molecules

• The best understood CD8+ T cells are cytotoxic T lymphocytes -


secrete molecules that destroy the cell to which they have bound.

• The role of the CD8+ T cells is to monitor all the cells of the body,
ready to destroy any that express foreign antigen fragments in their
class I molecules.

CD4+ T cells

• CD4+ T cells bind epitopes that are part of class II histocompatibility


molecules. Only specialized antigen-presenting cells express class II
molecules

• Essential for both the cell mediated and antibody-mediated branches of


the immune system

• Bind to antigen presented by antigen-presenting cells and


releaselymphokines that attract other cells to the area.

• Bind to antigen presented by B cells, resulting in the development


of clonesof plasma cells secreting antibodies against the antigenic
material.

ANTIGEN PRESENTING CELLS *****

• All nucleated cells can present antigens derived from microbes in the
cytoplasm to cytotoxic T-cells *

• Professional antigen-presenting cells include dendritic cells


(Langerhans cells are epidermal dendritic cells) and macrophages *

• B-cells ingest protein antigens and present them to helper T-cells *

Langerhans cells

• Professional antigen presenting cells

• Found in all tissues.

• Can be activated by local tissue injury


• This activation process induces Langerhans cells to migrate to
secondary lymphoid tissues where they present to T lymphocytes
antigen derived peptide/MHC complexes.

LYMPH NODES *****

• Capsulated structures which drain lymphatics

• Have a cortex and a medulla with sinuses

• Lymph drains from the efferent lymphatics into the sub-capsular sinus
and then through the medullary sinuses into the efferent lymph vessels

• B-cells are concentrated in the cortex where they form follicles. Un-
stimulated B-cells form primary follicles while secondary follicles are
formed following antigenic stimulation. Secondary follicles have a
germinal centre which is the site of B-cell maturation and the
generation of B-cell memory

• T-cells are concentrated in the para-cortical area

• Na-ve T-cells leave the circulation to enter the lymph nodes through
specialised post-capillary venules called high endothelial venules

SPLEEN ***

• Has a capsule, red pulp and white pulp

• The red pulp is the red cell area - the spleenic capillaries are very
porous and red cells leave the capillaries to enter the red pulp. They
then squeeze through the trabeculae which are lined by a vast number
of macrophages to re-enter the circulation at the venous end

• The white pulp contains follicles (B-cell area) and peri-arteriolar


lymphoid sheaths (T-cell area)

• Old red cells and platelets are removed from the circulation in the
spleen
Immune cells & distribution *****
Cell type Number/litre in blood Life-span after Distribution
of adults leaving bone
marrow

Basophils / trace numbers ~30 days Mainly in tissues

Mast cells

Neutrophils 2-8x109/l 5-7 hr in blood Blood & tissues

Eosinophils 0.04-0.4x109/l 14-18 hr in blood blood and tissues


Monocytes/ 0.2-0.8x109/l Days to weeks Most are in tissues -
spleen, liver and
macrophages lungs

Langerhans 5-14 days Mostly in tissues


cells and T zones of
secondary lymphoid
tissues

Blood spleen and


NK cells sites of
9
~0.2-1.0x10 /l inflammation

T 0.04-2.5x109/l 1 day - several years Most T cells are in


lymphocytes secondary lymphoid
tissues or epithelia,

<5% are in the


blood

B 0.03-0.7x109/l 2 days to several Most B cells are in


lymphocytes weeks; memory secondary lymphoid
clones up to years tissues<5% are in the
blood
THE MAJOR HISTOCOMPATIBILITY COMPLEX

• Made up of membrane bound proteins on antigen presenting cells that display peptide antigens
for recognition by T-cells *

• Play a central role in graft rejection *

• Human MHC molecules are called human leukocyte antigens (HLA) *

• MHC genes are highly polymorphic - there are many different alleles present in different
individuals. No two individuals in an outbred population have identical MHC genes or
molecules (except identical twins) *

• Human MHC located on chromosome 6 *

• Variations in MHC genes are not due to DNA recombinations

• There are two sets of highly polymorphic HLA genes called class I and class II genes

• The MHC also contains several non-polymorphic genes, some involved in antigen presentation
and others with unknown functions

• Inherited in an autosomal co-dominant fashion *

• Class I molecules are expressed by all nucleated cells while class II molecules are expressed
by professional antigen-presenting cells only (dendritic cells, macrophages and B-
lymphocytes) *

• Each MHC molecule can only present one antigen at a time, but is capable of presenting many
different peptides. The binding of peptides to MHC molecules is a low affinity, low specificity
interaction. Bound peptides are displayed for several days to enable recognition by T-cells

• Antigenic peptides are acquired during the synthesis and assembly of MHC molecules in the
cytoplasm - class I molecules display peptides acquired from cytosolic proteins while class II
molecules acquire peptides from intracellular vesicles

• MHC molecules cannot discriminate between self and non-self peptides


• MHC molecules cannot present non-peptide antigens

MHC CLASS I *****

• Three polymorphic class I genes: HLA-A, HLA-B and HLA-C *

• Each person inherits one set from each parent, therefore each cell can express 6 different
class I molecules *

• Each class I molecule is made up of an alpha chain non-covalently attached to beta-2-


microglobulin *

• The alpha chain has three domains: the alpha1 and alpha2 domains are highly polymorphic
and surround a peptide binding cleft which is capable of binding peptides with 8-11 amino acids

• The alpha3 domain is invariant and contains the binding site for the T-cell receptor CD8

• CD8 positive T cells can therefore only respond to peptide antigens displayed by class I MHC
molecules

MHC CLASS II *****

• Three polymorphic genes: HLA-DR, HLA-DQ, HLA-DP *

• Each person inherits one set from each parent, therefore each cell can express 6 different
class II molecules *

• Each class II molecule is made up of an alpha and a beta chain *

• The peptide binding site for class II molecules can bind peptides with 10-30 amino acids
• The beta2 domain on the beta chain of class II molecules is invariant and contains the binding
site for the T-cell receptor CD4

• CD4 positive cells can therefore only recognise peptide antigens displayed by MHC class II
molecules
Question 1: With respect to lymphocyte maturation and the generation of diversity

a. B-cell maturation is completed in the bone marrow

True False
b. T-cell maturation occurs in the thymus gland

True False
c. Proliferation of lymphocyte precursors is under the stimulation of TNF-alpha

True False
d. Positive selection occurs during B-cell maturation

True False

Question 2: With respect to lymphocytes

a. CD8 positive T-lymphocytes kill virally infected host cells

True False
b. T-lymphocytes mature in the thymus gland

True False
c. Na?ve lymphocytes do not express antigen receptors

True False
d. T-cells are MHC-restricted

True False

Question 3: With respect to TH1 and TH2 lymphocytes

a. TH1 cells stimulate phagocyte-mediated killing of microbes

True False
b. TH1 cells stimulate the production of IgE antibodies

True False
c. TH2 cells are CD8 positive

True False
d. TH1 cells stimulate phagocyte-mediated killing of microbes

True False

Question 4: With respect to TH1 and TH2 lymphocytes

a. TH2 cells stimulate immune responses against helminthic parasites

True False
b. Cytokines secreted by TH2 cells activate macrophages

True False
c. Interleukin-12 stimulates the differentiation of helper T-cells into TH2 cells

True False
d. TH2 cells secrete interleukin-4

True False

Question 5: Lymphocytes

a. Are the only immune cells with specific receptors for antigens

True False
b. Make up 5% of the total leukocyte count

True False
c. Count is increased in viral infections

True False
d. Count is increased following corticosteroid therapy

True False

Question 6: With respect to the major histocompatibility (MHC) antigens

a. Class I & II MHC molecules are membrane-bound proteins

True False
b. Class I & II MHC molecules contain a peptide-binding domain at the amino-terminal end

True False
c. MHC antigens are inherited in an autosomal dominant fashion

True False
d. Class I MHC antigens are expressed by professional antigen-presenting cells only

True False

Question 7: The T-cell receptor

a. Does not recognise polysaccharide antigens

True False
b. Does not recognise lipid antigens

True False
c. Only recognises antigens displayed by MHC molecules

True False
d. Recognises the three dimensional conformation of antigens

True False

Question 8: With respect to the major histocompatibility (MHC) antigens

a. Class II MHC antigens are expressed by all nucleated cells

True False
b. Class II molecules are expressed by B-lymphocytes

True False
c. Class II molecules are not expressed by macrophages

True False
d. Class I but not class II MHC molecules have polymorphic genes

True False

Question 9: With respect to the processing and presentation of peptide antigens

a. Antigens within the cytosol are displayed by class I MHC molecules only

True False
b. Antigens within phagocytic vesicles are processed and presented by both MHC class I and
class II molecules

True False
c. Only foreign cytosolic proteins are displayed on MHC molecules
True False
d. The peptide binding cleft of newly synthesised MHC class II molecules contains the class II
invariant chain peptide

True False

Question 10: With respect to the major histocompatibility (MHC) antigens

a. MHC molecules acquire antigenic peptides during their biosynthesis within the cell

True False
b. Class I MHC molecules acquire antigenic peptides from proteins in intracellular vesicles

True False
c. Class II MHC molecules acquire antigenic peptides from cytosolic proteins

True False
d. MHC molecules cannot discriminate between foreign and self antigens

True False

Question 11: With respect to the structure of the spleen

a. T-cells are located in the peri-arteriolar lymphoid sheaths

True False
b. B-cells are located in the red pulp

True False
c. The capillaries are lined by endothelial cells with tight junctions

True False
d. The trabeculae of the red pulp contain macrophages

True False

Question 12: With respect to lymphocytes

a. CD4 positive T-lymphocytes are cytotoxic T-cells

True False
b. Helper T-cells help B-lymphocytes to produce antibodies

True False
c. Helper T-cells help phagocytes to destroy ingested microbes

True False
d. CD8 positive T-lymphocytes are helper T-cells

True False

Question 13: With respect to the major histocompatibility (MHC) antigens

a. CD4 positive T-cells can only respond to antigens presented by class II MHC molecules

True False
b. There are three polymorphic MHC class I genes in humans

True False
c. Each human cell expresses three different MHC class I molecules

True False
d. Humans have four polymorphic MHC class II genes
True False

Question 14: With respect to TH1 and TH2 lymphocytes

a. TH1 cells are CD4 positive

True False
b. TH2 cells are CD8 positive

True False
c. TH1 cells produce interferon-gamma

True False
d. TH2 cells produce mainly interleukin-12

True False

Question 15: Antigen-presenting cells include

a. Dendritic cells

True False
b. Macrophages

True False
c. Langerhans cells

True False
d. B-cells

True False
GRAFT REJECTION *****

HYPERACUTE REJECTION

• Occurs within minutes of transplantation

• Characterised by thrombosis of graft vessels and ischaemic necrosis of the graft

• Antibody mediated response caused by pre-existing antibodies that may have developed as a
result of previous blood transfusion - therefore more likely to occur in those with previous
transfusion

• Uncommon problem as donor and recipient are cross-matched

• Corneal and cartilage transplants are not rejected as these are avascular structures and
therefore do not sensitize the recipient

ACUTE REJECTION

• Occurs within 10 days (early) or after 10 days (late)

• Early acute rejection is a cell mediated hypersensitivity reaction (T-cell) although antibody
mediated hypersensitivity also contributes

• Late acute rejection may be a result of break-through of immuno-suppression and is


characterised by the deposition of complement and immunoglobulins in arterioles and
capillaries

• Preventable by immuno-suppressive therapy

CHRONIC TRANSPLANT REJECTION

• Occurs over several months / years and manifests as a gradual loss of graft function

• Characterised by fibrosis and narrowing of graft vessels (graft arteriosclerosis)

• T-cell mediated response with the production of cytokines which induce fibrosis and
proliferation of vascular smooth muscle

GRAFT VERSUS HOST REACTION


• Occurs when competent T-cells are transplanted from a donor to a recipient who is unable to
reject them. The cells survive, recognise the host's antigens and mount an immunological
reaction

• May follow bone marrow transplant or occur as a result of T-cells present in transplanted
organs

• Causes fever, haemolytic anaemia, splenomegaly, weight loss, rash, diarrhoea

The reaction is greater the greater the HLA difference between the host and the donor.
May be fatal
IMMEDIATE (TYPE I) HYPERSENSITIVITY *****

• Rapid IgE and mast cell dependent vascular and smooth muscle reaction that is usually
followed by an inflammatory reaction in an individual who has been previously exposed to an
antigen (sensitized) *

• Antigen binds to IgE on mast cells, cross-linking receptors and inducing mast cell
degranulation *

• Released mast cell granules contain histamine, heparin, neutral proteases and cytokines
typical of TH2 responses - IL-3 and IL-4 *

• Cyclo-oxygenase and lipoxygenase are also activated with the production of vasodilator
prostaglandins, leukotrienes and thromboxanes *

• In the skin is characterised by a wheal and flare reaction *

• The result is increased capillary permeability, smooth muscle contraction and this manifests as
wheezing, oedema, hypotension - this is the acute phase and occurs within minutes *

• The latent phase peaks after 3-5 hours and is caused by cytokine mediated inflammatory
infiltration by neutrophils and eosinophils - this is responsible for tissue damage that occurs
after repeated episodes of immediate hypersensitivity *

• Typical disease processes include asthma, hay fever, drug and insect bite induced
anaphylaxis, penicillin allergy, food allergy, atopic dermatitis *

TYPE II (ANTIBODY MEDIATED CYTOTOXIC)


HYPERSENSITIVITY *****

• Caused by antibodies binding to cells or tissues and initiating damage by opsonisation or


activation of inflammatory cells -typically IgG. Tissue damage is caused by phagocytic and
non-phagocytic cells *

• Antibodies may be allo-immune (for instance in transfusion reactions) or auto-immune


(myasthenia gravis) *

• Disorders include *

1) Transfusion reaction and Rhesus disease

2) Auto-immune haemolytic anaemia

3) Auto-immune thrombocytopaenia purpura

4) Myasthenia gravis, Graves- and Hashimoto's disease


5) Acute rheumatic fever - anti-streptococcal antibodies cross-react with
myocardial antigen causing myocarditis and arthritis

6) Pernicious anaemia

7) Drug-induced haemolytic anaemia

8) Goodpasture's syndrome -deposition of antibodies and complement to


glomerular and lung basement membrane

IMMUNE COMPLEX MEDIATED (TYPE III) HYPERSENSITIVITY *****

• Caused by immune complex deposition in the blood vessels or tissues *

• These complexes activate macrophages, activate complement and cause the extracellular
release of neutrophil granules and tissue damage *

• The inflammatory response is characterised by the Arthus reaction which is characterised by a


neutrophilic infiltration and immunoglobulin + complement deposition *

• The inciting antigen may be -self- or -non-self- from infectious organisms

• Disease processes include *

1) Polyarteritis nodosa and SLE

2) Farmer's lung - occurring in patients sensitized to thermophilic actinomycetes;


pigeon-fancier's disease - these disorders are generally known as extrinsic
allergic alveolitis

3) Post-streptococcal glomerulonephritis

4) Jarisch-Herxheimer reaction

DELAYED CELL MEDIATED (TYPE IV) HYPERSENSITIVITY *****

• Occurs in allergic reactions to bacteria, fungi and viruses, contact dermatitis and in the
rejection of transplanted tissues *

• Typically occurs in the Mantoux reaction to tuberculin injected in the skin of a sensitized
individual - characterised by erythema and induration *

• Reaction occurs after several hours and peaks 24-48h after contact *
• The early phase is characterised by a perivascular cuffing with mononuclear cells, and then
polymorphs which later leave. The late phase is characterised by a lymphocyte and monocyte-
macrophage infiltration *

• T-lymphocyte dependent, and TH1 dependent in most cases, resulting in activation of


macrophages. TH2 responses do however occur, for instance in the hepatic lesions in
schistosomiasis *

• Antigen interacts with primed T-cells with tissue damage resulting from inappropriate cell
mediated immune reactions

• The chronic inflammatory response leads to the formation of granulomas, which may be
caseating in the case of TB *

Occurs in Crohn's disease, psoriasis, celiac disease, TB, leprosy, contact dermatitis,
tissue damage following measles and herpes infection; schistosomiasis *
PASSIVE IMMUNOTHERAPY *****

• The use of immunoglobulins to treat or prevent disease. Antibodies may be from human or
animal sources or may be genetically engineered

• Does not confer long term immunity

• Associated with serum sickness especially when antibodies are from an animal source

• Antibodies are used to treat:

1) Tetanus and diphtheria

2) Botulism, snake and scorpion bites

3) Hepatitis B

4) Auto-immune disorders including immune thrombocytopaenia

NEONATAL PASSIVE IMMUNITY

• Maternal IgG is actively transported by the placenta into the fetus. IgA is not taken up by the
placenta *

• Secreted IgA in breast milk is not absorbed but confers mucosal immunity in the GI tract *

VACCINATION *****

Types of Vaccines
• Killed organisms

• Live-attenuated organisms replicate in the host. Provide a larger –does- and have the
advantage of inducing immunity at the natural site of infection

• Sub-unit vaccines - make use of the most antigenic subunit of the organism. These can be
produced by genetic engineering

• Toxoid - bacterial exotoxins are first detoxified by treatment with formaldehyde

BACTERIAL

Live Attenuated

• BCG - tuberculosis

Inactivated

• Cholera vaccine

• Pertussis vaccine

• Typhoid vaccine

Sub-unit

• H. influenzae

• N. meningitides
• S. pneumoniae

Toxoid

• Tetanus

• Diphtheria

VIRUSES

Live Attenuated

• MMR - measles, mumps, rubella

• Polio (Sabin)

• Varicella zoster - not in general use

• Adenovirus

• Yellow fever
Inactivated

• Polio (Salk)

• Influenza

• Rabies

• Hepatitis A

Sub-unit

• Hepatitis B

• Influenza
Question 1: Hyperacute transplant rejection

a. Is mediated by T-lymphocytes

True False
b. Is a TH1 response

True False
c. Is more likely to occur in patients who have never had a previous blood transfusion

True False
d. Is the commonest type of transplant rejection

True False

Question 2: The following vaccines utilise inactivated microbes

a. Tetanus vaccine

True False
b. Diphtheria vaccine

True False
c. Mumps vaccine

True False
d. Hepatitis A vaccine

True False

Question 3: Immediate (type I) hypersensitivity is associated with

a. Increased capillary permeability

True False
b. Smooth muscle relaxation

True False
c. Hypertension

True False
d. Oedema

True False

Question 4: The following are live attenuated vaccines

a. Sabin polio vaccine

True False
b. Salk polio vaccine

True False
c. Hepatitis B vaccine

True False
d. Rubella vaccine

True False

Question 5: The graft-versus-host reaction

a. May occur when an immuno-suppressed individual receives a bone marrow transplant

True False
b. Only occurs following bone marrow transplant

True False
c. Occurs when transplanted leukocytes are rejected by the host

True False
d. Is of greater severity if the donor and the recipient have a greater HLA difference

True False

Question 6: The following are characteristic of graft-versus-host disease

a. Rash

True False
b. Diarrhoea

True False
c. Weight loss

True False
d. An immuno-competent transplant recipient

True False

Question 7: The following are typically released following mast cell degranulation

a. Histamine

True False
b. Vasoconstrictor prostaglandins

True False
c. Neutral proteases

True False
d. Leukotrienes

True False

Question 8: Immediate (type I) hypersensitivity occurs in the following disorders

a. Immune thrombocytopaenia purpura

True False
b. Extrinsic allergic alveolitis

True False
c. Goodpasture?s syndrome

True False
d. Drug-induced haemolytic anaemia

True False

Question 9: Neonatal passive immunity is acquired by

a. The trans-placental transfer of IgG

True False
b. The trans-placental transfer of IgA

True False
c. The uptake of IgA from the gut
True False
d. The uptake of IgG from the gut

True False

Question 10: The following are characteristic of graft-versus-host disease

a. Rejection of transplanted organ

True False
b. Fever

True False
c. Haemolytic anaemia

True False
d. Splenomegaly

True False

Question 11: Type 1 (immediate) hypersensitivity

a. Is mast cell dependent

True False
b. Is IgD dependent

True False
c. Occurs when an individual encounters an antigen for the first time

True False
d. Has an early and a late phase

True False

Question 12: The following disorders are currently treated by passive immunotherapy

a. Tetanus

True False
b. Diphtheria

True False
c. Botulism

True False
d. Hepatitis B

True False

Question 13: The following diseases are more common in females

a. Systemic lupus erythematosus

True False
b. Scleroderma

True False
c. Auto-immune thrombocytopaenia

True False
d. Graves disease
True False

Question 14: Antibody dependent (type II) hypersensitivity

a. Causes post-streptococcal glomerulonephritis

True False
b. Occurs in drug-induced haemolytic anaemia

True False
c. Is characterised by the Arthus reaction

True False
d. Occurs in myasthenia gravis

True False

Question 15: The following are sub-unit vaccines

a. Rubella vaccine

True False
b. Yellow fever vaccine

True False
c. Polio vaccine

True False
d. Adenovirus vaccine

True False
40 random questions for EMQ Mock Exam

Options for Questions 1-2

A Within 24h of fertilization B Day 25


C Day 4 D Day 27
E 6th week F 4th ? 5th week
G 10th week H 12th week
I 7th ? 8th week J 8th ? 12th week

Instructions: For each of the events below, choose the single most appropriate time period from the
above list (list indicates time after fertilisation). Each option may be used once, more than once or not at
all

Question 1 Closure of the anterior neuropore

Question 2 Closure of the posterior neuropore

Options for Questions 3-3

A Promethazine B Ranitidine
C Sodium chromoglycate D Methyldopa
E Chlorpheniramine F Cimetidine
G Cyproterone acetate H Chlorpromazine
I Omeprazole J Tubocurarine

Instructions: Histamine antagonists are widely used therapeutically. For each of the questions below,
choose the singlemost appropriate drug from the above list. Each option may be used once, more than
once or not at all

Question 3 Which drug induces histamine release from mast cells?

Options for Questions 4-5

A Rubella B Influenza A
C Human papilloma virus type 16 D Parvovirus B19
E Coxsackie B virus F Hepatitis A
G Hepatitis B H HIV
I Hepatitis C J Hepatitis E

Instructions: Infections in pregnancy may be associated with fetal / neonatal adverse effects. For each
of the scenarios below, choose the single most appropriate micro-organism from the above list. Each
option may be used once, more than once or not at all.

An RNA virus spread by sexual intercourse or blood


Question 4
products. Associated with a 50% risk of chronic liver disease

An RNA virus spread by the feco-oral route, causes more


Question 5 severe disease in pregnancy but is not associated with
chronic liver disease
Options for Questions 6-7

A Rubella B Influenza A
C Cytomegalovirus D Parvovirus B19
E Coxsackie B virus F Hepatitis A
G Hepatitis B H HIV
I Toxoplasmosis J Varicella zoster

Instructions: Infections in pregnancy may be associated with fetal adverse effects. For each of the
questions below, choose the single most appropriate micro-organism from the above list. Each option
may be used once, more than once or not at all.

Which virus is the commonest cause of congenital infection in


Question 6
the UK?

Which virus is associated with a 2% risk of congenital


Question 7
anomalies if infection occurs before 20 weeks gestation?

Options for Questions 8-9

A Staphylococcus aereus B Actinomycete israelii


C Neisseria Gonorrhoea D Chlamydia trachomatis
E Schistosoma haematobium F Pseudomonas aeruginosa
G E. coli H Pseudomonas aeruginosa
I Bacteriodes J Mycobacterium tuberculosis

Instructions: For each of the scenarios below, choose the single most appropriate micro-organism
from the above list. Each option may be used once, more than once or not at all.

Question 8 Oxidase negative gram negative bacillus

Question 9 Strictly aerobic gram negative bacillus

Options for Questions 10-11

A Inferior rectal nerve B Celiac plexus


C Pudendal nerve D Perineal branch of the pudendal nerve
E S2/3/4 sacral nerves F L1/2 lumbar nerves
G L2/3/4 lumbar nerves H Aortic plexus
I Superior mesenteric plexus J Inferior hypogastric plexus

Instructions: For each of the organs / structures listed below, select the single most appropriate nerve
supply from the above list. Each option may be used once, more than once or not at all

Question 10 The rectum

Question 11 The ovary

Options for Questions 12-13


A 7th ? 9th month B Day 25
C Day 4 D Day 27
E 6th week F 4th ? 5th week
G 10th week H 5th month
I 7th ? 8th week J 8th ? 12th week

Instructions: For each of the events below, choose the single most appropriate time period from the
above list (list indicates time after fertilisation). Each option may be used once, more than once or not at
all

Question 12 Descent of the testes into the scrotum

The number of oogonia in the developing ovary is at its


Question 13
maximum

Options for Questions 14-15

A Benzyl penicillin B Carbenicillin


C Metronidazole D Teicoplanin
E Flucloxacillin F Ampicillin
G Spiramycin H Isoniazid
I Doxycycline J Azythromycin

Instructions: For each of the scenarios below, choose the single most appropriate antibiotic from the
above list. Each option may be used once, more than once or not at all.

A 25 year old woman complains of vaginal discharge and a


Question 14
high vaginal swab is positive for Bacteriodes fragilis

A 25 year old woman develops a wound infection 3 days after


Question 15
elective caesarean section

Options for Questions 16-17

A Cimetidine B Ranitidine
C Sodium chromoglycate D Tubocurarine
E Chlorpheniramine F Atropine
G Cyproterone acetate H Chlorpromazine
I Lanzoprazole J Omeprazole

Instructions: Histamine antagonists are widely used therapeutically. For each of the questions below,
choose the singlemost appropriate drug from the above list. Each option may be used once, more than
once or not at all

Question 16 Which drug is a histamine H1 antagonist?

Which histamine antagonist is a recognised cause of


Question 17
gynaecomastia?

Options for Questions 18-18

A Rubella B Influenza A
C Human papilloma virus type 16 D Parvovirus B19
E Coxsackie B virus F Hepatitis A
G Hepatitis B H HIV
I Toxoplasmosis J Varicella zoster

Instructions: Infections in pregnancy may be associated with fetal adverse effects. For each of the
questions below, choose the single most appropriate micro-organism from the above list. Each option
may be used once, more than once or not at all.

Which virus is a DNA virus and a member of the Herpes


Question 18
family of viruses?

Options for Questions 19-20

A Transitional epithelium B Stratified squamous keratinising epithelium


C Stratified squamous non-keratinising epithelium D Columnar epithelium
E Pseudo-columnar epithelium F Ciliated columnar epithelium
G Simple cuboidal epithelium H Simple stratified epithelium
I Stratified cuboidal epithelium J Simple squamous epithelium

Instructions: For each of the structures / organs below, choose the single most appropriate type of
epithelium from the above list. Each option may be used once, more than once or not at all

Question 19 The ectocervix

Question 20 The endocervix

Options for Questions 21-22

A DNA polymerase alpha B DNA polymerase beta


C DNA polymerase delta D DNA polymerase gamma
E DNA helicase F DNA ligase
G RNA polymerase H RNAase
I Telomerase J Okazaki fragments

Instructions: Nucleic acid replication is vital for cell division. For each of the questions below, choose
the single most appropriate option from the above list. Each option may be used once, more than once
or not at all.

Which enzyme unwinds the DNA double helix forming


Question 21
replication units?

Question 22 Which enzyme synthesizes the leading strand of DNA?

Options for Questions 23-24

A Marfan?s syndrome B Hereditary spherocytosis


C Haemophilia B D Haemophilia A
E Hypertrophic cardiomyopathy F Galactosaemia
G Myotonic dystrophy H Von Willebrand disease
I Alpha-1 anti-trypsin deficiency J Congenital adrenal hyperplasia
Instructions: A 20 year old woman has been referred for pre-natal diagnosis at 11 weeks gestation. For
each of the scenarios below, choose the single most appropriate disorder from the above list. Each
option may be used once, more than once or not at all.

The woman is affected by an autosomal dominant condition


Question 23
associated with aortic aneurysms and lens subluxation

Question 24 The woman has an autosomal dominant bleeding disorder

Options for Questions 25-26

A Pituitary gland B Anterior pituitary gland


C Posterior pituitary gland D Hypothalamus
E Zona glomerulosa of the adrenal cortex F Zona fasciculata of the adrenal cortex
G Zona reticularis of the adrenal cortex H Thyroid gland
I Ovary J Islets of Langerhans

Instructions: For each of the hormones listed below, select the single most appropriate site of
synthesis from the above list. Each option may be used once, more than once or not at all.

Question 25 Adrenocorticotropic hormone

Question 26 Thyrotropin releasing hormone

Options for Questions 27-28

A Leutenising Hormone B Follicle stimulating hormone


C Inhibin D Activin
E Gonadotropin releasing hormone F Leptin
G Neuropeptide Y H Human chorionic gonadotrophin
I Dehydro-epiandrosterone sulphate (DHEA-S) J Human placental lactogen

Instructions: For each of the scenarios below, select the single most appropriate hormone from the
above list. Each option may be used once, more than once or not at all.

Glycoprotein hormone with alpha & beta sub-units, secreted


Question 27 by the anterior pituitary and stimulates granulosa cell
proliferation

Decapeptide hormone stimulates FSH and LH production by


Question 28
the anterior pituitary gland

Options for Questions 29-30

A Leutenising Hormone B Follicle stimulating hormone


C Inhibin D Activin
E Gonadotropin releasing hormone F Leptin
G Neuropeptide Y H Human chorionic gonadotrophin
I Dehydro-epiandrosterone sulphate (DHEA-S) J Human placental lactogen
Instructions: For each of the scenarios below, select the single most appropriate hormone from the
above list. Each option may be used once, more than once or not at all.

Glycoprotein hormone with alpha & beta sub-units, secreted


Question 29 by the anterior pituitary and stimulates progesterone
secretion and conversion of granulosa into lutein cells

Question 30 Ovarian protein hormone, inhibits pituitary FSH production

Options for Questions 31-32

A Interferons B IgA
C IgG D Immunoglobulins
E Cytokines F IgE
G Prostaglandins H Interleukins
I IgM J IgD

Instructions: For each of the statements below, choose the single most appropriate option from the
above list. Each option may be used once, more than once or not at all.

Question 31 Immunoglobulin present in secretions

Question 32 Pentameric immunoglobulin

Options for Questions 33-34

A Transitional epithelium B Stratified squamous keratinising epithelium


C Stratified squamous non-keratinising epithelium D Columnar epithelium
E Pseudo-columnar epithelium F Ciliated columnar epithelium
G Simple cuboidal epithelium H Simple stratified epithelium
I Stratified cuboidal epithelium J Simple squamous epithelium

Instructions: For each of the structures / organs below, choose the single most appropriate type of
epithelium from the above list. Each option may be used once, more than once or not at all

Question 33 Fallopian tube

Question 34 Urinary bladder

Options for Questions 35-36

A Staphylococcus aereus B Actinomycete israelii


C Neisseria Gonorrhoea D Chlamydia trachomatis
E Schistosoma haematobium F Pseudomonas aeruginosa
G E. coli H Pseudomonas aeruginosa
I Bacteriodes J Mycobacterium tuberculosis

Instructions: For each of the scenarios below, choose the single most appropriate micro-organism
from the above list. Each option may be used once, more than once or not at all.
Question 35 Coagulase positive gram positive bacteria

Obligate intra-cellular organism which does not infect


Question 36 squamous cells. Diagnosis confirmed by PCR of early
morning urine specimen

Options for Questions 37-38

A Uterine artery B Superior mesenteric artery


C Cervical branch of the uterine artery D Superior rectal artery
E Middle rectal artery F Inferior rectal artery
G Internal pudendal artery H Anterior division of the uterine artery
I Ovarian artery J Internal iliac artery

Instructions: A 30 year old woman suffered post-partum haemorrhage of 1,500ml following forceps
delivery for delay in the second stage. For each of the questions below, select the single most
appropriate option from the above list. Each option may be used once, more than once or not at all

Question 37 Which vessel supplies the clitoris?

Which vessel supplies the muscle of the superficial & deep


Question 38
perineal pouches?

Options for Questions 39-40

A Leutenising Hormone B Follicle stimulating hormone


C Inhibin D Activin
E Gonadotropin releasing hormone F Leptin
G Neuropeptide Y H Human chorionic gonadotrophin
I Dehydro-epiandrosterone sulphate (DHEA-S) J Human placental lactogen

Instructions: For each of the scenarios below, select the single most appropriate hormone from the
above list. Each option may be used once, more than once or not at all.

Ovarian peptide hormone, increases FSH binding to


Question 39
granulosa cells thereby augmenting FSH stimulation

Question 40 Polypeptide hormone produced mainly by adipose tissue


40 random questions for EMQ Test

Options for Questions 1-2

A Staphylococcus aereus B Actinomycete israelii


C Neisseria Gonorrhoea D Chlamydia trachomatis
E Schistosoma haematobium F Pseudomonas aeruginosa
G E. coli H Pseudomonas aeruginosa
I Bacteriodes J Mycobacterium tuberculosis

Instructions: For each of the scenarios below, choose the single most appropriate micro-organism
from the above list. Each option may be used once, more than once or not at all.

Question 1 Oxidase negative gram negative bacillus

Question 2 Strictly aerobic gram negative bacillus

Options for Questions 3-4

A Benzyl penicillin B Carbenicillin


C Metronidazole D Teicoplanin
E Flucloxacillin F Ampicillin
G Spiramycin H Isoniazid
I Doxycycline J Azythromycin

Instructions: For each of the scenarios below, choose the single most appropriate antibiotic from the
above list. Each option may be used once, more than once or not at all.

A 37 year old woman with a vaginal discharge has a high


Question 3
vaginal swab which is positive for Actinomycete isrealii

A 20 year old woman has a septic miscarriage at 16 weeks


Question 4
gestation. Blood cultures are positive Listeria monocytogenes

Options for Questions 5-6

A Benzyl penicillin B Carbenicillin


C Metronidazole D Teicoplanin
E Flucloxacillin F Ampicillin
G Pyramethamine H Rifampicin
I Doxycycline J Ciprofloxacin

Instructions: For each of the scenarios below, choose the single most appropriate antibiotic from the
above list. Each option may be used once, more than once or not at all.

A 32 year old woman requesting termination of pregnancy


Question 5 has an early morning urine specimen which is positive for
Chlamydia trachomatis

An 18 year old woman with a vaginal discharge has cervical


Question 6 and urethral swabs which show gram negative intra-cellular
diplococci
Options for Questions 7-8

A Interferons B Endotoxins
C Exotoxins D Immunoglobulins
E Cytokines F Histamine
G Prostaglandins H Interleukins
I Tumour necrosis factor J Lipopolysaccharide

Instructions: For each of the statements below, choose the single most appropriate option from the
above list. Each option may be used once, more than once or not at all.

Are typically produced by gram positive bacteria and


Question 7
destroyed by heat

Question 8 Toxic lipopolysaccharide molecules

Options for Questions 9-10

A Rubella B Influenza A
C Human papilloma virus type 16 D Parvovirus B19
E Coxsackie B virus F Hepatitis A
G Hepatitis B H HIV
I Hepatitis C J Hepatitis E

Instructions: Infections in pregnancy may be associated with fetal / neonatal adverse effects. For each
of the scenarios below, choose the single most appropriate micro-organism from the above list. Each
option may be used once, more than once or not at all.

An RNA virus spread by sexual intercourse or blood


Question 9
products. Associated with a 50% risk of chronic liver disease

An RNA virus spread by the feco-oral route, causes more


Question 10 severe disease in pregnancy but is not associated with
chronic liver disease

Options for Questions 11-12

A Immediate (Type I) hypersensitivity B Antibody mediated (Type II) hypersensitivity


Immune complex mediated (Type III) Delayed cell mediated (type IV)
C D
hypersensitivity hypersensitivity
E Graft versus host disease F Acute graft rejection
G Hyperacute graft rejection H Chronic graft rejection
I Allergic Reaction J Jerisch-Herxheimer reaction

Instructions: For each of the scenarios below, choose the single most appropriate option from the
above list. Each option may be used once, more than once or not at all.

IgE and mast cell dependent vascular and smooth muscle


Question 11
reaction in a sensitised individual

Mast cell degranulation typically occurs during this immune


Question 12
response
Options for Questions 13-14

A Benzyl penicillin B Carbenicillin


C Metronidazole D Teicoplanin
E Flucloxacillin F Ampicillin
G Spiramycin H Isoniazid
I Doxycycline J Azythromycin

Instructions: For each of the scenarios below, choose the single most appropriate antibiotic from the
above list. Each option may be used once, more than once or not at all.

A 47 year old woman develops MRSA infection following


Question 13
abdominal hysterectomy for cervical cancer

A 37 year old woman is found to have a urinary tract infection


Question 14
with Pseudomonas aeruginosa

Options for Questions 15-16

A Rubella B Influenza A
C Cytomegalovirus D Parvovirus B19
E Coxsackie B virus F Hepatitis A
G Hepatitis B H HIV
I Toxoplasmosis J Varicella zoster

Instructions: Infections in pregnancy may be associated with fetal adverse effects. For each of the
questions below, choose the single most appropriate micro-organism from the above list. Each option
may be used once, more than once or not at all.

Infection with which organism is typically associated with


Question 15
cataract and cardiac defects?

Question 16 Which organism is typically associated with fetal anaemia?

Options for Questions 17-18

A Interferons B IgA
C IgG D Immunoglobulins
E Cytokines F IgE
G Prostaglandins H Interleukins
I IgM J IgD

Instructions: For each of the statements below, choose the single most appropriate option from the
above list. Each option may be used once, more than once or not at all.

Immunoglobulin which makes up less than 0.01% of serum


Question 17
immunoglobulin and does not activate complement

Immunoglobulin which is not secreted, does not activate


Question 18
complement or sensitize mast cells
Options for Questions 19-20

A Interferons B IgA
C IgG D Immunoglobulins
E Cytokines F IgE
G Prostaglandins H Interleukins
I Exotoxins J IgD

Instructions: For each of the statements below, choose the single most appropriate option from the
above list. Each option may be used once, more than once or not at all.

Immunoglobulin which does not cross the placenta and does


Question 19
not activate mast cells

Makes up 75% of total immunoglobulin and causes Rhesus


Question 20
disease

Options for Questions 21-22

A Rubella B Influenza A
C Cytomegalovirus D Parvovirus B19
E Coxsackie B virus F Hepatitis A
G Hepatitis B H HIV
I Toxoplasmosis J Varicella zoster

Instructions: Infections in pregnancy may be associated with fetal adverse effects. For each of the
questions below, choose the single most appropriate micro-organism from the above list. Each option
may be used once, more than once or not at all.

If maternal infection occurs in the first trimester, which


Question 21
organism is associated with the highest risk of fetal infection?

Question 22 Which organism is typically associated with fetal hydrops?

Options for Questions 23-23

A Interferons B IgA
C IgG D Immunoglobulins
E Cytokines F IgE
G Prostaglandins H Interleukins
I IgM J IgD

Instructions: For each of the statements below, choose the single most appropriate option from the
above list. Each option may be used once, more than once or not at all.

The first immunoglobulin to be synthesised during B cell


Question 23
maturation

Options for Questions 24-25

A Staphylococcus aereus B Actinomycete israelii


C Neisseria Gonorrhoea D Chlamydia trachomatis
E Schistosoma haematobium F Pseudomonas aeruginosa
G E. coli H Pseudomonas aeruginosa
I Bacteriodes J Mycobacterium tuberculosis

Instructions: For each of the scenarios below, choose the single most appropriate micro-organism
from the above list. Each option may be used once, more than once or not at all.

Question 24 Gram negative intra-cellular diplococcus

Gram negative diplococcus, causes arthritis, urethritis and


Question 25
pharyngitis

Options for Questions 26-27

A Benzyl penicillin B Carbenicillin


C Metronidazole D Teicoplanin
E Flucloxacillin F Ampicillin
G Pyramethamine H Rifampicin
I Doxycycline J Azythromycin

Instructions: For each of the scenarios below, choose the single most appropriate antibiotic from the
above list. Each option may be used once, more than once or not at all.

A 45 year old woman has an abnormal pre-operative chest X-


Question 26
ray and is found to have active tuberculosis

A 30 year old farmer develops toxoplasmosis at 12 weeks


Question 27
gestation

Options for Questions 28-29

A Staphylococcus aereus B Actinomycete israelii


C Neisseria Gonorrhoea D Chlamydia trachomatis
E Schistosoma haematobium F Pseudomonas aeruginosa
G E. coli H Pseudomonas aeruginosa
I Bacteriodes J Mycobacterium tuberculosis

Instructions: For each of the scenarios below, choose the single most appropriate micro-organism
from the above list. Each option may be used once, more than once or not at all.

Question 28 Gram negative anaerobic bacteria

Acid-fast bacilli. Infection causes delayed hypersensitivity


Question 29
reaction

Options for Questions 30-30

A Rubella B Influenza A
C Human papilloma virus type 16 D Parvovirus B19
E Coxsackie B virus F Hepatitis A
G Hepatitis B H HIV
I Toxoplasmosis J Varicella zoster

Instructions: Infections in pregnancy may be associated with fetal adverse effects. For each of the
questions below, choose the single most appropriate micro-organism from the above list. Each option
may be used once, more than once or not at all.

Which virus is a DNA virus and a member of the Herpes


Question 30
family of viruses?

Options for Questions 31-32

A Benzyl penicillin B Carbenicillin


C Metronidazole D Teicoplanin
E Flucloxacillin F Amoxycillin
G Pyramethamine H Rifampicin
I Doxycycline J Azythromycin

Instructions: For each of the scenarios below, choose the single most appropriate antibiotic from the
above list. Each option may be used once, more than once or not at all.

A 32 year old woman complaining of vaginal discharge at 16


Question 31 weeks gestation has an endocervical swab which is positive
for Chlamydia trachomatis

An 18 year old woman with a vaginal discharge has cervical


Question 32
and urethral swabs positive for Neisseria gonorrhoea

Options for Questions 33-34

A Immediate (Type I) hypersensitivity B Antibody mediated (Type II) hypersensitivity

C Immune complex mediated (Type III) D Delayed cell mediated (type IV)
hypersensitivity hypersensitivity
E Graft versus host disease F Acute graft rejection
G Hyperacute graft rejection H Chronic graft rejection
I Allergic Reaction J Jerisch-Herxheimer reaction

Instructions: For each of the scenarios below, choose the single most appropriate option from the
above list. Each option may be used once, more than once or not at all.

Question 33 Immune response occurring during anaphylaxis

Question 34 Immune response occurring in atopic dermatitis

Options for Questions 35-36

A Rubella B Influenza A
C Human papilloma virus type 16 D Parvovirus B19
E Coxsackie B virus F Hepatitis A
G Hepatitis B H HIV
I Toxoplasmosis J Varicella zoster

Instructions: Infections in pregnancy may be associated with fetal / neonatal adverse effects. For each
of the scenarios below, choose the single most appropriate micro-organism from the above list. Each
option may be used once, more than once or not at all.

Vertical transmission of this virus is associated with a high


Question 35
risk of chronic carriage and a risk of chronic liver disease?

Question 36 A DNA virus spread by sexual intercourse or blood products

Options for Questions 37-38

A Interferons B IgA
C IgG D Immunoglobulins
E Cytokines F IgE
G Prostaglandins H Interleukins
I Exotoxins J IgD

Instructions: For each of the statements below, choose the single most appropriate option from the
above list. Each option may be used once, more than once or not at all.

Question 37 Immunoglobulin which is actively transported by the placenta

Immunoglobulin present in 4 sub-types and activates


Question 38
complement

Options for Questions 39-40

A Staphylococcus aereus B Actinomycete israelii


C Neisseria Gonorrhoea D Chlamydia trachomatis
E Schistosoma haematobium F Pseudomonas aeruginosa
G E. coli H Pseudomonas aeruginosa
I Bacteriodes J Mycobacterium tuberculosis

Instructions: For each of the scenarios below, choose the single most appropriate micro-organism
from the above list. Each option may be used once, more than once or not at all.

Question 39 Gram negative bacillus. Metabolises glucose by oxidation

Gram negative bacillus resistant to ampicillin and


Question 40
trimethoprim
Act at 4 main sites

• Cell wall synthesis

• Protein synthesis

• Nucleic acid synthesis

• Cell membrane function

BACTERIAL RESISTANCE

CAUSED BY:

• Single gene mutations - random events, with the mutant that results in resistance being
selected in the presence of the antibiotic

• Plasmid mediated resistance

• Transposons - 'jumping genes' - resistance genes that are capable of intergration into the
chromosome or plasmids

BETA-LACTAM ANTIBIOTICS *****

• Inhibit bacterial cell wall synthesis *


• Analogues of d-alanyl-d-alanine

• Cell wall synthesis is dependent on transpeptidation which links soluble peptidoglycan


precursors emerging from the cell membrane into the insoluble growing peptidoglycan chain.
This reaction is inhibited by beta-lactam antibiotics - act outside the cell membrane

• Effective against all gram positive bacteria and gram negative cocci

• Excreted by the kidneys - penicillin excretion inhibited by probenecid *

• Resistance mainly via beta-lactamase production *

• Include penicillins and cephalosporins

ANTIBIOTICS AND CELL WALL SYNTHESIS *****

• Beta lactams - inhibit transpeptidation (penicillins, monobactams, carbapenams and


ceplalosporins)

• Cycloserine - inhibits reactions incorporating alanine into cell wall precursor within the
cytoplasm

• Glycopeptides - vancomycin, teicoplanin - bind to terminal d-alanyl-d-alanine residue,


preventing incorporation into growing peptidoglycan chain

• Bacitracin - prevents dephophorylation of phosphilipid carrier, preventing regeneration of


carrier required for continuous synthesis

GLYCOPEPTIDES ****
• Vancomycin and teicoplanin

• Inhibit bacterial cell wall synthesis by binding to d-alanyl-d-alanine at the end of a pentapeptide
chain, preventing the incorporation of new subunits

• Large molecules which cannot penetrate gram negative cell wall. Only active against gram
positive bacteria

• Not absorbed from the GI tract. Oral administration used to treat Clostridium difficile diarrhoea

• Potentially ototoxic and nephrotoxic - vancomycin should be administered slowly to prevent


'red-man' syndrome. Teicoplanin is less toxic and can be given bolus
AMINOGLYCOSIDES *****

• Gentamicin, tobramycin, netilmicin, streptomycin

• Inhibit bacterial protein synthesis *- interfere with the binding of formylmethionyl-transfer RNA
to the ribosomes, preventing the formation of the initiation complex

• Not absorbed from the GI tract.* Do not cross the blood-brain barrier

• Not active against streptococci. Active against gram negative bacteria

• Excreted via the kidneys *

• Potentially ototoxic and nephrotoxic *

• Production of aminoglycoside modifying enzymes is the most important mechanism of acquired


bacterial resistance - usually plasmid mediated

TETRACYCLINES *****

• Inhibit bacterial protein synthesis by preventing aminoacyl tRNA from entering the acceptor
site on ribosomes *

• Absorbed from the gut. Doxycycline and minocycline are better absorbed than the others *

• Penetrate tissues well and are active against intracellular bacteria *

• Effective against chlamydial, mycoplasma and rickettsiae infections *


• Cross placenta - interferes with bone development and cause brown discolouration of teeth -
contraindicated in pregnancy and children *

• Widespread resistance as a result of their use as growth promoters in animal feed -


resistance gene carried on a transposon

CHLORAMPHENICOL

• Inhibits bacterial protein synthesis by blocking peptidyl transferase

• Well absorbed from the gut

• Active against a wide range of bacteria. Main indication is the treatment of Salmonella typhi

• Causes REVERSIBLE dose-dependent bone marrow suppression *

MACROGLIDES *****

• Erythromycin, azithromycin, clarithromycin and spiramycin

• Inhibit bacterial protein synthesis by binding to 23S rRNA, blocking translocation *

• Well absorbed from the gut; penetrate tissues well and are active against intracellular
organisms *
• Erythromycin is concentrated in the liver and excreted via bile

CLINDAMYCIN *****

• Clindamycin belongs to the group of Lincosamides

• Binds to 50S ribosomal subunit, inhibiting peptide bond formation.

• Similar antibiotic spectrum to erythromycin, excreted by the liver, active in faeces for up to 5
days after a dose

• More active against anaerobes than erythromycin. Clostridium difficile is resistant and may be
selected, causing pseudomembranous colitis
ANTI-FUNGAL AGENTS

• Azole compounds - clotrimazole, ketoconazole, fluconazole: inhibit


lanosterol C14 demethylase,an important enzyme in sterol biosynthesis

• Polyene compounds - amphotericin B, nystatin: bind ergosterol in cell


membrane resulting in leakage of cellular contents

• Flucytosine: pro-drug, taken up by fungal cells and converted into 5-


fluorouracil. Inhibits DNA synthesis

• Griseofulvin: inhibits mitosis and nucleic acid synthesis

ANTI-VIRAL AGENTS

• Acyclovir - acycloguanosine. Inhibits herpes virus DNA polymerase.


Effective against herpes simplex, varicalla zoster but NOT against CMV
or Epstein-Barr virus

• Ganciclovir - active against CMV

• Foscarnet - used for CMV retinitis in AIDS

• Zidovudine- HIV

• Ribavirin - aerosol for severe RSV infection; severe influenza B infection

• Amantidine -specifically inhibits replication of influenza A but has no


activity against influenza B
BACTERIAL VAGINOSIS *****

Polymicrobial infection characterised by:

• Copious frothy vaginal discharge with a fishy odour *

• Increased vaginal pH (>4.5) *

• Clue cells - vaginal epithelial cells coated with gram-variable bacteria on wet vaginal smear *

• Vaginosis - there is minimal inflammation *

• Positive amine test - pungent fishy odour on adding the vaginal discharge to sodium hydroxide

• Associated with an alteration in vaginal flora with decreased lactobacilli and increased
anaerobes and Gaednerella vaginalis

Gardnerella vaginalis

• Gram variable, can be isolated from the vaginal flora in 20% of asymptomatic women
and present in the urethra of male partners of women with bacterial vaginosis

• Treated with metronidazole

TRICHOMONAS VAGINALIS
• Flagellated protozoan, anaerobic, readily cultured on Diamond’s medium

• Sexually transmitted

• Infection is different from bacterial vaginosis although clinical differentiation is difficult

• Characteristic strawberry cervix only found in 3% of cases

• Diagnosed by microscopy of wet vaginal smear or by culture

• The organism is difficult to detect in males but can be isolated from prostatic fluid

• Also causes urethritis, cystitis and Bartholinitis

CANDIDA ALBICANS *****

• Commensal organism in the mouth, gut or vagina of 40-80% of individuals *

• Causes candidiasis

• Yeast-like fungus, grows by budding and may produce long non-branching filaments
called pseudohyphae. Candida is a polymorphic yeast, i.e., yeast cells, hyphae and
pseudohyphae are produced *

• Gram positive anaerobic organism *

• Causes local infection of the vagina, mouth and skin


• Increased risk of colonisation / infection in pregnancy (especially third trimester),
steroid / broad spectrum antibiotic use, diabetes mellitus. Increased risk in oral contraceptive
users is disputed. Causes disseminated infection in the immunocompromised *

• Culture - Sabouraud's medium - pH ~ 5.6 *

• Vaginal infection associated with a pH at the low end of normal (3-4)

• Sensitive to polyene antibiotics (nystatin / amphotericin B) *

• Sensitive to imidazole derivatives - clotrimazole, econazole, floconazole, miconazole *

• Rarely isolated from the vagina in pre-pubertal or post-menopausal women


CHLAMYDIA TRACHOMATIS *****

• Obligate intracellular organism - bacteria *

• Non-motile gram negative or gram variable although gram stain is not useful in diagnosis *

• Enter the host through abrasions in mucosal surfaces *

• Does not infect squamous cells - the vaginal epithelium and ectocervix are not infected.
Endocervical glands are susceptible *

• Produce infectious particles called Elementary Bodies which enter cells by endocytosis

• Differentiate into Reticulate Bodies within cells. These multiply by binary fission and then
produce other elementary bodies which are released from the cell

• Asymptomatic infection is common

• C. psittaci infects birds and non-human animals *

Various serovars:

• A - C: Trachoma (blindness)

• D - K: genital infection - cervicitis, urethritis, salpingitis, epididymitis, proctitis, Bartholinitis,


conjunctivitis, interstitial pneumonitis in neonates. *
• Consequences of infection include infertility, ectopic pregnancy, Fitz-Hugh-Curtis
syndrome, Reiter's syndrome, arthritis, dermatitis

• L1-3: Lymphogranuloma venereum

DIAGNOSIS

• ELISA test - endocervical (or other) swab - sensitivity ~70%

• Immuno-fluorescence

• PCR or LCR - early morning urine specimen - most sensitive test

• Culture - requires special (McCoy) cells

• Serology - unreliable

TREATMENT

• Doxycycline (tetracycline) - mainstay of treatment

• Erythromycin

• Amoxycillin or clindamycin - comparable cure rates (93-98%) to erythromycin in pregnancy


but better tolerated *

• Azythromycin - also effective treatment for Gonorrhoea


• Ofloxacin

LYMPHOGRANULOMA VENEREUM

• Caused by Chlamydia trachomatis serovars L1-3 *

• Inguinal lymphadenopathy with subsequend suppuration and ulceration. Recognised cause of


fistula-in-ano

• Chlamydia may cause systemic infection - hepatitis, pneumonitis and meningo-encephalitis

• Sensitive to tetracycline

NEISSERIA GONORRHOEA *****

• Gram negative intracellular diplococcus *

• Fragile organism - very sensitive to drying and low temperature- swabs should be collected
in Stuart's transport medium, maintained at room temperature * and cultured on selective VCT
medium (vancomycin, collistin, trimethoprim). Grows on blood or chocolate agar

• Infects mucous membranes lined by non-squamous epithelium *

• Glucose fermenting (N. meningitides is Maltose fermenting)

Serological diagnosis is unreliable.


• Has pili which are important in virulence

• Neisseria meningitides is carried in the nasopharynx of 5-30%of adults and is one of the
three primary pathogens causing bacterial meningitis

INFECTION

• Incubation period 2-7 days, majority of infected females are asymptomatic *

• Males: 20% chance; Females: 50% chance of being infected after one episode of unprotected
intercourse with infected partner

• Causes urethritis, cervicitis, epididymitis, pharyngitis, proctitis, endocarditis, skin infection,


arthritis * and vertical neonatal infection (ophthalmia neonatorum). Salpingitis (PID) may lead to
infertility.

TREATMENT *

• Procaine penicillin im + probenecid

• Third generation cephalosporin

• Ciprofloxacin

• Spectinomycin

• Azythromycin
• Contrat tracing required

REITER’S SYNDROME *

• Arthritis +/- conjunctivitis following GI or GU infection

• Commoner in males

• 50% associated with Chlamydia infection

• Arthritis is reactive, aseptic and sero-negative

• Abnormal response to the infection in genetically predisposed individuals


COLIFORMS

• Include E. coli, Klebsiella, Proteus, Salmonella and Shigella

• Gram negative bacilli

• Facultative anaerobes

• Grow on blood agar or selective media

• Oxidase negative (differentiates from P. aeruginosa which is oxidase positive)

• E. coli and Klebsiella are lactose fermenting

• E. coli and Salmonella are motile (flagellated)

• Produce endotoxins (gram positive bacteria produce exotoxins) but E. coli also produces an
enterotoxin (exotoxin) which causes diarrhoea by stimulating epithelial adenylate cyclase

• E. coli is part of normal fecal flora

• Salmonella and shigella are not part of normal flora

• Salmonella typhi may be present permanently in the biliary tract of carriers

• Klebsiella is resistant to ampicillin. Other community acquired colifirms are generally sensitive
to ampicillin
• Hospital acquired coloforms are generally resistant to ampicillin but sensitive to gentamicin and
third generation cephalosporins

PSEUDOMONAS AERUGINOSA *****

• Gram negative bacilli, indistinguishable from coliforms on gram staining *

• Strict aerobes *

• The bacterium is ubiquitous in soil and water, and on surfaces in contact with soil or
water. They occur regularly on the surfaces of plants and occassionally on the surfaces of
animals.

• Its metabolism is respiratory and never fermentative *

• Almost all strains are motile by means of a single polar flagellum *

• The bacterium almost never infects uncompromised tissues, yet there is hardly any tissue that
it cannot infect if the tissue defenses are compromised in some manner

• P. aeruginosa strains produce two types of soluble pigments, the fluorescent


pigment pyoverdin and the blue pigment pyocyanin *. The latter is produced abundantly in
media of low-iron content and functions in iron metabolism in the bacterium

• Grow well on selective agar containing the disinfectant cetrimide

• Strongly oxidase positive (coliforms are oxidase negative)


• Metabolises glucose by oxidation

• Not part of normal gut flora

• Causes chronic UTIs, wound infections, chronic osteomyelitis and opportunistic infections

• Resistant to ampicillin, trimethoprim, tetracycline, sulphonamides *

• Sensitive to aminoglycosides gentamicin and netilmicin *

• Sensitive to anti-pseudomonas penicillins carbenicillin, ticarcillin, piperacillin *

• Most are sensitive to ceftazidime, ciprofloxacin and imipenam *

VIBRIOS

• Include Vibrio cholerae and parahaemolyticus and Campylobacter

• Gram negative bacilli

• Not distinguishable from coliforms on gram staining


• V. cholerae produces exotoxin which stimulates intestinal epithelial adenylate cyclase causing
diarrhoea

• Campylobacter is a common cause of gastro-enteritis and food poisoning

BORDETELLA PERTUSSIS ****

• Gram negative bacilli; member of family of Parvobacteria which includes H. influenzae *

• Non-motile

• Does not grow on blood agar

• Causes whopping cough *

• Incubation period 5-14 days

• Spread by droplet infection *

• Infected individual is infectious for up to three weeks after the onset of symptoms

• Infection may occur at any age but most common and more severe in infants below the age of
1 year *

• In children, lymphocytosis is often profound (>70% of the total WBC count). The WBC count
often rises to 20-40,000 cells/mm2; and may rise to 100,000 cells/mm2 *

• Infection is more common in females than in males


• Disease has Catarrhal, spasmodic (paroxysmal) and recovery stages

• Cultures are typically negative soon after the onset of the paroxysmal stage

• Infection is preventable by vaccination using killed organism. Does not produce a toxin

• Severity of infection may be reduced by erythromycin

HAEMOPHILUS INFLUENZAE

• Gram negative bacillus - parvobacterium

• Grows on blood agar

• Causes respiratory tract infections in childhood, meningitis and infective exacerbations of


COAD

• Frequently found in normal throat flora

• Sensitive to ampicillin though up to 10% may be resistant. Sensitive to chloramphenicol

LEGIONELLA PNEUMOPHILIA

• Gram negative bacilli


• Does not grow on ordinary media

• Grows on media containing added cysteine and iron salts

• Is spread by droplet infection via contaminated air conditioning systems and shower mixers

• Causes Legionnaires disease - up to 15% mortality

• Sensitive to erythromycin

• Resistant to penicillin and aminoglycosides

BACTERIODES *****

• Gram negative bacilli, strict ANAEROBES *

• Non-spore forming *

• Readily killed by heat or chemical disinfectants

• Do not produce exotoxins

• Part of normal mouth, vaginal and rectal flora

• Some are motile while others are non-motile *


• Important cause of post-operative infection, intra-abdominal and pelvic sepsis. Synergistic
action with aerobic organisms such as coliforms, coliforms being responsible for acute
infection, septicaemia and shock while bacteriodes contribute to abscess formation *

• Sensitive to metronidazole and clindamycin *


CONGENITAL INFECTIONS

Organism Infection Trans-placental Acquired from


route mother shortly
before or during
delivery

Treponema Congenital syphilis Yes No


pallidum *

Rubella virus * Congenital rubella Yes No

Cytomegalovirus* Congenital CMV Yes No


disease

Toxoplasma gondii Congenital Yes No


toxoplasmosis

Hepatitis B Neonatal Hep B Possibly Yes


infection

Herpes simplex Neonatal herpes Yes Yes

Varicella zoster Congenital Yes Yes


varicella

HIV Neonatal AIDS Yes Yes

Listeria Listeriosis Yes yes


monocytogenes *

Neisseria Ophthalmia No Yes


gonorrhoea neonatorium

Chlamydia Ophthalmia No Yes


trachomatis * neonatorium,
neonatal
pneumonia

Candida albicans Neonatal oral No Yes


thrush
EXOTOXINS *****

• Highly toxic polypeptides produced by living bacteria *

• Act at specific target sites *

• Are highly antigenic (except Cl. Tetani toxin)

• Are converted into antigenic non-toxic toxoids by formalin *

• Are often destroyed by heat

• Are neutralised by anti-toxins *

• Typically produced by gram positive bacteria *

ENDOTOXINS

• Are lipopolysaccharide molecules

• Present in the outer layer of the cell wall of gram negative bacteria,
released on disintergration

• Relatively heat stable

• Not converted into toxoid by formalin

• Act non-specifically on the reticulo-endothelial system causing


increased vascular permeability and prostaglandin release

• May cause DIC and cardiovascular collapse

• Detectable by the Limulus test

• Do not stimulate production of natural anti-toxins

• Produced by E. coli, N. meningitides, S. typhi, P. aeruginosa


HEPATITIS A

• RNA virus
• Spread by the feco-oral route although sexual transmission may
occur

• Not vertically transmissible

• Incubation period 2-3 weeks

• Does not result in chronic carriage or chronic liver disease

• 20-50% of adults in UK have been infected

• formaldehyde inactivated vaccine is available

HEPATITIS B *****

• DNA virus

• Spread by sexual intercourse / blood products *

• Incubation period 1-5 months


• Multiplies only in the liver
• Vertically transmissible * - 90% of vertically acquired infection
result in chronic carriage - preventable by passive + active
immunisation

• 10% of horizontally acquired infections result in chronic carriage

• There is no evidence that mosquitoes can transmit hepatitis B

• 60-70% of acute hepatitis B infections are sub-clinical or


associated with mild flu-like illness. Severity of symptoms is similar
in pregnancy

• Chronic infection associated with hepatocellular carcinoma *

• Horizontal and vertical infection preventable by active and


passive immunisation *

Serology

• HBsAg detected before HBeAg - both detectable before the


onset of jaundice. HBeAg levels fall rapidly

• Anti-HBs antibody detectable at the onset of jaundice and


before anti-HBe antibodies
• Anti HBs antibody detected about 4 weeks after the onset of
jaundice

• Detection of HBeAg indicates high infectivity *


• Detection of anti-HBs IgG indicates previous infection or
vaccination

• Chronic carriage associated with persistence of HBsAg

HEPATITIS C

• RNA virus

• Spread by blood products / sexual and vertical transmission

• Incubation period ~2 months

• 50% of patients develop chronic liver disease

• Associated with hepatocellular carcinoma

• treatment with interferon-gamma / Ribavirin may be effective


HEPATITIS D

• DefectiveRNA virus

• Requires co-infection with Hepatitis B for propagation

• Spread by blood products and sexual transmission

• Incubation period 1-3 months

• Associated with chronic carriage and chronic liver disease

HEPATITIS E

• RNA virus

• Spread by the feco-oral route

• Incubation period 6-8 weeks

• Causes more severe disease in pregnancy with DIC and 20%


mortality
• Does not cause chronic carriage or chronic liver disease
EX VIRUS *****

• DNA virus

• Type I classically causes oro-labial, and type II genital infection *.

• However, the prevalence of oro-genital sexual practices means that this distinction is
less evident

• Incubation period 3-7 days

• Primary infection may be asymptomatic. Symptoms include genital vesicles which


progress into painful ulcers that eventually crust over. Dysuria, urinary retention, bilateral
inguinal lymphadenopathy, systemic illness may occur. Aseptic meningitis / encephalitis
are rare complications *

• Primary infection involves cervix in 70% of symptomatic cases, causing purulent


discharge and necrotic ulceration

• Virus remains dormant in dorsal root ganglia, causing recurrent infection. Average
number of recurrences per year = 4. Less severe symptoms *

• Genital infection can be acquired from oro-genital contact.

• Virus can be shed from the genital tract and transmit infection in the absence of
symptoms or a prodrome

• Trauma (such as dental treatment, facial peels), exposure to sunlight or ultra-violet


light (sun beds) may trigger recurrent cold sores *

• Vertical transmission causes neonatal disseminated herpes which has high mortality /
morbidity. Does not cause congenital anomalies.
Diagnosis

• Clinical

• Viral culture - negative culture does not exclude infection. The longer the duration of
symptoms, the less likely is a swab to be positive

• Electron microscopy

• ELISA / PCR

• Serology - unreliable cannot distinguish between oro-labial and genital infection

Treatment

• Symptomatic

• Acyclovir

• Prophylactic acyclovir reduces the frequency of recurrence.

VARICELLA ZOSTER

• DNA virus, member of the Herpes family of viruses


• Incubation period 10-21 days

• Spread by droplet infection

• Contagious 48h before onset of rash until all vesicles have crusted over
• Severs infection may cause pneumonitis, encephalitis, myocarditis and adrenal
insufficiency

• Virus remains dormant in the dorsal root ganglia, causing recurrent infection
(shingles, Herpes zoster). Shingles is contagious

• Infection in pregnancy is more severe, with 10% risk of pneumonitis and up to 6%


mortality

• 2% risk of congenital varicella syndrome (dermatomal scarring, cataract, limb


hypoplasia, microcephaly, learning disability..) if infection occurs BEFORE 20 weeks
gestation

• Infection results in life-long immunity.

• There is no effective vaccine currently available

• Infection can be prevented by passive immunisation with varicella zoster immune


globulin - not life-long

• 85% of pregnant women are immune (UK)


• Severe neonatal disease likely (30%) if maternal disease develops within 4 days of
delivery - neonate has not acquired maternal antibodies

• Sensitive to acyclovir - useful in severe disease

CYTOMEGALOVIRUS *****
• DNA virus, member of the Herpes family of viruses *
• Primary infection usually asymptomatic but may produce an illness that is clinically
indistinguishable from Epstein-Barr mononucleosis (glandular fever)

• CMV mononucleosis is heterophile negative (EBV is positive)

• Can be cultured from urine of infected individuals. Excretion may persist for years *

• Causes self-limiting illness with fever, malaise, hepatitis

• Severe disease in immunocompromised (AIDS / transplant patients)

• Infection is more prevalent in lower socio-economic classes

• Primary infection occurs in 2% of pregnant women

• Virus can be isolated in 1% of neonates - commonest congenital infection

• Recurrences are common and fetal infection may occur during recurrence. Maternal
immunity (presence of IgG does not prevent recurrence or fetal infection)

• Results in congenital anomalies including hepatosplenomegaly, jaundice,


thrombocytopaenia, intra-cranial calcification, microcephaly, ventriculomegaly, optic
atrophy, IUGR, cerebral atrophy with psychomotor delay, neonatal haemolytic anaemia *

• Sexually transmitted. Present in saliva, semen, cervical secretions and breast milk *
• Gancyclovir is often effective in CMV retinitis / pneumonia
Human infections transmitted via urine

Infection

Details

Parasite eggs excreted in urine


Schistosomiasis *

Bacterial persistence in bladder scarred


by schistosomiasis

Typhoid

Excreted in urine of infected children

Cytomegalovirus

Infected rats and dogs excrete bacteria in


urine

Leptospirosis *
Infected rodents excrete virus in urine

Lassa fever

Human infections transmitted via saliva


Infection

Details

Infection in childhood

Herpes simplex *

Mumps

Adolescent / adult infection

CMV
Epstein-Barr virus *

Saliva of infected dogs, wolves, vampire


bats

Rabies

Saliva of dogs & cats. Transmitted by


bites or scratches

Pasteurella multocida

Human infections transmitted via milk


Infection

Type of milk

Human milk
Mumps virus

Human milk

CMV *

Human milk

HIV

Cow / goat / sheep

Brucella
Cow milk

Mycobacterium bovis

Cow milk

Salmonella

Cow milk

Listeriosis *
Cow milk

Streptococcal infection
HUMAN PAPILLOMA VIRUS *****
• DNA virus

• Over 80 different genotypes

• Cannot be cultured in-vitro

• Commonest sexually transmitted infection

• Types 2,6,11 cause exophytic epithelial proliferation (genital warts;


condylomata accuminata)

• Types 16 and 18 are associated with CIN and cervical cancer - high
risk HPV. Also associated with oral and vulval carcinoma and VIN

• Some women are able to clear the infection - dependent on host


immunity

• Vertically acquired infection can occur, causing juvenile laryngeal


papilomatosis

• There is no recognised anti-viral treatment

HUMAN IMMUNO-DEFICIENCY VIRUS *****

• RNA virus

• Infects CD4 positive cells - uses CD4 antigen as receptor


• Sero-conversion may take up to three months
• Transmitted by breastfeeding, sexual intercourse, needle stick
injuries (including sharing needles), blood and blood products and
vertival transmission mainly during passage through the genital
tract
• Risk of infection after a needle stick injury involving an infected
patient is <1% and lower than risk of hepatitis B transmission
• Risk of infection after single episode of unprotected intercourse
with infected partner <1%

Vertical Transmission
• 15-20% vertical transmission rate without treatment
• 2/3rd transmission occurs around the time of delivery

• Breast-feeding increases risk of transmission

• High viral load, low CD4 count, advanced disease, prolonged


ROM (>4h), invasive procedures, vaginal delivery, smoking,
intercurrent STD, vitamin A deficiency - associated with increased
risk of vertical transmission

• Prophylaxis against opportunistic infections if CD4 count < 200/ml


-
co-trimoxazole + 5mg folic acid
• Antenatal + intrapartum + neonatal AZT shown in randomised
trials to reduce risk of vertical transmission (25% to 8%)

• Delivery by caesarean section + avoidance of breast feeding further


reduces risk to < 2%
• No evidence that caesarean section reduces the risk of vertical
transmission if performed after the onset of labour or after ROM

• If vaginal delivery, avoid invasive monitoring and keep membranes


intact for as long as possible. Use chlorhexidine to cleanse vagina -
may reduce risk of transmission. Appropriate infection control
measures. Early clamping of the cord + washing off maternal blood
from baby may be helpful
INFECTION CONTROL *****

• Disinfection - removes or kills most, but not all viable organisms. Uses chemical
or physical processes

• Sterilisation - removes or kills all viable organisms

• Pasteurisation - reduces total number of viable organisms in fluids without


affecting flavour. Effective against intracellular organisms and some viruses but
not against spores

STERILISATION / DISINFECTION *****

• Ultr-violet irradiation - inefficient as a sterilant. Used to inhibit bacterial growth in


complex apparatus

• Dry heat -160-180C. Useful for heat resistant equipment

• Moist heat (autoclaving) - most effective method of sterilisation. Usual cycle is 121C
for 15 minutes - sufficient to kill spores of Clostridium botulinum. Heat-sensitive
instruments such as endoscopes -use low temperature steam + formaldehyde

• Boiling - disinfects but does not sterilise -some spores are not killed

• Pasteurisation - reduce microbial load in fluids - 62.8 - 65.6C for 30 minutes

• Gamma irradiation - useful for sterilising large batches of small items such as
needles

Filtration - produces particle-free fluid

• Ethylene oxide -effective sterilant but toxic and explosive

• Formaldehyde -disinfectant. Irritant.

• Iodine / iodophors -kills some spores, inactivated by blood and other organic
matter; iodine stains skin and can cause local sensitivity

• Mercuric chloride - topical skin preparation

• Benzalkalonium chloride / cetrimide - useful as skin disinfectants, mainly active


against gram positive organisms. Solutions can become contaminated with gram
negative rods

• Chlorhexidine - useful for skin and mucous membranes. Combination with


detergents highly effective for hand disinfection. Poor activity against TB, spores,
fungi and viruses. Inactivated by many materials including plastic, soap, pus

• Alcohols - effective only in the presence of water (70% alcohol). Isopropyl alcohol
better for skin. Inactive against spores and fungi, inflammable

• Hexachlorophene - more effective against gram negative organisms. Used in soap


or dusting powder but may be toxic

• Phenolics (carbolic acid, hexachlorophene, chloroxylenol) - active against a wide


range of bacteria including mycobacteria. Do not kill spores. May cause irritation
• Glutaraldehyde - less irritant than formaldehyde, exposure of at least 3h required to
kill all microbes. Most killed within 10 minutes. Disinfects but does not sterilise
heat-sensitive instruments such as endoscopes

• Hypochlorite and chlorine -active against fungi, viruses (including HepB and HIV),
and spores as well as bacteria. Easily inactivated by organic material- not used for
skin disinfection
INFLUENZA

• Caused by a myxovirus - RNA virus


• Spread by droplet infection

• Infection confined to the respiratory tract - there is no


viraemia
• Incubation period 1-3 days

Three types

• Influenza A: causes epidemics and pandemics; has a


reservoir in birds, undergoes antigenic shift - sudden major
change in antigenicity based on recombination between
different strains
• Influenza B: causes epidemics. No animal reservoir
• Influenza C: causes mild illness only
• Virus envelope has haemagglutinin and neuraminidase
antigens which are used for nomenclature
• All three types undergo antigenic drift- minor changes in
antigenicity due to mutations
• Infection predisposes to bacterial pneumonia
• Viral replication inhibited by rimantadine (amantadine). Can
reduce disease severity if given within 2 days of onset

• Vaccination provides protection in 70% of individuals for


about one year
• New vaccines produced each year dependent on the strains
circulating in the previous year

PRION DISEASE *****

• Caused by host-encoded protein that is slightly altered in


infected brain leading to accumulation in nerve cells
• Similar size to viruses, contain neither DNA nor RNA and
replicate very slowly with a doubling time of over one week
• Resistant to heat, chemical agents and irradiation
• Cannot be cultured in vitro

• Do not induce an immune response no antibody production


• Cause microscopic vacuoles in the brain giving a spongiform
appearance

• Incubation period may be up to 20 years

• Transmissible by infected surgical instruments, tissue / organ


transplantation and possibly vertically
• Cause Creutzfeldt-Jakob disease and Kuru in humans and
BSE in cattle
MRSA

• Methicillin-resistant Staphylococcus aureus.

• The treatment of Staphylococcus aureus infections was revolutionised by the


introduction of penicillin. Most strains of Staphylococcus aureus are now resistant to
penicillin because of -lactamase production but remain sensitive to the anti-
staphylococcal penicillin flucloxacillin

• Staph aureus strains known as MRSA have now also become resistant to methicillin
and flucloxacillin. Infections most common in hospitals in-patients and are rarely seen
in the community.

• Colonisation with Staph. aureus is very common. Some individuals are colonised
with MRSA (skin or nose / pharynx usually). Wounds (including surgical), indwelling
catheters or burns and immuno-deficiency increase the risk of MRSA infection.

• MRSA infection may be endogenous or acquired from a different patient (cross-


infection). Some MRSA strains are particularly successful in spreading between
patients and may spread between hospital units -epidemic MRSA (EMRSA). Two
strains have been identified - EMRSA 15 & 16

• Spread by direct physical contact although organisms in the nose or lungs can be
spread by droplet infection

• Rarely causes infection in healthy people such as health care staff and their families.
However, MRSA may be introduced into unaffected areas by colonised staff

• Between 1 and 5 per cent of people being admitted to acute hospitals are probably
currently colonised with MRSA, and at least 50 per cent of these are not known about
prior to admission screening

SCREENING FOR MRSA COLONISATION

• The sensitivity of detection of MRSA carriage depends on many the methods used
and the number and sites sampled.

• The perineum is the main carriage site on normal skin. Nose, lesions or wounds,
manipulated sites, intravenous and stoma sites, tracheostomies, perineum/groin, urine
from catheterised patients and sputum if available are usually tested
• Other swabs may be taken from the umbilicus in infants, faeces and vagina if
indicated. Other possible sites of carriage which may be relevant to certain outbreaks
include the axilla, vagina and skin of the buttock area in the elderly.

• The sensitivity of various sampling sites for detecting carriage is: nose alone 78.5%;
nose and throat 85.6%; nose and perineum 93.4%; nose, throat and perineum 98.3%.
Positive groin (15.6%) and axillary samples (10.1%) were less frequent than perineal
(38.1%) and throat (30.8%).

• Perineal carriage, the presence of large numbers of colonies from hair and fingers or
positive contact are all suggestive of heavy dispersal from carriers. The organisms are
commonly present in the air near the disperser and on the floor around the bed.

CONTROLLING MRSA

• Scrupulous hand washing by hospital staff between patients. Antiseptic detergents are
commonly used but a 70% alcohol handrub is more effective

• Rapid identification of colonised patients on admission to hospital - the use of


microbiological swabs for culture & sensitivity takes too long to provide results.

• Colonised or infected patients should be informed of the finding and its implications
and identified for infection control purposes.

• This can be done by tagging their medical records, in a manner which preserve
confidentiality, so that they are recognised immediately on admission to hospital

• Infection control team should be notified on admission

• Isolation of colonised / infected patients

• Infection control measures such as hand washing, the use of gloves, protective
clothing and damp dusting are of even greater importance if there are no isolation
cubicles, few wash hand basins and an open -Nightingale- ward design without
separate bays. However, the establishment of an isolation ward even on a temporary
basis is necessary during an outbreak

• The individual rooms used for isolation should preferably have an extraction system
removing air to the outside. This, and keeping the door closed, reduces spread
between cubicles
• Restricted access to colonised / infected patients - essential staff only

• Appropriate barrier precautions (gloves, disposable gowns) during physical contact


with MRSA patients. The gowns and gloves should be discarded safely and hands
washed before leaving the room

• Similar measures for visitors / relatives

• Cleaning of the room with disinfectant as MRSA can survive on inanimate objects or
surfaces such as linen, sinks, floors and even mops used for cleaning. The organism
may survive in a domestic environment for anything from 3-4 days to several months.
Skin scales which in floor coverings may favour survival for 6 months or more.

• Antibiotic therapy should be reviewed as antibiotics have been identified as risk factors
for acquiring MRSA, although some recent data suggest that this might have been over-
estimated

• Regular cleaning of wards and hospital beds

• However, cultures from hospital environment are generally negative even during
MRSA outbreaks.

• Ward closure to further admissions may be necessary if existing measures, such as


patient isolation, cohort nursing etc. fail to control spread

• Staphylococcus aureus carriage among pregnant women and newborn infants,


including MRSA is recognised and the possibility of mother-to-infant infection has
been confirmed

• Data on the persistence of MRSA colonisation vary, some showing that the majority
of patients remained colonised for 2-3 years while other studies showed that the
majority of patients became clear within a year provided they did not have
predisposing lesions such as open wounds or prostheses. Open skin lesions appear to
be the main risk factor for prolonged MRSA colonisation

• MRSA infection is a notifiable infection

• Hospitals may keep a list of known affected patients with the admissions or accident
and emergency department or have them identified on the hospital computer system,
so they are admitted directly to an isolation room.
• Patients may also be given cards indicating that they have been infected or colonised
with MRSA in the past. This has the benefit of informing staff if the patient is
admitted to a different hospital or health care centre for treatment.

TREATMENT

Colonisation

• Carriage of MRSA by patients and hospital personnel provide


important sources of MRSA for subsequent spread. Therefore an
attempt should be made to eradicate colonisation in most hospitalised
patients and health care staff.

• It may, however, be decided that in patients in low risk clinical


areas of the hospital, where the situation is similar to that in
community residential homes and there is not extensive interaction
with the acute units, that treatment of colonisation is not necessary.

• Eradication of carriage may also be necessary in patients who are


to be transferred to other units.

• Patient compliance, heavy skin dispersal and the impact of


possible side-effects also influence the decision to eradicate
colonisation. Frail, elderly patients recovering from surgery or serious
illness may not be able to tolerate combinations such as rifampicin and
fusidic acid to treat throat carriage, whilst topical antiseptics may
exacerbate pre-existing skin conditions or cause irritation

• Intranasal or topical mupirocin ointment (three times a day for 5 days) is effective in
the control of MRSA

• Mupirocin is a naturally occurring antibiotic produced by fermentation


ofPseudomonas fluorescens. It inhibits bacterial protein synthesis by binding
reversibly and specifically to isoleucyl-tRNA synthetase

• Skin and hair can be washed with disinfectants, such as chlorhexidine


• 1% chlorhexidine is also an alternative for nasal colonisation but is less effective

• MRSA strains resistant to mupirocin have emerged in many hospitals in the UK

• Animal data indicates that mupirocin is not terratogenic but human data are not
available

• Systemic therapy with rifampicin plus fusidic acid , ciprofloxacin or trimethoprim


may be considered if the benefits outweigh the potential risks associated with
treatment

• Upon discharge, no additional precautions are needed for colonised patients so long
as their contacts are healthy

Infection

• Most MRSA strains remain sensitive to vancomycin and teicoplanin which are the
main agents used to treat infections

• Vancomycin must be given by slow intravenous infusion to avoid -red man


syndrome-. Teicoplanin can be administered by intravenous or intramuscular injection

• Linezolid (Zyvox) has been shown to be effective in the treatment of MRSA


infections resistant to vancomycin and teicoplanin
MYCOBACTERIA *****

• Cause tuberculosis, leprosy *in addition to other opportunistic infections

• Present as 2-4 micron long acid-fast bacilli on the Ziehl-Neelsen stain

• Multiply intracellularly in macrophages but also divide extra-cellularly*

• Infection results in delayed hypersensitivity response *

• Not all mycobacteria are pathogenic in humans *

• All mycobacteria are:

1. Acid fast- i.e. they do not destain with acid and alcohol once stained with
arylmethane dyes

2. Aerobic

3. Contain mycolic acids

4. Intracellular pathogens- the wall helps the organism to survive within the
macrophage by resisting oxidative damage *

Mycobacterium tuberculosis
• Slender, straight or slightly curved bacillus, non-motile, non-encapsulated and does not form
spores *

• Acid fast bacillus (AFB) *

• Aerobic

• Slow growing- divides every 18-24 hr.

• Resistant to drying and chemical disinfectants

• Sensitive to heat (Pasteurization) and UV light

Tuberculosis

• Transmission occurs aerosol / droplet infection

• one organism may be enough to establish an infection

• TB is a chronic disease and an infected person can spread the infective organism, M.
tuberculosis, to several contacts before being diagnosed

• The main site of infection is the lung (>80%)

• Any organ can be infected via lymphatic drainage or haematological spread

• Symptoms include severe weight loss, night sweats, chronic cough, haemoptis
• A person may be infected without developing disease or remain healthy and free of disease for
decades. The infecting organisms remain viable. Disease may be activated by malignancy,
immune suppression, old age or chronic ill-health

• Diagnosis based on:

1) Clinical presentation

2) Radiology -CXR

3)Microbiology - demonstration of M. tuberculosis in a clinical specimen:


microscopic (Ziehl-Neelsen or Auramine O stain for acid-fast bacilli) and / or
Culture- may take 2-6 weeks for the isolation of M. tuberculosis

4)Negative culture does not exclude tuberculosis

Treatment

• Treatment is for 6 months

• Combinations of at least 3 anti-tuberculous agents are used

• First-line treatment: Isoniazid, Rifampicin and Pyrazinamide +/-ethambutol

Tuberculin test

• Tuberculin is a partially purified extract of M. tuberculosis proteins and induces a delayed


hypersensitivity response when injected into the skin
• Tuberculin skin testing can be used to identify individuals, especially children, with active
tuberculosis, to trace contacts of patients with active tuberculosis

• Mantoux and Heaf tests are different types of tuberculin skin tests. The Heaf gun is used to
make multiple punctures at the skin site and is more convenient when large populations are to
be screened. However, there is a risk of transmitting blood-borne infections

• The results of both tests are best read on day 3 although the Mantoux test may become
positive on day 2

• Positive Mantoux: area of induration >5mm at 72h

• Positive Heaf: at least 4 papules at the multiple puncture site at 72h

BCG vaccination *****

Use a live, attenuated M. bovis strain -Bacillus Calmette-Gu?rin (BCG)

• Administered intra-dermally *

• Efficacy questioned. Prevents disseminated disease

• Should be administered in neonatal period if high risk of contact with TB *

• Following intra-dermal vaccination a small indurated papule appears within one to three
weeks *
• Enlargement of the regional lymph nodes may develop after vaccination. Spontaneous
regression usually occurs within a few months *

• Disseminated BCG infection has been reported rarely after BCG vaccination, principally in
immunocompromised individuals
SPIROCHAETES *****

• Thin-walled spiralled flexible motile organisms

• Not seen on gram stain except Borrelia vincenti

• Seen on dark ground microscopy or silver stain

• Include Treponema (T. pallidum - syphilis, T. pertenue - Yaws, T. carateum - Pinta), Leptospira
(L. icterohaemorrhagica – Weil’s disease) and Borrelia (B. recurrentis - relapsing fever, B.
vincenti – Vincent’s infection) *

SYPHILIS
• Sexually transmitted infection caused by Treponema pallidum

• Spirocheate which cannot be cultured in-vitro. Can be visualised by dark-ground


microscopy, silver staining or immunofluorescence

• Incubation period 10-90 days *

• PRIMARY: Painless chancre on genital or extra-genital sites. Regional


lymphadenopathy may occur *

• SECONDARY - generalised malaise, sore throat, serpiginous ulcers in the mouth,


maculo-papular rash and generalised lymphadenopathy. Primary and secondary
syphilis are infectious. Occurs 6-8 weeks after the primary stage *

• LATENT - may last 3-30 years

• TERTIARY - characteristic lesion is the GUMMA - a granuloma involving the skin,


mucous membranes, bones and joints 9Charcot’s joints)

• QUATERNARY

1) CVS - aortic incompetence, coronary ostia stenosis, aortic aneurysm

2) CNS - meningo-vascular syphilis, Tabes Dorsalis, Generalised paralysis of the


insane

GUMMA

• Granuloma with ghost cell necrosis

• Characteristic lesion of tertiary syphilis

• Typically involves the skin and sub-cutaneous tissue, mucous membranes of the
upper respiratory tract, sub-periosteal sites of long bones and joint

DIAGNOSIS

• Cardiolipin antibody tests (such as Wassermann reaction, rapid plasma regain


test * and VDRL - Venereal Disease Reference Laboratory) which are relatively non-
specific and Treponemal antibody tests such as the Fluorescent Treponemal
Antibody test -Absorbed (FTA-Abs) which are specific and used as verification tests
• Cardiolipin antibody tests are positive in the serum in untreated secondary, latent
and tertiary syphilis and may be false positive

• Cardiolipin antibody tests become negative with treatment

• Primary: VDRL (+ve or -ve), FTA-Abs (+ve). FTA-Abs usually becomes positive
before VDRL

• Secondary : VDRL and FTA-Abs (+ve)

• Latent : VDRL (usually +ve), FTA-Abs (+ve)

• Tertiary / quaternary : VDRL (+ve or ?ve), FTA-Abs (+ve)

• Treated : VDRL (-ve), FTA-Abs (+ve)

• The VDRL test becomes negative with treatment and can be used to monitor
treatment. The FTA-Abs test can remain positive for several years after adequate
treatment *

• Treponema pallidum immobilisation test - live bacteria become immobile by the


action of treponemal antibodies. Highly specificbut potentially dangerous for staff *

Causes of biological false positive test for syphilis

• Recent immunisation, pneumonia, malaria, pregnancy - cause acute false positives


(last less than 6 months)

• SLE, and other auto-immune disorders, acquired haemolytic anaemia, leprosy and
drug addiction cause chronic biological false positives

TREATMENT

• Procaine penicillin is first line treatment

• Treponema pallidum is also sensitive to erythromycin and tetracycline although the


results of treatment are less satisfactory

• The Jerisch-Herxheimer reaction is an inflammatory response to the increased


release of treponemal antigens during treatment -managed with steroids.

LEPTOSPIROSIS

Organisms

• L. interrogans- infects puppies and children may be infected from


contamination with dog urine

• L. icterohaemorrhagica - may be contracted by sewer workers from


infected rat urine. Causes Weil’s disease *

• Survive for long periods in stagnant water / wet soil and infection
occurs through abraded or intact skin, conjunctiva or mucous
membranes *
• Spread through blood to kidneys causing leptospiuria which
perpetuates the cycle of infection *

Clinical features (Weil’s disease) *

• Fever, jaundice, sub-conjunctival haemorrhage, proteinuria, hepato-renal


failure

Diagnosis

• Blood - dark-ground microscopy

• Urine - dark-ground microscopy

• Serology -ok paired samples to demonstrate a rise in titres. Become


positive during the second week of illness

Treatment

• Benzyl penicillin

• Tetracycline in penicillin allergy


STREPTOCOCCI

• Gram positive

• Capsulated, aerobic

• May form chains (S. viridans) or diplococci (S. pneumoniae)

• Grow on blood agar - may cause haemolysis

Alpha-Haemolytic

• Green colour around each colony due to altered haemoglobin

• S. viridans and S. pneumoniae

• Differentiated by Optochin disc test (S. pneumoniae shows zone of inhibition) and bile solubility
test (S. pneumoniae is soluble)

• S. pneumoniae isolated from nose / throat in 70% of healthy individuals

• S. viridans - isolated from the mouth, causes bacterial endocarditis

Beta-Haemolytic
• Complete lysis of red cells

• S. pyogenes, S. agalactiae, S. fecalis

• Differentiated by Lancefield grouping and Bacitracin test

• Lancefield group A: S. pyogenes- causes sore throat, skin and wound infections (erysipelas,
impetigo), rheumatic fever and acute glomerulonephritis. Strains causing scarlet fever produce
an exotoxin

• Lancefield group B: S. algalactiae - isolated from vagina, perineum and rectum; causes
neonatal septicaemia and meningitis

• Lancefield group D: S. fecalis - usually non-haemolytic. Isolated from gut, causes UTIs and
bacterial endocarditis

Gamma Haemolytic

• Non haemolytic

• S. fecalis

• Streptococci are characteristically sensitive to penicillin / erythromycin

STAPHYLOCOCCI ****

• Gram positive
• Grow on blood agar

• Differentiated by coagulase test

• S. aureus: coagulase positive *- produce clot when mixed with serum. Also catalase positive
and produces exotoxin (toxic shock syndrome, toxic epidermal necrolysis). Non-motile *

• S. aureus produces yellow - orange pigmented colonies *

• Plasmid mediated antibiotic resistance - can spread between different strains by transducing
phages. Produces beta-lactamase (penicillinase)

• Methicillin-resistant S. aureus is resistant to flucloxacillin but usually sensitive to vancomycin

• S. aureus diseases include wound, bone and joint infections, breast abscess, skin and food-
borne infections, toxic shock syndrome. Isolated from the nose in 10-30% of healthy individuals

• S. epidermidis is coagulase negative

TOXIC SHOCK SYNDROME (TSS) *****

• Rare, life-threatening inflammatory response syndrome characterized by fever, rash,


hypotension, constitutional symptoms, and multi-organ involvement

• Most often associated with the use of super-absorbent tampons and occasionally with the
use of contraceptive sponges. *

• Associated with infection with toxigenic strains of Staphylococcus aureus*

• The infection can also affect men, children and postmenopausal women.
• Signs and symptoms develop suddenly

Fatal ~ 5%.

• TSS has been linked to many bacterial infections, including pneumonia, osteomyelitis,
sinusitis, and skin and gynecologic infections

• Endotoxin toxic shock syndrome toxin-1 (TSST-1) is the major toxin produced by strains
of S aureus that are responsible for causing TSS.

• Streptococcus pyogenes exotoxin A (SPEA) and S pyogenes exotoxin B (SPEB) are the
major toxins produced by group A beta-hemolytic streptococci.

• The toxins activate production of cytokines, such as tumor necrosis factor, interleukin-1, M
protein, and gamma-interferon and induction of nitric oxide production which contributes to
hypotension. *

BACILLUS SPECIES

• B. anthracis - gram positive aerobic spore-forming bacilli; capsulated

• Grows on blood agar

• Spores may remain dormant in the environment for several years

• Infects many animals including sheep


• Causes anthrax

• Produces an exotoxin

• B. cereus contaminates food such as boiled rice, causing food poisoning

• Sensitive to penicillin

CORYNEBACTERIUM DIPHTHERIAE

• Gram positive bacillus

• Isolated on Loeffler's serum agar slop

• Ferments glucose and maltose but rarely sucrose

• Produces an EXOTOXIN causing diphtheria

• Sensitive to penicillin; treatment with anti-toxin should be started once infection is suspected

• Not part of normal throat flora

• Incubation period 1-7 days

• Complications include polyneuritis (including cranial and peripheral nerve lesions),


cardiovascular collapse and renal involvement causing proteinuria
ACTINOMYCETE ISRAELII *****

• Gram positive branching filamentous bacillus *

• Anaerobic *

• Form part of the normal flora in the mouth, vagina and rectum.*

• Non-acid -fast (Nocardia are weakly acid-fast)

• Infection is characterised by sulphur (yellow) granules in pus *

• Does not involve the lymphatics *

• Causes cervico-facial, pulmonary and abdominal infection, causing multiple discharging


sinuses

• Genital tract infection associated with the IUCD *

• Sensitive to penicillin *

CLOSTRIDIA

• Gram positive anaerobic bacilli


• Spore-forming - the spores are resistant to boiling and chemical disinfectants

• Commonly found in the environment and as part of normal fecal flora

• Produce EXOTOXINS

• Infection common in gangrenous / ischaemic tissue and in association with foreign bodies

• Sensitive to metronidazole and penicillin

• CLOSTRIDIUN DIFFICILE - causes pseudo-membranous colitis - diarrhoea with collapse


following broad spectrum antibiotic treatment - treated with oral metronidazole or vancomycin

• CLOSTRIDIUM PERFRINGENS - Causes gas gangrene (causing crepitus) and septic abortion
(rare)

• Produces exotoxin causing septic shock, jaundice, haemolysis and acute renal failure

• Sensitive to penicillin - surgical debridement usually required

Clostridium Tetani *****

• Flagellated bacilli. Spores are resistant to boiling and autoclaving is required

• Incubation period 3-21 days

• Produces EXOTOXIN which is non-immunogenic therefore infection does not confer immunity.
Neurotoxin, inhibits release of inhibitory neurotransmitters in the CNS causing tetany / muscle
spasms
• There is no local inflammation associated with tissue infection

• Infection prevented by vaccination with tetanus toxoid in infancy followed by a booster dose
every 5-10 years

• Passive immunisation with human tetanus immune globulin + active immunisation required in
at-risk individuals following injury

• Antibiotic treatment is an ancillary measure and should include penicillin +/- flucloxacillin.
Removal of devitalised tissue, good tissue perfusion and oxygenation are essential *

LISTERIA MONOCYTOGENES *****

• Gram positive coccobaccilus *

• Motile. Ubiquitous in the environment but is not a gut commensal

• Wide-spread among animals and in the environment

• Facultative anaerobe, non-spore-forming *

• Beta-haemolytic

• Produces exotoxin (haemolysin) which damages leukoytes

• May grow within macrophages


• Multiplies slowly at refrigeration temperatures *

• Present in undercooked meats, soft cheeses, pate, coleslaw, contaminated milk. Can be
contracted from animal manure, silage and abortus material *

• Produces disease in circumstances associated with depressed T-cell immunity -


malignancy, steroids, AIDS, pregnancy

• Disease more common in third trimester of pregnancy. Infection in first trimester associated
with miscarriage. Third trimester infection associated with fetal death, meconium stained liquor,
chorio-amnionitis *

• Crosses placenta *

• Causes early (vertically acquired) and late (horizontally acquired) onset neonatal disease *

• May be isolated from vagina in infected women *

• Listeriosis involves many organs with micro-abscesses and granulomas

• Sensitive to ampicillin - main therapy *

• Also sensitive to trimethoprim, gentamicin, terracycline, rifampicin and chloramphenicol.


Resistant to cephalosporins *

• In non-pregnant, typically causes meningitis


SULPHONAMIDES *****

• Derivatives of p-aminobenzoic acid

• Inhibit bacterial synthesis of tetrahydrofolic acid *

• Active orally, metabolised by the liver and excreted by the kidneys

• Recognised cause of Stevens-Johnson syndrome

• Resistance is widespread - plasmid mediated

• Administered in combination with trimethoprim as


COTRIMOXAZOLE *

TRIMETHOPRIM *****

• Folic acid analogue

• Inhibits dihydrofolate reductase

• Similar structure and mechanism of action to methotrexate and the


anti-malarial agent pyrimethamine

• Absorbed from the gut

• Excreted by the kidneys

• Effective against gram negative bacilli EXCEPT pseudomonas

• Resistance is mediated by a plasmid-encoded dihydrofolate


reductase which has a much reduced affinity for trimethoprim

QUINOLONES *****

• Nalidixic acid, ciprofloxacin, norfloxacin

• Inhibit the activity of DNA gyrase, preventing supercoiling of


bacterial chromosome

• Active orally although significant systemic concentrations are not


achieved with nalidixic acid

• Excreted by the kidneys

• Nalidixic acid is only effective against enterobacteria and its use is


confined to the treatment of UTIs

• Other quinilones are effective against intracellular organisms -


rickettsiae, Chlamydia, L. pneumophilia, S. typhi

• Ciprofloxacin effective against P. aeruginosa *


• Resistance to quinolones is chromosomally mediated - no
evidence for plasmid mediated resistance

ANTI TB DRUGS *****

• Rifampicin - side effects - hypersensitivity, hepatitis

• Isoniazide - neuropathy (prevented with pyridoxine), hepatitis

• Ethambutol - optic neuritis (reduced visual acuity, red-green colour


blindness), peripheral neuropathy

• Pyrazinamide - arthralgia, hepatitis

• Streptomycin

Rifampicin

• Binds to bacterial RNA polymerase preventing mRNA synthesis

• Well absorbed from the gut

• Crosses blood-brain barrier

• Has an affinity for plastic - effective in patients with prosthesis

• Metabolised and excreted by the liver / bile

• Results in orange colour in urine, sweat and saliva

• Reserved for the treatment of TB

METRONIDAZOLE *****

• A nitroimidazole

• Pro-drug, converted to active metabolite by reduction after entering


microbial cell *

• Well absorbed from the gut / rectum *

• Effective only against anaerobes and protozoa such as Entamoeba


histolitica, trichomonas vaginalis, Gardnerella vaginalis, and
Giardia lamblia *
• Used for surgical prophylaxis following abdominal / pelvic surgery

• Effective treatment for pseudomembranous colitis

• Rectal administration is an effective alternative to iv therapy *

• Causes a disulfiram-like reaction with alcohol *

• Excreted by the kidneys

NITROFURANTOIN

• Absorbed from the gut and excreted in the uring in high enough
concentrations to inhibit bacteria

• Effective only in acid urine

Effective in UTI prophylaxis


TOXOPLASMA GONDII

• Obligate intracellular protozoan

• Completes part of its life cycle in animals (mainly cats but also sheep, cattle and pigs)

• Contracted by contact with cat litter, infected soil or eating undercooked beef, lamb or pork.
Prevented by wearing gloves during gardening, thorough food hygiene and avoiding contact
with cat litter

• Oocysts or tissue cysts invades the intestinal epithelium , develop into trophozoites which
spread via the lymphatic system

• Infection is usually mild or asymptomatic with lymphadenopathy (posterior cervical) being the
main presentation. Causes severe disease with encephalitis and chorioretinitis in the
immunocompromised

• Risk of fetal infection increases with increasing gestation age (17% first, 25% second and 65%
third trimester)

• Risk of fetus being affected FALLS with increasing gestation age

• Recognised cause of congenital anomalies with chorioretinitis, intra-cranial calcification,


microcephaly, hydrocephalus. Increased risk of miscarriage with first trimester infection

• Diagnosis can be made by culture in laboratory rodents

• Detection of IgM or a 4 fold rise in IgG titres in samples 3 weeks apart is indicative of acute
infection

• Sensitive to spiramycin and pyramethamine


ZOONOSES

• Diseases of vertebrate animals that can be transmitted to man.

• Mode of transmission includes direct contact, food and milk products, saliva, faeces, urine,
blood or tissue

• Other diseases are transmitted by arthropod vectors.

Bacterial zoonoses

1) Tuberculosis - bovine tuberculosis via infected unpasteurised milk

2) Salmonellosis - infect intestines of poultry, cattle, pigs. Cause food


poisoning

3) Brucellosis - Brucella abortus from cattle and other species from sheep and
pigs. Infection acquired through contact with infected vaginal secretions or
products of conception (vets / farm workers)

4) Campylobacter infection -farm animals, cause food poisoning

5) Leptospirosis - infected urine of farm animals or small wild rodents

6) Anthrax - Bacillus anthracis - gram positive, spore-forming bacillus, causes


disease in cattle and sheep. Infects workers handling contaminated hides and
other animal products
7) Listeriosis - Listeria monocytogenes -infects a variety of farm animals.
Causes important infection in pregnancy and neonates

8) Plague - Yersinia pestis, gram negative bacillus. Rats are animal reservoir
and the infection is passed to humans through the rat flea

9) Tetanus - Clostridium tetani, frequently found in intestines of many animals


including horses and cattle and occasionally humans. Humans infected when
spores germinate under anaerobic conditions

10) Lyme disease - Borrelia burgdorferi, animal reservoir is the deer, passed to
humans by the hard ixodes tick

Rickettsial zoonoses

• These include Q fever and typhus

Chlamydial zoonoses

• Ornithosis and Psittacosis

Viral zoonoses

• Rabies

• Lassa fever
Protozoal zoonoses

• Toxoplasmosis

• Leishmaniasis

• Trypanosomiasis - sleeping sickness and Chagas disease

Arthropod-borne infections include:

• Yellow fever, dengue fever, haemorrhagic fever (viral) - housefly, sand fly, mosquitoe, ticks &
mites *

• Plague, tularaemia (bacterial) - lice, fleas, housefly, ticks & mites*

• Q-fever, Epidemic typhus (rickettsial) - lice, fleas, housefly, ticks & mites *

• Relapsing fever, Lyme disease (spirochaetes) - lice, fleas, housefly, ticks & mites

• Sleeping sickness, Chagas disease (trypanosomes -protozoan) - sand flies, mosquitoe, ticks *

• Leishmaniasis (leishmania, protozoan) - sand fly, mosquitoe, ticks

• Malaria (plasmodium, protozoan) - sand fly, mosquitoe, ticks *


• Filariasis and onchocerciasis (nematodes, helminthic) - mosquitoe, black flies, midges

TREMATODES (FLUKES) *****

Schistosomiasis

• Caused by a worm - Schistosoma haematobium, S. mansoni and S. japonicum

• Vector is the fresh water snail *

• Infects hundreds of millions of humans world wide

• Adult schistosomes live in the pelvic, mesenteric and portal venous systems of
humans

• Disease caused by deposition of ova in the liver, walls of the large bowel, bladder and
ureter *

• Fibrosis and granulomas develop around the ova causing polyps, strictures and pre-
malignant change *

• S. haematobium endemic in Middle East and Africa, S. mansoni found in Latin


America also and S. japonicum in Far East only *

• Ova penetrate the bladder / rectal wall to be excreted in the urine or faeces. Hatch in
water and larvae infect a specific species of snails. Cercariae emerge from the snail and
penetrate the skin of humans

• Complications include fibrosis and calcification of the bladder, granulomatous polyps


and strictures of the bladder and ureter, haematuria, hydronephrosis, cirrhosis, portal
hypertension with oesophageal varices, strictures of the large bowel *
• Diagnosis - microscopy of urine / faeces or rectal biopsy for ova *

• Serology - becomes positive before ova become apparent - positive result may persist
for a long time after treatment

• Treatment - praziquantel as a single oral dose


Question 1: With respect to Varicella zoster infection

a. There is a 2% risk of causing congenital varicella syndrome if infection occurs after 20 weeks
gestation

True False
b. Infection is preventable by passive immunisation

True False
c. Passive immunisation results in life-long immunity to infection

True False
d. Neonatal infection is unlikely if the baby is delivered within 4 days of maternal disease

True False

Question 2: Herpes simplex virus

a. Is an RNA virus

True False
b. Type II herpes simplex virus typically causes oro-labial infection

True False
c. Type I herpes simplex virus can be isolated from genital sites

True False
d. Has an incubation period of 10-21 days

True False

Question 3: Prion agent

a. Is transmissible by tissue / organ transplantation

True False
b. Has an incubation period of 3-6 months

True False
c. Can be cultured in vitro

True False
d. Is vertically transmissible

True False

Question 4: With respect to infection with Hepatitis B virus

a. During infection, HBeAg is detectable before jaundice develops

True False
b. Anti-HBs antibody is detectable before jaundice develops

True False
c. The presence of HBeAg indicates that the infection has been cleared

True False
d. Detection of HBsAg may indicate previous immunisation

True False

Question 5: The following are caused by herpes simplex virus

a. Acute gingivostomatitis
True False
b. Cold sores

True False
c. Cervical warts

True False
d. Meningoencephalitis

True False

Question 6: Hepatitis B virus

a. Is sensitive to acyclovir

True False
b. The majority of infections are associated with clinical disease

True False
c. Chronic carriage is associated with persistence of anti-HBs IgG

True False
d. Causes more severe disease in pregnancy

True False

Question 7: Cytomegalovirus

a. Causes hepatitis

True False
b. Is found in 1% of all neonates

True False
c. Is a recognised cause of fetal congenital anomalies

True False
d. Is not present in breast milk

True False

Question 8: Features of congenital rubella include

a. Excretion of virus by the neonate

True False
b. Hepatomegaly

True False
c. Excessive production of growth hormone

True False
d. Cataract

True False

Question 9: Cytomegalovirus

a. Causes a heterophile positive mononucleosis

True False
b. Causes post-auricular and sub-occipital lymphadenopathy

True False
c. Primary infection occurs in 0.2% of pregnant women

True False
d. Is the most common congenital infection

True False

Question 10: The human immuno-deficiency virus

a. Can be transmitted by infusion of blood products

True False
b. Sero-conversion occurs within 6 weeks of infection

True False
c. Needle stick injury involving an infected patient carries a 10% risk of transmitting the infection

True False
d. Risk of infection following needle stick injury is higher than for hepatitis B virus

True False

Question 11: Human papilloma virus

a. Causes juvenile laryngeal papilomatosis

True False
b. Is not sexually transmissible

True False
c. Is associated with oral carcinoma

True False
d. Is sensitive to acyclovir

True False

Question 12: Rubella

a. Is caused by at least 6 different viral serotypes

True False
b. Infection is treated with famcyclovir

True False
c. Is spread by droplet infection

True False
d. Vaccination is necessary only in females

True False

Question 13: The human immuno-deficiency virus

a. Is a retro-virus

True False
b. Contains DNA

True False
c. Infects CD4 positive cells only

True False
d. Can be transmitted by breastfeeding
True False

Question 14: Cytomegalovirus

a. Can be cultured from the urine of infected patients

True False
b. Remains dormant within the host, causing re-infection

True False
c. Is excreted for up to 7 days after infection

True False
d. Causes severe disease in transplant patients

True False

Question 15: Hepatitis C virus

a. Is a DNA virus

True False
b. Is spread by the feco-oral route

True False
c. Is not vertically transmissible

True False
d. Is associated with chronic carriage

True False
Question 1: Vibrio cholerae

a. Is a gram negative baccilus

True False
b. Produces an exotoxin

True False
c. Is distinguishable from coliforms by gram staining

True False
d. Is sensitive to trtracycline

True False

Question 2: Streptococci

a. Are gram positive organisma

True False
b. Are anaerobic

True False
c. Have a capsule

True False
d. Can be cultured on blood agar

True False

Question 3: CLOSTRIDIA

a. Are anaerobic bacilli

True False
b. Are non-spore-forming

True False
c. Are readily killed by boiling water or chemical disinfectants

True False
d. Are commonly found in the environment

True False

Question 4: Legionella pneumophilia

a. Is spread by contaminated shower mixers

True False
b. Is sensitive to penicillin

True False
c. Infection is treated with gentamicin

True False
d. Is resistant to erythromycin

True False

Question 5: Staphylococci

a. Are gram negative

True False
b. Coagulase positive cocci produce a clot when mixed with plasma

True False
c. S. aureus is coagulase negative

True False
d. Produce penicliinnase (beta-lactamase)

True False

Question 6: Schistosoma haematobium

a. Is a snail

True False
b. Is prevalent in China

True False
c. Causes chronic granulomatous lesions

True False
d. Infestation predisposes to cancer

True False

Question 7: Endotoxins

a. Are polypeptides

True False
b. Are released by living bacteria

True False
c. Act non-specifically on the reticulo-endothelial system

True False
d. Are converted into non-toxic toxoids by formalin

True False

Question 8: Baccilus anthracis

a. Is gram positive

True False
b. Is anaerobic

True False
c. Is non-spore forming

True False
d. Is capsulated

True False

Question 9: Actinomyces israelii

a. Is a rickettsia

True False
b. Forms yellow granules in pus

True False
c. Is a commensal in the mouth
True False
d. Is a commensal in the vagina

True False

Question 10: Exotoxins

a. Are lipopolysaccharide molecules

True False
b. Are converted into non-toxic toxoids by formaline

True False
c. Act at specific target sites

True False
d. Are highly antigenic

True False

Question 11: With respect to Streptococci

a. Beta-haemolytic streptococci can be differentiated by bile solubility test

True False
b. Alpha-haemolytic streptococci are differentiated by Lancefield grouping

True False
c. Lancefield group B streptococci are usually isolated from the vagina

True False
d. Lancefield group B streptococci are a recognised cause of neonatal septicaemia and
meningitis

True False

Question 12: With respect to Coliforms

a. Shigella is part of normal fecal flora

True False
b. Salmonella typhi may be found permanently in the biliary tract of carriers

True False
c. Klebsiella is characteristically sensitive to ampicillin

True False
d. Coliforms are facultative anaerobes

True False

Question 13: Bacteriodes organisms

a. Do not produce spores

True False
b. Grow in aerobic culture

True False
c. Are synergistic with coliforms

True False
d. Are typically resistant to penicillin
True False

Question 14: With respect to coliform bacteria

a. Coliforms are gram negative cocci

True False
b. E. coli is lactose fermenting

True False
c. Klebsiella is non-lactose fermenting

True False
d. Salmonella and shigella are lactose fermenting

True False

Question 15: Corynebacterium diphtheriae

a. Is a gram positive coccus

True False
b. Ferments glucose and maltose

True False
c. Produces an endotoxin

True False
d. Is sensitive to penicillin

True False
Question 1: The following are recognised treatment for gonococcal infection

a. Penicillin + probenecid

True False
b. Doxycycline

True False
c. Azythromycin

True False
d. Spectinomycin

True False

Question 2: Chlamydia organisms

a. Are intracellular

True False
b. Infect squamous cells

True False
c. Are found in birds

True False
d. Cause trachoma

True False

Question 3: Chlamydia trachomatis

a. Can be detected by culture of endo-cervical swabs on blood agar

True False
b. Can be detected by culture of high vaginal swabs on blood agar

True False
c. Is an intracellular organism

True False
d. Produces infectious particles called elementary bodies

True False

Question 4: Bacterial vaginosis

a. Can be treated with metronidazole

True False
b. Causes an inflammation of the vaginal skin

True False
c. Is characterised by a positive amine test

True False
d. Is associated with candida albicans infection

True False

Question 5: Candida albicans

a. Is cultured on alkaline media

True False
b. Forms pseudohyphae

True False
c. Reproduces by budding

True False
d. Is associated with diabetes mellitus

True False

Question 6: Leptospirosis (Weil?s disease)

a. Is associated with jaundice

True False
b. Is transmitted to humans from cats

True False
c. Is transmitted to humans from rats

True False
d. Infection is usually via skin

True False

Question 7: Treponema pallidum

a. Is sensitive to penicillin

True False
b. Is sensitive to erythromycin

True False
c. Is sensitive to tetracycline

True False
d. Treatment may result in the Jarisch-Herxheimer reaction

True False

Question 8: Bacterial vaginosis

a. Is characterised by a fishy vaginal discharge

True False
b. The vaginal pH is < 4.5

True False
c. Is characterised by clue cells on wet vaginal smear

True False
d. Is associated with an increased number of lactobacilli in the vaginal flora

True False

Question 9: The following are recognised causes of a biological false positive serological test for
syphilis

a. Drug addiction

True False
b. Leprosy

True False
c. Recent malaria infection
True False
d. Recent immunisation

True False

Question 10: Leptospirosis (Weil?s disease)

a. Is a riskettsial infection

True False
b. Is caused by a gram negative cocobacillus

True False
c. Is frequently transmitted to man from inanimate objects

True False
d. Is transmitted in pasteurised cow?s milk

True False

Question 11: The following are recognised consequences of Chlamydia trachomatis infection

a. Interstitial pneumonitis

True False
b. Infertility

True False
c. Cervical intra-epithelial neoplasia

True False
d. Bartholinitis

True False

Question 12: Candida albicans

a. Infection of the vagina can be treated with oral fluconazole

True False
b. Infection of the vagina can be treated with topical co-trimoxazole

True False
c. Is commonly isolated from the vagina in post-menopausal women

True False
d. Is commonly isolated from the vagina in pre-pubertal girls

True False

Question 13: Chlamydia trachomatis

a. Produces reticulate bodies which can survive outside cells

True False
b. Can be detected using the polymerase chain reaction on early morning urine specimens

True False
c. Can be detected by the ELISA test on endocervical swabs

True False
d. Infection can reliably be diagnosed by the Chlamydia antibody test
True False

Question 14: Candida albicans

a. Is gram positive

True False
b. Is a commensal in the bowel

True False
c. Is sensitive to miconazole

True False
d. Causes secondary infection after treatment with broad spectrum antibiotics

True False

Question 15: The following are recognised consequences of Chlamydia trachomatis infection

a. Conjunctivitis

True False
b. Fitz-Hugh-Curtis syndrome

True False
c. Cervicitis

True False
d. Vaginitis

True False
Question 1: The following are effective disinfectants for use on the skin

a. Isopropyl alcohol

True False
b. Chlorhexidine

True False
c. Formaldehyde

True False
d. Benzalkonium chloride

True False

Question 2: The following are the mechanisms of action of anti-fungal agents

a. Amphotericin B ? binds ergosterol producing leakage of cellular contents

True False
b. Nystatin ? inhibit fungal DNA synthesis

True False
c. Flucytosine ? inhibit fungal DNA synthesis

True False
d. Griseofulvin ? bind ergosterol, producing leakage of cellular contents

True False

Question 3: The following anti-viral agents are effective against the viruses listed

a. Acyclovir ? herpes simplex virus

True False
b. Acyclovir ? varicella zoster virus

True False
c. Acyclovir ? cytomegalovirus

True False
d. Acyclovir ? Epstein-Barr virus

True False

Question 4: Microglide antibiotics

a. Include erythromycin

True False
b. Include gentamicin

True False
c. Include azithromycin and clarithromycin

True False
d. Include clindamycin

True False

Question 5: Tetracyclines

a. Inhibit bacterial protein synthesis

True False
b. Are not active against intracellular bacteria

True False
c. Are excreted via the liver

True False
d. Are poorly absorbed from the gut

True False

Question 6: Tetracyclines

a. Are effective against rickettsiae infections

True False
b. Are effective against mycoplasma infections

True False
c. Do not cross the placenta

True False
d. Are contra-indicated in women of child-bearing age

True False

Question 7: Sulphonamide antibiotics

a. Inhibit bacterial protein synthesis

True False
b. Inhibit bacterial cell wall synthesis

True False
c. Are derivatives of p-aminobenzoic acid

True False
d. Inhibit the synthesis of tetrahydrofolic acid

True False

Question 8: The following are effective disinfectants for use on the skin

a. 100% alcohol

True False
b. 70% alcohol

True False
c. Iodine

True False
d. Mercuric chloride

True False

Question 9: Beta-lactam antibiotics

a. Inhibit bacterial cell wall synthesis

True False
b. Inhibit peptidoglycan formation

True False
c. Act within the cell membrane
True False
d. Are effective against gram negative bacilli

True False

Question 10: The following are target sites for the action of anti-bacterial agents

a. Exotoxin synthesis

True False
b. Nucleic acid synthesis

True False
c. Cell membrane function

True False
d. Amino acid transport

True False

Question 11: The following drugs are effective against Mycobacterium tuberculosis

a. Gentamicin

True False
b. Pyridoxine

True False
c. Pyrimethamine

True False
d. Streptomycin

True False

Question 12: The following are effective methods of sterilising surgical instruments

a. Pasteurisation

True False
b. Boiling

True False
c. Ethylene oxide

True False
d. Liquid glutaraldehyde

True False

Question 13: Microglide antibiotics

a. Inhibit bacterial protein synthesis

True False
b. Are well absorbed from the gut

True False
c. Are excreted by the kidneys

True False
d. Are not effective against intracellular organisms
True False

Question 14: The following anti-viral agents are effective against the viruses listed

a. Ganciclovir ? cytomegalovirus

True False
b. Acyclovir ? HIV

True False
c. Foscarnet ? cytomegalovirus

True False
d. Zidovudine ? HIV

True False

Question 15: The following are recognised side-effects of anti-tuberculous drugs

a. Isoniazide ? peripheral neuropathy

True False
b. Ethambutol ? optic neuritis

True False
c. Rifampicin ? hypersensitivity reactions

True False
d. Rifampicin ? reduced visual acuity

True False
NORADRENALINE SYNTHESIS *****

• L-tyrosine to l-dihydrophenylalanine (levodopa; rate limiting step -


tyrosine hydroxylase) *

• Levodopa to dopamine (aromatic l-amino acid decarboxylase)

• Dopamine taken up into vesicles

• Dopamine to noradrenaline (dopamine beta-oxidase) *

• Noradrenaline to adrenaline

ALPHA ADRENERGIC EFFECTS *****

• Gut - reduced motility, peristalsis and secretion *

• Pupils - dilate *

• Bladder neck - contraction of smooth muscle *

• Seminal vesicles and vas deferens - contraction of smooth muscle

• Blood vessels - vasoconstriction *

• Pilomotor smooth muscle - contraction

BETA-ADRENERGIC EFFECTS *****

• HEART - tachycardia, increased cardiac output *

• GUT - decreased motility and peristalsis /


decreased secretion *

• ADIPOSE TISSUE - increased lipolysis *

• LIVER - glycogenolysis, increased


gluconeogenesis, hyperglycaemia *

• RESPIRATORY TRACT - bronchodilatation and


decreased secretions *

• BLOOD VESSELS - vasodilatation *

• UTERUS - smooth muscle relaxation *

1
• SKELETAL MUSCLE - tremor, glycolysis,
vasodilatation *

RITODRINE - SIDE-EFFECTS *****

These are the side-effects of beta-adrenergic agonists

• CVS - tachycardia, palpitations, hypotension, pulmonary oedema,


chest pain and tightness

• Gut - nausea, vomiting

• Skeletal muscle - tremor

• Skin - sweating

• Electrolytes - hypokalaemia

• Hyperglycaemia

BETA-BLOCKERS - SIDE EFFECTS *****

• CVS - hypotension (therapeutic effect), bradycardia, peripheral


vasoconstriction *

• Bronchospasm *

• Fatigue

• Sleep disturbance

METHYLDOPA *****

• Converted to alpha methyldopamine by aromatic l-amino acid


decarboxylase

• Alpha methyldopamine converted to alpha methylnoradrenaline

• Acts centrally, reducing the activity of central noradrenergic and


dopaminergic neurons with minimal peripheral effects. Alpha-
methlynoradrenaline is as effective as noradrenaline on peripheral
tissue

Side-effects

• CNS - depression, mild psychosis, Parkinsonism, nightmares, sedation

• GI - dry mouth, stomatitis, hepatitis, abnormal LFTs

2
• Neuromuscular - myalgia, arthralgia, paraesthesia

• BLOOD - Haemolytic anaemia, thrombocytopaenia, leucopaenia, bone


marrow depression

• Hypersensitivity - drug fever, rash, SLE-like syndrome

• Hyperprolactinaemia, loss of libido, impotence, gynaecomastia,


amenorrhoea

• Oedema

3
CHOLINERGIC NEURONS
• Synthesize, store and release acetylcholine as their
neurotransmitter. Include:

• All pre-ganglionic autonomic neurons

• All post-ganglionic parasympathetic neurons

• All lower motor neurons

• Some neurons within the CNS

• Sympathetic post-ganglionic neurons have noradrenaline as the


neurotransmitter except those supplying the eccrine sweat glands and
the arterioles of skeletal muscles which are cholinergic

The following steps occur during cholinergic transmission

• Conduction of action potential down the axon

• Release of acetylcholine from axon terminal

• Interaction of acetylcholine with receptors on membrane of post-


junctional cell, resulting in post-junctional event

• Hydrolysis of acetylcholine by acetylcholinesterase, forming


choline and acetic acid

• Re-uptake of choline for synthesis of acetylcholine

NICOTINIC RECEPTORS

• Located over the whole of the cell bodies of autonomic ganglia

• Found only in the motor end plate of skeletal muscle

Nicotinic Agonists

• Acetylcholine

• Carbachol

• Nicotine

• Suxamethonium

1
EFFECTS - Skeletal muscle contraction followed by depolarising blockade of
neuromuscular transmission; indirect sympathetic and parasympathetic
effects by action at nicotinic receptors of autonomic ganglia

SUXAMETHONIUM *****

• Nicotinic agonist

• Initial iv administration causes asynchronous twitches of


individual muscle fibres (fasciculation) followed by depolarisation
blockade *

• Paralysis lasts ~6 minutes - hydrolysed by cholinesterase

• Cholinesterase deficiency causes prolonged paralysis NOT


loss of consciousness *

NICOTINIC ANTAGONISTS

• Tubocurarine *
• Pancuronium
• Atracurium - useful in patients with hepatic / liver impairment as
inactivated by spontaneous dissociation
• Hexamethonium *

These are competitive antagonists. Alpha-bungarotoxin is a non-competitive


nicotinic antagonist

TUBOCURARINE *****

• Competitive nicotinic antagonist *

• Causes histamine release and ganglion blockade with


hypotension *

• Causes muscle paralysis - use requires the patient to be ventilated

• Reversible with anti-cholinesterases like neostigmine *

• Has important interaction with gentamicin which potentiates its


effects

2
MUSCARINIC AGONISTS

• Methacholine

• Carbachol

• Pilocarpine

EFFECTS OF MUSCARINIC AGONISTS *****

• Gut - increased motility and increased secretion *

• Heart - bradycardia *

• Eye - contraction of ciliary muscle and circular smooth


muscle of the iris *

• Lacrimal, eccrine sweat and salivary glands - increased secretion

• Respiratory tract - increased secretion and smooth muscle


contraction *

• Detrusor muscle - contraction *

• Erectile tissue - vasodilatation *

MUSCARINIC ANTAGONISTS *****

• Atropine *

• Hyoscine

• Tropicamide

• Ipratropium bromide

• Benzhexol - used in Parkinson’s disease

• Imipramine - tricyclic anti-depressant

• Quinidine - anti-dysrhythmic drug

3
All competitive antagonists. There are at least three sub-types
of muscarinic receptors with Pirenzipine being a specific M1
antagonist (gastric parietal cells)

EFFECTS OF MUSCARINIC ANTAGONISTS *****

• Gut - decreased motility and secretion *

• Lacrimal, salivary and eccrine sweat glands - decreased


secretion *

• Heart - tachycardia *

• Pupils - dilatation *

• Ciliary muscles - relaxation *

• Respiratory tract - smooth muscle relaxation and decreased


secretion

• Detrusor - relaxation *

MUSCARINIC ANTAGONISTS - SIDE-EFFECTS *****

• GUT - constipation, dry mouth, nausea, vomiting, abdominal


discomfort

• EYES - blurred vision

• URINARY TRACT - difficulty in micturiction / urinary retention

• HEART - palpitations, tachycardia, arrhythmia

• CNS -stimulation - restlessness and convulsion

• SKIN - reduced sweating, rash

ANTI-MUSCARINIC DRUGS FOR URINARY FREQUENCY, ENURESIS


AND URGE INCONTINENCE

• Oxybutynin is the standard

• Tolterodine

4
• Flavoxate hydrochloride

• Propantheline bromide

• Propiverine hydrochloride

• Trosopium chloride
• Tricyclic antidepressants such as imipramine and amitriptyline

Benzhexol is an anti-muscarinic agent used to control tremor and excessive


salivation in Parkinson’s disease

CHOLINESTERASE *****

• Found in blood

• 1 in 3000 individuals are homozygous for gene coding atypical


enzyme which hydrolyses suxamethonium very slowly resulting in
prolonged paralysis (not loss of consciousness)

• Atypical cholinesterase is resistant to inhibition by cinchocaine


(dibucaine) and assessment is made using the dibucaine number

CHOLINESTERASE INHIBITORS *****

COMPETITIVE

• Neostigmine *

• Physostigmine, pyridostigmine, edrophonium, carbaryl

NON-COMPETITIVE

• Malaoxon and Malathion

• Used in the diagnosis and treatment of myasthenia gravis:


Edrophonium leads to a transient increase in muscle strength in
myasthenia. Treatment is with neostigmine with muscarinic antagonists
to minimise side-effects

• Neostigmine used to reverse neuromuscular blockade


induced by nicotinic antagonists tubocurarine / pancuronium *

5
• Organophosphorus compounds are non-competitive
cholinesterase inhibitors - poisoning is treated with atropine and
pralidoxime

• Cholinesterase inhibitors cause increased sweating, salivation,


bronchial secretion, miosis, bradycardia and diarrhoea.

6
LOCAL ANAESTHETICS *****

Lignocaine, bupivacaine

• Lipid soluble drugs which act by inhibiting the increase in


sodium *permeability which normally gives rise to action
potentials

• Usually weak bases combined with a strong acid (such as


lignocaine hydrochloride) *

• Small diameter neurons more readily blocked than large


diameter neurons

• Pain sensation is blocked first (small diameter fibres with a


high firing rate) followed by temperature, touch and pressure
and finally motor fibres

• Recovery from local anaesthesia is a result of diffusion from


the site of administration. The drugs are then metabolised by
plasma esterases or in hepatic microsomes

• Cause vasodilatation and myocardial depression

• Used in combination with adrenaline to cause


vasoconstriction and prolong the duration of local anaesthesia.
The concentration of adrenaline should NOT exceed
1:200,000 (5micrograms /ml) and the total dose of adrenaline
should NOT exceed 500 micrograms. Combinations with
adrenaline should not be used for digits or appendages as
there is a risk of ischaemic necrosis *

Side-effects

• CNS stimulation initially, causing restlessness, tremor and


convulsion

• CNS depression with drowsiness, slurred speech

• Cardiovascular collapse - hypotension, bradycardia and


myocardial depression *

• Circumoral paraesthaesia

• Respiratory failure

1
Lignocaine

• Has a more rapid onset but a shorter duration of action


compared to bupivacaine (duration of action up to 8 hours) *

• Lignocain is useful in treating cardiac arrhythmias

• Following the onset of regional anaesthetic procedure,


maximal arterial concentration of local anaesthetic (therefore
likelihood of toxicity) is attained at 10-25 minutes -therefore
careful surveillance required for the first 30 minutes.

2
INHALATION ANAESTHETICS *****

• Gases (Nitrous Oxide) or volatile liquids (Halothane,


Enflurane, Isoflurane
• Used for both induction and maintenance of anaesthesia
• should be administered with oxygen concentrations
greater than in air to avoid hypoxia
• All may cause malignant hyperpyrexia
• All cause respiratory depression and hypotension
• Cause muscle relaxation but muscle relaxants usually
required for major abdominal surgery
• All should be administered with concentrations of oxygen
greater than air to prevent hypoxia

• HALOTHANE - causes hepatotoxicity especially with


repeated exposure. Avoid use of adrenaline infiltration as
ventricular arrhythmias may occur. Inhibits uterine
contractility. Vapour is non-irritant, pleasant to inhale and
rarely causes coughing or breath-holding. *
• ENFLURANE - less potent than halothane; powerful
cardio-respiratory depressant
• NITROUS OXIDE - 50-70% in oxygen - anaesthetic; 50%
in oxygen used as analgesia in labour / minor surgery
(causes release of opioid peptides). Prolonged exposure
may cause macrocytic anaemia.

INTRAVENOUS ANAESTHETIC AGENTS *****

• Advantageous in providing rapid and pleasant induction,


no respiratory irritation, requiring simple equipment and
causing little post-operative vomiting

• Reversal of anaesthesia is secondary to drug redistribution

• Produce their effect in one arm-brain circulation time

• Not muscle relaxants and muscle relaxants are required to


facilitate intubation if iv anaesthetics used for induction *

• Highly lipid soluble and cross the placenta *

1
• THIOPENTONE SODIUM - barbiturate, narrow therapeutic
index, contraindicated in porphyria. No analgesic effects.
Highly alkaline (pH >10) and irritant if injection misplaced
(not suitable for im injection) *.

• Because of the alkalinity, thiopentone should not be mixed


in the same syringe as other drugs, as it may cause
formation of a cloudy precipitate and inactivate the drug.
Slowly metabolised by the liver. ~80% protein bound*.

• Its main use is for induction of anaesthesia. Following


induction anaesthesia is usually maintained by an
anaesthetic vapour

• An iv injection of thiopentone causes loss of


consciousness within 15 to 30 seconds, and lasts for 5 to 10
minutes. The onset time of thiopentone is approximately the
time it takes for the drug to travel from the vein in the arm to
the brain.

• It does not have any direct toxic effects on the liver or


kidney, but patients with liver or kidney disease may require
a lower dose range than 3 to 7 mg/kg. Although it crosses
the placenta it is a safe agent for induction in obstetric
anaesthesia

• Thiopentone is metabolised slowly, and the rapid recovery


is due to redistribution of the drug firstly into muscle and
skin, and later into body fat stores.

• PROPOFOL - rapid recovery with no hangover effect. May


cause severe bradycardia

• ETOMIDATE - used for induction only. Continuous


administration can cause adrenocortical suppression

KETAMINE - analgesic at sub-anaesthetic dose; associated with


increased muscle tone, hypertension and tachycardia.

2
GLUCOCORTICOIDS *****
• Anti-inflammatory agents; enhance fetal lung maturity and reduce
perinatal mortality and morbidity when administered to the mother
24h to 7 days before pre-term delivery (up to 36 weeks gestation) *

• Cortisone and hydrocortisone have significant miniralocorticoid activity


while this is minimal with prednisolone, betamethasone and
dexamethasone

• All are active orally

• Dexamethasone and betamethasone cross the placenta well. 88% of


prednisolone and most of hydrocortisone is inactivated as it crosses
the placenta. These drugs are not terratogenic. However, long term
treatment increases the risk of IUGR *

• Contraindicated in active systemic infection in the absence of anti-


microbial therapy. Increased risk of disseminated viral infection
especially varicella. Presentation of sepsis may be atypical

• Long term therapy associated with risk of pituitary-adrenal suppression.


This risk is reduced by administering the dose in the morning, using
short courses or alternate day therapy *

• Gradual withdrawal of therapy is recommended in patients treated for


over 3 weeks, receiving doses in the evening, taking over 40mg
prednisolone (or equivalent) daily, received repeated courses or taking
a short course within 1 year of stopping long term treatment

• Replacement required for patient undergoing major / minor surgery

• Steroid treatment cards should be carried by all patients on long term


treatment

SIDE-EFFECTS

1) GI - peptic ulceration, abdominal distension, acute pancreatitis

2) MUSCULOSKELETAL - proximal myopathy, osteoporosis, a vascular


necrosis of the femoral head

3) ENDOCRINE - adrenal suppression, Cushing's, menstrual irregularity,


amenorrhoea, hirsutism *

4) NEURO-PSYCHIATRIC - euphoria, depression, psychosis

5) EYE - cataract, glaucoma

6) Increased risk of sepsis *

1
7) Impaired wound healing, skin strophy, striae, acne *

8) Fliud and electrolyte disturbance *

ANTI-ANDROGENS *****

• Cyproterone acetate - progestogenic anti-androgen. Used in


combination with ethinyloestradiol as a combined oral contraceptive to
treat acne and hirsutism. Inhibits spermatogenesis (but is not a male
contraceptive) & libido and reduces sebum production *

• Spironolactone is an aldosterone antagonist with anti-androgenic


effects

• Finasteride is a 5-alpha reductase inhibitor - prevents the conversion of


testosterone to the active dihydrotestosterone

• Flutamide is an androgen antagonist


• Cimetidine displaces testosterone from its receptor
• Nandrolone is an anabolic steroid with androgenic effects while
danazol and norethisterone have androgenic effects

ANTI-THYROID DRUGS *****

• Carbimazole, propylthiouracil *

• Inhibit thyroxine synthesis *

• Propylthiouracil also inhibits the peripheral conversion of T4 to the


more active T3 *

• Not terratogenic; cross the placenta and enter breast milk. May cause
neonatal goitre / hypothyroidism therefore lowest effective dose should
be used and the neonata should be monitored *

• Principal side-effect is bone marrow suppression with agranulocytosis -


patients should be warned to report sore throats / fever / non-specific
illness

• Also associated with rash

• Iodine is occasionally used prior to thyroid surgery. Radioactive iodine


is contraindicated in pregnancy

2
HISTAMINE ANTAGONISTS *****

H1

• Smooth muscle contraction (GI / Respiratory), increased capillary


permeability, hypotension and stimulation of sensoey neurons are
mediated via H1 receptors

• Chlorpheniramine (Promethazine, Terfenadine)

• Side-effects - sedation, anorexia, nausea and vomiting, dizziness,


blurred vision, dry mouth. However, promethazine is an effective anti-
emetic

H2

• Mediate stimulation of gastric acid secretion, stimulation of the


heart and inhibition of antigenic release of histamine from basophils

• Cimetidine, Ranitidine

• Cimetidine (not ranitidine) binds androgen receptors and may


cause gynaecomastia. Also inhibits cytochrome P450 and may
potentiate drugs such as phenytoin and warfarin

• Sodium chromoglycate inhibits histamine release from mast cells.

DRUG-INDUCED HISTAMINE RELEASE *****

Caused by basic drugs:

• Morphine, pethidine, codeine phosphate *

• Tubocurarine *

• Vancomycin

• Atropine *

Caused by high molecular weight polymers such as dextran

Sodium chromoglycate inhibits histamine release

1
5HT ANTAGONISTS

• Pizotifen - 5HT 1&2 - effective prophylactic in migraine. Also


reduces increased gastric motility associated with carcinoid tumour.

• LSD - antagonist at 5HT2 but partial agonist at central 5HT1 and


dopamine receptors - hallucinogen

• Ondansetron - 5HT3 antagonist, useful antiemetic especially in


patients undergoing chemotherapy
• Ergotamine is a 5HT agonist

VOMITING AND ANTI-EMETICS

• Vomiting is typically caused by drugs that stimulate the


chemosensitive trigger zone (CTZ) or impair gastric function: dopamine
agonists (levodopa, apomorphine), opiates, oestrogens, digoxin,
chemotherapeutic agents especially cisplatin

ANTI-EMETICS *****

• Dopamine antagonists - Phenothiazines (prochlorperazine -


stemetil); metoclopramide, domperidone *

• Anti-histamines - Cyclizine, promethazine, betahistine (vomiting


associated with Meniere's disease) *

• Anti-muscarinic agents - Hyoscine (motion sickness)

• 5HT-3 antagonists - Ondansetron *

2
COMBINED ORAL CONTRACEPTIVE PILL *****

• Contain ethinyloestradiol (20-50mcg) or mestranol (a prodrug which


is converted to ethinyloestradiol) and a progestogen *

• Progestogen is second generation (Norethisterone or


levonorgestrel) or third generation (gestodene, desogestrel or
norgestimate)

• Side-effects include nausea and vomiting, weight change, headache,


DVT, chloasma, hypertension, impaired liver function *

• Contraindications - pregnancy, personal or strong family history of


thrombosis, focal migraine, active liver disease, undiagnosed vaginal
bleeding, porphyria, hormone dependent cancer, heart disease
associated with pulmonary hypertension *

• Important drug interactions with hepatic enzyme inducers, broad


spectrum antibiotics, St John’s wort (see drug interactions) *

NORETHISTERONE *****

• 19-Nortestosterone derivative

• Second generation progestogen

• The prodrug ethynodiol is converted to norethisterone in vivo

• Has androgenic effects

• Used in combined oral contraceptive pill formulations, progesterone-


only pill, depot-progesterone contraceptives (norethisterone enanthate
or medroxyprogesterone acetate)

CLOMIPHENE CITRATE *****

• Non-steroidal anti-oestrogen with weak oestrogenic effects *

• Active orally, has long half life and significant plasma concentrations
can be detected 1 month after a single 50mg dose

• Results in increased GnRH, FSH and LH levels which stimulate the


ovaries (therefore has indirect effect on ovaries) *

• Has no effects on androgen, progesterone or corticosteroid


receptors *

1
• Administered usually from days 2-6 of the cycle for ovulation
induction. Main use is ovulation induction in PCOS *

• Use should be limited to a 6 months course

• Contraindications - hepatic disease, ovarian cysts and hormone


dependent tumours *

• Side-effects - visual disturbance, hot flushes, ovarian


hyperstimulation, multiple pregnancy, hair loss, breast tenderness,
headache *

TAMOXIFEN *****

• Non-steroidal; anti-oestrogen in breast and used in adjuvant


treatment and secondary prevention of breast cancer

• Anti-oestrogenic on the pre-menopausal uterus

• Oestrogenic effects on the post-menopausal uterus - increased risk


of endometrial carcinoma / hyperplasia; fibroids

• Associated with increased risk of thromboembolism and ovarian


cysts

• Risk of osteoporosis is not increased despite anti-oestrogenic effect

• Side-effects include hot flushes, visual disturbance, amenorrhoea,


irregular vaginal bleeding, GI disturbance, hair loss, thromboembolism
(antagonise effects of warfarin) *

RALOXIFEN *****

• Selective oestrogen receptor modulator (SERM)

• Non-steroidal; active orally

• Used in the treatment and prevention of post-menopausal


osteoporosis

• Oestrogen-antagonist in breast

• Oestrogen antagonist / neutral on the uterus - may reduce the risk of


endometrial cancer in treated women

• Does not relieve hot flushes

• Side-effects include hot flushes, leg cramps, thromboembolism, GI


upset and flu-like illness

2
TIBOLONE *****

• Combines oestrogenic, progestogenic and weak androgenic activity

• Used in prophylaxis for postmenopausal osteoporosis and in the


treatment of menopausal symptoms and as add-back during GnRH
therapy

• Given orally and continuously without cyclical progesterone

3
BROMOCRIPTINE - DOPAMINE AGONISTS *****

• Ergot derivative

• Inhibits pituitary prolactin and growth hormone release and


results in shrinkage of pituitary adenomas. Useful as
treatment for galactorrhoea

• Rapid onset of action with reduction in prolactin levels


within 24-48h of initiating treatment

• Should be continued during pregnancy in women with


pituitary adenomas. Not terratogenic *

• Side-effects include postural hypotension with dizziness,


nausea, vomiting and constipation; peripheral vasospasm
with exacerbation of Raynaud’s phenomenon, dyskinesia,
hallucinations, dry mouth, leg cramps, retroperitoneal
fibrosis, pleural effusion *

• Other dopamine agonists with similar therapeutic


indications include carbegoline and quinagolide *

• Carbegoline has longer duration of action

DRUG-INDUCED HYPERPROLACTINAEMIA *****

• Pituitary prolactin production is under inhibitory control of


hypothalamic dopamine *

• Dopamine antagonists therefore increase prolactin


production - metoclopramide, phenothiazines, opiates,
methyldopa, reserpine (depletes dopamine stores).
Oestrogens stimulate prolactin synthesis and release *

• Cimetidine displaces testosterone from its receptor,


allowing background oestrogenic effects to be manifested

• Bromocriptine and carbegolline are dopamine agonists and


are used in the treatment of hyperprolactinaemia *

DRUG-INDUCED MOVEMENT DISORDERS

• Dopamine antagonists - metoclopramide - extra-pyramidal


side-effects

• Anti-psychotic agents such as haloperidol, fluphenazine


and chlorpromazine have anti-dopaminergic effects and

1
cause drug-induced Parkinsonism. They also cause tardive
dyskinesia

• Dopamine agonists such as bromocriptine and


carbegolline can cause dyskinesia

• Levodopa may cause abnormal involuntary movements

GnRH ANALOGUES *****

• GnRH is a decapeptide which is released in short pulses


from the hypothalamus, stimulating pituitary FSH / LH
production *

• Continuous administration causes down-regulation of


pituitary receptors, inhibiting FSH / LH production and
therefore ovarian / testicular activity and inducing a
menopausal state in women *

• Risk of osteoporosis with long-term use restricted to 6


months

• Used for pituitary desensitization prior to induction of


ovulation with gonadotrophins. Also used to treat
endometriosis, prostate cancer, fibroids and prepare the
endometrium prior to uterine surgery

• Analogues are administered by intramuscular, subdermal


injection and as nasal spray *

• Contraindicated in osteoporosis, pregnancy, breastfeeding

2
PROSTAGLANDINS *****

• Synthesized from arachidonic acid *

• Conversion of arachidonic acid to prostaglandin


endoperoxide by cyclo-oxygenase is inhibited by aspirin and
NSAIDs . Specific cyclo-oxygenase 2 inhibitors are less likely
to cause GI side-effects (meloxicam, sulindac) *

• Metabolised by the liver and tissues and excreted by the


kidney *

• Dinoprostone (PGE2) is used for induction of labour


(Prostin, propess). It is active orally (less active than following
vaginal administration) but side-effects are more frequent

• Side -effects: Nausea & vomiting, diarrhoea (increased


peristalsis), fever, hypertension, bronchospasm, uterine
hypertonus or hyperstimulation, fetal distress, placental
abruption, amniotic fluid embolism. *

• Dinoprost (PGF2-alpha) - used for pregnancy termination.


Similar side-effects to dinoprostone but may be more severe

• Gemeprost (PGE1) - used for cervical preparation prior to


surgical evacuation of the uterus and for medical termination of
pregnancy.

• Side-effects similar to PGE2

• Carboprost - administered by deep intramuscular or intra-


myometrial injection to treat severe post-partum haemorrhage

• Misoprostol (PGE1) - synthetic analogue of PGE1 - inhibits


gastric acid secretion and is used in the treatment of gastric
ulcers.

• Side-effects - diarrhoea, nausea, vomiting, abdominal pain,


abnormal vaginal bleeding including menorrhagia, inter-
menstrual and post-menopausal bleeding. *

• Unlicensed use for cervical preparation prior to surgical


evacuation of the uterus, medical termination of pregnancy and
induction of labour.

• Alprostadil (PGE1) - used to maintain a patent ductus


arteriosus in neonates with congenital heart disease.

1
Question 1: Norethisterone

a. Is a third generation progestogen


True False
b. Is a 19-nortestosterone derivative
True False
c. Is released from the prodrug ethynodiol
True False
d. Has anti-androgenic effects
True False

Question 2: Clomiphene citrate

a. Is a steroid
True False
b. Is an oestrogen antagonist
True False
c. Is active when administered orally
True False
d. Decreases hypothalamic GnRH release
True False

Question 3: Tibolone

a. Causes hot flushes


True False
b. Causes cyclical bleeding
True False
c. Can be used as add-back therapy in women treated with GnRH
analogues
True False
d. Is administered with progesterone in the last 10-14 days of the cycle
True False

1
Question 4: Aspirin

a. Does not cross the placenta


True False
b. Is contra-indicated in the first trimester of pregnancy
True False
c. Is a specific cyclo-oxygenase 2 inhibitor
True False
d. Is analgesic
True False

Question 5: Tamoxifen

a. Is a steroid
True False
b. Is an oestrogen antagonist in breast tissue
True False
c. Is an oestrogen agonist in the post-menopausal uterus
True False
d. Is associated with an increased risk of venous thromboembolism
True False

Question 6: The following drugs have anti-androgenic effects

a. Cimetidine
True False
b. Nandrolone
True False
c. Finasteride
True False
d. Flutamide
True False

2
Question 7: Raloxifen

a. Is associated with an increased risk of endometrial carcinoma


True False
b. Is associated with an increased risk of osteoporosis
True False
c. Is inactive when administered orally
True False
d. Causes leg cramps
True False

Question 8: Cyproterone acetate

a. Is a testosterone antagonist
True False
b. Inhibits spermatogenesis
True False
c. Is a progestogen
True False
d. Increases sebum production
True False

Question 9: The following are recognised side-effects of prednisolone

a. Muscle wasting
True False
b. Post-menopausal bleeding
True False
c. Peptic ulceration
True False
d. Amenorrhoea
True False

3
Question 10: The following drugs cause hyperprolactinaemia

a. Metoclopramide
True False
b. Bromocriptine
True False
c. Carbegolline
True False
d. Oestrogen
True False

Question 11: The following are recognised side-efects of anti-histamine


drugs

a. Ranitidine ? inhibition of hepatic cytochrome P450 enzymes


True False
b. Promethazine ? drowsiness
True False
c. Cimetidine ? gynaecomastia
True False
d. Promethazine ? nausea and vomiting
True False

Question 12: Aspirin

a. Is anti-pyretic
True False
b. Inhibits uric acid excretion by the kidneys
True False
c. Inhibits platelet aggregation
True False
d. Inhibits prothrombin formation
True False

4
Question 13: With respect to drug therapy with glucocorticoids

a. Corticosteroid replacement is not necessary in patients undergoing


minor surgical procedures who have been taking 10mg of prednisolone
daily within the last 3 months
True False
b. Steroid treatment cards should be carried by pregnant women
receiving betamethasone for fetal lung maturity
True False
c. Therapy is contraindicated in women with osteoporosis
True False
d. Hydrocortisone is inactive when administered orally
True False

Question 14: Tamoxifen

a. Increases the risk of osteoporosis


True False
b. Causes hot flushes in pre-menopausal women
True False
c. Is associated with an increased risk of ovarian cysts
True False
d. Is associated with a decreased risk of endometrial carcinoma
True False

Question 15: The following are recognised side-effects of the


prostaglandins listed

a. Dinoprostone ? exacerbation of asthma


True False
b. Misoprostol ? constipation
True False
c. Misoprostol ? exacerbation of asthma
True False
d. Misoprostol ? post-menopausal bleeding
True False

5
HEPARIN *****

• Sulphated mucopolysaccharide of varying molecular


weight

• Present naturally in the liver, lungs and mast cells *

• Requires the presence of plasma antithrombin III for


anticoagulant activity *

• High net negative charge - not absorbed orally and


does not cross the placenta. Not contraindicated in
breastfeeding / pregnancy. Should NOT be given
intramuscularly - risk of haematoma *

• Commercial preparations of heparin are


heterogeneous, their components having molecular
weights (mw) ranging from 3,000 to 30,000 (mean,
15,000).

• Only about one third of the heparin binds to


antithrombin III and this fraction is responsible for
most of its anticoagulant effect. The remaining two
thirds has minimal anticoagulant activity at therapeutic
concentrations.

• The heparin-antithrombin III complex inactivates a


number of coagulation enzymes, including thrombin
and activated factors X, XII, XI and IX. *

• Thrombin and activated factor X (factor Xa) are the


most sensitive to inactivation.

• Low molecular weight heparin (<5400 mw) do not


bind thrombin and antithrombin III simultaneously and
therefore are unable to accelerate the inactivation of
thrombin by ATIII, but retain the ability to catalyze the
inhibition of factor Xa by ATIII *

• Heparin sodium also prevents the formation of a


stable fibrin clot by inhibiting the activation of the fibrin
stabilizing factor.

• Bleeding time is usually unaffected by heparin


sodium. Clotting time is prolonged by full therapeutic
doses of heparin sodium; in most cases it is not
measurably affected by low doses of heparin.

1
• Patients over 60 years of age, following similar doses of
heparin, may have higher plasma levels of heparin and
longer APTTs compared with patients under 60 years of
age.

• Peak plasma levels of heparin sodium are achieved two


to four hours following subcutaneous administration. *

• Liver and the reticulo-endothelial system are the sites of


metabolism.

• Heparin sodium does not have fibrinolytic activity;


therefore, it will not lyse existing clots

• Low molecular weight heparins have a longer half life


than standard heparin *

• Causes thrombocytopaenia which is usually immune


mediated and does not occur until after 6-10 days of
therapy. Check platelet count in all patients on heparin for
more than 5 days.

• Therapy monitored by the APTT. Therapy with low


molecular weight heparin is monitored by the anti-Xa
activity

• Inhibits aldosterone secretion and may cause


hyperkalaemia - caution in diabetics, potassium sparing
diuretics, renal failure *

• Anticoagulant effect reversed by protamine sulphate

• Risk of alopecia and osteoporosis with long term use

• Used for DVT prophylaxis after major surgery and in the


treatment of DVT / PE

WARFARIN *****

 Vitamin K antagonist, inhibiting the synthesis of vitamin K


dependent clotting factors VII, IX, X and prothrombin *

 Absorbed orally, protein bound and metabolised by


hepatic MFO enzymes *

Crosses the placenta, is terratogenic and causes fetal


haemorrhage. Not contraindicated in breastfeeding *

2
 Therapy monitored by the prothrombin time. Severe
bleeding treated with clotting factors (FFP)

 Several important drug interactions including broad-


spectrum antibiotics which reduce vitamin K absorbtion
and potentiate anticoagulant effect *

 Oestrogens (including tamoxifen) are pro-thrombotic and


antagonise warfarin effects *

ANTI-PLATELET AGENTS

 Heparin - low molecular weight heparin has less effect on


platelet function

 Aspirin / NSAIDs

• Dipyridamole - inhibits phosphodiesterase,


resulting in increased platelet cAMP concentration
and decreased adhesiveness

ASPIRIN *****

 Non-selective cyclo-oxygenase inhibitor- inhibits


conversion of arachidonic acid to prostaglandin
endoperoxide *

 Analgesic, anti-pyretic

 Crosses the placenta and increases the risk of neonatal


and post-partum haemorrhage. Not contra-indicated in
the first trimester *

 Increases uric acid excretion by the kidneys and inhibits


platelet aggregation and prothrombin formation *

• Protein bound and has important drug interactions


with other protein-bound drugs such as warfarin
and methotrexate *

• Contraindicated in neonates and children because


of risk of Rye syndrome *

3
DIGOXIN

• Cardiac glycoside

• Inhibits magnesium dependent Na/K ATPase, resulting in increased


intracellular Na and calcium with decreased K

• Has positive inotropic effect (increases strength of myocardial


contraction) and negative chronotropic effect (reduces heart rate)

• Has low therapeutic index - small variations in dose can cause


toxicity

• Excreted mainly by the kidneys (80%), although hepatic clearance


becomes more important in renal failure. *Half life ~1-2 days with
normal renal function.

• Toxicity increased by hypokalaemia (diuretics, beta-agonists,


glucocorticoids) and hypomagnesaemia *

• Useful in the management of atrial fibrillation

TOXICITY

• Nausea, vomiting, diarrhoea

• Cardiac arrhythmias

• Blurred vision and disturbed colour vision

NIFEDIPINE - SIDE-EFFECTS *****

• Headache

• Hypotension and tachycardia

• Myocardial depression

• Constipation / diarrhoea

• Impotence and gynaecomastia

• Flushing (peripheral vasodilation) and peripheral oedema

• Increased frequency of micturiction

• Rash

• Gum hyperplasia and visual disturbance

1
NITRATES

• Glyceryl trinitrite, Isosorbide mononitrate

• Lipid soluble; nitric oxide (NO) donors

• NO acts by activating guanylate cyclase, resulting in increased cyclic


GMP concentrations

• Cause smooth muscle relaxation - effect on veins / venules greater


than the effect on arterioles, resulting in a greater reduction in cardiac
pre-load

• Side-effects - flushing, sweating, headache, tachycardia, fainting

• Consumption of SH groups results in the development of tolerance

• GTN is rapidly inactivated by hepatic enzymes - sub-lingual


administration is effective. ISMO is more stable and active orally

2
DIURETICS

THIAZIDES - Bendoflumethiazide (Bendrofluazide) *****

• Reduce Na reabsorbtion in the distal tubule*. Also increase K loss

• Increased Mg loss but decreased Ca loss

• Induce hyperglycaemia and may exacerbate pre-existing diabetes mellitus

• Produce anti-hypertensive effect which is not secondary to diuresis but


probably due to vasodilation

LOOP DIURETICS - Furosemide (Frusemide), Bumetanide *****


• Rapid onset and short duration of action, cause marked diuresis

• Protein-bound

• Site of action - ascending limb of the loop of Henle *

• Cause loss of Na, Cl as well as K, Ca and Mg

POTASSIUM-SPARING DIURETICS*****
Amiloride, Triamterene, Spironolactone

• Amiloride & triamterene* - decreases K secretion in the collecting ducts

• Spironolactone *- aldosterone antagonist - inhibits Na reabsorbtion and K


secretion in the distal tubule. Also androgen antagonist may cause
gynaecomastia

• These diuretics may cause hyperkalaemia

• Acetazolamide is a carbonic anhydrase inhibitor which results in bicarbonate


loss and can cause metabolic acidosis while

All diuretics (apart from the potassium sparing diuretics) may cause hypokalaemia
and precipitate digoxin toxicity

Mannitol * is an osmotic diuretic and may be used following retention of hypotonic


fluid, for instance in hyponatraemia following trans-cervical resection of
endometrium

1
Josejosejosejocase

Question 1: Heparin

a. Anti-coagulant therapy with low molecular weight heparin is monitored


by the APTT
True False
b. Long term therapy may cause osteoporosis
True False
c. Reduces the risk of DVT following major gynaecological surgery
True False
d. May cause hyperkalaemia
True False

Question 2: The following are recognised effects of beta-adrenergic


agonists

a. Increased glycogenolysis in the liver


True False
b. Decreased bronchial secretions
True False
c. Bronchodilation
True False
d. Increased uterine tone
True False

Question 3: The following are side-effects of nifedipine

a. Headache
True False
b. Hypertension
True False
c. Bradycardia
True False
d. Constipation
True False

1
Question 4: Heparin

a. May be given by intramuscular injection


True False
b. Requires the presence of plasma antithrombin III for anticoagulant
activity
True False
c. May cause thrombocytopaenia
True False
d. May cause allopaecia
True False

Question 5: Heparin

a. Is a glycoprotein
True False
b. Occurs naturally in the body
True False
c. Crosses the placenta
True False
d. Is contra-indicated in breastfeeding women
True False

Question 6: Warfarin

a. Reduces factor IX production in the liver


True False
b. Increases plasma factor VII concentration
True False
c. Is protein bound
True False
d. Is mainly excreted by the kidneys
True False

2
Question 7: The following are characteristic of the diuretics listed

a. Frusemide ? inactive on oral administration


True False
b. Frusemide ? rapid onset and short duration of diuresis
True False
c. Frusemide ? protein bound
True False
d. Frusemide ? site of action is the distal convoluted tubule
True False

Question 8: With respect to Heparin

a. Therapy is monitored by the prothrombin time


True False
b. Anticoagulant effect is reversed with vitamin K
True False
c. Anticoagulant effect is reversed by protamine sulphate
True False
d. Low molecular weight heparin does not cause thrombocytopaenia
True False

Question 9: Warfarin

a. Inhibits vitamin K absorbtion


True False
b. Does not cross the placenta
True False
c. Is terratogenic
True False
d. May safely be used in the third trimester of pregnancy
True False

3
Question 10: The following are recognised effects of beta-adrenergic
agonists

a. Hypertension
True False
b. Tachycardia
True False
c. Decreased cardiac output
True False
d. Increased gut motility
True False

Question 11: Glyceryl trinitrate

a. Is a nitric oxide inhibitor


True False
b. Functions by activating guanylate cyclase
True False
c. Causes vasodilation and tachycardia
True False
d. Is an effective tocolytic
True False

Question 12: The following are side-effects of nifedipine

a. Peripheral oedema
True False
b. Pre-mature closure of the ductus arteriosus
True False
c. Impotence
True False
d. Gynaecomastia
True False

Question 13: The following are characteristic of the diuretics listed

4
a. Bendrofluazide ? effective anti-hypertensive agent (
True False
b. Bendrofluazide ? exacerbation of pre-existing diabetes mellitus
True False
c. Bendrofluazide ? site of action is the proximal convoluted tubule
True False
d. Bendrofluazide ? precipitation of digoxin toxicity
True False

Question 14: Labetolol

a. Is an alpha-adrenergic antagonist
True False
b. Is contraindicated in diabetics
True False
c. Causes uterine smooth muscle relaxation
True False
d. Is contraindicated in asthmatics
True False

Question 15: The following are recognised side-effects of ritodrine

a. Tachycardia
True False
b. Hyperkalaemia
True False
c. Hypoglycaemia
True False
d. Hypertension
True False

5
BENZODIAZEPINES *****

• Anxiolytic, sedative and cause amnesia

• Cross the placenta and may cause neonatal respiratory


depression and hypotonia

• Not terratogenic. Present in breast milk

• Absorbed orally

• May cause severe respiratory depression which should be


treated with the benzodiazepine antagonist flumazenil

• TEMAZEPAM - more rapid onset and shorter duration of


action than diazepam *

• LORAZEPAM - more prolonged sedation than temazepam;


causes marked amnesia *

• MIDAZOLAM - water soluble (diazepam is relatively water


insoluble) and therefore preferred for intravenous
administration *

• DIAZEPAM - causes skeletal muscle relaxation *

TRICYCLIC ANTIDEPRESSANTS

• Amitriptyline, Imipramine, Dothiepin

• Prevent noradrenaline re-uptake in the CNS. Also have anti-


muscarinic effects / side-effects

• Antidepressant effect may take 2-3 weeks to manifest

• Metabolised by demethylation into active metabolites which


have twice the potency of the parent compound

• Cross the placenta, are not terratogenic but may cause


neonatal withdrawal - tachycardia and irritability. Not contra-
indicated in breastfeeding *

• Sedative potential varies but amitriptyline is sedative

Side-Effects

1
1) ANTI-CHOLINERGIC - dry mouth, blurred vision, constipation, urinary
retention *

2) CARDIOVASCULAR - arrhythmias (contraindication), tachycardia, postural


hypotension

3) NEURO-PSYCHIATRIC - hypomania, dyskinesia, convulsion

4) Testicular enlargement, gynaecomastia and galactorrhoea

SELECTIVE SEROTONIN RE-UPTAKE INHIBITORS (SSRIs)

• Fluoxetine, Citalopram, Paroxetine

• Inhibit serotonin re-uptake

• Not terratogenic *

• Present in breast milk but not contraindicated *

• Lower risk of cardiovascular side-effects compared to tricyclic


antidepressants - not contraindicated in cardiac arrhythmias

Side-Effects

1) GI - nausea, vomiting, diarrhoea, abdominal pain, anorexia and weight loss


(weight gain with citalopram)

2) Hypersensitivity - rash, urticaria, angioedema

3) Convulsions, galactorrhoea, movement disorder, hypomania

ANTI-PSYCHOTIC DRUGS *****

• Chlorpromazine, Haloperidol, Promethazine, Trifluoperazine*

• Dopamine antagonist but also have anti-muscarinic, anti-


histamine (H1) and anti-adrenergic (alpha) effects / side-effects

• Not terratogenic. Breastfeeding should be discontinued during


treatment*
• Recognised cause of galactorrhoea (dopamine antagonist) *

Side-Effects

1) Movement disorders - Parkinsonism, tardive dyskinesia, akathisia, dystonia

2) Postural hypotension, tachycardia, arrhythmias

2
3) Impaired temperature regulation causing hypothermia / hyperthermia

4) Dry mouth, blurred vision, constipation, urinary retention

5) Menstrual irregularity, galactorrhoea, gynaecomastia, impotence *

6) Photosensitization and skin pigmentation

LITHIUM CARBONATE *****

• Effective in bipolar disorders and mania

• Crosses placenta and is associated with cardiac anomaly


(Ebstein's anomaly)

• Associated with neonatal hypotonia, poor feeding, cyanosis


and hypothyroidism

• May cause hypothyroidism

• Requires monitoring of serum lithium concentrations every 3


months and thyroid function tests every 6-12 months

• Toxicity: hyper reflexia, hyperextension of limbs, convulsion,


oliguria, coma

Side-effects

1) GI - anorexia, vomiting, diarrhoea

2) Polydypsia & polyuria

3) Muscle weakness, tremor, sluggishness, ataxia.

3
DULOXETINE FOR URINARY STRESS INCONTINENCE

• Combined serotonin and noradrenaline re-uptake inhibitor

• Increased synaptic concentrations of noradrenaline and 5HT within the


pudendal nerve results in increased stimulation of the urethral
sphincter

• Increases sphincter activity in the storage phase of the micturiction


cycle

• Recommended dose 40mg twice daily

• Meta-analysis of placebo - controlled trials show that duloxetine


resulted in:

1) A significant reduction in incontinence episodes per week

2) A significant reduction in social embarrassment and psychological impact of


incontinence and a significant improvement in quality of lif

• The maximum duration of treatment in the trials was 12 weeks and the
age of women treated was 22 - 83 years

Side-effects

• GI disturbance particularly nausea and dry mouth

• Headache, decreased libido, anorgasmia

• Withdrawal reaction is characterised by headache, nausea,


paraesthesia, dizziness and anxiety - drug should not be stopped
abruptly and dose should be reduced over a 2 week period

Contraindications

• Pregnancy, lactation
• Hepatic impairment
• Monoamine oxidase therapy
• Lowers seizure threshold therefore avoid in epilepsy
• Can enhance the anti-coagulant effects of warfarin
• Metabolised by the same enzymes as ciprofloxacin and fluvoxamine -
avoid co-prescription
• Avoid co-prescription with SSRIs and tricyclic anti-depressants

1
Role of duloxetine

• Long-term data are currently unavailable

• Potentially useful in women awaiting surgery or women who have not


completed their family - contraindicated in pregnancy

• May be used in women who wish to avoid surgical intervention -


physiotherapy remains a useful first line option

• Cannot be considered as an alternative to surgery as long-term data


not available

2
OPIOID ANALGESICS *****

• Morphine, diamorphine, pethidine, codeine phosphate, tramadol,


methadone, fentanyl

• Three receptors - mu, kappa and delta

• Have analgesic, sedative, anaesthetic effects and cause cough suppression

• Analgesia: diamorphine > morphine > pethidine > codeine phosphate

• Pethidine better absorbed orally than morphine

• Methadone is longer acting than morphine and causes less


sedation

• Tramadol causes less constipation and respiratory depression than


morphine

• Loperamide is a selective opioid constipating agent

• Tolerance (decreased response to continuous drug use) can develop after


short term treatment such as post-operative pain releive) but physical
dependence only occurs after long term therapy

Side-Effects:

• Nausea & vomiting, constipation, sedation, respiratory depression,


itching (morphine causes histamine release - may be fatal in asthmatics),
miosis (constrict pupils), hypotension *

• Analgesia and respiratory depression are reversed by the opioid


antagonist naloxone (duration of action 30 minutes) *

Morphine *****

• Increases the tone and decreases peristalsis. This increases gastric stasis
and causes constipation. *

• Increase biliary-tract pressure - some patients with biliary colic may


experience worsening rather than relief of pain

• Increases the tone of urinary-tract smooth muscle. Net effect is variable


and may cause urgency / retention

• Rapid iv injection may cause hypotension

1
• Cause histamine release - dilation of cutaneous blood vessels, with
resulting flushing of the face and neck, pruritus, and sweating.

• Only about 40% of the administered oral dose reaches the central
compartment because of first-pass metabolism in the gut and liver

• Crosses the placental membrane and has been found in breast milk *

• Converted to glucuronide metabolites; only a small fraction (less than


5%) of absorbed morphine is demethylated.

• Elimination half-life is approximately 2 to 4 hours

• Maximum analgesic effects occur 60 mins after oral dose *

PARACETAMOL *****

• Analgesic, anti-pyretic but without anti-inflammatory properties

• Similar analgesic efficacy to aspirin

• Inhibits central cyclo-oxygenase in preference to the peripheral enzyme

• Maximum daily dose if 4g for adults

• Metabolised by the liver

• Overdosage causes hepatic and renal damage. Liver damage


prevented by anti-oxidants such as acetylcysteine

2
Quejosestion 1: The following are opioid drugs

a. Morphine
True False
b. Apomorphine
True False
c. Loperamide
True False
d. Codein phosphate
True False

Question 2: The following drugs have an anti-emetic effect

a. Paclitaxel
True False
b. Carboplatin
True False
c. Ondansetron
True False
d. Domperidone
True False

Question 3: With respect to inhalation anaesthetic agents

a. These agents are used for induction but not for maintenance of
anaesthesia
True False
b. These agents can be administered with 5% oxygen
True False
c. Halothane is a volatile liquid at room temperature
True False
d. Diethyl ether is highly inflammable
True False

1
Question 4: With respect to inhalation anaesthetic agents

a. Nitrous oxide is analgesic at sub-anaesthetic concentrations


True False
b. Prolonged exposure to nitrous oxide may cause macrocytic anaemia
True False
c. Isoflurane is a gas at room temperature
True False
d. Muscle relaxants are usually not required for major abdominal surgery
True False

Question 5: The following drugs typically cause nausea and vomiting

a. Oestrogen
True False
b. Metoclopramide
True False
c. Levodopa
True False
d. Cisplatin
True False

Question 6: Chlorpromazine

a. Is a recognised cause of hypothermia


True False
b. Is a regognised cause of movement disorder
True False
c. Is a recognised cause of postural hypotension
True False
d. Has anti-nicotinic side-effects
True False

2
Question 7: Lignocaine

a. When used with adrenaline, the concentration of adrenaline should be


1:20,000
True False
b. Has a more rapid onset of anaesthesia than bupivacaine
True False
c. Has a longer duration of action than bupivacaine
True False
d. Causes cardiac arrhythmia
True False

Question 8: With respect to opioid drugs

a. Therapy is associated with dilatation of the pupils


True False
b. Respiratory depression, but not analgesia are reversed by naloxone
True False
c. Methadone is longer acting and causes more sedation compared to
morphine
True False
d. Tramadol causes less constipation and respiratory depression
compared to morphine
True False

Question 9: Lignocaine

a. Increases sodium permeability in excitable cells (


True False
b. Is lipid soluble
True False
c. Blocks small diameter neurons more readily than large diameter
neurons
True False
d. Blocks the sensation of temperature and touch before the sensation of
pain
True False

3
Question 10: With respect to intravenous anaesthetic agents

a. Thiopentone sodium is a barbiturate


True False
b. Thiopentone sodium has analgesic properties
True False
c. Reversal of anaesthesia is secondary to drug metabolism to inactive
metabolites
True False
d. Propofol typically causes a ?hangover? effect
True False

Question 11: Fluoxetine

a. Does not enter breast milk


True False
b. Is contraindicated in patients with cardiac arrhythmias
True False
c. Causes nausea and vomiting as side-effects
True False
d. Causes anorexia and weight loss
True False

Question 12: With respect to Paracetamol

a. The maximum daily dose for adults is 8g


True False
b. Overdosage causes renal damage
True False
c. Overdosage causes liver damage
True False
d. The analgesic effects mediated via the opiate receptors
True False

Question 13: Lithium carbonate

4
a. Therapy requires regular assessment of serum lithium concentrations
True False
b. Clearance and therefore therapeutic dose is decreased in pregnancy
True False
c. Toxicity is associated with hyporeflexia
True False
d. Causes polyuria and polydypsia
True False

Question 14: The following are side-effects of local anaesthetic agents

a. Convulsion
True False
b. Respiratory failure
True False
c. Hypotension
True False
d. Ischaemic necrosis of digits
True False

Question 15: Methyldopa

a. Is a substrate for aromatic l-amino acid decarboxylase


True False
b. Is eventually converted to alpha-methylnoradrenaline
True False
c. Reduces the effects of adrenergic neurons in the CNS
True False
d. Reduces the effects of dopaminergic neurons in the CNS
True False

5
DRUG INTERACTIONS

A) INDUCERS OF HEPATIC MIXED FUNCTION OXIDASE ENZYME *****

• Carbamazepine

• Ethanol

• Groseofulvin

• Phenobarbitone

• Phenytoin

• Rifampicin

These drugs will INCREASE the metabolism of drugs that are inactivated via
the MFO system (glucocorticoids, oral contraceptives, theophylline and
warfarin) and REDUCE their efficacy. Higher doses are therefore required to
attain the same therapeutic effect. *

B) INHIBITORS OF HEPATIC ENZYMES *****

• MONO-AMINE OXIDASE - inhibited by phenelzine and


tranylcypromine - results in impaired metabolism of tyramine

• ALDEHYDE DEHYDROGENASE - inhibited by disulfiram,


chlorpropamide, metronidazole - results in impaired alcohol
metabolism

• XANTHINE OXIDASE - inhibited by allopurinol - impairs


metabolism of azathioprine and mercaptopurine

• MIXED FUNCTION OXIDASE - inhibited by:

1)Phenelzine

2)Cimetidine

3)Isoniazide

4)Sodium valproate

5)Sulphonamides

6)Erythromycin

7)Theophylline
Lead to impaired metabolism of drugs such as warfarin, increasing its activity.

1
C) METHOTREXATE INTERACTIONS *****

• Analgesics - Renal excretion reduced by aspirin and


NSAIDs which also displace methotrexate from binding sites *

• Antibacterial agents - antifolate agents (trimethoprim / anti-


malarials such as pyrimethamine) and sulphonamides
increase toxicity. Penicillins and probenecid reduce renal
excretion and increase toxicity *

• Phenytoin increases anti-folate effects

• Corticosteroids increase haematological toxicity

D) OTHER INTERACTIONS

• PROTEIN BINDING -Aspirin can displace warfarin and


methotrexate from binding sites on albumin *

• GUT FLORA - Broad spectrum antibiotics reduce gut flora,


reducing vitamin K synthesis and entero-hepatic circulation of
oestrogen. As a result, the anticoagulant effects of warfarin
are increased and the efficacy of oral contraceptives is
reduced *

• AMINOGLYCOSIDE antibiotics such as gentamicin


potentiate the activity of competitive nicotinic antagonists such
as tubocurarine

• DIGOXIN toxicity is exacerbated by hypokalaemia -this is


exacerbated by diuretics (not potassium sparing diuretics)

• Beta agonists and corticosteroids are used to reduce


perinatal mortality / morbidity from pre-term delivery. Both
drugs have a diabetogenic effect and should be used with
care in diabetics *

• Insulin and beta agonists can both cause hypokalaemia *

E) ATYPICAL ACETYLATION

• Autosomal recessive defect in acetylation enzyme (hepatic


N-acetyltransferase), produces fast and slow acetylators

• Slow acetylators - accumulate drugs that are metabolised


by acetylation: Isoniazide, dapsone, hydralazine, phenelzine,
sulphasalazine, nitrazepam and procainamide

2
F) MALIGNANT HYPERTHERMIA

• Impaired ability of skeletal muscles to sequester calcium,


precipitated by Halothane and Suxamethonium

• Results in muscle contraction, increased metabolic heat


production and lactic acidosis

• Mortality is reduced by dantrolene - interferes with calcium


release from the sarcoplasmic reticulum

G) GLUCOSE-6-PHOSPHATE DEHYDROGENASE DEFICIENCY *****

Drug induced haemolysis precipitated by:

• Nalidixic acid

• Salicylic acid

• Sulphonamides

• Chloroquine

• Primaquine

H) DRUG INTERACTIONS - COMBINED ORAL CONTRACEPTIVE *****

• Broad spectrum antibiotics - reduce enterohepatic


circulation of oestrogen *

• Anti-epileptics (EXCEPT VALPROATE) - induce hepatic


enzymes *

• Rifampicin - induces hepatic enzymes

• Griseofulvine - induces hepatic enzymes

• Anti-viral agents - nevirapine, nelfinavir, ritonavir - induce


hepatic enzymes *

• Modafinil - induces hepatic enzymes

• St. John’s wort reduces the efficacy of COCP *

• Combined oral contraceptives delay the excretion of


theophylline, antagonise the anti-coagulant effect of warfarin
(tamoxifen also *) and increase plasma concentration of
corticosteroids

3
I) DRUG INTERACTIONS - LITHIUM

• ACE-inhibitors - reduce excretion, increase toxicity

• NSAIDs - reduce excretion, increase toxicity

• Thiazide / loop diuretics - reduce excretion, increase toxicity

• Anti-epileptics - may cause neurotoxicity without increased


Li concentrations

• Methyldopa - may cause neurotoxicity without increased Li


concentration *

• Theophylline and sodium bicarbonate increase Li excretion


and reduce toxicity

J) DRUG INTERACTIONS -ERGOMETRINE *****

• Halothane - reduces the effects of ergometrine on the


uterus *

• Antibiotics such as erythromycin and azithromycin;


terracycline - increased risk of ergotism

• Anti-virals - nelfinavir, ritonavir - increased risk of ergotism

• Sympathetic agonists - increased risk of ergotism

K) SURGERY AND LONG-TERM MEDICATION *****

DRUGS THAT SHOULD BE CONTINUED

• Anti-epileptics

• Anti-psychotics

• Bronchodilators

• Cardiac drugs

• Anti-parkinsonian agents

• Immunosuppressants

• Thyroid and anti-thyroid drugs

• Anti-glaucoma drugs
• Corticosteroids (additional cover may be required)

4
DRUGS THAT SHOULD BE DISCONTINUED *****

• Anti-coagulants

• Combined oral contraceptive pill

• Monoamine oxidase inhibitors (at least 2 weeks pre-op)

• Lithium (24h pre-op)

5
Question 1: The following drugs should be discontinued before general
anaesthesia for major elective surgery

a. Combined oral contraceptive pill


True False
b. Anti-epileptic drugs
True False
c. Warfarin
True False
d. Lithium
True False

Question 2: The following drugs reduce the efficacy of the combined


oral contraceptive pill

a. St. John?s Wort


True False
b. Carbamazepine
True False
c. Sodium valproate
True False
d. Griseofulvine
True False

Question 3: The following drugs precipitate haemolysis in individuals


with glucose-6-phosphate dehydrogenase deficiency

a. Warfarin
True False
b. Nalidixic acid
True False
c. Salicylic acid
True False
d. Chloroquine
True False

1
Question 4: The following drugs reduce the efficacy of the combined
oral contraceptive pill

a. Nevirapine
True False
b. Prednisolone
True False
c. Modafinil
True False
d. Theophylline
True False

Question 5: The following drugs are terratogenic in humans

a. Stilboestrol
True False
b. Carbimazole
True False
c. Phenytoin
True False
d. Sodium valproate
True False

Question 6: The following drugs inhibit hepatic mixed function oxidase


enzyme

a. Metronidazole
True False
b. Warfarin
True False
c. Griseofulvin
True False
d. Disulfiram
True False

2
Question 7: The following drug interactions are correct

a. Broad spectrum antibiotics reduce efficacy of combined oral


contraceptive pill
True False
b. Aspirin displaces warfarin from its binding sites on albumin
True False
c. Sodium valproate induces hepatic enzymes and reduces the efficacy of
the combined oral contraceptive pill
True False
d. Insulin exacerbates hypokalaemia caused by beta agonist
True False

Question 8: The following drugs can precipitate methotrexate toxicity

a. Aspirin
True False
b. Diclofenac
True False
c. Folic acid
True False
d. Sulphonamides
True False

Question 9: The following drugs increase lithium toxicity

a. Captopril
True False
b. Diclofenac
True False
c. Sodium bicarbonate
True False
d. Methyldopa
True False

3
Question 10: The following drugs inhibit hepatic mixed function oxidase
enzyme

a. Sodium valproate
True False
b. Phenytoin
True False
c. Isoniazide
True False
d. Erythromycin
True False

Question 11: The following drugs should be discontinued before general


anaesthesia for major elective surgery

a. Tricyclic antidepressants
True False
b. Monoamine oxidase inhibitors
True False
c. Bronchodilators
True False
d. Immunosuppressive agents
True False

Question 12: The following drugs increase lithium toxicity

a. Frusemide
True False
b. Theophylline
True False
c. Probenecid
True False
d. Carbamazepine
True False

4
Question 13: The following drugs induce hepatic mixed function oxidase
enzyme

a. Cimetidine
True False
b. Isoniazide
True False
c. Phenobarbitone
True False
d. Rifampicin
True False

Question 14: The following drugs are terratogenic in humans

a. Cytotoxic drugs
True False
b. Isotrentinoin
True False
c. Heparin
True False
d. Warfarin
True False

Question 15: The following drugs increase the risk of ergotism when
used in combination with ergometrine

a. Azithromycin
True False
b. Nelfinavir
True False
c. Noradrenaline
True False
d. Oxytocin
True False

5
CHEMOTHERAPEUTIC AGENTS

CELL CYCLE DEPENDENCE

• Cell cycle dependent drugs have a killing effect on cells in


a particular phase of the cell cycle

• These drugs are usually anti-metabolites and are most


effective in tumours with a large proportion of actively
dividing cells

• Methotrexate - S-phase

• Mercaptopurine, flurouracil - S-phase

• Vinca alkaloids - M-phase

• Women should be formally assessed by a medical


oncologist for their suitability for chemotherapy. Such
assessment would include FBC, renal and hepatic function
and ECG.

• Drugs used in gynaecological malignancies include


alkylating agents, platinum agents, taxanes, anti-metabolites
(methotrexate) and vinca alkaloids. The side-effect profiles
of these drugs vary but include nausea & vomiting, myelo-
suppression with neutropaenia and risk of sepsis,
neuropathy and long term risk of leukaemia.
• Nausea & vomiting can be reduced by use of 5HT
antagonist ondansetron, H2 antagonists and corticosteroid
• Other side-effects can be monitored by regular FBC, renal
and liver function tests
• No proven mechanisms to reduce the risk of alopecia -
appropriate drug selection

ALKYLATING AGENTS *****

• Form covalent bonds with DNA side chains, causing


depurination, strand breaks and cross-linking.
• Active throughout all phases of the cell cycle; kill both
rapidly and slowly proliferating cells, causing delayed,
prolonged and even permanent bone marrow failure.
• Cause amenorrhoea and oligospermia / azospermia
• Mutagenic and carcinogenic effect on bone marrow,
causing acute myeloid leukaemia - risk related to drug dose
and peak incidence is between 5-10 years post-treatment.
Risk may be as high as 5-10% following treatment for
ovarian cancer

1
• Aactive orally

• Drugs include Cyclophosphamide, chlorambucil,


melphalan, treosulfan, ifosfamide *
• Cyclophosphamide * - inactive, converted in liver to active
drug 4-hydroxy-cyclophosphamide. Decomposes within
cells to form phosphoramine mustard (anti-tumour activity)
and acrolein (excreted in urine and responsible for urothelial
toxicity). Less myelotoxic with lower risk of leukaemia
compared to other alkylating agents. Associated with acute
haemorrhagic cystitis (as with its analogue ifosfamide) - can
be prevented by maintaining high urine out-put or by the co-
administration of N-acetylcysteine or mesna - neutralise
acrolein.
• Other side effects include reversible alopecia, darkened
skin and nails, pneumonitis and pulmonary fibrosis; renal
failure.

2
PLATINUM AGENTS *****

Mode of action - cross-link DNA strands

CISPLATIN

• Most active in the G1 phase of the cell cycle but functions in a


phase non-specific manner

• Administered inrea-venously, 90% protein bound

• Excreted via the kidneys - rate of clearance increased by


diuresis

• Direct renal tubular toxin and renal damage is the major dose-
limiting toxicity and is associated with hypomagnesaemia. Renal
damage can be reduced by forced diuresis using saline or
mannitol or by administration in hypertonic saline.

• Ototoxic - tinnitus / high frequency hearing loss

• Peripheral neuropathy - stocking & glove distribution

• Severe nausea & vomiting

CARBOPLATIN

• Causes less nausea & vomiting, renal or to-toxicity than


cisplatin

• Dose-limiting toxicity is myelosupression

• Excreted via the kidneys - dose adjusted according to patient’s


creatinine clearance

1
TAXANES *****

• Act by binding to micro-tubules and preventing de-polymerisation into


tubulin dimmers and disrupting mitosis - opposite effect to vinca
alkaloids

PACLITAXEL

• Administered by slow intravenous infusion over 3-24h

• Associated with hypersensitivity reactions - risk reduced by pre-


treatment with corticosteroids, H1 and H2 antagonists

• 95% protein bound

• Metabolised by hepatic cytochrome P450 dependent pathways. 5-


10% excreted by the kidneys

• Hepatic enzyme inducers accelerate paclitaxel metabolism

• In combination chemotherapy with platinum agents, paclitaxel should


be administered first as this reduces risk of myelotoxicity

• Principal toxicity is neutropaenia - onset 8-10 days after


administration with complete recovery bay 15-21 days

• Other side-effects include asymptomatic bradycardia during


administration and other more serioud brady-arrhythmias including
heart block; mucositis and sensory peripheral neuropathy. Alopecia is
almost universal and involves all body hair sites including eye brows,
pubic and axillary hair.

• Very little tendency to cause nausea & vomiting

1
2
TOPOISOMERASE I INHIBITORS

TOPOTECAN

• Administered intra-venously, excreted primarily by the kidneys,


reduce dose if impaired renal function

• Dose-limiting toxicity is neutropaenia

• Other side -effects include mucositis, nausea & vomiting,


alopecia, rash and elevated liver enzymes.

ANTIMETABOLITES *****

• Folic acid analogues - Methotrexate

• Inhibits dihydrofolate reductase

• 50% protein bound, excreted principally by the kidneys. Renal


excretion inhibited by the administration of weak organic acids
such as aspirin and penicillin. Aspirin also displaces drug from its
binding sites compounding toxicity. *

• When given in high doses may precipitate in urine - prevented


by diuresis or alkalinisation of urine

• First line drug in the treatment of choriocarcinoma

• Mucositis is a significant complication with risk of secondary


infection or intestinal perforation

• Nausea & vomiting usually absent, myelo-suppression is


frequent but mild.

• Methotrexate - oral ulceration

• 5-flurouracil - myelosupression / mucositis

• Other agents include mercaptopurine, gemcitabine,


cytarabine *

1
VINCA ALKALOIDS

VINCRISTINE / VINBLASTINE

• Bind to tubulin preventing polymerisation and disrupting mitosis with


arrest of cells in G2 and M phase.

• Associated with neurotoxicity as a result of interaction with tubulin -


depressed tendon reflexes and paraesthaesia, motor weakness,
cranial nerve palsies and paralytic ileus

• Administered intravenously, highly protein bound and metabolised by


the liver

• Myelo-suppression very common with vinblastine but extremely rare


with vincristine

• Extravasations may cause skin damage

2
AAaAaja

Question 1: Cyclophosphamide

a. Is an alkylating agent
True False
b. Causes amenorrhoea and oligospermia
True False
c. Is an anti-metabolite
True False
d. Is active only during the S phase of the cell cycle
True False

Question 2: The following chemotherapeutic agents are cell-cycle phase


dependent

a. Methotrexate
True False
b. Vincristine
True False
c. Flurouracil
True False
d. Mercaptopurine
True False

Question 3: The following chemotherapeutic agents are cell-cycle phase


dependent

a. Cisplatin
True False
b. Carboplatin
True False
c. Cyclophosphamide
True False
d. Doxorubicin
True False

1
Question 4: Methotrexate

a. May be used in the management of ectopic pregnancy


True False
b. Is used in the treatment of choriocarcinoma
True False
c. Excretion is inhibited by penicillin administration
True False
d. Is associated with mucositis
True False

Question 5: The following are recognised side-effects of paclitaxel

a. Bradycardia
True False
b. Loss of eye-brows and axillary hair
True False
c. Hyper-sensitivity
True False
d. Neutropaenia
True False

Question 6: Paclitaxel

a. Metabolism is inhibited by hepatic enzyme inducers


True False
b. Should be administered by intravenous injection over 30 minutes
True False
c. Prevents the de-polymerisation of micro-tubules into tubulin dimmers
True False
d. Has a mechanism of action opposite to that of the vinca alkaloids
True False

2
Question 7: The following drugs are alkylating agents

a. Ifosfamide
True False
b. Carboplatin
True False
c. Vincristine
True False
d. Etoposide
True False

Question 8: With respect to chemotherapeutic agents used in


gynaecological oncology

a. Cis-platin is more nephro-toxic than carboplatin


True False
b. Cis-platin causes less nausea and vomiting than carboplatin
True False
c. Forced diuresis is usually employed during paclitaxel therapy
True False
d. Intestinal perforation is a recognised complication of methotrexate
True False

Question 9: Cyclophosphamide

a. Is converted by the liver to the active metabolite 4-hydroxy


cyclophosphamide
True False
b. Decomposes within cells releasing acrolein
True False
c. Is toxic to the urothelium
True False
d. Can cause renal failure
True False

3
Question 10: Cis-platin

a. Acts by cross-linking DNA


True False
b. Is most active when administered in the G1 phase of the cell cycle
True False
c. Is protein bound
True False
d. Clearance is inhibited by diuresis
True False

Question 11: Methotrexate

a. Inhibits protein synthesis


True False
b. Is protein-bound
True False
c. Is excreted mainly by the liver
True False
d. Toxicity is prevented by aspirin
True False

Question 12: Cyclophosphamide

a. Is a pro-drug
True False
b. Causes acute haemorrhagic cystitis
True False
c. Is inactivated by the liver
True False
d. Is associated with an increased long-term risk of acute myeloid
leukaemia
True False

4
Question 13: Paclitaxel

a. Is a taxane
True False
b. Is a pro-drug
True False
c. Is protein-bound
True False
d. Is associated with hypersensitivity reactions
True False

Question 14: The following are recognised side-effects of


cyclophosphamide

a. Darkened skin and nails


True False
b. Hypersensitivity reactions
True False
c. Oto-toxicity
True False
d. Pneumonitis and pulmonary fibrosis
True False

Question 15: With respect to the side-effects of chemo-therapeutic


agents for ovarian cancer

a. Carboplatin is more nephrotoxic than cis-platin


True False
b. Forced diuresis reduces the incidence of nephrotoxicity in women
receiving platinum agents
True False
c. Ondansetron has been shown to significantly reduce the incidence of
vomiting in women receiving platinum agents
True False
d. Dexamethasone is useful in reducing chemotherapy associated
vomiting
True False

5
Act at 4 main sites

• Cell wall synthesis

• Protein synthesis

• Nucleic acid synthesis

• Cell membrane function

BACTERIAL RESISTANCE

CAUSED BY:

• Single gene mutations - random events, with the mutant that results in resistance being
selected in the presence of the antibiotic

• Plasmid mediated resistance

• Transposons - 'jumping genes' - resistance genes that are capable of intergration into the
chromosome or plasmids

BETA-LACTAM ANTIBIOTICS *****

• Inhibit bacterial cell wall synthesis *

• Analogues of d-alanyl-d-alanine

• Cell wall synthesis is dependent on transpeptidation which links soluble peptidoglycan


precursors emerging from the cell membrane into the insoluble growing peptidoglycan chain.
This reaction is inhibited by beta-lactam antibiotics - act outside the cell membrane

• Effective against all gram positive bacteria and gram negative cocci

• Excreted by the kidneys - penicillin excretion inhibited by probenecid *

• Resistance mainly via beta-lactamase production *

• Include penicillins and cephalosporins

ANTIBIOTICS AND CELL WALL SYNTHESIS *****

• Beta lactams - inhibit transpeptidation (penicillins, monobactams, carbapenams and


ceplalosporins)

• Cycloserine - inhibits reactions incorporating alanine into cell wall precursor within the
cytoplasm

• Glycopeptides - vancomycin, teicoplanin - bind to terminal d-alanyl-d-alanine residue,


preventing incorporation into growing peptidoglycan chain

• Bacitracin - prevents dephophorylation of phosphilipid carrier, preventing regeneration of


carrier required for continuous synthesis
GLYCOPEPTIDES ****

• Vancomycin and teicoplanin

• Inhibit bacterial cell wall synthesis by binding to d-alanyl-d-alanine at the end of a pentapeptide
chain, preventing the incorporation of new subunits

• Large molecules which cannot penetrate gram negative cell wall. Only active against gram
positive bacteria

• Not absorbed from the GI tract. Oral administration used to treat Clostridium difficile diarrhoea

• Potentially ototoxic and nephrotoxic - vancomycin should be administered slowly to prevent


'red-man' syndrome. Teicoplanin is less toxic and can be given bolus
AMINOGLYCOSIDES *****

• Gentamicin, tobramycin, netilmicin, streptomycin

• Inhibit bacterial protein synthesis *- interfere with the binding of


formylmethionyl-transfer RNA to the ribosomes, preventing the
formation of the initiation complex

• Not absorbed from the GI tract.* Do not cross the blood-brain barrier

• Not active against streptococci. Active against gram negative bacteria

• Excreted via the kidneys *

• Potentially ototoxic and nephrotoxic *

• Production of aminoglycoside modifying enzymes is the most important


mechanism of acquired bacterial resistance - usually plasmid mediated

TETRACYCLINES *****

• Inhibit bacterial protein synthesis by preventing aminoacyl tRNA from


entering the acceptor site on ribosomes *

• Absorbed from the gut. Doxycycline and minocycline are better


absorbed than the others *

• Penetrate tissues well and are active against intracellular bacteria *

• Effective against chlamydial, mycoplasma and rickettsiae infections *

• Cross placenta - interferes with bone development and cause brown


discolouration of teeth - contraindicated in pregnancy and children *

• Widespread resistance as a result of their use as growth promoters in


animal feed - resistance gene carried on a transposon

CHLORAMPHENICOL

• Inhibits bacterial protein synthesis by blocking peptidyl transferase

• Well absorbed from the gut

• Active against a wide range of bacteria. Main indication is the treatment


of Salmonella typhi
• Causes REVERSIBLE dose-dependent bone marrow suppression *

MACROGLIDES *****

• Erythromycin, azithromycin, clarithromycin and spiramycin

• Inhibit bacterial protein synthesis by binding to 23S rRNA, blocking


translocation *

• Well absorbed from the gut; penetrate tissues well and are active
against intracellular organisms *

• Erythromycin is concentrated in the liver and excreted via bile

CLINDAMYCIN *****

• Clindamycin belongs to the group of Lincosamides

• Binds to 50S ribosomal subunit, inhibiting peptide bond formation.

• Similar antibiotic spectrum to erythromycin, excreted by the liver, active


in faeces for up to 5 days after a dose

• More active against anaerobes than erythromycin. Clostridium difficile


is resistant and may be selected, causing pseudomembranous colitis
ANTI-FUNGAL AGENTS

• Azole compounds - clotrimazole, ketoconazole, fluconazole: inhibit


lanosterol C14 demethylase,an important enzyme in sterol biosynthesis

• Polyene compounds - amphotericin B, nystatin: bind ergosterol in cell


membrane resulting in leakage of cellular contents

• Flucytosine: pro-drug, taken up by fungal cells and converted into 5-


fluorouracil. Inhibits DNA synthesis

• Griseofulvin: inhibits mitosis and nucleic acid synthesis

ANTI-VIRAL AGENTS

• Acyclovir - acycloguanosine. Inhibits herpes virus DNA polymerase.


Effective against herpes simplex, varicalla zoster but NOT against CMV
or Epstein-Barr virus

• Ganciclovir - active against CMV

• Foscarnet - used for CMV retinitis in AIDS

• Zidovudine- HIV

• Ribavirin - aerosol for severe RSV infection; severe influenza B infection

• Amantidine -specifically inhibits replication of influenza A but has no


activity against influenza B
eft] Pharmacology
The following antibiotics are usually effective against pseudomonas
aeruginosa: 1
F Cephradine. A
F Amoxicillin. B
T Carbenicillin. C
T Gentamicin. D
F Trimethoprim. E

√ The following antibiotics act on bacterial cell walls: 2


T Penicillin A
T Ceftazidime B
F Metronidazole C
F Clindamycin D
F Gentamicin E
Comment:
Metranidazole inhibit nucleic acid synthesis.
Clindamycin and gentamycin inhibit protein synthesis.

listeria monocytogenes: 3
T Can grow at 6c. A
F Is gut commensal B
F Is a gram-negative bacillus C
F Infection is best treated with benzylpenicillin D
T Is a cause of septicaemia in neonates E

Metronidazole: 4
T Is effective against giardia lamblia A
T Is effective when administered per rectum B
F Should not be administered intravenously C
T Is usually effective against entamoeba histolytica D
T Interferes with ethanol metabolism E

Metronidazole: 5
F Is a folic acid antagonist A
F Is ineffective when given per rectum B
F Should not be administered intravenously C
T Is effective against Entamoeba histolytica D
T Interferes with ethanol metabolism E

Gentamicin: 6
T Is ineffective systemically when given by mouth A
F Is metabolized prior to excretion by the kidney B
T May cause damage to the eighth cranial nerve C
T Is a bacteriostatic drug D
T Toxicity is potentiated by frusemide E

Co-trimoxazole: 7
T Contains two different drugs. A
T Inhibits folic acid synthesis. B
T Potentiates the action of warfarin. C
F Is bacteriostatic D
T Displaces methotrexate from protein binding sites. E
Comments 2:
Combination of sulphonamide and trimethorpim, with synergistic
bactericidal effect through inhibition of folic acid synthesis. It
potentiates the effects of warfarin (like erythromycin, isoniazid) and
the sulphonamide component can displace methotrexate from its
plasma protein binding site.+
The following substances are sympathomimetic amines: 8
T Amphetamines A
T Ephedrine B
F Histamine C
T Isoprenaline D
F Chlorpromazine E

Sympathomimetic drugs in therapeuticc doses: 9


T Cause tachycardia A
F Cause hypotension B
F Cause a decrease in cardiac output C
T Cause arrhythmias in association with hydrocarbon anaesthetics D
F Are contraindicated in thyrotoxicosis E
Comments 2:
Beta-sympathomimetic drugs in therapeutic doses causes a direct
inotropic and chronotropic effect on the heart. There is little or no
effect on the mean blood pressure because the increase in blood
pressure resulting from increased heart rate and contractility is
counteracted by the decrease in total peripheral resistance due to
vasodilation in blood vessels perfusing skeletal muscle. Arrhythmias
can occur in large doses – tachycardia is commonly associated with
their use. Beta-sympathomimetic drugs are not contraindicated in
thyrotoxicosis but may produce an increase in heart rate eg you can
use beta agonists to treat asthma in a patient with thyrotoxicosis.
The following drugs are beta-sympathomimetic agonists: 10
T Adrenaline A
T Noradrenaline B
F Phenylephrine C
T Ritodrine hydrochloride D
F Oxprenolol hydrochloride E
Comment:
Phenylephrine is α– agonist
Beta sympathomimetic drugs may: 11
F Cause bronchospasm A
T Reduce the frequency of uterine contractions B
F Cause heart block C
T Reduce diastolic blood pressure D
T Increase blood glucose concentration E
Comments 2:
These drugs have the following effects vasodilation, bronchial
relaxation, intestinal and genitourinary wall relaxation, cardiac
stimulation, renin release, glycogenolysis, gluconeogenesis,
lipolysis.
The following are beta-mimetic effects 12
F Constriction of bronchioles A
T Increased heart rate B
F A decrease in the force of cardiac contraction C
F Constriction of arterioles in the skin D
T Increased glycogenolysis in skeletal muscle E

The following agents are bronchodilators: 13


T Salbutamol A
F Atenolol B
F Prostaglandin F2α C
F Morphine D
T Prednisolone E

The following drugs can cause bronchoconstriction: 14


T Propranolol A
F Atropine B
T Morphine C
F Ritordine D
T Aspirin E
Comments 2:
These bronchoconstrictors act in the following manner - Porpanolol
by being a beta blocker and Morphine causes histamine release.
Aspirin induced bronchoconstriction is thought to be due to shunting
of arachidonic acid into the lipooxygenase pathway or removal of
cyclooxygenase product which prevent bronchospasm. Ritodrine is a
beta symphatomimetic and atropine is an anticholinergic agent that
causes broncodilation.
Ventilation is increased due to stimulation of central receptors by 15
T Nikethamide A
T Hypoxia B
T Doxapram C
F Phenobarbitone D
F Salbutamol E

The following drugs stimulate myometrial contractility: 16


T Vasopressin A
F Nifedipine B
F Hydralazine hydrochloride C
F Salbutamol D
F Indomethacin E

The following statements describe the action of drugs on the


myometrium: 17
T Ergometrine stimulates sympathetic alpha receptors. A
F Indomethacin inhibits contractions by blocking prostaglandin
receptors. B
F Prostaglandin E1 is a stimulant of isolated uterine tissue in vitro. C
T Oxytocin requires ionised calcium as a co-factor. D
F Magnesium sulphate is a myometrial stimulant. E

The following are features of Ergometrine maleate: 18


F It is inactive when administered orally A
F The onset of action after intravenous injection occurs in
approximately 5 minutes. B
T Transient hypertension may occur after its administration C
T Parenteral administration may result in vomiting D
F Its use is contraindicated in patients with migraine E

The following agents inhibit uterine activity: 19


T Magnesium sulphate A
T Ritodrine hydrochloride B
F Oxprenolol hydrochloride C
Fenoterol hydrobromide D
T Indomethacin E
Recognised unwanted effects of prostaglandin E include 20
F Water retention A
F Increased uterine contractility B
T Increased small bowel peristalsis C
T Flushing of the skin D
T Vomiting E

Concerning heparins: 21
T Heparin is synthesized in the lungs A
T Antithrombin III is necessary for standard heparins to exert their
anticoagulant effect B
T Factor X is inhibited by low-molecular-weight heparins C
T Low-molecular-weight heparins have a longer half-life than
standard heparins D
T Penicillins potentiate the action of low-molecular-weight heparins
E

The following statements about drug interactions are correct: 22


T Antacids decrease intestinal absorption of tetracycline. A
F The effects of warfarin are potentiated by combined oral
contraceptives. B
F The action of heparin is opposed by vitamin K C
T Alcohol metabolism is impaired by metronidazole D
F The effects of bromocriptine are potentiated by chlorpromazine. E

The following statements about anticoagulants are correct: 23


T Heparin inhibits the action of thrombin A
F The action of heparin is antagonized by vitamin k B
T Heparin increases antithrrombin III activity C
F The effects of coumarin anticoagulants are decreased by
metronidazole D
T Warfarin is greater than 80%protein-bound in plasma E
Aspirin: 24
T Inhibits cyclooxegenase. A
F Is the treatment of choice in childhood fever. B
T Is contraindicated in gout. C
T Should be avoided by women in anticoagulant therapy. D
T Has little anti –platelet activity when given in low dosage. E

Propanolol: 25
F Is a selective beta adrenergic blocking agent A
F Is not secreted in breast milk B
T Given in pregnancy slows the maternal heart rate C
T Causes bad dreams D
T Antagonises the tocolytic effect of salbutamol E

The following anti-hypertensive agents are correctly paired with


their mode of action: 26
T Captopril : angiotensin –converting enzyme inhibition A
T Phentolamine : alpha adrenoceptor blockade B
F Methyldopa : ganglion blockade C
F Hydralazine hydrochloride : angiotensin II inhibition D
T Sodium nitroprusside: vasodilatation E

The following drugs are potassium sparing diuretics: 27


T Amiloride hydrochloride A
T Triameterene B
T Spironolactone C
F Chlorothiazide D
F Frusemide E

Hypokalaemia may be caused by: 28


T Bendrofluazide A
F Digoxin B
F Spironolactone C
T Carbenoxolone D
F amiloride E
Comments 2:
Carbenoxolone may cause hypokalaemic hypertension like liquorice
through inhibition of 11beta hydroxysteroid dehydrogenase
(11bHSD). Bendroflumethiazide is a thiazide diuretic and promotes
potassium excretion. Amiloride, like spironolactone is a potassium
sparing diuretic and causes hyperkalaemia. Digoxin toxicity is
exacerbated by hypokalaemia but it does not cause hypokalaemia.
The following substances exert their diuretic actions upon the distal
convoluted tubule: 29
T Bendrofluaside A
F Fusemide B
F Bumetanide C
F Mannitol D
F Alcohol E

Fluid retention may be caused by the administration of: 30


F Spironolactone A
F Chlorothiazide B
T Diethylstilboestrol C
T Carbenoxolone D
T Prednisolone E

Parenteral administration of atropine in therapeutic doses to a


normal person causes: 31
T Impaired visual accommodation. A
F Diarrhoea. B
F Constriction of the bronchi. C
T An increase in heart rate. D
T A reduction in bronchial secretions. E

Subcutaneous atropine injection characteristically produces 32


T An increase in heart rate A
F An increase in salivation B
F Constriction of the pupil C
F A hypnotic effect D
T Decreased bronchiolar secretion. E

The following drugs have anti-cholinergic effects: 33


T Propantheline bromide. A
F Carbachol. B
F Distigmine bromide. C
T Benzhexol. D
T Atropine. E
Comment:
Carbachol- Nicotinic Agonists.
Benzhexol - used in Parkinson’s disease and is MUSCARINIC
ANTAGONISTS.
Atropine is MUSCARINIC ANTAGONISTS.
Propantheline bromide is ANTI-MUSCARINIC DRUGS FOR URINARY
FREQUENCY, ENURESIS AND URGE INCONTINENCE.
The following have an antiemetic action: 34
T Hyoscine hydrobromide A
F Morphine sulphate B
T Chlorpropamide C
T Promethazine hydrochloride D
T Perphenazine E
Comments 2:
Hyoscine is an alkaloid, Promethazine is an antihistamine,
Perphenazine is an antipsychotic and all 3 have antiemetic
properties. The sulphonyl urea chloropropramide may have
antiemetic properties and has thus been marked as true. Morphine
has an emetic action.
The following statements are true: 35
F Suxamethonium is non-depolarising muscle relaxant. A
F Huxamethonium is a ganglion blocker. B
T Tubocurarine is reversed by neostigmine C
F Streptomycin is absorbed from the gastrointestinal tract D
F Thiopentone can be given intramuscularly. E

The following stimulate peristalsis in the large bowel: 36


F Opiates A
T Liquid paraffin B
F Suxamethonium chloride C
T Neostigmine D
T Senna glycoside E

Treatment with morphine: 37


T Causes respiratory depression A
F Increases gastric motility B
T Causes side effects which may be reversed by naloxone C
T Increases the secretion of antidiuretic hormone D
F Causes papillary dilatation E

Treatment with morphine: 38


T Causes respiratory depression A
F Increases gastric motility B
T Causes side effects, all of which may be reversed C
T Increases the secretion of anti-diuretic hormone D
F Causes papillary dilatation E

The following statements about morphine are true: 39


T All of its pharmacological actions are reversed by naloxone A
T It is transferred into breast milk B
F It does not cross the placenta in significant quantites C
F Its analgesic effects last about 1 hour D
F It causes papillary dilatation E

Neostigmine in therapeutic doses: 40


F Acts for several days A
T Inhibits hydrolysis of acetylcholine B
F Causes paralytic ileus C
F Reverses the action of carbachol D
T Relieves the effects of myasthenia E

Lignocaine used as a local anaesthetic: 41


T Cause tachycardia if given as systemic injection A
F Has a longer lasting action than bupivicaine B
F Is used in combination with adrenaline for ring block C
F Causes vasoconstriction D
T Is a week base E
Comments 2:
Lidocaine may cause tachycardia and rise in blood pressure on IV
administration. Bupivicaine has a longer lasting action than
lidocaine. Adenaline is a vasoconstrictor and cannot be used in
organs with end-arteries. Local anaesthetics cause vasodilataion. All
local anaesthetics are weak bases.
Halothane produces: 42
T Cardiac arrhythmias A
F Explosive mixtures with air B
T Liver damage if given repeatedly C
T Myometrial relaxation D
F Bronchial irritation E

Thiopentone sodium administered intravenously: 43


F Is a potent muscle relaxant A
F Is predominantly excreted by the kidney B
T Binds to protein C
T Is fat soluble D
T Crosses the placenta E

Oestrogen therapy raises the plasma concentrations of: 44


T Thyroxine binding globulin. A
F Free cortisol. B
T Transferring. C
F Albumin. D
F Folate. E

The following compounds are predominantly progestogens: 45


F Buserelin A
T Dydrogesterone B
T Norethisterone C
T 17 α -hydroxyprogesterone D
F Androstenedione E

The effectiveness of a combined oral contraceptive may be reduced


by: 46
F Bromocriptine A
T Phenytoin B
T Rifampicin C
T Ampicillin D
F Sodium valproate E

Clomifene citrate: 47
F Is an anti-androgen A
T Does not stimulate ovulation directly B
T Can produce visual disturbances C
T Is generally prescribed throughout the proliferative phase of the
menstrual cycle D
T In the treatment of an ovulation, increases the risk of multiple
pregnancy E

Cyproterone acetate 48
F Is an oestrogen A
F Is used for the treatment of amenorrhea B
T Bind to androgen receptors C
F Increases libido D
T Inhibits spermatogensis E
The following are cytotoxic alkylating agents 49
T Cyclophosphamide A
F Mercaptopuracil B
T Chloroambucil C
F Fluorouracil D
F Methotrexate E

The following drugs may cause enlargement of the fetal thyroid


gland: 50
F Methyldopa A
F Thyroxine B
T Carbimazole C
F Propranolol D
T Propylthiouracil E

The following substances lower the blood glucose concentration: 51


F Adrenaline A
T Chlorpropamide B
F Chlorothiazide C
T Metformin D
F Thyroxine E

The following drugs and side effects are associated: 52


T Carbenoxolone : sodium retention A
F Chlorothiazide: hypoglycaemia B
F Salbutamol: bronchospasm C
T Clonidine: rebound hypertension D
T Phenytoin: folate deficiency E

The following drugs and side effects are associated: 53


T Methyldopa:depression A
F Paracetamol: thromboembolism B
T Indomethacin:peptic ulcer C
T Prednisolone :osteoporosis D
F Ritodrine:hypoglycaemia E

The following substances increase the serum uric acid


concentration: 54
F Colchicine A
T Cholorothiazide B
F Allopurinol C
F Probenecid D
F Phenylbutazone E

The therapeutic effect of the first drug is enhanced by the second


drug: 55
F Phenytoin: ethinyloestradiol A
F Bromocriptine: metoclopramide B
T Penicillin: probenecid C
T Ritodrine: dexamethasone D
F Warfarin: phenobarbitone E

The following factors enhance the transplacental passage of drugs:


56
T Lipid solubility A
F A high degree of ionic dissociation B
F High molecular weight C
F Protein binding D
F Uterine contractions E

The following drugs diminish detrusor contractions: 57


T Atropine. A
F Carbachol. B
T Propantheline. C
T Nifedipine. D
F Distigmine. E
Comments 2:
Atropine has anticholinergic effects. Carbachol causes the detrusor
muscle to contract. Propantheline has antimuscarinic and
ganglionic-blocking effects Nifedipine – calcium antagonist - Influx
of extracellular calcium is important for detrusor muscle
contractions and this can be blocked by calcium antagonists.
Distigmine does not have any effect on detrusor contractility.
The following drug combinations have been shown to interact to
produce the stated effects: 58
T Ampicillin enhances the anticoagulant action of warfarin. A
T Aciclovir diminishes the effect of oral contraceptives. B
F Cimetidine inhibits the metabolism of phenytoin. C
T Aspirin diminishes the action of ritodrine. D
F Tamoxifen increases the anticoagulant effect of warfarin. E
Comment:
Oestrogens (including tamoxifen) are pro-thrombotic and
antagonise warfarin effects *.
Anti-viral agents - nevirapine, nelfinavir, ritonavir - induce hepatic
enzymes *.
The following cytotoxic drugs are correctly classified: 59
F Methotrexate: alkylating agent A
T Cyclophosphomide: alkylating agent B
F Vinblastine: antimetabolite C
T Mercaptopurine: antimetabolite D
F Fluoruracil: antibiotic E

The following drugs should be avoided in renal impairment: 60


T Cephalothin A
T Cisplatin B
F Norethisterone C
F Dopamine D
T Gentamycin E
INFECTION CONTROL *****

• Disinfection - removes or kills most, but not all viable


organisms. Uses chemical or physical processes

• Sterilisation - removes or kills all viable organisms

• Pasteurisation - reduces total number of viable organisms


in fluids without affecting flavour. Effective against
intracellular organisms and some viruses but not against
spores

STERILISATION / DISINFECTION *****

• Ultr-violet irradiation - inefficient as a sterilant. Used to inhibit


bacterial growth in complex apparatus

• Dry heat -160-180C. Useful for heat resistant equipment

• Moist heat (autoclaving) - most effective method of


sterilisation. Usual cycle is 121C for 15 minutes - sufficient to
kill spores of Clostridium botulinum. Heat-sensitive instruments
such as endoscopes -use low temperature steam +
formaldehyde

• Boiling - disinfects but does not sterilise -some spores are


not killed

• Pasteurisation - reduce microbial load in fluids - 62.8 - 65.6C


for 30 minutes

• Gamma irradiation - useful for sterilising large batches of


small items such as needles

Filtration - produces particle-free fluid

• Ethylene oxide -effective sterilant but toxic and explosive

• Formaldehyde -disinfectant. Irritant.

• Iodine / iodophors -kills some spores, inactivated by blood


and other organic matter; iodine stains skin and can cause
local sensitivity

• Mercuric chloride - topical skin preparation

• Benzalkalonium chloride / cetrimide - useful as skin


disinfectants, mainly active against gram positive organisms.
Solutions can become contaminated with gram negative rods
• Chlorhexidine - useful for skin and mucous membranes.
Combination with detergents highly effective for hand
disinfection. Poor activity against TB, spores, fungi and
viruses. Inactivated by many materials including plastic, soap,
pus

• Alcohols - effective only in the presence of water (70%


alcohol). Isopropyl alcohol better for skin. Inactive against
spores and fungi, inflammable

• Hexachlorophene - more effective against gram negative


organisms. Used in soap or dusting powder but may be toxic

• Phenolics (carbolic acid, hexachlorophene, chloroxylenol) -


active against a wide range of bacteria including
mycobacteria. Do not kill spores. May cause irritation

• Glutaraldehyde - less irritant than formaldehyde, exposure


of at least 3h required to kill all microbes. Most killed within 10
minutes. Disinfects but does not sterilise heat-sensitive
instruments such as endoscopes

• Hypochlorite and chlorine -active against fungi, viruses


(including HepB and HIV), and spores as well as bacteria.
Easily inactivated by organic material- not used for skin
disinfection
Question 1: The following are effective disinfectants for use on the skin

a. Isopropyl alcohol

True False
b. Chlorhexidine

True False
c. Formaldehyde

True False
d. Benzalkonium chloride

True False

Question 2: The following are the mechanisms of action of anti-fungal agents

a. Amphotericin B ? binds ergosterol producing leakage of cellular contents

True False
b. Nystatin ? inhibit fungal DNA synthesis

True False
c. Flucytosine ? inhibit fungal DNA synthesis

True False
d. Griseofulvin ? bind ergosterol, producing leakage of cellular contents

True False

Question 3: The following anti-viral agents are effective against the viruses listed

a. Acyclovir ? herpes simplex virus

True False
b. Acyclovir ? varicella zoster virus

True False
c. Acyclovir ? cytomegalovirus

True False
d. Acyclovir ? Epstein-Barr virus

True False

Question 4: Microglide antibiotics

a. Include erythromycin

True False
b. Include gentamicin

True False
c. Include azithromycin and clarithromycin

True False
d. Include clindamycin

True False

Question 5: Tetracyclines

a. Inhibit bacterial protein synthesis

True False
b. Are not active against intracellular bacteria

True False
c. Are excreted via the liver

True False
d. Are poorly absorbed from the gut

True False

Question 6: Tetracyclines

a. Are effective against rickettsiae infections

True False
b. Are effective against mycoplasma infections

True False
c. Do not cross the placenta

True False
d. Are contra-indicated in women of child-bearing age

True False

Question 7: Sulphonamide antibiotics

a. Inhibit bacterial protein synthesis

True False
b. Inhibit bacterial cell wall synthesis

True False
c. Are derivatives of p-aminobenzoic acid

True False
d. Inhibit the synthesis of tetrahydrofolic acid

True False

Question 8: The following are effective disinfectants for use on the skin

a. 100% alcohol

True False
b. 70% alcohol

True False
c. Iodine

True False
d. Mercuric chloride

True False

Question 9: Beta-lactam antibiotics

a. Inhibit bacterial cell wall synthesis

True False
b. Inhibit peptidoglycan formation

True False
c. Act within the cell membrane
True False
d. Are effective against gram negative bacilli

True False

Question 10: The following are target sites for the action of anti-bacterial agents

a. Exotoxin synthesis

True False
b. Nucleic acid synthesis

True False
c. Cell membrane function

True False
d. Amino acid transport

True False

Question 11: The following drugs are effective against Mycobacterium tuberculosis

a. Gentamicin

True False
b. Pyridoxine

True False
c. Pyrimethamine

True False
d. Streptomycin

True False

Question 12: The following are effective methods of sterilising surgical instruments

a. Pasteurisation

True False
b. Boiling

True False
c. Ethylene oxide

True False
d. Liquid glutaraldehyde

True False

Question 13: Microglide antibiotics

a. Inhibit bacterial protein synthesis

True False
b. Are well absorbed from the gut

True False
c. Are excreted by the kidneys

True False
d. Are not effective against intracellular organisms
True False

Question 14: The following anti-viral agents are effective against the viruses listed

a. Ganciclovir ? cytomegalovirus

True False
b. Acyclovir ? HIV

True False
c. Foscarnet ? cytomegalovirus

True False
d. Zidovudine ? HIV

True False

Question 15: The following are recognised side-effects of anti-tuberculous drugs

a. Isoniazide ? peripheral neuropathy

True False
b. Ethambutol ? optic neuritis

True False
c. Rifampicin ? hypersensitivity reactions

True False
d. Rifampicin ? reduced visual acuity

True False
SULPHONAMIDES *****

• Derivatives of p-aminobenzoic acid

• Inhibit bacterial synthesis of tetrahydrofolic acid *

• Active orally, metabolised by the liver and excreted by the


kidneys

• Recognised cause of Stevens-Johnson syndrome

• Resistance is widespread - plasmid mediated

• Administered in combination with trimethoprim as


COTRIMOXAZOLE *

TRIMETHOPRIM *****

• Folic acid analogue

• Inhibits dihydrofolate reductase

• Similar structure and mechanism of action to methotrexate


and the anti-malarial agent pyrimethamine

• Absorbed from the gut

• Excreted by the kidneys

• Effective against gram negative bacilli EXCEPT


pseudomonas

• Resistance is mediated by a plasmid-encoded


dihydrofolate reductase which has a much reduced affinity
for trimethoprim

QUINOLONES *****

• Nalidixic acid, ciprofloxacin, norfloxacin

• Inhibit the activity of DNA gyrase, preventing supercoiling


of bacterial chromosome

• Active orally although significant systemic concentrations


are not achieved with nalidixic acid

• Excreted by the kidneys


• Nalidixic acid is only effective against enterobacteria and
its use is confined to the treatment of UTIs

• Other quinilones are effective against intracellular


organisms - rickettsiae, Chlamydia, L. pneumophilia, S. typhi

• Ciprofloxacin effective against P. aeruginosa *

• Resistance to quinolones is chromosomally mediated - no


evidence for plasmid mediated resistance

ANTI TB DRUGS *****

• Rifampicin - side effects - hypersensitivity, hepatitis

• Isoniazide - neuropathy (prevented with pyridoxine),


hepatitis

• Ethambutol - optic neuritis (reduced visual acuity, red-


green colour blindness), peripheral neuropathy

• Pyrazinamide - arthralgia, hepatitis

• Streptomycin

Rifampicin

• Binds to bacterial RNA polymerase preventing mRNA


synthesis

• Well absorbed from the gut

• Crosses blood-brain barrier

• Has an affinity for plastic - effective in patients with


prosthesis

• Metabolised and excreted by the liver / bile

• Results in orange colour in urine, sweat and saliva

• Reserved for the treatment of TB

METRONIDAZOLE *****

• A nitroimidazole

• Pro-drug, converted to active metabolite by reduction after


entering microbial cell *
• Well absorbed from the gut / rectum *

• Effective only against anaerobes and protozoa such as


Entamoeba histolitica, trichomonas vaginalis, Gardnerella
vaginalis, and Giardia lamblia *

• Used for surgical prophylaxis following abdominal / pelvic


surgery

• Effective treatment for pseudomembranous colitis

• Rectal administration is an effective alternative to iv


therapy *

• Causes a disulfiram-like reaction with alcohol *

• Excreted by the kidneys

NITROFURANTOIN

• Absorbed from the gut and excreted in the uring in high


enough concentrations to inhibit bacteria

• Effective only in acid urine

Effective in UTI prophylaxis


jose

Question 1: Tubocurarine

a. Is a non-competitive nicotinic antagonist


True False
b. Causes histamine release
True False
c. Use does not require the patient to be ventilated
True False
d. Effects can be reversed by anti-cholinesterase drugs
True False

Question 2: The following are recognised side-effects of muscarinic


antagonists

a. Urinary retention
True False
b. Palpitations
True False
c. Skeletal muscle weakness
True False
d. Urge incontinence
True False

Question 3: Cholinesterase inhibitors

a. Cause excessive sweating and salivation


True False
b. Cause bradycardia
True False
c. Cause bronchodilation and decreased bronchial secretion
True False
d. Are used in the treatment of myasthenia gravis
True False

1
Question 4: Muscarinic antagonists have the following effects on the
organs / tissues described

a. Pupils ? dilatation
True False
b. Ciliary muscles ? contraction
True False
c. Lacrimal glands ? increased secretion
True False
d. Salivary glands ? decreased secretion
True False

Question 5: The following are muscarinic antagonists

a. Atropine
True False
b. Carbachol
True False
c. Pilocarpine
True False
d. Hyoscine
True False

Question 6: The following are muscarinic antagonists

a. Tropicamide
True False
b. Ipratropium bromide
True False
c. Benzhexol
True False
d. Tubocurarine
True False

2
Question 7: With respect to Cholinesterase

a. The enzyme is present in blood


True False
b. Patients with atypical cholinesterase have a delay in regaining
consciousness after general anaesthesia
True False
c. Atypical cholinesterase is relatively resistant to inhibition by
cinchocaine
True False
d. Patients with atypical cholinesterase suffer prolonged paralysis
following tubocurarine administration
True False

Question 8: Cholinesterase inhibitors

a. Are used to reverse neuromuscular blockade caused by


suxamethonium
True False
b. Are used in conjunction with atropine to reduce muscarinic side-effects
True False
c. Cause respiratory paralysis
True False
d. Have their effects exacerbated by pralidoxime
True False

Question 9: The following drugs have anti-muscarinic effects on the


detrusor muscle

a. Oxybutynin
True False
b. Tolterodine
True False
c. Imipramine
True False
d. Amitriptyline
True False

3
Question 10: Muscarinic agonists have the following effects on the
organs / tissues listed

a. Ciliary muscle ? relaxation


True False
b. Heart ? tachycardia
True False
c. Bronchial glands ? increased secretion
True False
d. Bronchial smooth muscle ? relaxation
True False

Question 11: Suxamethonium

a. Is a muscarinic agonist
True False
b. Causes skeletal muscle fasciculation on intravenous administration
True False
c. Causes depolarisation blockade of neuromuscular transmission
True False
d. Causes prolonged loss of consciousness in patients with
cholinesterase deficiency
True False

Question 12: Muscarinic antagonists have the following effects on the


organs / tissues described

a. Heart ? tachycardia
True False
b. Respiratory tract ? increased bronchial secretions
True False
c. Respiratory tract ? smooth muscle relaxation
True False
d. Gut ? decreased motility
True False

Question 13: The following drugs are competitive cholinesterase

4
inhibitors

a. Neostigmine
True False
b. Benxhexol
True False
c. Malaoxon
True False
d. Edrophonium
True False

Question 14: The following are recognised side-effects of muscarinic


antagonists

a. Constipation
True False
b. Blurred vision
True False
c. Increased sweating
True False
d. Dry mouth
True False

Question 15: The following are nicotinic agonists

a. Carbachol
True False
b. Suxamethonium
True False
c. Atropine
True False
d. Tubocurarine
True False

5
TYPES OF DATA *****

• Nominal / categorical - names or categories that do not use /


require an order - eye colour, sex, ethnic group. The values of the
scale have no 'numeric' meaning

• Ordinal - numbers, names or categories

• Data are ordered - for instance, pain scores of 1-10

• The magnitude of the difference between the numbers is


unimportant - the difference between a score of 1 and 2 is not
necessarily the same as that between 4 and 5 - the intervals
between adjacent scale values are indeterminate

• Interval - data are ordered

• Intervals between adjacent scale values are equal with respect the
attribute being measured - the difference between 8 and 9 is the
same as the difference between 80 and 81

• Ratio - data are ordered with mathematically meaningful intervals

• The ratio of the numbers is mathematically meaningful

• There is a rational zero point for the scale - zero actually means
none

• Example is length / distance in cm

• Nominal and ordinal data cannot be normally distributed and their


analysis require non-parametric tests
• Interval / ratio data can be analysed by both parametric and non-
paremetric tests

DESCRIPTIVE STATISTICS *****

• Mean - numerical middle / average = sum of all the values divided


by the number of values. Data must be interval or ratio
(continuous) *

• Median - middle of the frequency distribution - half of the data


entries lie above and half below the median. Data must be ordinal,
interval or ratio *

• Mode - the value that is reported most frequently for a variable -


valid for ordinal, interval or ratio data *

• Normal distribution: mean = median = mode *

• Positively skewed data: mean > median > mode *

• Negatively skewed data: mean < median < mode *

• Variance *- measure of how spread out a distribution is. However,


all the negative values that are lower than the mean will cancel out
all the positive values that are greater. To avoid this, the average
squared deviation of each number from its mean is calculated. For
example, for the numbers 1, 2, and 3, the mean is 2 and the
variance is:

σ2 =[ (1-2) squared + (2-2) squared + (3-2) squared ] divided by 3

• Standard Deviation - square root of the variance. It is the most


commonly used measure of spread *
• Sandard error of a sample of sample size n is the sample's
standard deviation divided by the square root of n .It estimates
the standard deviation of the sample mean based on the
population mean *

• Confidence intervals *

If a series of samples are selected from a population and the mean of


each calculated, 95% of the means would be expected to fall within the
range of two standard errors above / below the mean of these means.

This common mean would be expected to lie very close to the


population mean.

The standard error of a mean provides a statement of probability about


the difference between the mean of the population and the mean of the
sample.

Mean + 2(standard error) and mean - 2(standard error) give the range
which has a 95% chance of including the population mean = 95%
confidence interval

Mean +/- 3(standard error) = 99.7% confidence interval


NORMAL DISTRIBUTION *****

Bffxfrytdfytthpae[\pirtrtyo
• Family of distributions with the same general shape.

• Symmetrical and bell-shaped

• The distribution is defined by two parameters: the mean (μ) and the
standard deviation (σ). If these are known then one knows essentially
as much as if one had access to all the data *

• Mean = median = mode *

• Data are unimodal (have one highest value) *

• Area under the curve = 1

• 68% of the data fall within 1 standard deviation of the mean (34%
above and 34% below) *

• 95% of the data fall within 2 (1.96) standard deviations of the mean *

• 99.7% of the data falls within 3 standard deviations (SD)of the mean *

• The standard normal distribution has a mean of 0 and SD of 1 *

TESTS FOR NORMALITY *****

• Visual inspection of the frequency distribution histogram -number of


peaks and symmetry

• Use statistical packages for values of


a) Skew (A distribution is skewed if one of its tails is longer than the
other) and

b) Kurtosis (degree of peakedness of a distribution). The kurtosis value


for a normal distribution is 0. Negative value -observations cluster more
closely to the centre; positive value - observations cluster less closely
to the centre

• Kolmogorov-Smirnov test - principal goodness of fit test for normal


and uniform data sets.

• Shapiro-Wilk test - reliable when n<50.

Limitations of normality tests *****

• Small samples almost always pass a normality test.

• With large samples, minor deviations from normality may be


statistically significant. However, such small deviations will not affect
the results of most statistical tests

• Decisions on which statistical tests to use should not be based on a


normality test of one data set.

PARAMETRIC & NON-PARAMETRIC TESTS *****

Parametric Tests *

Assumption

• Data are interval or ratio


• Normally distributed data (skew and kurtosis are between -1 and +1)

• Variance is equal between groups (variance can be up to 4 times


different from each other, but no more than that). This assumption only
applies to independent design

Examples include:

• Independent t-test

• Paired or repeated measures t-test

• ANOVA (with posthoc tests to compare group means)

Non-Parametric Tests *

• Do not require parametric assumptions because interval data are


converted to rank-ordered data. All tests involving ranked data, i.e.
data that can be put in order, are non-parametric

• Mann-Whitney U Test - one of the most powerful non-parametric tests


for comparing two populations. It is used to test the null hypothesis that
two populations have identical distribution functions against the
alternative hypothesis that the two distribution functions differ only with
respect to location (median), if at all

• Kruskal-Wallis Test - non-parametric test used to compare three or


more samples

• Wilcoxon Signed Ranks Test - designed to test a hypothesis about


the location (median) of a population distribution. It often involves the
use of matched pairs, for example, before and after data, in which case
it tests for a median difference of zero.
• Chi squared test
p Values *****

• Statistical data are typically accompanied by a p value, which is the


mathematical probability that the data are the result of random
chance *.

• Data with a low P value (less than or equal to 0.05) are said to be
statistically significant. p = 0.05 means that there is a 1 in 20 chance
that the data is the result of random chance *

• p = 1 means there is a 100% certainty that any association or


difference occurred purely by chance

• Type I error - incorrectly rejecting a true statistical null hypothesis. The


experimental hypothesis is supported by the data when it is in fact false

• Type II error - incorrectly accepting a null hypothesis when it is in fact


false.For example, in a clinical trial of a new drug, the null hypothesis
might be that the new drug is no better, on average, than the current
drug. A Type II error would occur if it were concluded that the two
drugs produced the same effect when, in fact, they produced different
ones.
RISK & ODDS *****

• Attributable risk: Disease rate in exposed persons minus rate in


unexposed persons

• Relative risk: ratio of rate of disease in exposed persons to rate in


unexposed persons

• Attributable risk = rate of disease in unexposed persons (relative risk -


1)

• Odds *- a ratio of the probability that an event will occur versus the
probability that the event will not occur = probability / (1-probability).

• The probability of pulling a jack of hearts is 1/52 whereas the odds of


pulling the jack of hearts is 1/51

• For example, if you draw cards and have 3 spades and 1 diamond,
then the odds of drawing a diamond = [(1/4)/ (3/4)] = 1/3 = 0.33.

• This differs from risk (or probability): the risk drawing a diamond =

(No. of diamonds) / (total No. of cards drawn) = 1/4 = 0.25.

• Odds ratio *- the ratio of odds - the ratio of the odds of an event
occurring in the exposed group versus the unexposed group.

• Most RCTs express results as risk ratios. Case-control studies (when


data are obtained retrospectively) and meta-analyses typically report
results as odds ratios

• When the primary outcome is rare, relative risk and odds ratio are ~
equal.

• Odds ratio and relative risks are presented with 95% confidence
intervals, for instance, relative risk (RR = 2.5; 95% CI 0.6 to 4.4)
• When the 95% CI of the odds ratio or relative risk includes 1 (as above)
then the groups are NOT statistically significantly different (p > 0.05)
TYPES OF STUDIES *****

• Observational - Sampling the target population and simply observing


which risk factors are present in each subject.

• Can be prospective - samples consists of subjects who possess / do


not posses the risk factor and the occurrence of the primary outcome is
observed.

• Can be retrospective - samples consists of subjects who have


exhibited the primary outcome (cases) or have not exhibited the
primary outcome (called controls) and the investigator looks
backwards to identify the risk factors in the two groups

• Experimental study - the investigator specifies the exposure for each


subject then follows the subjects to detect the effects of the exposure

• Cohort study - observational study in which outcomes in a group of


patients that received an intervention are compared with outcomes in a
similar group of patients that did not receive the intervention. Involves
identification of two groups (cohorts) of patients, one which did receive
the exposure of interest, and one which did not, and following these
cohorts forward for the outcome of interest.

• Randomised study *- participants are assigned by chance to separate


groups that compare different treatments; neither the researchers nor
the participants can choose which group. Using chance to assign
people to groups means that the groups will be similar and that the
treatments they receive can be compared objectively.

• Blind assessment *- The investigator does not know which treatment


the subject is receiving. When the subject knows which treatment they
are receiving, this is a single blind study. When neither the subject nor
the investigator know, this is a double blind study
Randomisation *****

• Process of assigning clinical trial participants to treatment


groups. Gives each participant a know chance of being assigned
to any of the groups.

• Group assignment cannot be predicted in advance.

• Removes the possibility that any differences observed


between the treatment groups are a consequence of a
systematic difference (or bias) between the groups due to factors
other than the intervention.

• Ensures similar levels of all risk factors (known and unknown)


in each group

• Successful randomisation does not, however, guarantee


perfect balance in risk factors between groups (due to the play of
chance)

• There are a variety of acceptable methods of randomisation


including random number generators and block randomisation
depending on the sample size.
• Some methods may result in unequal number of patients in the
study groups

• Methods of allocation such as alternate allocation to treatment


group, or methods based on patient characteristics such as date
of birth, order of entry into the clinic or day of clinic attendance,
are not reliably random and are therefore not acceptable
methods of randomisation

Epidemiological definitions *****

• Prevalence * - The number of people in a given population affected by


a particular disease at a given time as a proportion of the total
population at risk of developing the disease - a snapshot of all existing
cases at a specified time

• Incidence *- The number of new disease cases reported in a


population over a certain period of time, usually 1 year, as a proportion
of the total population at risk of developing the disease

• Centile (percentile) *- any of the 99 numbered points that divide an


ordered set of scores into 100 parts each of which contains one-
hundredth of the total. The 50th centile is the median

• Sensitivity *- the ability of a test to detect a disease when it is truly


present. Determined as the number of true positives divided by the
sum of true positives + false negatives (True positives / True positives
+ False negatives)

• Specificity *- the ability of a test to exclude the presence of a disease


when it is truly not present. The proportion of non-diseased patients for
whom there is a correctly negative test. (True negatives / True
negatives + False positives)

• Positive predictive value *- the probability that an individual with a


positive test has, or will develop, a particular disease that the test is
designed to detect. True positives / all positives = True positives / True
positives + False positives

• Negative predictive value *- the probability that a subject with a


negative test result actually does not have the disease. True negatives
/ all negatives = True negatives / True negatives + false negatives

• Numbers needed to treat *- numbers of patients needed to undergo


treatment to prevent one bad outcome, as compared to the alternative
treatment arm or placebo. Numerical representation of the
effectiveness of an intervention. For example, the NNT to prevent one
stroke with aspirin over two years is 38 - if you treat 38 people with
aspirin for two years one stroke will be prevented that would otherwise
have occurred

• Metaanalysis *- statistical method of combining the results of a


number of studies in an attempt to overcome the problem of reduced
statistical power in studies with small sample sizes. Analysing the
results from a group of studies can allow more accurate estimation of
effects

• Correlation *- the degree to which one phenomenon or variable is


associated with or can be predicted from another. The degree to which
a linear predictive relationship exists between random variables, as
measured by a correlation coefficient (r). May be positive (but never
larger than 1) - both variables increase or decrease together OR
negative (but never smaller than -1) - one variable increases when the
other decreases; OR zero - a change in one variable does not affect
the other

• Regression *- technique used to establish the relationship of a


dependent variable and one or more independent variables.
Regression analysis attempts to measure the degree of correlation
between the dependent and independent variables, thereby
establishing the latter's predictive values. For example, regression
analysis could predict your life expectancy by combining your
grandparents' age at death, whether you smoke...

• Statistical power * - a gauge of the sensitivity of a statistical test - its


ability to detect relationships. The probability of rejecting a null
hypothesis when it is false. In general, the statistical power increases
with your sample size. An index of the probability a study has of
obtaining a statistically significant effect. A high power of 80 percent, or
0.8, indicates that the study - if conducted repeatedly-would produce a
statistically significant effect 80 percent of the time, if one exists. On
the other hand, a power of only 0.1 means there would be a 90 percent
chance that the research missed the effect-if one exists at all. Also
called "Power"

• Perinatal mortality rate * - The total number of deaths of a fetus or


infant between the end of the 20th week gestation and the end of the
6th day of life in a calendar year per 1,000 total births (live and still) in
the same calendar year.

• Neonatal mortality rate * - the number of children dying under 28


days of age divided by the number of live births that year.

• Post-neonatal death * - includes deaths after 28 days of life but before


one year

• Infant mortality rate - number of live newborns dying under a year of


age per one thousand live births

• Maternal mortality rate *- The number of women who die while


pregnant or during the first 42 days following termination of the
pregnancy per 100,000 women of reproductive age in a given year for
any cause related to or aggravated by pregnancy, but not from
accidental or incidental causes. This is irrespective of the gestation age
or site of pregnancy. The 10th revision of the International Classification
of Diseases makes provision for including late maternal deaths
occurring between six weeks and one year afterchildbirth

• Maternal deaths are divided into Direct obstetric deaths resulting


from obstetric complications of pregnancy, from interventions,
omissions or incorrect treatment; or from a chain of events resulting
from any of these.Indirect obstetric deaths result from previously
existing disease or disease that developed during pregnancy and that
was not directly due to obstetric causes but was aggravated by the
physiologic effects of pregnancy
Question 1: The following statements are correct

a. The Student t-test is designed to correct for skew distribution

True False
b. The Chi squared test may be used when data are not normally distributed

True False
c. In a distribution that is skewed to the right, the mean is less than the median

True False
d. In a distribution that is skewed to the right, the mean lies to the left of the median

True False

Question 2: The following statements are correct

a. In a frequency distribution, the mode is the numerical middle of the distribution

True False
b. The incidence of a disease is the proportion of the population suffering from the disease at any
one time

True False
c. The mode is always greater than the mean in a normal distribution

True False
d. P = 0.001 indicates that the observation is more likely to have occurred by chance compared to
p = 0.05

True False

Question 3: In the statistical analysis of any group of numbers

a. The mean is always less than the mode

True False
b. The median always lies at the mid-point of the range

True False
c. The standard deviation is always greater than the standard error of the mean

True False
d. There are the same number of observations greater than and less than the median

True False

Question 4: The following statements are correct

a. Sensitivity is the ability of a test to correctly predict those with a disease

True False
b. Sensitivity is equal to positive predictive value

True False
c. The 95% confidence intervals must cross 1 to prove statistical significance

True False
d. An odds ratio of 1:4 means a risk of 25%

True False

Question 5: The standard deviation of a normally distributed population

a. Can be used to calculate the standard error of the mean


True False
b. Can have a negative value

True False
c. Is a test of statistical significance

True False
d. Is expressed as the square of the original units

True False

Question 6: In a randomised double-blind trial of two cholesterol lowering drugs, 400 patients
received drug A and 398 received drug B. Drug A was found to cause a greater reduction in
serum cholesterol (p = 0.001).

a. Randomisation is best achieved by alternate allocation of subjects to treatment groups

True False
b. Randomisation controls for unknown confounding variables

True False
c. Randomisation controls for known confounding variables

True False
d. It is likely that the patients receiving drug A had different characteristics from those receiving
drug B

True False

Question 7: In a sample of 1000 pregnant women, birth weight was found to be normally
distributed with a mean of 3.2kg and a standard deviation of 600g

a. 95% of babies will have birth weights between 2.0 and 4.4kg

True False
b. The standard error for birth weight can be calculated from 3.2 divided by the square root of
1000

True False
c. The variance for birth weight would be 36kg squared

True False
d. The standard error of the mean is independent of the sample size

True False

Question 8: A normal distribution

a. Allows calculation of the standard deviation

True False
b. Has a variance which is measured in units squared

True False
c. Kurtosis = 0

True False
d. 2.5% of the values in the sample lie more than 2SD below the mean

True False

Question 9: In a randomised double-blind trial comparing a drug with placebo

a. Each subject will take either the drug or placebo but not both
True False
b. The subjects can choose whether they take the drug or placebo

True False
c. The investigator decides which subject takes the drug or the placebo

True False
d. The likelihood of the trial giving statistically significant results is independent of the number of
subjects

True False

Question 10: In a randomised double-blind trial of two cholesterol lowering drugs, 400 patients
received drug A and 398 received drug B. Drug A was found to cause a greater reduction in
serum cholesterol (p = 0.001).

a. The data should have been analysed by non-parametric tests if they are not normally
distributed

True False
b. The results reach an acceptable level of statistical significance

True False
c. Unequal numbers in the two groups invalidate the trial

True False
d. Drug A has been shown to be 1000 times more effective than drug B

True False

Question 11: The WHO definition of perinatal mortality rate

a. Includes all fetuses and infants weighing 1000g or more

True False
b. Includes all fetuses and infants of gestation age more than 20 weeks

True False
c. Is expressed as deaths per 1000 live births

True False
d. Includes deaths occurring in the first month of life

True False

Question 12: The following conditions must be met before parametric tests can be used

a. Data are nominal or ordinal

True False
b. Data are normally distributed

True False
c. The variance of the groups is very different

True False
d. The data are from a randomised study

True False

Question 13: The following are correct

a. The 95% confidence interval indicates the range within which 19 out of 20 values will lie
True False
b. The p value indicates how often a result will occur by chance

True False
c. The conventionally accepted level of statistical significance is p < 0.05

True False
d. A relative risk of 0.5 is statistically significant irrespective of the 95% confidence interval

True False

Question 14: The standard deviation of a normally distributed population

a. Is a measure of the median value

True False
b. May be used to estimate the range of the distribution

True False
c. Is a measure of the scatter

True False
d. Is an estimate of skewness

True False

Question 15: In a randomised double-blind trial of two cholesterol lowering drugs, 400 patients
received drug A and 398 received drug B. Drug A was found to cause a greater reduction in
serum cholesterol (p = 0.001).

a. Randomisation eliminates investigator bias

True False
b. Randomisation has been unsuccessful because there is an unequal number of patients in the
two groups

True False
c. Randomisation may be undertaken using the date of birth of the patients

True False
d. A computer based random number generator may be used for randomisation

True False

You might also like